Harvard-MIT Mathematics Tournament 1998-2006 With Solutions

Download as pdf or txt
Download as pdf or txt
You are on page 1of 470

1998 HMMT Algebra Event

1. The cost of 3 hamburgers, 5 milk shakes, and 1 order of fries at a certain fast food
restaurant is $23.50. At the same restaurant, the cost of 5 hamburgers, 9 milk shakes,
and 1 order of fries is $39.50. What is the cost of 2 hamburgers, 2 milk shakes and
2 orders of fries at this restaurant?

2. Bobbo starts swimming at 2 feet/s across a 100 foot wide river with a current of 5
feet/s. Bobbo doesn’t know that there is a waterfall 175 feet from where he entered
the river. He realizes his predicament midway across the river. What is the minimum
speed that Bobbo must increase to make it to the other side of the river safely?

3. Find the sum of every even positive integer less than 233 not divisible by 10.

r 2( 2 + 10 )
4. Given that r and s are relatively prime positive integers such that = ,
s 5( (3 + 5 )
find r and s.

5. A man named Juan has three rectangular solids, each having volume 128. Two of the
faces of one solid have areas 4 and 32. Two faces of another solid have areas 64 and
16. Finally, two faces of the last solid have areas 8 and 32. What is the minimum
possible exposed surface area of the tallest tower Juan can construct by stacking his
solids one on top of the other, face to face? (Assume that the base of the tower is not
exposed.)

1 1 1
6. How many pairs of positive integers ( a , b ) with a ≤ b satisfy + = ?
a b 6

7. Given that three roots of f ( x ) = x 4 + ax 2 + bx + c are 2, -3, and 5, what is the value
of a + b + c ?

x + 1 3x + 4
8. Find the set of solutions for x in the inequality > when x ≠ −2, x ≠ − 92 .
x + 2 2x + 9

2 f (x)
9. Suppose f ( x ) is a rational function such that 3 f ( 1x ) + = x 2 for x ≠ 0 .
x
Find f ( −2 ) .

10. G. H. Hardy once went to visit Srinivasa Ramanujan in the hospital, and he started the
conversation with: “I came here in taxi-cab number 1729. That number seems dull to
me, which I hope isn’t a bad omen.” “Nonsense,” said Ramanujan. “The number isn’t
dull at all. It’s quite interesting. It’s the smallest number that can be expressed as the
sum of two cubes in two different ways.” Ramujan had immediately seen that
1729 = 12 3 + 13 = 10 3 + 9 3 . What is the smallest positive integer representable as the
sum of the cubes of three positive integers in two different ways?
1998 Harvard/MIT Math Tournament
CALCULUS Answer Sheet

Name:

School: Grade:

1 6

2 7

3 8

4 9

5 10

TOTAL:
CALCULUS

Question One. [3 points]


Farmer Tim is lost in the densely-forested Cartesian plane. Starting
from the origin he walks a sinusoidal path in search of home; that
is, after t minutes he is at position (t,sint) .
Five minutes after he sets out, Alex enters the forest at the origin
and sets out in search of Tim. He walks in such a way that after he
has been in the forest for m minutes, his position is (m,cos t).
What is the greatest distance between Alex and Farmer Tim while
they are walking in these paths?

Question Two. [3 points]


A cube with sides 1m in length is filled with water, and has a tiny
hole through which the water drains into a cylinder of radius 1m.
If the water level in the cube is falling at a rate of 1 cm s , at what
rate is the water level in the cylinder rising?

Question Three. [4 points]


2
Find the area of the region bounded by the graphs of y = x , y = x ,
and x = 2 .

Question Four. [4 points]


1

x x x 2 3 ∫ f(x)dx
Let f(x) = 1 + + + +…, for −1 ≤ x ≤ 1. Find e0 .
2 4 8

Question Five. [5 points]


x
sin(1−x)
Evaluate lim x .
x→1
Question Six. [5 points]
Edward, the author of this test, had to escape from prison to work
in the grading room today. He stopped to rest at a place 1,875 feet
from the prison and was spotted by a guard with a crossbow.
The guard fired an arrow with an initial velocity of 100 ft s . At the
same time, Edward started running away with an acceleration of
1 ft s 2 . Assuming that air resistance causes the arrow to decelerate at
1 ft s 2 and that it does hit Edward, how fast was the arrow moving at
ft
the moment of impact (in s )?

Question Seven. [5 points]


A parabola is inscribed in equilateral triangle ABC of side length 1 in
the sense that AC and BC are tangent to the parabola at A and B,
respectively:
Find the area between AB and the parabola.

Question Eight. [6 points]


Find the slopes of all lines passing through the origin and tangent to
2 3
the curve y = x + 39 x − 35.

Question Nine. [7 points]



1
Evaluate ∑ n⋅ 2
n= 1
n −1

Question Ten. [8 points]


Let S be the locus of all points (x, y) in the first quadrant such that
x y
+ = 1 for some t with 0<t<1. Find the area of S.
t 1− t
1998 Harvard/MIT Math Tournament
GEOMETRY Answer Sheet

Name:

School: Grade:

1 7

2 8

3 9

4 10a

5 10b

6 10c

TOTAL:
GEOMETRY

Question One. [3 points]


Quadrilateral ALEX, pictured below (but not necessarily to scale!)
can be inscribed in a circle; with m∠LAX = 20° and m∠AXE = 100° :
L

N
A E

X D
Calculate m∠EDX .

Question Two. [3 points]


Anne and Lisa enter a park that has
two concentric circular paths joined
by two radial paths, one of which is at
the point where they enter. Anne goes
in to the inner circle along the first
radial path, around by the shorter way
to the second radial path and out exit
along it to the exit. Walking at a the
same rate, Lisa goes around the outer
circle to the exit, taking the shorter enter
of the two directions around the park.

They arrive at the exit at the same time. The radial paths meet at
the center of the park; what is the angle between them?
Question Three. [4 points]
MD is a chord of length 2 in a circle of radius 1, and L is chosen on
the circle so that the area of triangle MLD is the maximized.
Find m∠MLD .

Question Four. [4 points]


A cube with side length 100cm is filled with water and has a hole
through which the water drains into a cylinder of radius 100cm. If
the water level in the cube is falling at a rate of 1 cm s , how fast is
the water level in the cylinder rising?

Question Five. [5 points]


Square SEAN has side length 2 and a N S
quarter-circle of radius 1 around E is cut
out.
Find the radius of the largest circle that
can be inscribed in the remaining figure.

A E

Question Six. [5 points]


A circle is inscribed in an equilateral
triangle of side length 1. Tangents to
the circle are drawn that cut off
equilateral triangles at each corner.
Circles are inscribed in each of these
equilateral triangles. If this process is
repeated infinitely many times, what is
the sum of the areas of all the circles?
Question Seven. [5 points]
Pyramid EARLY has rectangular base EARL and apex Y, and all of its
edges are of integer length. The four edges from the apex have
lengths 1, 4, 7, 8 (in no particular order), and EY is perpendicular to
YR.
Find the area of rectangle EARL.

Question Eight. [6 points]


It is not possible to construct a segment of length π using a
straightedge, compass, and a given segment of length 1. The
following construction, given in 1685 by Adam Kochansky, yields a
segment whose length agrees with π to five decimal places:
Construct a circle of radius 1 and call its center O. Construct a
diameter AB of this circle and a line l tangent to the circle at A.
Next, draw a circle with radius 1 centered at A, and call one of the
intersections with the original circle C. Now from C draw an arc of
radius 1 intersecting the circle around A at D, where D lies outside
of the circle centered at O. Draw OD and let E be its point of
intersection with l . Construct H on AE such that A is between
H and E, and HE=3.
The distance between B and H is then close to π; calculate its exact
value.
Question Nine. [7 points]
Let T be the intersection of the common internal tangents of circles
C1, C2 with centers O1, O2 respectively. Let P be one of the points
of tangency on C1 and let line l bisect angle O1TP . Label the
intersection of l with C1 that is farthest from T, R, and label the
intersection of l with C2 that is closest to T, S. If C1 has radius 4,
C2 has radius 6, and O1O2 = 20 , calculate (TR)(TS).

C2
R
P

T
O1 S O2

C1
l

Question Ten [8 points].


Lukas is playing pool on a table shaped like an equilateral triangle.
The pockets are at the corners of the triangle and are labeled C, H,
and T. Each side of the table is 16 feet long. Lukas shoots a ball
from corner C of the table in such a way that on the second bounce,
the ball hits 2 feet away from him along side CH.
a. [5 points]
How many times will the ball bounce before hitting a pocket?
b. [2 points]
Which pocket will the ball hit?
c. [1 points]
How far will the ball travel before hitting the pocket?
1998 HMMT Advanced Topics Event
1. Evaluate sin(1998°+237° ) sin(1998°−1653° ) .

2. How many values of x, -19 < x < 98 satisfy cos 2 x + 2 sin 2 x = 1 ?

1
3. Find the sum of the infinite series 1 + 2( 1998 1
) + 3( 1998 )2 + 4( 1998
1
)3 +Κ

sin A(3 cos 2 A + cos 4 A + 3 sin 2 A + sin 2 A cos 2 A)


4. Find the range of f ( A) = if
tan A(sec A − sin A tan A)

A≠ .
2

5. How many positive integers less than 1998 are relatively prime to 1547? (Two
integers are relatively prime if they have no common factors besides 1.)

6. In the diagram below, how many distinct paths are there from January 1 to December
31, moving from one adjacent dot to the next either to the right, down, or diagonally
down to the right?

Jan. 1 -> * * * * * * * * * *

* * * * * * *

* * * * * * * * * *

* * * * *

* * * * * * * * * * <-Dec. 31

7. The Houson Association of Mathematics Educators decides to hold a grand forum on


mathematics education and invites a number of politicians from around the United
States to participate. Around lunch time the politicians decide to play a game. In this
game, players can score 19 points for pegging the coordinator of the gathering with a
spit ball, 9 points for downing an entire cup of the forum’s interpretation of coffee, or
8 points for quoting more than three consecutive words from the speech Senator
Bobbo delivered before lunch. What is the product of the two greatest scores that a
player cannot score in this game?

8. Given any two positive real numbers x and y, then x◊y is a positive real number
defined in terms of x and y by some fixed rule. Suppose the operation x◊y satisfies
the equations
( x ⋅ y )◊y = x ( y◊y ) and ( x◊1)◊x = x◊1 for all x , y > 0 . Given that 1◊1 = 1 , find
19◊98 .

9. Bob’s Rice ID number has six digits, each a number from 1 to 9, and any digit can be
used any number of times. The ID number satifies the following property: the first
two digits is a number divisible by 2, the first three digits is a number divisible by 3,
etc. so that the ID number itself is divisible by 6. One ID number that satifies this
condition is 123252. How many different possibilities are there for Bob’s ID number?

10. In the fourth annual Swirled Series, the Oakland Alphas are playing the San Francisco
Gammas. The first game is played in San Francisco and succeeding games alternate in
location. San Francisco has a 50% chance of winning their home games, while
Oakland has a probability of 60% of winning at home. Normally, the series will
stretch on forever until one team gets a three game lead, in which case they are
declared the winners. However, after each game in San Francisco there is a 50%
chance of an earthquake, which will cause the series to end with the team that has
won more games declared the winner. What is the probability that the Gammas will
win?
Power Question - Coloring Graphs
Perhaps you have heard of the Four Color Theorem (if not, don’t panic!), which essentially says that any
map (e.g. a map of the United States) can be colored with four or fewer colors without giving neighboring
regions (e.g. Massachusetts and New Hampshire) the same color. It was, until 22 years ago, one of the
most famous unsolved problems in mathematics. The only known proof, however, is so long that it
requires a computer to carry it out. This power question will lead you through the basic definitions and
theorems of graph theory necessary to prove the Five Color Theorem, a weaker and much easier (though
still quite challenging) statement.

Part I - Graphs

Definitions: A graph is a collection of points and lines (or curves), called


vertices and edges, respectively, where each edge connects exactly two
distinct vertices and any two vertices are connected by at most one edge. We
say that two vertices are adjacent if they are connected by an edge.
Notation: For this problem we will denote vertices by capital letters and the
edge connecting vertices X and Y by XY. Vertices will be drawn as dots so that
they will be distinguishable from edge crossings. We will denote graphs by
capital script letters, usually G or H.

Example: Here is a graph with vertices A, B, C, D, and edges AB, AC, BC.
B D

A C

a. Which of the following are graphs? List the vertices and edges of each graph
and explain why the others are not graphs. [1 point each]
i. A B iv. A B C

D C D E

ii. v. B C

A B C A

iii. B vi. A B

A C E
C
D

Definition: Graph G is said to be isomorphic (from Greek roots meaning “same


structure”) to H, written G ≅ H, if the vertices of G can be given the names of
the vertices of H in such a way that G and H have the same edges. This
relabling is called an isomorphism and is reversible, thus implying H ≅ G as
well.

Example: A B B L G ≅ H since we can relable the


vertices of G as follows:
A↔B, B↔L, C↔U, D↔E
D C E U (we write the double arrows to
emphasize the reversibility of
G H the isomorphism).

b. [1 point each]
i. List the edges of H.
ii. Show that H ≅ G in the example by writing an isomorphism from H to G.

From now on the diagrams in the problem will not have the vertices explicitly
labled unless necessary. Thus to show two unlabled graphs are isomorphic we
can arbitrarily lable the vertices of one and then show that the other can be
labled with the same names to yield the same edges.

c. For each of the following pairs of graphs, state whether or not they are
isomorphic (you do not need to justify your answer). [parts i-iii are 1 point
each, parts iv-vi are 3 points each]
i. iv.

and and

ii. v.

and and

iii. vi.
and and

A couple more definitions are best given in this section, though there won’t be
any questions specifically about them until later.
Definitions: A graph H is a subgraph of a graph G if every vertex of H is a
vertex of G and every edge of H is an edge of G.
The degree of a vertex is the number of edges connected to it.
Example: B B
H is a subgraph of G. Vertex B has
degree 2 in G and degree 1 in H.

A C A C
G H

Part II - Planar Graphs


There are not many nontrivial properties possessed by all graphs, so we will restrict our attention to
graphs with certain nice properties that we can exploit to prove cool theorems. So now we need one
more round of definitions, then the real fun begins!

Definitions: A graph is planar if it is isomorphic to a graph that has been drawn


in a plane without edge crossings. Otherwise a graph is nonplanar. It shouldn’t
be too hard to see that every subgraph of a planar graph is planar.
A walk in a graph is a sequence A1 A2 ... An of not necessarily distinct vertices
in which Ak is adjacent to Ak+1 for k = 1, 2, ..., n-1.
A graph is connected if every pair of vertices is joined by a walk, or
equivalently if there is a walk that passes through every vertex at least once.
Otherwise a graph is said to be disconnected.
When a planar graph is actually drawn in a plane without edge crossings, it
cuts the plane into regions called faces of the graph.

Example: A R L Is connected since the walk


EDWARDEARLY passes through every
Y vertex at least once. It has 3 faces
(don’t forget the exterior region
when counting).
W D E
a. How many vertices, edges, and faces do each of the following graphs have?
[3 points each]
i. ii.

b. For each of the following graphs, state whether or not it is planar. You do
not need to justify your answer. [3 points each]
i. iii.

ii. iv.

Euler’s formula states if a connected planar graph has v vertices, e edges, and
f faces, then v-e+f=2. Proving this would go beyond the scope of this problem,
so you may take this formula as given.

c. [15 points] Prove that if G is planar and connected with v≥3, then
3
f ≤ e ≤ 3v − 6 .
2

d. Prove that the following graphs are nonplanar.


i. [5 points] ii. [10 points]

e. [12 points] Prove that every planar graph has at least one vertex of degree 5
or less.

Part III - Coloring


Now we can begin talking about coloring planar graphs.
Definition: We say a graph has been colored if a color has been assigned to
each vertex in such a way that adjacent vertices have different colors.
The chromatic number χ (the Greek letter chi, pronounced like sky without the
s) of a graph is the smallest number of colors with which it can be colored.

Example: Here is a graph with chromatic number 3, colored in black, green,


and red.

b g

b b

a. Calculate χ for each of the following graphs. [3 points each]


i. iii.

ii. iv.

b. [14 points] Given a graph G, let χ be the chromatic number of the graph
obtained by taking the vertices of G and drawing edges only between those
that are not adjacent in G. Prove that χ + χ ≥ 2 v .

c. Assume G is a planar graph with χ>5.


i. [5 points] Prove that at least one of the following must be isomorphic to a
subgraph of G (where the vertex V is only adjacent to the vertices shown).
V
V V
V
V V
ii. [20 points] Prove that if one of the above is a subgraph of G (where the
vertex V is only adjacent to the vertices shown) then we can remove one
vertex (and all edges touching it) from G to obtain a graph with fewer vertices
which also has χ>5 (you may use the Jordan Curve Theorem, which states that
a closed loop that does not intersect itself divides the plane into an inside and
outside, and if a continuous curve joins a point on the inside to one on the
outside then it must cross the loop).
iii. [5 points] Prove the Five Color Theorem: Every planar graph has χ≤5.
Algebra
Harvard-MIT Math Tournament
February 27, 1999

a3 −b3
1. If a@b = a−b , for how many real values of a does a@1 = 0?

2. For what single digit n does 91 divide the 9-digit number 12345n789?

3. Alex is stuck on a platform floating over an abyss at 1 ft/s. An evil physicist has arranged for
the platform to fall in (taking Alex with it) after traveling 100ft. One minute after the platform
was launched, Edward arrives with a second platform capable of floating all the way across the
abyss. He calculates for 5 seconds, then launches the second platform in such a way as to maximize
the time that one end of Alex’s platform is between the two ends of the new platform, thus giving
Alex as much time as possible to switch. If both platforms are 5 ft long and move with constant
velocity once launched, what is the speed of the second platform (in ft/s)?

d
4. Find all possible values of a where a2 − 6ad + 8d2 = 0, a 6= 0.

5. You are trapped in a room with only one exit, a long hallway with a series of doors and land
mines. To get out you must open all the doors and disarm all the mines. In the room is a panel
with 3 buttons, which conveniently contains an instruction manual. The red button arms a mine,
the yellow button disarms two mines and closes a door, and the green button opens two doors.
Initially 3 doors are closed and 3 mines are armed. The manual warns that attempting to disarm
two mines or open two doors when only one is armed/closed will reset the system to its initial state.
What is the minimum number of buttons you must push to get out?

6. Carl and Bob can demolish a building in 6 days, Anne and Bob can do it in 3, Anne and Carl
in 5. How many days does it take all of them working together if Carl gets injured at the end of
the first day and can’t come back? Express your answer as a fraction in lowest terms.

7. Matt has somewhere between 1000 and 2000 pieces of paper he’s trying to divide into piles of
the same size (but not all in one pile or piles of one sheet each). He tries 2, 3, 4, 5, 6, 7, and 8 piles
but ends up with one sheet left over each time. How many piles does he need?

8. If f (x) is a monic quartic polynomial such that f (−1) = −1, f (2) = −4, f (−3) = −9, and
f (4) = −16, find f (1).

9. How many ways are there to cover a 3 × 8 rectangle with 12 identical dominoes?

10. Pyramid EARLY is placed in (x, y, z) coordinates so that E = (10, 10, 0), A = (10, −10, 0),
R = (−10, −10, 0), L = (−10, 10, 0), and Y = (0, 0, 10). Tunnels are drilled through the pyramid
in such a way that one can move from (x, y, z) to any of the 9 points (x, y, z − 1), (x ± 1, y, z − 1),
(x, y ± 1, z − 1),(x ± 1, y ± 1, z − 1). Sean starts at Y and moves randomly down to the base of the
pyramid, choosing each of the possible paths with probability 19 each time. What is the probability
that he ends up at the point (8, 9, 0)?
Calculus
Harvard-MIT Math Tournament
February 27, 1999

1. Find all twice differentiable functions f (x) such that f 00 (x) = 0, f (0) = 19, and f (1) = 99.

2. A rectangle has sides of length sin x and cos x for some x. What is the largest possible area of
such a rectangle?

3. Find √
Z 4π 2 sin x
√ ( + 1)dx.
−4π 2 1 + x4

4. f is a continuous real-valued function such that f (x + y) = f (x)f (y) for all real x, y. If f (2) = 5,
find f (5).

5. Let f (x) = x + 2x+


1
1 for x > 0. Find f (99)f 0 (99).
2x+ 1
..
2x+ .

d
6. Evaluate dx (sin x − 43 sin3 x) when x = 15.

7. If a right triangle is drawn in a semicircle of radius 1/2 with one leg (not the hypotenuse) along
the diameter, what is the triangle’s maximum possible area?

8. A circle is randomly chosen in a circle of radius 1 in the sense that a point is randomly chosen
for its center, then a radius is chosen at random so that the new circle is contained in the original
circle. What is the probability that the new circle contains the center of the original circle?

9. What fraction of the Earth’s volume lies above the 45 degrees north parallel? You may assume
the Earth is a perfect sphere. The volume in question is the smaller piece that we would get if the
sphere were sliced into two pieces by a plane.

10. Let An be the area outside a regular n-gon of side length 1 but inside its circumscribed circle,
let Bn be the area inside the n-gon but outside its inscribed circle. Find the limit as n tends to
An
infinity of B n
.
Geometry
Harvard-MIT Math Tournament
February 27, 1999

1. Two 10 × 24 rectangles are inscribed in a circle as shown. Find the shaded area.

2. A semicircle is inscribed in a semicircle of radius 2 as shown. Find the radius of the smaller
semicircle.

3. In a cube with side length 6, what is the volume of the tetrahedron formed by any vertex and
the three vertices connected to that vertex by edges of the cube?

4. A cross-section of a river is a trapezoid with bases 10 and 16 and slanted sides of length 5. At
this section the water is flowing at π mph. A little ways downstream is a dam where the water
flows through 4 identical circular holes at 16 mph. What is the radius of the holes?

5. In triangle BEN shown below with its altitudes intersecting at X, N A = 7, EA = 3, AX = 4,


and N S = 8. Find the area of BEN .
B H E

S A

6. A sphere of radius 1 is covered in ink and rolling around between concentric spheres of radii 3
and 5. If this process traces a region of area 1 on the larger sphere, what is the area of the region
traced on the smaller sphere?

7. A dart is thrown at a square dartboard of side length 2 so that it hits completely randomly.
What is the probability that it hits closer to the center than any corner, but within a distance 1 of
a corner?

8. Squares ABKL, BCM N, CAOP are drawn externally on the sides of a triangle ABC. The line
segments KL, M N, OP , when extended, form a triangle A′ B ′ C ′ . Find the area of A′ B ′ C ′ if ABC
is an equilateral triangle of side length 2.

1
9. A regular tetrahedron has two vertices on the body diagonal of a cube with side length 12. The
other two vertices lie on one of the face diagonals not intersecting that body diagonal. Find the
side length of the tetrahedron.

10. In the figure below, AB = 15, BD = 18, AF = 15, DF = 12, BE = 24, and CF = 17. Find
BG : F G.
B C

A
D
F

2
Advanced Topics
Harvard-MIT Math Tournament
February 27, 1999

1. One of the receipts for a math tournament showed that 72 identical trophies were purchased for
$-99.9-, where the first and last digits were illegible. How much did each trophy cost?

2. Stacy has d dollars. She enters a mall with 10 shops and a lottery stall. First she goes to the
lottery and her money is doubled, then she goes into the first shop and spends 1024 dollars. After
that she alternates playing the lottery and getting her money doubled (Stacy always wins) then
going into a new shop and spending $1024. When she comes out of the last shop she has no money
left. What is the minimum possible value of d?

3. An unfair coin has the property that when flipped four times, it has the same nonzero probability
of turning up 2 heads and 2 tails (in any order) as 3 heads and 1 tail (in any order). What is the
probability of getting a head in any one flip?

4. You are given 16 pieces of paper numbered 16, 15, ..., 2, 1 in that order. You want to put them
in the order 1, 2, ..., 15, 16 switching only two adjacent pieces of paper at a time. What is the
minimum number of switches necessary?

5. For any finite set S, let f (S) be the sum of the elements of S (if S is empty then f (S) = 0).
Find the sum over all subsets E of S of ff (E)
(S) for S = {1, 2, ..., 1999}.

6. Matt has somewhere between 1000 and 2000 pieces of paper he’s trying to divide into piles of
the same size (but not all in one pile or piles of one sheet each). He tries 2, 3, 4, 5, 6, 7, and 8 piles
but ends up with one sheet left over each time. How many piles does he need?

7. Find an ordered pair (a, b) of real numbers for which x2 + ax + b has a non-real root whose cube
is 343.

8. Let C be a circle with two diameters intersecting at an angle of 30 degrees. A circle S is tangent
to both diameters and to C, and has radius 1. Find the largest possible radius of C.

9. As part of his effort to take over the world, Edward starts producing his own currency. As part
of an effort to stop Edward, Alex works in the mint and produces 1 counterfeit coin for every 99
real ones. Alex isn’t very good at this, so none of the counterfeit coins are the right weight. Since
the mint is not perfect, each coin is weighed before leaving. If the coin is not the right weight, then
it is sent to a lab for testing. The scale is accurate 95% of the time, 5% of all the coins minted are
sent to the lab, and the lab’s test is accurate 90% of the time. If the lab says a coin is counterfeit,
what is the probability that it really is?

10. Find the minimum possible value of the largest of xy, 1 − x − y + xy, and x + y − 2xy if
0 ≤ x ≤ y ≤ 1.
Oral Event
Harvard-MIT Math Tournament
February 27, 1999

1. [25] Start with an angle of 60◦ and bisect it, then bisect the lower 30◦ angle, then the upper
15◦ angle, and so on, always alternating between the upper and lower of the previous two angles
constructed. This process approaches a limiting line that divides the original 60◦ angle into two
angles. Find the measure (degrees) of the smaller angle.

2. [25] Alex, Pei-Hsin, and Edward got together before the contest to send a mailing to all the
invited schools. Pei-Hsin usually just stuffs the envelopes, but if Alex leaves the room she has to
lick them as well and has a 25% chance of dying from an allergic reaction before he gets back.
Licking the glue makes Edward a bit psychotic, so if Alex leaves the room there is a 20% chance
that Edward will kill Pei-Hsin before she can start licking envelopes. Alex leaves the room and
comes back to find Pei-Hsin dead. What is the probability that Edward was responsible?

3. [30] If x, y, and z are distinct positive integers such that x2 + y 2 = z 3 , what is the smallest
possible value of x + y + z.
P∞ cos nθ
4. [35] Evaluate n=0 2n , where cos θ = 15 .

5. [45] Let, r be the inradius of triangle ABC. Take a point D on side BC, and let r1 and r2 be
the inradii of triangles ABD and ACD. Prove that r, r1 , and r2 can always be the side lengths of
a triangle.

6. [45] You want to sort the numbers 5 4 3 2 1 using block moves. In other words, you can take
any set of numbers that appear consecutively and put them back in at any spot as a block. For
example, 6 5 3 4 2 1 → 4 2 6 5 3 1 is a valid block move for 6 numbers. What is the minimum
number of block moves necessary to get 1 2 3 4 5?
P∞ n5
7. [55] Evaluate n=1 n! .

8. [55] What is the smallest square-free composite number that can divide a number of the form
4242 . . . 42 ± 1?

9. [60] You are somewhere on a ladder with 5 rungs. You have a fair coin and an envelope that
contains either a double-headed coin or a double-tailed coin, each with probability 1/2. Every
minute you flip a coin. If it lands heads you go up a rung, if it lands tails you go down a rung. If
you move up from the top rung you win, if you move down from the bottom rung you lose. You
can open the envelope at any time, but if you do then you must immediately flip that coin once,
after which you can use it or the fair coin whenever you want. What is the best strategy (i.e. on
what rung(s) should you open the envelope)?

10. [75] A, B, C, D, and E are relatively prime integers (i.e., have no single common factor) such
that the polynomials 5Ax4 + 4Bx3 + 3Cx2 + 2Dx + E and 10Ax3 + 6Bx2 + 3Cx + D together have
7 distinct integer roots. What are all possible values of A? Your team has been given a sealed
envelope that contains a hint for this problem. If you open the envelope, the value of this problem
decreases by 20 points. To get full credit, give the sealed envelope to the judge before presenting
your solution.
Team
Harvard-MIT Math Tournament
February 27, 1999

1. A combination lock has a 3 number combination, with each number an integer between 0 and 39
inclusive. Call the numbers n1 , n2 , and n3 . If you know that n1 and n3 leave the same remainder
when divided by 4, and n2 and n1 + 2 leave the same remainder when divided by 4, how many
possible combinations are there?

2. A ladder is leaning against a house with its lower end 15 feet from the house. When the lower
end is pulled 9 feet farther from the house, the upper end slides 13 feet down. How long is the
ladder (in feet)?

3. How many non-empty subsets of {1, 2, 3, 4, 5, 6, 7, 8} have exactly k elements and do not contain
the element k for some k = 1, 2, ..., 8.

4. Consider the equation F ORT Y + T EN + T EN = SIXT Y , where each of the ten letters
represents a distinct digit from 0 to 9. Find all possible values of SIXT Y .

5. If a and b are randomly selected real numbers between 0 and 1, find the probability that the
nearest integer to a−b
a+b is odd.

q
3
√ q
3

6. Reduce the number 2+ 5+ 2− 5.

1 P∞ n
7. Let 1−x−x2 −x3
= i=0 an x , for what positive integers n does an−1 = n2 ?

8. Find all the roots of (x2 + 3x + 2)(x2 − 7x + 12)(x2 − 2x − 1) + 24 = 0.


P17 n2 +n+1
9. Evaluate n=2 n4 +2n3 −n2 −2n .

10. If 5 points are placed in the plane at lattice points (i.e. points (x, y) where x and y are both
integers) such that no three are collinear, then there are 10 triangles whose vertices are among
these points. What is the minimum possible number of these triangles that have area greater than
1/2?

11. Circles C1 , C2 , C3 have radius 1 and centers O, P, Q respectively. C1 and C2 intersect at A, C2


and C3 intersect at B, C3 and C1 intersect at C, in such a way that 6 AP B = 60◦ , 6 BQC = 36◦ ,
and 6 COA = 72◦ . Find angle ABC (degrees).

12. A fair coin is flipped every second and the results are recorded with 1 meaning heads and 0
meaning tails. What is the probability that the sequence 10101 occurs before the first occurance
of the sequence 010101?
Algebra
Ri e Mathemati s Tournament 2000

1. How many integers x satisfy jxj + 5 < 7 and jx 3j > 2?


2. Evaluate 20003 1999  20002 19992  2000 + 19993 .
3. Five students take a test on whi h any integer s ore from 0 to 100 in lusive is possible.
What is the largest possible di eren e between the median and the mean of the s ores?
4. What is the fewest number of multipli ations required to rea h x2000 from x, using only
previously generated powers of x? For example, x ! x2 ! x4 ! x8 ! x16 ! x32 !
x64 ! x128 ! x256 ! x512 ! x1024 ! x1536 ! x1792 ! x1920 ! x1984 ! x2000 uses 15
multipli ations.
5. A ja ket was originally pri ed $100. The pri e was redu ed by 10% three times and
in reased by 10% four times in some order. To the nearest ent, what was the nal
pri e?
6. Barbara, Edward, Abhinav, and Alex took turns writing this test. Working alone, they
ould nish it in 10, 9, 11, and 12 days, respe tively. If only one person works on the
test per day, and nobody works on it unless everyone else has spent at least as many
days working on it, how many days (an integer) did it take to write this test?
7. A number n is alled multipli atively perfe t if the produ t of all the positive divisors
of n is n2 . Determine the number of positive multipli atively perfe t numbers less than
100.
8. A man has three daughters. The produ t of their ages is 168, and he remembers that
the sum of their ages is the number of trees in his yard. He ounts the trees but annot
determine any of their ages. What are all possible ages of his oldest daughter?
p2 2 p2 2
9. a
a +
= db = 34 , b + d = 15. Find a + bd ad b .
10. Find the smallest positive integer a su h that x4 + a2 is not prime for any integer x.
Cal ulus
Ri e Mathemati s Tournament 2000

1. Find the slope of the tangent at the point of in e tion of y = x3 3x2 + 6x + 2000.
2. Karen is attempting to limb a rope that is not se urely fastened. If she pulls herself
up x feet at on e, then the rope slips x3 feet down. How many feet at a time must she
pull herself up to limb as eÆ iently as possible?
3. A re tangle of length 41  and height 4 is bise ted by the x-axis and is in the rst and
fourth quadrants. The graph of y = sin(x) + C divides the area of the square in half.
What is C?
4. For what value of x (0 < x < 2 ) does tan x + ot x a hieve its minimum?
P i
5. For 1 < x < 1, let f (x) = i=1 xi . Find a losed form expression (a losed form
1

expression is one not involving summation) for f .


p
6. A hallway of width 6 feet meets a hallway of width 6 5 feet at right angles. Find the
length of the longest pipe that an be arried horizontally around this orner.
7. An envelope of a set of lines is a urve tangent to all of them. What is the envelope of
the family of lines y = x20 + x(1 x12 ), with x0 ranging over the positive real numbers?
0
Hint: slope at a point P on a urve is dxdy
jP .
8. Find
R  ln sin d.
2
0
v
u s r q
u
t p
9. Let f (x) = x + 0 + x + 0 + x + : : :. If f (a) = 4, then nd f (a). 0

p
10. A mirror is onstru ted in the shape of y equals  x for 0  x  1, and 1 for
1 < x < 9. A ray of light enters at (10,1) with slope 1. How many times does it boun e
before leaving?
Geometry
Ri e Mathemati s Tournament 2000

1. How many re tangles are there on an 8x8 he kerboard?

2. In a triangle the sum of squares of the sides is 96. What is the maximum possible value
of the sum of the medians?
3. Find P B , given that P A = 15; P C = 20; P D = 7, and ABC D is a square.
A B
C #
C #
#
C #
C#
P
a
 aa
 aa
D C

4. Find the total area of the non-triangle regions in the gure below (the shaded area).
a/4

b/3

a/3
a

5. Side AB = 3. 4ABF is an equilateral triangle. Side DE = AB = AF = GE .


6 F ED = 60 Æ. F G = 1. Cal ulate the area of ABC DE .
A

G
F C B

E D
6. What is the area of the largest ir le ontained in an equilateral triangle of area 8 3?
p
x2 2
7. Let ABC be a triangle ins ribed in the ellipse 4
+ y9 = 1. If its entroid is the origin
(0,0), nd its area.
p
8. A sphere is ins ribed inside a pyramid with a square as a base whose height is 215 times
the length of one edge of the base. A ube is ins ribed inside the sphere. What is the
ratio of the volume of the pyramid to the volume of the ube?
9. How many hexagons are in the gure below with verti es on the given verti es? (Note
that a hexagon need not be onvex, and edges may ross!)

10. Let C1 and C2 be two on entri re e tive hollow metal spheres of radius R and R 3
p
respe tively. From a point P on the surfa e of C2 , a ray of light is emitted inward at 30Æ
from the radial dire tion. The ray eventually returns to P. How many total re e tions
o of C1 and C2 does it take?
General
Ri e Mathemati s Tournament 2000

1. If a = 2b + , b = 2 + d, 2 = d + a 1, d = a , what is b?
2. The temperatures f Æ F and Æ C are equal when f = 95 + 32. What temperature is the
same in both Æ F and Æ C?
3. A twelve foot tree asts a ve foot shadow. How long is Henry's shadow (at the same
time of day) if he is ve and a half feet tall?
4. Ti kets for the football game are $10 for students and $15 for non-students. If 3000
fans attend and pay $36250, how many students went?
5. Find the interior angle between two sides of a regular o tagon (degrees).
6. Three ards, only one of whi h is an a e, are pla ed fa e down on a table. You sele t
one, but do not look at it. The dealer turns over one of the other ards, whi h is not
the a e (if neither are, he pi ks one of them randomly to turn over). You get a han e
to hange your hoi e and pi k either of the remaining two fa e-down ards. If you
sele ted the ards so as to maximize the han e of nding the a e on the se ond try,
what is the probability that you sele ted it on the
(a) rst try?
(b) se ond try?
p
7. Find [ 19992000℄ where [x℄ is the greatest integer less than or equal to x.
8. Bobo the lown was juggling his spheri al ows again when he realized that when he
drops a ow is related to how many ows he started o juggling. If he juggles 1, he
drops it after 64 se onds. When juggling 2, he drops one after 55 se onds, and the other
55 se onds later. In fa t, he was able to reate the following table:
ows started juggling 1 2 3 4 5 6 7 8 9 10 11
se onds he drops after 64 55 47 40 33 27 22 18 14 13 12
ows started juggling 12 13 14 15 16 17 18 19 20 21 22
se onds he drops after 11 10 9 8 7 6 5 4 3 2 1
He an only juggle up to 22 ows. To juggle the ows the longest, what number of ows
should he start o juggling? How long (in minutes) an he juggle for?
9. Edward's formula for the sto k market predi ts orre tly that the pri e of HMMT is
dire tly proportional to a se ret quantity x and inversely proportional to y , the number
of hours he slept the night before. If the pri e of HMMT is $12 when x = 8 and y = 4,
how many dollars does it ost when x = 4 and y = 8?
10. Bob has a 12 foot by 20 foot garden. He wants to put fen ing around it to keep out
the neighbor's dog. Normal fen eposts ost $2 ea h while strong ones ost $3 ea h. If
he needs one fen epost for every 2 feet and has $70 to spend on the fen eposts, what is
the largest number of strong fen eposts he an buy?
11. If ab = a+b
a b, nd n su h that 3n = 3.
12. In 2020, the United States admits North Mathemati a as the 51st state. It onsists
of 5 islands joined by bridges as shown. Is it possible to ross all the bridges without
doubling over? If so, what is the di eren e (positive) between the number of the start
island and the number of the end island?
1 2 3

4 5

13. How many permutations of 123456 have exa tly one number in the orre t pla e?
14. The author of this question was born on April 24, 1977. What day of the week was
that?
3 5
15. Whi h is greater: (35 )(5 ) or (53 )(3 ) ?
16. Joe bikes x miles East at 20 mph to his friend's house. He then turns South and bikes
x miles at 20 mph to the store. Then, Joe turns East again and goes to his grandma's
house at 14 mph. On this last leg, he has to arry our he bought for her at the store.
Her house is 2 more miles from the store than Joe's friend's house is from the store.
Joe spends a total of 1 hour on the bike to get to his grandma's house. If Joe then rides
straight home in his grandma's heli opter at 78 mph, how many minutes does it take
Joe to get home from his grandma's house?
17. In how many distin t ways an the letters of STANTON be arranged?
18. You use a lo k with four dials, ea h of whi h is set to a number between 0 and 9
(in lusive). You an never remember your ode, so normally you just leave the lo k
with ea h dial one higher than the orre t value. Unfortunately, last night someone
hanged all the values to 5. All you remember about your ode is that none of the
digits are prime, 0, or 1, and that the average value of the digits is 5. How many
ombinations will you have to try?
19. Eleven pirates nd a treasure hest. When they split up the oins in it, they nd that
there are 5 oins left. They throw one pirate overboard and split the oins again, only
to nd that there are 3 oins left over. So, they throw another pirate over and try again.
This time, the oins split evenly. What is the least number of oins there ould have
been?
20. Given: AC has length 5, semi ir le AB has radius 1, semi ir le BC has diameter 3.
What per ent of the the big ir le is shaded?

A B C
21. Find the area of the six-pointed star if all edges are of length s, all a ute angles are 60 Æ
and all obtuse angles are 240 Æ.
22. An equilateral triangle with sides of length 4 has an isos eles triangle with the same
base and half the height ut out of it. Find the remaining area.
77
23. What are the last two digits of 77 ?
24. Peter is randomly lling boxes with andy. If he has 10 pie es of andy and 5 boxes in
a row labeled A, B, C, D, and E, how many ways an he distribute the andy so that
no two adja ent boxes are empty?
25. How many points does one have to pla e on a unit square to guarantee that two of them
are stri tly less than 1/2 unit apart?
p p
p is 5 2 miles from (0,0),
26. Janet is trying to ndpTim in a Cartesian forest. Janet p 41
miles from (1,0),pand 61 miles from (0,1). Tim is 65 miles from (0,0), 2 13 miles
from (1,0), and 58 miles from (0,1). How many miles apart are Janet and Tim?
Advan ed Topi s
Ri e Mathemati s Tournament 2000

1. How many di erent ways are there to paint the sides of a tetrahedron with exa tly
4 olors? Ea h side gets its own olor, and two olorings are the same if one an be
rotated to get the other.
p 6 p 6
2. Simplify ( 1+2i 3 ) + ( 1 2i 3 ) to the form a + bi.
3. Evaluate 1
P 21 .
n=1 n +2n

4. Five positive integers from 1 to 15 are hosen without repla ement. What is the prob-
ability that their sum is divisible by 3?
5. Find all 3-digit numbers whi h are the sums of the ubes of their digits.
6. 6 people ea h have a hat. If they shue their hats and redistribute them, what is the
probability that exa tly one person gets their own hat ba k?
p p2 2
7. Assume that a; b; ; d are positive integers, and a = db = 34 , a2 + 2 b + d = 15.

Find a + bd ad b .
8. How many non-isomorphi graphs with 9 verti es, with ea h vertex onne ted to exa tly
6 other verti es, are there? (Two graphs are isomorphi if one an relabel the verti es
of one graph to make all edges be exa tly the same.)
9. The Cin innati Reals are playing the Houston Alphas in the last game of the Swirled
Series. The Alphas are leading by 1 run in the bottom of the 9th (last) inning, and
the Reals are at bat. Ea h batter has a 31 han e of hitting a single and a 32 han e of
making an out. If the Reals hit 5 or more singles before they make 3 outs, they will
win. If the Reals hit exa tly 4 singles before making 3 outs, they will tie the game and
send it into extra innings, and they will have a 53 han e of eventually winning the game
(sin e they have the added momentum of oming from behind). If the Reals hit fewer
than 4 singles, they will LOSE! What is the probability that the Alphas hold o the
Reals and win, sending the pa ked Alphadome into a frenzy? Express the answer as a
fra tion.
10. I all two people A and B and think of a natural number n. Then I give the number
n to A and the number n + 1 to B. I tell them that they have both been given natural

numbers, and further that they are onse utive natural numbers. However, I don't tell
A what B's number is and vi e versa. I start by asking A if he knows B's number. He
says \no". Then I ask B if he knows A's number, and he says \no" too. I go ba k to
A and ask, and so on. A and B an both hear ea h other's responses. Do I ever get a
\yes" in response? If so, who responds rst with \yes" and how many times does he
say \no" before this? Assume that both A and B are very intelligent and logi al. You
may need to onsider multiple ases.
Guts
HMMT 2000
School:

Problem Gu1 [4]

The sum of 3 real numbers is known to be zero. If the sum of their cubes is π e , what is their
product equal to?

Problem Gu2 [5]

If X = 1 + x + x2 + x3 + ... and Y = 1 + y + y 2 + y 3 + ..., what is 1 + xy + x2 y 2 + x3 y 3 + ...


in terms of X and Y only?

School:

Problem Gu3 [±7]

Using 3 colors, red, blue and yellow, how many different ways can you color a cube (modulo
rigid rotations)?

Problem Gu4 [5]

Let ABC be a triangle and H be its orthocentre. If it is given that B is (0, 0), C is (1, 2)
and H is (5, 0), find A.

School:

Problem Gu5 [3]

Find all natural numbers n such that n equals the cube of the sum of its digits.

Problem Gu6 [±10]

If integers m, n, k satisfy m2 + n2 + 1 = kmn, what values can k have?

1
School:

Problem Gu7 [7]

Suppose you are give a fair coin and a sheet of paper with the polynomial xm written on it.
Now for each toss of the coin, if heads show up, you much erase the polynomial xr (where r
is going to change with time – initially it is m) written on the paper and replace it by xr−1 .
If tails show up, replace it by xr+1 . What is the expected value of the polynomial I get after
m such tosses? (Note: this is a different concept from the most probable value)

Problem Gu8 [±4]

Johny’s father tells him : “I am twice as old as you will be seven years from the time I was
thrice as old as you were”. What is Johny’s age?

School:

Problem Gu9 [6]

A cubic polynomial f satisfies f (0) = 0, f (1) = 1, f (2) = 2, f (3) = 4. What is f (5)?

Problem Gu10 [7]

What is the total surface area of an ice cream cone, radius R, height H, with a spherical
scoop of ice cream of radius r on top? (Given R < r)

School:

Problem Gu11 [6]

Let M be the maximum possible value of x1 x2 + x2 x3 + ... + x5 x1 where x1, x2, ..., x5 is
a permutation of (1, 2, 3, 4, 5) and let N be the number of permutations for which this
maximum is attained. Evaluate M + N .

Problem Gu12 [9]

Calculate the number of ways of choosing 4 numbers from the set {1, 2, ..., 11} such that at
least 2 of the numbers are consecutive.

2
School:

Problem Gu13 [±4]

Determine the remainder when (x4 − 1)(x2 − 1) is divided by 1 + x + x2 .

Problem Gu14 [7]

ABCD is a cyclic quadrilateral inscribed in a circle of radius 5, with AB = 6, BC = 7, CD =


8. Find AD.

School:

Problem Gu15 [8]

Find the number of ways of filling a 8 × 8 grid with 0’s and X’s so that the number of 0’s in
each row and each column is odd.

Problem Gu16 [5]

Solve for real x, y:

x+y = 2
x + y 5 = 82
5

School:

Problem Gu17 [5]


 
666
Find the highest power of 3 dividing 333

Problem Gu18 [±5]


∞ √ √
(tan−1 n − tan−1
P
What is the value of n + 1)?
n=1

3
School:

Problem Gu19 [3]


a−b
Define a ∗ b = 1−ab
. What is (1 ∗ (2 ∗ (3 ∗ ...(n ∗ (n + 1))...)))?

Problem Gu20 [6]

What is the minimum possible perimeter of a triangle two of whose sides are along the x-and
y-axes and such that the third contains the point (1,2)?

School:

Problem Gu21 [8]

How many ways can you color a necklace of 7 beads with 4 colors so that no two adjacent
beads have the same color?

Problem Gu22 [6]

Find the smallest n such that 22000 divides n!

School:

Problem Gu23 [5]

How many 7-digit numbers with distinct digits can be made that are divisible by 3?

Problem Gu24 [±3]

At least how many moves must a knight make to get from one corner of a chessboard to the
opposite corner?

4
School:

Problem Gu25 [4]

Find the next number in the sequence 131, 111311, 311321, 1321131211,

Problem Gu26 [5]

What are the last 3 digits of 1! + 2! + ... + 100!

School:

Problem Gu27 [±6]

What is the smallest number that can be written as a sum of 2 squares in 3 ways?

Problem Gu28 [8]

What is the smallest possible volume to surface ratio of a solid cone with height = 1 unit?

School:

Problem Gu29 [±9]


v s
u

u r q
What is the value of 1 + 2 1 + 3 1 + 4 1 + 5 1 + ...
t

Problem Gu30 [7]

ABCD is a unit square. If 6 P AC = 6 P CD, find the length BP .

5
School:

Problem Gu31 [10]

Given collinear points A, B, C such that AB = BC. How can you construct a point D on
AB such that AD = 2DB, using only a straightedge? (You are not allowed to measure
distances)

Problem Gu32 [7]

How many (nondegenerate) tetrahedrons can be formed from the vertices of an n-dimensional
hypercube?

School:

Problem Gu33 [±5]

Characterise all numbers that cannot be written as a sum of 1 or more consecutive odd
numbers.

Problem Gu34 [±6]

What is the largest n such that n! + 1 is a square?

School:

Problem Gu35 [4]

If 1 + 2x + 3x2 + ... = 9, find x.

Problem Gu36 [6]


b+c−a
If, in a triangle of sides a, b, c, the incircle has radius 2
, what is the magnitude of angle
A?

6
School:

Problem Gu37 [9]

A cone with semivertical angle 30◦ is half filled with water. What is the angle it must be
tilted by so that water starts spilling?

Problem Gu38 [4]

What is the largest number you can write with three 3’s and three 8’s, using only symbols
+,-,/,× and exponentiation?

School:

Problem Gu39 [±8]

If r = 1/3, what is the value, rounded to 100 decimal digits, of


7
X 2n
2n
n=0 1 + x

Problem Gu40 [±10]

Let φ(n) denote the number of positive integers less than equal to n and relatively prime to
n. find all natural numbers n and primes p such that φ(n) = φ(np).

School:

Problem Gu41 [7]

A observes a building of height h at an angle of inclination α from a point on the ground.


After walking a distance a toward it, the angle is now 2α, and walking a further distance b
causes to increase to 3α. Find h in terms of a and b.

Problem Gu42 [4]

A n × n magic square contains numbers from 1 to n2 such that the sum of every row and
every column is the same. What is this sum?

7
School:

Problem Gu43 [6]

Box A contains 3 black and 4 blue marbles. Box B has 7 black and 1 blue, whereas Box C
has 2 black, 3 blue and 1 green marble. I close my eyes and pick two marbles from 2 different
boxes. If it turns out that I get 1 black and 1 blue marble, what is the probability that the
black marble is from box A and the blue one is from C?

Problem Gu44 [6]

A function f : Z → Z satisfies

f (x + 4) − f (x) = 8x + 20
f (x − 1) = (f (x) − x)2 + x2 − 2
2

(1)

Find f (0) and f (1).

School:

Problem Gu45 [7]


 
x
Find all positive integers x for which there exists a positive integer y such that y
= 1999000

Problem Gu46 [6]

For what integer values of n is 1 + n + n2 /2 + ... + nn /n! an integer?

School:

Problem Gu47

Find an n < 100 such that n · 2n − 1 is prime. Score will be n − 5 for correct n, 5 − n for
incorrect n (0 points for answer < 5).

8
Oral Event
HMMT 2000

1. [25] Find all integer solutions to m2 = n6 + 1.

2. [30] How many positive solutions are there to x10 + 7x9 + 14x8 + 1729x7 − 1379x6 = 0 ?
How many positive integer solutions ?

3. [35] Suppose the positive integers a, b, c satisfy an + bn = cn , where n is a positive integer


greater than 1. Prove that a, b, c > n. (Note: Fermat’s Last Theorem may not be used)

4. [40] On an n × n chessboard, numbers are written on each square so that the number in
a square is the average of the numbers on the adjacent squares. Show that all the numbers
are the same.

5. [45] Show that it is impossible to find a triangle in the plane with all integer coordinates
such that the lengths of the sides are all odd.

6. [45] Prove that every multiple of 3 can be written as a sum of four cubes (positive or
negative).

7. [45] A regular tetrahedron of volume 1 is filled with water of total volume 7/16. Is it
possible that the center of the tetrahedron lies on the surface of the water? How about in a
cube of volume 1?

8. [55] f is a polynomial of degree n with integer coefficients and f (x) = x2 + 1 for x =


1, 2, ..., n. What are the possible values for f (0) ?

9. [60] Let − →
v1 , −

v2 , −

v3 , −

v4 and →

v5 be vectors in three dimensions. Show that for some i, j in

− →

1, 2, 3, 4, 5, vi · vj ≥ 0.

10. [75] 23 frat brothers are sitting in a circle. One, call him Alex, starts with a gallon of
water. On the first turn, Alex gives each person in the circle some rational fraction of his
water. On each subsequent turn, every person with water uses the same scheme as Alex
did to distribute his water, but in relation to themselves. For instance, suppose Alex gave
1/2 and 1/6 of his water to his left and right neighbors respectively on the first turn and
kept 1/3 for himself. On each subsequent turn everyone gives 1/2 and 1/6 of the water
they started the turn with to their left and right neighbours (resp.) and keep the final third
for themselves. After 23 turns, Alex again has a gallon of water. What possibilities are
there for the scheme he used in the first turn? (Note: you may find it useful to know that
1 + x + x2 + ... + x22 has no polynomial factors with rational coefficients.)
Team Test
Ri e Mathemati s Tournament 2000

1. You are given a number, and round it to the nearest thousandth, round this result to
nearest hundredth, and round this result to the nearest tenth. If the nal result is .7,
what is the smallest number you ould have been given? As is ustomary, 5's are always
rounded up. Give the answer as a de imal.
2. The pri e of a gold ring in a ertain universe is proportional to the square of its purity
and the ube of its diameter. The purity is inversely proportional to the square of the
depth of the gold mine and dire tly proportional to the square of the pri e, while the
diameter is determined so that it is proportional to the ube root of the pri e and also
dire tly proportional to the depth of the mine. How does the pri e vary solely in terms
of the depth of the gold mine?
3. Find the sum of all integers from 1 to 1000 in lusive whi h ontain at least one 7 in
their digits, i.e. nd 7 + 17 + ::: + 979 + 987 + 997.
4. All arrangements of letters VNNWHTAAIE are listed in lexi ographi (di tionary) or-
der. If AAEHINNTVW is the rst entry, what entry number is VANNAWHITE?
1 , nd tan .
5. Given os ( + ) + sin ( ) = 0, tan = 2000
6. If is a root of x3 x 1 = 0, ompute the value of 10 + 2 8 7 3 6 3 5 +
4 4 + 2 3 4 4 6 17.
7. 8712 is an integral multiple of its reversal, 2178, as 8712=4*2178. Find another 4-digit
number whi h is a non-trivial integral multiple of its reversal.
8. A woman has $1.58 in pennies, ni kels, dimes, quarters, half-dollars and silver dollars.
If she has a di erent number of oins of ea h denomination, how many oins does she
have?
9. Find all positive primes of the form 4x4 + 1, for x an integer.
10. How many times per day do at least two of the three hands on a lo k oin ide?
11. Find all polynomials f (x) with integer oeÆ ients su h that the oeÆ ients of both
f (x) and [f (x)℄3 lie in the set f0; 1; 1g
12. At a dan e, Abhinav starts from point (a; 0) and moves along the negative x dire tion
with speed va , while Pei-Hsin starts from (0; b) and glides in the negative y-dire tion
with speed vb . What is the distan e of losest approa h between the two?
13. Let P1 ; P2 ; : : : ; Pn be a onvex n-gon. If all lines Pi Pj are joined, what is the maximum
possible number of interse tions in terms of n obtained from stri tly inside the polygon?
14. De ne a sequen e <xn> of real numbers by spe ifying an initial x0 and by the re urren e
xn+1 = 11+xxnn . Find xn as a fun tion of x0 and n, in losed form. There may be multiple
ases.
q
15. limn!1 nr 2 1 os 2 =?
n
Power Test
Ri e Mathemati s Tournament 2000

1. A derangement of a string of distin t elements is a rearrangement of the string su h


that no element appears in its original position. For example, BCA is a derangement of
ABC. D represents the number of derangements of any string omposed of n distin t
n

elements. D = 1 and D = 2.
2 3

(a) What are D and D ?


4 5

(b) How many derangments are there of the string ABCDEFG?


( ) Find a re ursive relationship for D in terms of the previous two terms (D
n n 1 and
D n ).
2

(d) Find a re ursive relationship for D in terms of only the previous term, D
n n 1 .
2. Find the number of 3-letter "words" that use letters from the 10-letter set fA; B; C; :::; J g
in whi h all letters are di erent and the letters appear in alphabeti al order.
3. Assume that a hand of thirteen ards is dealt from a randomized de k of 52 ards. Let
A be the probability that the hand ontains two a es. Let B be the probability that

it ontains two a es if you already know it ontains at least one a e. Let C be the
probability that it ontains at least two a es if you already know it ontains an a e of
hearts. Write down the inequality relationship between A,B and C , i.e. one possibility
is A = B > C .
4. Find the number of rearrangments of 12345 (in luding 12345) su h that none of the
following is true: 1 is in position 5, 2 is in position 1, 3 is in position 2, 4 is in position
4, and 5 is in position 3.
5. Assume that nobody has a birthday on February 29th. How large must a group be so
that there is a greater than 50% han e that at least 2 members have the same birthday?
6. Find the number of ombinations of length k that use elements from a set of n distin t
elements, allowing repetition.
7. Find the number of ombinations of length k that use elements from a given set of n
distin t elements, allowing repetition and with no missing elements. (Obviously, k must
be greater than n)
8. An ele tion takes pla e between two andidates. Candidate A wins by a vote of 1032
to 971. If the votes are ounted one at a time and in a random order, determine the
probability that the winner was never behind at any point in the ounting.
       
9. Evaluate the sum 100
0
+ 1
2
100
1
+ 1
3
100
2
+:::+ 1
101
100
100
.
10. Find the number of distributions of a given set of m identi al balls into a given set of
n distin t boxes.

11. Find the number of distributions of a given set of m distin t balls into a given set of
n distin t boxes if ea h box must ontain a spe i number of balls (m is the number i

of balls to be put into box i). Please state the answer with only fa torials (not in
ombinatorial notation).
12. Find the oeÆ ient of X 2
Y
3
in ea h of the following.
(a) (X + Y + 1) 7

(b) (X + Y 1)
2 7

13. Find the number of "words" of length m from a set of n letters, if ea h letter must
o ur at least on e in ea h word.
14. Find the number of ways to distribute seven distin t balls into three distin t boxes if
ea h box must ontain a di erent number of balls, allowing an empty box.
15. How many ways an a lass of 10 students be divided into two groups of 3 and 1 group
of 4?
16. Find the number of subsets A of the set of digits f0; 1; 2; 3; : : : ; 9g su h that A ontains
no two onse utive digits. Hint: Find a better statement of the problem; nd a re ursive
formula, and then attempt to solve the problem for the number of digits given.
17. If we are trying to nd the number of words of length m from a given set of n letters,
with ea h letter o uring at least on e in ea h word, let us all the answer T (m; n).
This is equivalent to nding the number of distribution of a set of m distin t balls into
a set of n distin t boxes, if no boxes an be empty. T (m; n) is the sum of all possible
partitions of the balls (i.e. we sum all possible ways of putting the balls into boxes (4
in box 1, 2 in box 2, 1 in box 3 for example)).
P 1 More pre isely, if we all m to be the i

number of balls in box i, then T (m; n) = 2


1 2 3
3m +m +m =m
m ;m ;m >=1
1 2 3 n
. For example,
m!
m !m !m !:::m !

T (3; 2) = + = 3 + 3 = 6. Find a re ursive pattern for T (m; n) in terms of


3! 3!
1!2! 2!1!

previous terms (previous meaning a smaller m, a smaller n, or both). Hint: set up a


sort of "Pas al's Triangle" for T (m; n). Prove your answer using words.
18. You have an in nite number of 1 ent, 2 ent, and 5 ent stamps. You are trying to
post a letter that requires n ents of postage stamps, where n > 8. Let a(n) be the
number of sequen es of stamps that give exa tly the required postage of n ents (i.e.
order matters). Find a(n) in terms of previous terms of the sequen e of a's, using as
few previous terms as possible.
19. Suppose we have n lines in a plane in general position, whi h means that none are
parallel to ea h other and that no three of these lines interse t at a single point. Find
the number of regions that these lines divide the plane into...
(a) in a re ursive form.
(b) in a nonre ursive formula.
20. Find the 2000th positive integer that is not the di eren e between any two integer
squares.
Algebra Test
Harvard-MIT Math Tournament
March 3, 2001

1. Find x − y, given that x4 = y 4 + 24, x2 + y 2 = 6, and x + y = 3.

2. Find (x + 1)(x2 + 1)(x4 + 1)(x8 + 1) · · · , where |x| < 1.

3. How many times does 24 divide into 100! (factorial)?

4. Given that 7, 999, 999, 999 has at most two prime factors, find its largest prime factor.

5. Find the 6-digit number beginning and ending in the digit 2 that is the product of
three consecutive even integers.

6. What is the last digit of 11 + 22 + 33 + · · · + 100100 ?

7. A polynomial P has four roots, 41 , 12 , 2, 4. The product of the roots is 1, and P (1) = 1.
Find P (0).

8. How many integers between 1 and 2000 inclusive share no common factors with 2001?

9. Find the number of positive integer solutions to nx + ny = nz with nz < 2001.

10. Find the real solutions of (2x + 1)(3x + 1)(5x + 1)(30x + 1) = 10.
Calculus Test
Harvard-MIT Math Tournament
March 3, 2001

1. A sequence of ants walk from (0, 0) to (1, 0) in the plane. The nth ant walks along
n semicircles of radius n1 with diameters lying along the line from (0, 0) to (1, 0). Let Ln be
the length of the path walked by the nth ant. Compute lim Ln .
n→∞

2. The polynomial 3x5 −250x3 +735x is interesting because it has the maximum possible
number of relative extrema and points of inflection at integer lattice points for a quintic
polynomial. What is the sum of the x-coordinates of these points?

3. A balloon that blows up in the shape of a perfect cube is being blown up at a rate such
that at time t fortnights, it has surface area 6t square furlongs. At how many cubic furlongs
per fortnight is the air being pumped in when the surface area is 144 square furlongs?

4. What is the size of the largest rectangle that can be drawn inside of a 3-4-5 right
triangle with one of the rectangle’s sides along one of the legs of the triangle?

5. Same as question 4, but now we want one of the rectangle’s sides to be along the
hypotenuse.

6. The graph of x2 − (y − 1)2 = 1 has one tangent line with positive slope that passes
through (x, y) = (0, 0). If the point of tangency is (a, b), find sin−1 ( ab ) in radians.

7. Find the coefficient of x12 in the Maclaurin series (i.e. Taylor series around x = 0) for
1
1−3x+2x2
.


cot−1 (n2 + n + 1).
P
8. Evaluate
n=0

9. On the planet Lemniscate, the people use the elliptic table of elements, a far more
advanced version of our periodic table. They’re not very good at calculus, though, so they’ve
asked for your help. They know that Kr is somewhat radioactive and deteriorates into Pl, a
very unstable element that deteriorates to form the stable element As. They started with a
block of Kr of size 10 and nothing else. (Their units don’t translate into English, sorry.) and
nothing else. At time t, they let x(t) be the amount of Kr, y(t) the amount of Pl, and z(t)
the amount of As. They know that x0 (t) = −x, and that, in the absence of Kr, y 0 (t) = −2y.
Your job is to find at what time t the quantity of Pl will be largest. You should assume that
the entire amount of Kr that deteriorates has turned into Pl.

+1
2u332 +u998 +4u1664 sin u691
R
10. Evaluate the definite integral 1+u666
du.
−1
Geometry Test
Harvard-MIT Math Tournament
March 3, 2001

1. A circle of radius 3 crosses the center of a square of side length 2. Find the positive
difference between the areas of the nonoverlapping portions of the figures.

2. Call three sides of an opaque cube adjacent if someone can see them all at once. Draw
a plane through the centers of each triple of adjacent sides of a cube with edge length 1.
Find the volume of the closed figure bounded by the resulting planes.

3. Square ABCD is drawn. Isosceles Triangle CDE is drawn with E a right angle.
Square DEF G is drawn. Isosceles triangle F GH is drawn with H a right angle. This
process is repeated infinitely so that no two figures overlap each other. If square ABCD has
area 1, compute the area of the entire figure.

4. A circle has two parallel chords of length x that are x units apart. If the part of the
circle included between the chords has area 2 + π, find x.

5. Find the volume of the tetrahedron with vertices (5, 8, 10), (10, 10, 17), (4, 45, 46), (2, 5, 4).

6. A point on a circle inscribed in a square is 1 and 2 units from the two closest sides of
the square. Find the area of the square.

7. Equilateral triangle ABC with side length 1 is drawn. A square is drawn such that its
vertex at A is opposite to its vertex at the midpoint of BC. Find the area √ √
enclosed within
2( 3+1)
the intersection of the insides of the triangle and square. Hint: sin 75 = 4
.

8. Point D is drawn on side BC of equilateral triangle ABC, and AD is extended past


D to E such that angles EAC and EBC are equal. If BE = 5 and CE = 12, determine the
length of AE.

9. Parallelogram AECF is inscribed in square ABCD. It is reflected across diagonal


AC to form another parallelogram AE 0 CF 0 . The region common to both parallelograms has
area m and perimeter n. Compute the value of nm2 if AF : AD = 1 : 4.
10. A is the center of a semicircle, with radius AD lying on the base. B lies on the base
between A and D, and E is on the circular portion of the semicircle such that EBA is a
right angle. Extend EA
√ through A√ to C, and√put√ F on line CD such √ √that EBF is a line.
Now EA = 1, AC = 2, BF = 2−4 2 , CF = 2 5+4 10 , and DF = 2 5−4 10 . Find DE.
General Test, First Half
Harvard-MIT Math Tournament
March 3, 2001

1. What is the last digit of 17103 + 5?

2. Find x + y, given that x2 − y 2 = 10 and x − y = 2.

3. There are some red and blue marbles in a box. We are told that there are twelve
more red marbles than blue marbles, and we experimentally determine that when we pick
a marble randomly we get a blue marble one quarter of the time. How many marbles are
there in the box?

4. Find a + b + c + d + e if

3a + 2b + 4d = 10,
6a + 5b + 4c + 3d + 2e = 8,
a + b + 2c + 5e = 3,
2c + 3d + 3e = 4, and
a + 2b + 3c + d = 7.

5. What is the sum of the coefficients of the expansion (x + 2y − 1)6 ?

6. A right triangle has a hypotenuse of length 2, and one of its legs has length 1. The
altitude to its hypotenuse is drawn. What is the area of the rectangle whose diagonal is this
altitude?

7. Find (x + 1)(x2 + 1)(x4 + 1)(x8 + 1) · · · , where |x| < 1.

8. How many times does 24 divide into 100!?

9. Boris was given a Connect Four game set for his birthday, but his color-blindness makes
it hard to play the game. Still, he enjoys the shapes he can make by dropping checkers into
the set. If the number of shapes possible modulo (horizontal) flips about the vertical axis of
symmetry is expressed as 9(1 + 2 + · · · + n), find n. (Note: the board is a vertical grid with
seven columns and eight rows. A checker is placed into the grid by dropping it from the top
of a column, and it falls until it hits either the bottom of the grid or another checker already
in that column. Also, 9(1 + 2 + · · · + n) is the number of shapes possible, with two shapes
that are horizontal flips of each other counted as one. In other words, the shape that consists
solely of 3 checkers in the rightmost row and the shape that consists solely of 3 checkers in
the leftmost row are to be considered the same shape.)

10. Find the 6-digit number beginning and ending in the digit 2 that is the product of
three consecutive even integers.
General Test Solutions (Second Half)
Harvard-MIT Math Tournament
March 3, 2001

1. A circle of radius 3 crosses the center of a square of side length 2. Find the difference
between the areas of the nonoverlapping portions of the figures.

2. Call three sides of an opaque cube adjacent if someone can see them all at once. Draw
a plane through the centers of each triple of adjacent sides of a cube with edge length 1.
Find the volume of the closed figure bounded by the resulting planes.

..
x.
3. Find x if xx = 2.
4. Some students are taking a math contest, in which each student takes one of four
tests. One third of the students take one test, one fourth take another test, one fifth take
the next test, and 26 students take the last test. How many students are taking the contest
in total?

5. What is the area of a square inscribed in a semicircle of radius 1, with one of its sides
flush with the diameter of the semicircle?

6. You take a wrong turn on the way to MIT and end up in Transylvania, where 99% of
the inhabitants are vampires and the rest are regular humans. For obvious reasons, you want
to be able to figure out who’s who. On average, nine-tenths of the vampires are correctly
identified as vampires and nine-tenths of humans are correct identified as humans. What is
the probability that someone identified as a human is actually a human?

7. A real numbers x is randomly chosen in the interval −15 12 , 15 12 . Find the probability
 

that the closest integer to x is odd.

8. A point on a circle inscribed in a square is 1 and 2 units from the two closest sides of
the square. Find the area of the square.

9. Two circles are concentric. The area of the ring between them is A. In terms of A,
find the length of the longest chord contained entirely within the ring.

10. Find the volume of the tetrahedron with vertices (5, 8, 10), (10, 10, 17), (4, 45, 46), (2, 5, 4).
Advanced Topics Test
Harvard-MIT Math Tournament
March 3, 2001

1. Find x − y, given that x4 = y 4 + 24, x2 + y 2 = 6, and x + y = 3.

1 1 1
  
2. Find logn 2
logn−1 3
· · · log2 n
in terms of n.

3. Calculate the sum of the coefficients of P (x) if (20x27 + 2x2 + 1)P (x) = 2001x2001 .

4. Boris was given a Connect Four game set for his birthday, but his color-blindness makes
it hard to play the game. Still, he enjoys the shapes he can make by dropping checkers into
the set. If the number of shapes possible modulo (horizontal) flips about the vertical axis of
symmetry is expressed as 9(1 + 2 + · · · + n), find n. (Note: the board is a vertical grid with
seven columns and eight rows. A checker is placed into the grid by dropping it from the top
of a column, and it falls until it hits either the bottom of the grid or another checker already
in that column. Also, 9(1 + 2 + · · · + n) is the number of shapes possible, with two shapes
that are horizontal flips of each other counted as one. In other words, the shape that consists
solely of 3 checkers in the rightmost row and the shape that consists solely of 3 checkers in
the leftmost row are to be considered the same shape.)

5. Find the 6-digit number beginning and ending in the digit 2 that is the product of
three consecutive even integers.

6. There are two red, two black, two white, and a positive but unknown number of blue
socks in a drawer. It is empirically determined that if two socks are taken from the drawer
without replacement, the probability they are of the same color is 15 . How many blue socks
are there in the drawer?

7. Order these four numbers from least to greatest: 556 , 1051 , 1735 , 3128 .

8. Find the number of positive integer solutions to nx + ny = nz with nz < 2001.

9. Find the real solutions of (2x + 1)(3x + 1)(5x + 1)(30x + 1) = 10.

10. Alex picks his favorite point (x, y) in the first quadrant on the unit circle x2 + y 2 = 1,
such that a ray from the origin through(x, y) is θ radians counterclockwise from the positive
x-axis. He then computes cos−1 4x+3y5
and is surprised to get θ. What is tan(θ)?
Guts Round Questions
Harvard-MIT Math Tournament
March 3, 2001

1. [5] January 3, 1911 was an odd date as its abbreviated representation, 1/3/1911, can
be written using only odd digits (note all four digits are written for the year). To the nearest
month, how many months will have elapsed between the most recent odd date and the next
odd date (today is 3/3/2001, an even date).

2. [4] Ken is the best sugar cube retailer in the nation. Trevor, who loves sugar, is
coming over to make an order. Ken knows Trevor cannot afford more than 127 sugar cubes,
but might ask for any number of cubes less than or equal to that. Ken prepares seven cups
of cubes, with which he can satisfy any order Trevor might make. How many cubes are in
the cup with the most sugar?

3. [7] Find the number of triangulations of a general convex 7-gon into 5 triangles by 4
diagonals that do not intersect in their interiors.


1
Q 
4. [7] Find 1− n2
.
n=2

5. [± 6] Let ABC be a triangle with incenter I and circumcenter O. Let the circumradius
be R. What is the least upper bound of all possible values of IO?

6. [8] Six students taking a test sit in a row of seats with aisles only on the two sides of
the row. If they finish the test at random times, what is the probability that some student
will have to pass by another student to get to an aisle?

7. [5] Suppose a, b, c, d, and e are objects that we can multiply together, but the
multiplication doesn’t necessarily satisfy the associative law, i.e. (xy)z does not necessarily
equal x(yz). How many different ways are there to interpret the product abcde?

8. [10] Compute 1 · 2 + 2 · 3 + · · · + (n − 1)n.

9. [5] Suppose x satisfies x3 + x2 + x + 1 = 0. What are all possible values of x4 + 2x3 +


2
2x + 2x + 1?
10. [8] Two concentric circles have radii r and R > r. Three new circles are drawn so
that they are each tangent to the big two circles and tangent to the other two new circles.
Find Rr .

11. [8] 12 points are placed around the circumference of a circle. How many ways are
there to draw 6 non-intersecting chords joining these points in pairs?

12. [± 6] How many distinct sets of 8 positive odd integers sum to 20?

13. [5] Find the number of real zeros of x3 − x2 − x + 2.

1
14. [8] Find the exact value of 1 + 1+ 2 .
1+ 1
2
1+ 1+···

15. [6] A beaver walks from (0, 0) to (4, 4) in the plane, walking one unit in the positive
x direction or one unit in the positive y direction at each step. Moreover, he never goes to
a point (x, y) with y > x. How many different paths can he walk?

16. [6] After walking so much that his feet get really tired, the beaver staggers so that,
at each step, his coordinates change by either (+1, +1) or (+1, −1). Now he walks from
(0, 0) to (8, 0) without ever going below the x-axis. How many such paths are there?

17. [4] Frank and Joe are playing ping pong. For each game, there is a 30% chance that
Frank wins and a 70% chance Joe wins. During a match, they play games until someone
wins a total of 21 games. What is the expected value of number of games played per match?

10 8 6 4 2
√ [5] Find the largest prime factor of −x − x − x − x − x − 1, where x = 2i,
18.
i = −1.

2001
n3 .
P
19. [9] Calculate
n=1

20. [± 4] Karen has seven envelopes and seven letters of congratulations to various
HMMT coaches. If she places the letters in the envelopes at random with each possible
configuration having an equal probability, what is the probability that exactly six of the
letters are in the correct envelopes?

P (i+1)(i+2)(i+3)
21. [10] Evaluate (−2)i
.
i=1

22. [6] A man is standing on a platform and sees his train move such that after t
seconds it is 2t2 + d0 feet from his original position, where d0 is some number. Call the
smallest (constant) speed at which the man have to run so that he catches the train v. In
terms of n, find the nth smallest value of d0 that makes v a perfect square.

23. [5] Alice, Bob, and Charlie each pick a 2-digit number at random. What is the
probability that all of their numbers’ tens’ digits are different from each others’ tens’ digits
and all of their numbers’ ones digits are different from each others’ ones’ digits?

24. [6] Square ABCD has side length 1. A dilation is performed about point A, creating
square AB 0 C 0 D0 . If BC 0 = 29, determine the area of triangle BDC 0 .

25. [± 10] What is the remainder when 100! is divided by 101?

26. [6] A circle with center at O has radius 1. Points P and Q outside the circle are
placed such that P Q passes through O. Tangent lines to the circle through P hit the circle
at P1 and P2 , and tangent lines to the circle through Q hit the circle at Q1 and Q2 . If
∠P1 P P2 = 45◦ and angleQ1 QQ2 = 30◦ , find the minimum possible length of arc P2 Q2 .

27. [5] Mona has 12 match sticks of length 1, and she has to use them to make regular
polygons, with each match being a side or a fraction of a side of a polygon, and no two
matches overlapping or crossing each other. What is the smallest total area of the polygons
Mona can make?

28. [4] How many different combinations of 4 marbles can be made from 5 indistinguish-
able red marbles, 4 indistinguishable blue marbles, and 2 indistinguishable black marbles?

29. [10] Count the number of sequences a1 , a2 , a3 , a4 , a5 of integers such that ai ≤ 1 for
all i and all partial sums (a1 , a1 + a2 , etc.) are non-negative.

30. [± 4] How many roots does arctan x = x2 − 1.6 have, where the arctan function is
defined in the range − pi2 < arctan x < pi2 ?

31. [5] If two fair dice are tossed, what is the probability that their sum is divisible by
5?
32. [10] Count the number of permutations a1 a2 . . . a7 of 1234567 with longest decreasing
subsequence of length at most two (i.e. there does not exist i < j < k such that ai > aj > ak ).

33. [± 5] A line of soldiers 1 mile long is jogging. The drill sergeant, in a car, moving
at twice their speed, repeatedly drives from the back of the line to the front of the line and
back again. When each soldier has marched 15 miles, how much mileage has been added to
the car, to the nearest mile?

34. [8] Find all the values of m for which the zeros of 2x2 − mx − 8 differ by m − 1.

35. [7] Find the largest integer that divides m5 − 5m3 + 4m for all m ≥ 5.
. 5! = 120 .

36. [4] Count the number of sequences 1 ≤ a1 ≤ a2 ≤ · · · ≤ a5 of integers with ai ≤ i


for all i.

37. [5] Alex and Bob have 30 matches. Alex picks up somewhere between one and six
matches (inclusive), then Bob picks up somewhere between one and six matches, and so on.
The player who picks up the last match wins. How many matches should Alex pick up at
the beginning to guarantee that he will be able to win?

38. [9] The cafeteria in a certain laboratory is open from noon until 2 in the afternoon
every Monday for lunch. Two professors eat 15 minute lunches sometime between noon
and 2. What is the probability that they are in the cafeteria simultaneously on any given
Monday?

39. [9] What is the remainder when 22001 is divided by 27 − 1?

40. [5] A product of five primes is of the form ABC, ABC, where A, B, and C represent
digits. If one of the primes is 491, find the product ABC, ABC.

1 2 1 3
 
41. [4] If a + a
= 3, find a + a
in terms of a.

q q
1 1
42. [10] Solve x = x − x
+ 1− x
for x.
43. [4] When a single number is added to each member of the seqence 20, 50, 100, the
sequence becomes expressable as x, ax, a2 x. Find a.

44. [7] Through a point in the interior of a triangle ABC, three lines are drawn, one
parallel to each side. These lines divide the sides of the triangle into three regions each.
Let a, b, and c be the lengths of the sides opposite ∠A, ∠B, and ∠C, respectively, and let
a0 , b0 , and c0 be the lengths of the middle regions of the sides opposite ∠A, ∠B, and ∠C,
respectively. Find the numerical value of a0 /a + b0 /b + c0 /c.

45. [4] A stacking of circles in the plane consists of a base, or some number of unit
circles centered on the x-axis in a row without overlap or gaps, and circles above the x-axis
that must be tangent to two circles below them (so that if the ends of the base were secured
and gravity were applied from below, then nothing would move). How many stackings of
circles in the plane have 4 circles in the base?

46. [± 5] Draw a rectangle. Connect the midpoints of the opposite sides to get 4
congruent rectangles. Connect the midpoints of the lower right rectangle for a total of 7
rectangles. Repeat this process infinitely. Let n be the minimum number of colors we can
assign to the rectangles so that no two rectangles sharing an edge have the same color and m
be the minimum number of colors we can assign to the rectangles so that no two rectangles
sharing a corner have the same color. Find the ordered pair (n, m).

47. [7] For the sequence of numbers n1 , n2 , n3 , . . . , the relation ni = 2ni−1 + a holds for
all i > 1. If n2 = 5 and n8 = 257, what is n5 ?

48. [8] What is the smallest positive integer x for which x2 + x + 41 is not a prime?

1 1
49. [5] If 9
of 60 is 5, what is 20
of 80?

50. [9] The Fibonacci sequence F1 , F2 , F3 , . . . is defined by F1 = F2 = 1 and Fn+2 =


Fn+1 + Fn . Find the least positive integer t such that for all n > 0, Fn = Fn+t .

51. [5] Some people like to write with larger pencils than others. Ed, for instance, likes
to write with the longest pencils he can find. However, the halls of MIT are of limited height
L and width L. What is the longest pencil Ed can bring through the halls so that he can
negotiate a square turn?

52. [6] Find all ordered pairs (m, n) of integers such that 231m2 = 130n2 .
1 1 1 1 1 1 1 1 1
  
53. [7] Find the sum of the infinite series 32 −12 12
− 32
+ 52 −3 2 32
− 52
+ 72 −5 2 52
− 72
+
···.

54. [10] The set of points (x1 , x2 , x3 , x4 ) in R4 such that x1 ≥ x2 ≥ x3 ≥ x4 is a cone


(or hypercone, if you insist). Into how many regions is this cone sliced by the hyperplanes
xi − xj = 1 for 1 ≤ i < j ≤ n?

55. [7] How many multiples of 7 between 106 and 109 are perfect squares?

56. [6] A triangle has sides of length 888, 925, and x > 0. Find the value of x that
minimizes the area of the circle circumscribed about the triangle.

57. [5] Let x = 20011002 − 2001−1002 and y = 20011002 + 2001−1002 . Find x2 − y 2 .

58. [9] Let (x, y) be a point in the cartesian plane, x, y > 0. Find a formula in terms of
x and y for the minimal area of a right triangle with hypotenuse passing through (x, y) and
legs contained in the x and y axes.

59. [10] Trevor and Edward play a game in which they take turns adding or removing
beans from a pile. On each turn, a player must either add or remove the largest perfect
square number of beans that is in the heap. The player who empties the pile wins. For
example, if Trevor goes first with a pile of 5 beans, he can either add 4 to make the total
9, or remove 4 to make the total 1, and either way Edward wins by removing all the beans.
There is no limit to how large the pile can grow; it just starts with some finite number of
beans in it, say fewer than 1000.
Before the game begins, Edward dispatches a spy to find out how many beans will be in
the opening pile, call this n, then “graciously” offers to let Trevor go first. Knowing that the
first player is more likely to win, but not knowing n, Trevor logically but unwisely accepts,
and Edward goes on to win the game. Find a number n less than 1000 that would prompt
this scenario, assuming both players are perfect logicians. A correct answer is worth the
nearest integer to log2 (n − 4) points.

60. [∞] Find an n such that n! −n (n − 1)! + (n − 2)! − (n − 3)! + · · · ± 1! is prime. Be


n, n≤25
prepared to justify your answer for [ n+225 ], n>25 points, where [N ] is the greatest integer
10
less than N .
Team Round Solutions
Harvard-MIT Math Tournament
March 3, 2001

1. How many digits are in the base two representation of 10! (factorial)?

2. On a certain unidirectional highway, trucks move steadily at 60 miles per hour spaced
1/4 of a mile apart. Cars move steadily at 75 miles per hour spaced 3 seconds apart. A lone
sports car weaving through traffic at a steady forward speed passes two cars between each
truck it passes. How quickly is it moving in miles per hour?

10000
3. What is the 18th digit after the decimal point of 9899
?

4. P is a polynomial. When P is divided by x − 1, the remainder is −4. When P is


divided by x − 2, the remainder is −1. When P is divided by x − 3, the remainder is 4.
Determine the remainder when P is divided by x3 − 6x2 + 11x − 6.

5. Find all x between − π2 and π


2
such that 1 − sin4 x − cos2 x = 1
16
.

6. What is the radius of the smallest sphere in which 4 spheres of radius 1 will fit?

7. The Fibonacci numbers are defined by F1 = F2 = 1 and Fn+2 = Fn+1 + Fn for n ≥ 1.


The Lucas numbers are defined by L1 = 1, L2 = 2, and Ln+2 = Ln+1 + Ln for n ≥ 1.
15
Q F2n
Fn
n=1
Calculate 13
Q
.
Ln
n=1

sin 10+sin 20+sin 30+sin 40+sin 50+sin 60+sin 70+sin 80


8. Express cos 5 cos 10 cos 20
without using trigonometric func-
tions.


ai
P
9. Compute ai
for a > 1.
i=1

10. Define a monic irreducible polynomial with integral coefficients to be a polynomial


with leading coefficient 1 that cannot be factored, and the prime factorization of a polynomial
with leading coefficient 1 as the factorization into monic irreducible polynomials. How many
not necessarily distinct monic irreducible polynomials are there in the prime factorization of
(x8 + x4 + 1)(x8 + x + 1) (for instance, (x + 1)2 has two prime factors)?

11. Define a? = (a − 1)/(a + 1) for a 6= −1. Determine all real values N for which
(N ?)? = tan 15.

12. All subscripts in this problem are to be considered modulo 6, that means for example
that ω7 is the same as ω1 . Let ω1 , . . . ω6 be circles of radius r, whose centers lie on a regular
hexagon of side length 1. Let Pi be the intersection of ωi and ωi+1 that lies further from the
center of the hexagon, for i = 1, . . . 6. Let Qi , i = 1 . . . 6, lie on ωi such that Qi , Pi , Qi+1
are colinear. Find the number of possible values of r.
Harvard-MIT Math Tournament
March 17, 2002
Individual Subject Test: Algebra

1. Nine nonnegative numbers have average 10. What is the greatest possible value for their
median?

2. p and q are primes such that the numbers p + q and p + 7q are both squares. Find the
value of p.

3. Real numbers a, b, c satisfy the equations a + b + c = 26, 1/a + 1/b + 1/c = 28. Find the
value of
a b c a c b
+ + + + + .
b c a c b a

4. If a positive integer multiple of 864 is picked randomly, with each multiple having the
same probability of being picked, what is the probability that it is divisible by 1944?

5. Find the greatest common divisor of the numbers 2002 + 2, 20022 + 2, 20023 + 2, . . ..

6. Find the sum of the even positive divisors of 1000.

7. The real numbers x, y, z, w satisfy

2x + y + z + w = 1
x + 3y + z + w = 2
x + y + 4z + w = 3
x + y + z + 5w = 25.

Find the value of w.

8. Determine the value of the sum


3 5 7 29
+ + + · · · + .
12 · 22 22 · 32 32 · 42 142 · 152

9. For any positive integer n, let f (n) denote the number of 1’s in the base-2 representation
of n. For how many values of n with 1 ≤ n ≤ 2002 do we have f (n) = f (n + 1)?

10. Determine the value of


1 1 1 1
2002 + (2001 + (2000 + · · · + (3 + · 2)) · · · ).
2 2 2 2

1
Harvard-MIT Math Tournament
March 17, 2002
Individual Subject Test: Calculus


1. Two circles have centers that are d units apart, and each has diameter d. For any d,
let A(d) be the area of the smallest circle that contains both of these circles. Find lim A(d)
d2
.
d→∞

x2 −(x+h)2
2. Find lim h
.
h→0

3. We are given the values of the differentiable real functions f, g, h, as well as the derivatives
of their pairwise products, at x = 0:

f (0) = 1; g(0) = 2; h(0) = 3; (gh)0 (0) = 4; (hf )0 (0) = 5; (f g)0 (0) = 6.

Find the value of (f gh)0 (0).

4. Find the area of the region in the first quadrant x > 0, y > 0 bounded above the graph
of y = arcsin(x) and below the graph of the y = arccos(x).

5. What is the minimum vertical distance between the graphs of 2 + sin(x) and cos(x)?

6. Determine the positive value of a such that the parabola y = x2 + 1 bisects the area of
the rectangle with vertices (0, 0), (a, 0), (0, a2 + 1), and (a, a2 + 1).

7. Denote by hxi the fractional part of the real number x (for instance, h3.2i = 0.2). A
{1, 2, 3, . . . , M }, with each integer having
positive integer N is selected randomly from
the set
87
the same probability of being picked, and 303 N is calculated. This procedure is repeated
M times and the average value A(M ) is obtained. What is lim A(M )?
M →∞


( 2−1)/2
dx
R
8. Evaluate √
(2x+1) x2 +x
.
0

9. Suppose f is a differentiable real function such that f (x) + f 0 (x) ≤ 1 for all x, and
f (0) = 0. What is the largest possible value of f (1)? (Hint: consider the function ex f (x).)
R1
10. A continuous real function f satisfies the identity f (2x) = 3f (x) for all x. If 0 f (x) dx =
R2
1, what is 1 f (x) dx?

1
Harvard-MIT Math Tournament
March 17, 2002
Individual Subject Test: Geometry

1. A man, standing on a lawn, is wearing a circular sombrero of radius 3 feet. Unfortunately,


the hat blocks the sunlight so effectively that the grass directly under it dies instantly. If
the man walks in a circle of radius 5 feet, what area of dead grass will result?
2. Dan is holding one end of a 26 inch long piece of light string that has a heavy bead on
it with each hand (so that the string lies along straight lines). If he starts with his hands
together at the start and leaves his hands at the same height, how far does he need to pull
his hands apart so that the bead moves upward by 8 inches?
3. A square
√ and a regular hexagon are drawn with the same side length. If the area of the
square is 3, what is the area of the hexagon?
4. We call a set of professors and committees on which they serve a university if
(1) given two distinct professors there is one and only one committee on which they both
serve,
(2) given any committee, C, and any professor, P , not on that committee, there is exactly
one committee on which P serves and no professors on committee C serve, and
(3) there are at least two professors on each committee; there are at least two committees.
What is the smallest number of committees a university can have?
5. Consider a square of side length 1. Draw four lines that each connect a midpoint of a
side with a corner not on that side, such that each midpoint and each corner is touched by
only one line. Find the area of the region completely bounded by these lines.
6. If we pick (uniformly) a random square of area 1 with sides parallel to the x− and y−axes
that lies entirely within the 5-by-5 square bounded by the lines x = 0, x = 5, y = 0, y = 5
(the corners of the square need not have integer coordinates), what is the probability that
the point (x, y) = (4.5, 0.5) lies within the square of area 1?
7. Equilateral triangle ABC of side length 2 is drawn. Three squares external to the
triangle, ABDE, BCF G, and CAHI, are drawn. What is the area of the smallest triangle
that contains these squares?
8. Equilateral triangle ABC of side length 2 is drawn. Three squares containing the triangle,
ABDE, BCF G, and CAHI, are drawn. What is the area of the smallest triangle that
contains these squares?
9. A and B are two points on a circle with center O, and C lies outside the circle, on ray
AB. Given that AB = 24, BC = 28, OA = 15, find OC.
10. Let 4ABC be equilateral, and let D, E, F be points on sides BC, CA, AB respectively,
with F A = 9, AE = EC = 6, CD = 4. Determine the measure (in degrees) of ∠DEF .

1
Harvard-MIT Math Tournament
March 17, 2002
Individual General Test: Part 1

1. What is the maximum number of lattice points (i.e. points with integer coordinates) in
the plane that can be contained strictly inside a circle of radius 1?
2. Eight knights are randomly placed on a chessboard (not necessarily on distinct squares).
A knight on a given square attacks all the squares that can be reached by moving either (1)
two squares up or down followed by one squares left or right, or (2) two squares left or right
followed by one square up or down. Find the probability that every square, occupied or not,
is attacked by some knight.
3. How many triples (A, B, C) of positive integers (positive integers are the numbers
1, 2, 3, 4, . . .) are there such that A + B + C = 10, where order does not matter (for in-
stance the triples (2, 3, 5) and (3, 2, 5) are considered to be the same triple) and where two
of the integers in a triple could be the same (for instance (3, 3, 4) is a valid triple).
4. We call a set of professors and committees on which they serve a university if
(1) given two distinct professors there is one and only one committee on which they both
serve,
(2) given any committee, C, and any professor, P , not on that committee, there is exactly
one committee on which P serves and no professors on committee C serve, and
(3) there are at least two professors on each committee; there are at least two committees.
What is the smallest number of committees a university can have?
5. A square
√ and a regular hexagon are drawn with the same side length. If the area of the
square is 3, what is the area of the hexagon?
6. A man, standing on a lawn, is wearing a circular sombrero of radius 3 feet. Unfortunately,
the hat blocks the sunlight so effectively that the grass directly under it dies instantly. If
the man walks in a circle of radius 5 feet, what area of dead grass will result?
7. A circle is inscribed in a square dartboard. If a dart is thrown at the dartboard and hits
the dartboard in a random location, with all locations having the same probability of being
hit, what is the probability that it lands within the circle?
8. Count the number of triangles with positive area whose vertices are points whose (x, y)-
coordinates lie in the set {(0, 0), (0, 1), (0, 2), (1, 0), (1, 1), (1, 2), (2, 0), (2, 1), (2, 2)}.
9. Real numbers a, b, c satisfy the equations a + b + c = 26, 1/a + 1/b + 1/c = 28. Find the
value of
a b c a c b
+ + + + + .
b c a c b a
10. A certain cafeteria serves ham and cheese sandwiches, ham and tomato sandwiches,
and tomato and cheese sandwiches. It is common for one meal to include multiple types of
sandwiches. On a certain day, it was found that 80 customers had meals which contained
both ham and cheese; 90 had meals containing both ham and tomatoes; 100 had meals
containing both tomatoes and cheese. 20 customers’ meals included all three ingredients.
How many customers were there?

1
Harvard-MIT Math Tournament
March 17, 2002
Individual General Test: Part 2

1. The squares of a chessboard are numbered from left to right and top to bottom (so that
the first row reads 1, 2, . . . , 8, the second reads 9, 10, . . . , 16, and so forth). The number 1 is
on a black square. How many black squares contain odd numbers?

2. You are in a completely dark room with a drawer containing 10 red, 20 blue, 30 green,
and 40 khaki socks. What is the smallest number of socks you must randomly pull out in
order to be sure of having at least one of each color?

q p
3. Solve for x in 3 = x + x + x + · · ·.

4. Dan is holding one end of a 26 inch long piece of light string that has a heavy bead on it
with each hand (so that the string lies along two straight lines). If he starts with his hands
together at the start and leaves his hands at the same height, how far does he need to pull
his hands apart so that the bead moves upward by 8 inches?

5. A square
√ and a regular hexagon are drawn with the same side length. If the area of the
square is 3, what is the area of the hexagon?

6. Nine nonnegative numbers have average 10. What is the greatest possible value for their
median?

7. p and q are primes such that the numbers p + q and p + 7q are both squares. Find the
value of p.

8. Two fair coins are simultaneously flipped. This is done repeatedly until at least one of
the coins comes up heads, at which point the process stops. What is the probability that
the other coin also came up heads on this last flip?

9. A and B are two points on a circle with center O, and C lies outside the circle, on ray
AB. Given that AB = 24, BC = 28, OA = 15, find OC.

10. How many four-digit numbers are there in which at least one digit occurs more than
once?

1
Harvard-MIT Math Tournament
March 17, 2002
Individual Subject Test: Advanced Topics

1. Eight knights are randomly placed on a chessboard (not necessarily on distinct squares).
A knight on a given square attacks all the squares that can be reached by moving either (1)
two squares up or down followed by one squares left or right, or (2) two squares left or right
followed by one square up or down. Find the probability that every square, occupied or not,
is attacked by some knight.
2. A certain cafeteria serves ham and cheese sandwiches, ham and tomato sandwiches, and
tomato and cheese sandwiches. It is common for one meal to include multiple types of
sandwiches. On a certain day, it was found that 80 customers had meals which contained
both ham and cheese; 90 had meals containing both ham and tomatoes; 100 had meals
containing both tomatoes and cheese. 20 customers’ meals included all three ingredients.
How many customers were there?
3. How many four-digit numbers are there in which at least one digit occurs more than
once?
4. Two fair coins are simultaneously flipped. This is done repeatedly until at least one of
the coins comes up heads, at which point the process stops. What is the probability that
the other coin also came up heads on this last flip?
5. Determine the number of subsets S of {1, 2, 3, . . . , 10} with the following property: there
exist integers a < b < c with a ∈ S, b ∈
/ S, c ∈ S.
6. In how many ways can the numbers 1, 2, . . . , 2002 be placed at the vertices of a regular
2002-gon so that no two adjacent numbers differ by more than 2? (Rotations and reflections
are considered distinct.)
7. A manufacturer of airplane parts makes a certain engine that has a probability p of failing
on any given flight. Their are two planes that can be made with this sort of engine, one that
has 3 engines and one that has 5. A plane crashes if more than half its engines fail. For
what values of p do the two plane models have the same probability of crashing?
8. Given a 9 × 9 chess board, we consider all the rectangles whose edges lie along grid
lines (the board consists of 81 unit squares, and the grid lines lie on the borders of the unit
squares). For each such rectangle, we put a mark in every one of the unit squares inside
it. When this process is completed, how many unit squares will contain an even number of
marks?
9. Given that a, b, c are positive real numbers and loga b + logb c + logc a = 0, find the value
of (loga b)3 + (logb c)3 + (logc a)3 .
10. One fair die is rolled; let a denote the number that comes up. We then roll a dice; let
the sum of the resulting a numbers be b. Finally, we roll b dice, and let c be the sum of the
resulting b numbers. Find the expected (average) value of c.

1
1. [4] An (l, a)-design of a set is a collection of subsets of that set such that each subset
contains exactly l elements and that no two of the subsets share more than a elements. How
many (2,1)-designs are there of a set containing 8 elements?

2. [5] A lattice point in the plane is a point of the form (n, m), where n and m are integers.
Consider a set S of lattice points. We construct the transform of S, denoted by S 0 , by the
following rule: the pair (n, m) is in S 0 if and only if any of (n, m − 1), (n, m + 1), (n − 1, m),
(n + 1, m), and (n, m) is in S. How many elements are in the set obtained by successively
transforming {(0, 0)} 14 times?

3. [5] How many elements are in the set obtained by transforming {(0, 0), (2, 0)} 14 times?

4. [4] How many ways are there of using diagonals to divide a regular 6-sided polygon into
triangles such that at least one side of each triangle is a side of the original polygon and that
each vertex of each triangle is a vertex of the original polygon?

5. [6] Two 4 × 4 squares are randomly placed on an 8 × 8 chessboard so that their sides lie
along the grid lines of the board. What is the probability that the two squares overlap?

6. [±6] Find all values of x that satisfy x = 1 − x + x2 − x3 + x4 − x5 + · · · (be careful; this


is tricky).

7. [5] A rubber band is 4 inches long. An ant begins at the left end. Every minute, the
ant walks one inch along rightwards along the rubber band, but then the band is stretched
(uniformly) by one inch. For what value of n will the ant reach the right end during the nth
minute?

8. [7] Draw a square of side length 1. Connect its sides’ midpoints to form a second square.
Connect the midpoints of the sides of the second square to form a third square. Connect the
midpoints of the sides of the third square to form a fourth square. And so forth. What is
the sum of the areas of all the squares in this infinite series?

9. [4] Find all values of x with 0 ≤ x < 2π that satisfy sin x + cos x = 2.

1
10. [6] The mathematician John is having trouble remembering his girlfriend Alicia’s 7-
digit phone number. He remembers that the first four digits consist of one 1, one 2, and two
3s. He also remembers that the fifth digit is either a 4 or 5. While he has no memory of the
sixth digit, he remembers that the seventh digit is 9 minus the sixth digit. If this is all the
information he has, how many phone numbers does he have to try if he is to make sure he
dials the correct number?

11. [7] How many real solutions are there to the equation

||||x| − 2| − 2| − 2| = ||||x| − 3| − 3| − 3| ?

12. [±7] This question forms a three question multiple choice test. After each question,
there are 4 choices, each preceded by a letter. Please write down your answer as the ordered
triple (letter of the answer of Question #1, letter of the answer of Question #2, letter of the
answer of Question #3). If you find that all such ordered triples are logically impossible,
then write “no answer” as your answer. If you find more than one possible sets of answers,
then provide all ordered triples as your answer.
When we refer to “the correct answer to Question X” it is the actual answer, not the
letter, to which we refer. When we refer to “the letter of the correct answer to question X”
it is the letter contained in parentheses that precedes the answer to which we refer.
You are given the following condition: No two correct answers to questions on the test
may have the same letter.
Question 1. If a fourth question were added to this test, and if the letter of its correct answer
were (C), then:
(A) This test would have no logically possible set of answers.
(B) This test would have one logically possible set of answers.
(C) This test would have more than one logically possible set of answers.
(D) This test would have more than one logically possible set of answers.
Question 2. If the answer to Question 2 were “Letter (D)” and if Question 1 were not on
this multiple-choice test (still keeping Questions 2 and 3 on the test), then the letter of the
answer to Question 3 would be:
(A) Letter (B)
(B) Letter (C)
(C) Letter (D)
(D) Letter (A)
Question 3. Let P1 = 1. Let P2 = 3. For all i > 2, define Pi = Pi−1 Pi−2 − Pi−2 . Which is a
factor of P2002 ?
(A) 3
(B) 4
(C) 7
(D) 9

2
13. [7] A domino is a 1-by-2 or 2-by-1 rectangle. A domino tiling of a region of the plane
is a way of covering it (and only it) completely by nonoverlapping dominoes. For instance,
there is one domino tiling of a 2-by-1 rectangle and there are 2 tilings of a 2-by-2 rectangle
(one consisting of two horizontal dominoes and one consisting of two vertical dominoes).
How many domino tilings are there of a 2-by-10 rectangle?

14. [7] An omino is a 1-by-1 square or a 1-by-2 horizontal rectangle. An omino tiling of a
region of the plane is a way of covering it (and only it) by ominoes. How many omino tilings
are there of a 2-by-10 horizontal rectangle?

15. [7] How many sequences of 0s and 1s are there of length 10 such that there are no three
0s or 1s consecutively anywhere in the sequence?

16. [5] Divide an m-by-n rectangle into mn nonoverlapping 1-by-1 squares. A polyomino
of this rectangle is a subset of these unit squares such that for any two unit squares S, T in
the polyomino, either
(1) S and T share an edge or
(2) there exists a positive integer n such that the polyomino contains unit squares
S1 , S2 , S3 , . . . , Sn such that S and S1 share an edge, Sn and T share an edge, and for all
positive integers k < n, Sk and Sk+1 share an edge.
We say a polyomino of a given rectangle spans the rectangle if for each of the four edges
of the rectangle the polyomino contains a square whose edge lies on it.
What is the minimum number of unit squares a polyomino can have if it spans a 128-by-
343 rectangle?

17. [±8] Find the number of pentominoes (5-square polyominoes) that span a 3-by-3 rect-
angle, where polyominoes that are flips or rotations of each other are considered the same
polyomino.

18. [±5] Call the pentominoes found in the last problem square pentominoes. Just like
dominos and ominos can be used to tile regions of the plane, so can square pentominoes. In
particular, a square pentomino tiling of a region of the plane is a way of covering it (and only
it) completely by nonoverlapping square pentominoes. How many square pentomino tilings
are there of a 12-by-12 rectangle?

3
19. [8] For how many integers a (1 ≤ a ≤ 200) is the number aa a square?

20. [7] The Antarctican language has an alphabet of just 16 letters. Interestingly, every
word in the language has exactly 3 letters, and it is known that no word’s first letter equals
any word’s last letter (for instance, if the alphabet were {a, b} then aab and aaa could not
both be words in the language because a is the first letter of a word and the last letter
of a word; in fact, just aaa alone couldn’t be in the language). Given this, determine the
maximum possible number of words in the language.

21. [7] The Dyslexian alphabet consists of consonants and vowels. It so happens that a
finite sequence of letters is a word in Dyslexian precisely if it alternates between consonants
and vowels (it may begin with either). There are 4800 five-letter words in Dyslexian. How
many letters are in the alphabet?

22. [5] A path of length n is a sequence of points (x1 , y1 ), (x2 , y2 ), . . . , (xn , yn ) with integer
coordinates such that for all i between 1 and n − 1 inclusive, either
(1) xi+1 = xi + 1 and yi+1 = yi (in which case we say the ith step is rightward ) or
(2) xi+1 = xi and yi+1 = yi + 1 (in which case we say that the ith step is upward ).
This path is said to start at (x1 , y1 ) and end at (xn , yn ). Let P (a, b), for a and b nonneg-
ative integers, be the number of paths that start at (0, 0) and end at (a, b).
10
P
Find P (i, 10 − i).
i=0

23. [5] Find P (7, 3).

24. [7] A restricted path of length n is a path of length n such that for all i between 1 and
n − 2 inclusive, if the ith step is upward, the i + 1st step must be rightward.
Find the number of restricted paths that start at (0, 0) and end at (7, 3).

4
25. [±4] A math professor stands up in front of a room containing 100 very smart math
students and says, “Each of you has to write down an integer between 0 and 100, inclusive, to
guess ‘two-thirds of the average of all the responses.’ Each student who guesses the highest
integer that is not higher than two-thirds of the average of all responses will receive a prize.”
If among all the students it is common knowledge that everyone will write down the best
response, and there is no communication between students, what single integer should each
of the 100 students write down?

26. [±4] Another professor enters the same room and says, “Each of you has to write down
an integer between 0 and 200. I will then compute X, the number that is 3 greater than half
the average of all the numbers that you will have written down. Each student who writes
down the number closest to X (either above or below X) will receive a prize.” One student,
who misunderstood the question, announces to the class that he will write the number 107.
If among the other 99 students it is common knowledge that all 99 of them will write down
the best response, and there is no further communication between students, what single
integer should each of the 99 students write down?

27. [7] Consider the two hands of an analog clock, each of which moves with constant
angular velocity. Certain positions of these hands are possible (e.g. the hour hand halfway
between the 5 and 6 and the minute hand exactly at the 6), while others are impossible (e.g.
the hour hand exactly at the 5 and the minute hand exactly at the 6). How many different
positions are there that would remain possible if the hour and minute hands were switched?

28. [6] Count how many 8-digit numbers there are that contain exactly four nines as digits.

29. [8] A sequence s0 , s1 , s2 , s3 , . . . is defined by s0 = s1 = 1 and, for every positive integer


n, s2n = sn , s4n+1 = s2n+1 , s4n−1 = s2n−1 + s22n−1 /sn−1 . What is the value of s1000 ?

30. [9] A conical flask contains some water. When the flask is oriented so that its base is
horizontal and lies at the bottom (so that the vertex is at the top), the water is 1 inch deep.
When the flask is turned upside-down, so that the vertex is at the bottom, the water is 2
inches deep. What is the height of the cone?

5
31. [6] Express, as concisely as possible, the value of the product

(03 − 350)(13 − 349)(23 − 348)(33 − 347) · · · (3493 − 1)(3503 − 0).

32. [9] Two circles have radii 13 and 30, and their centers are 41 units apart. The line
through the centers of the two circles intersects the smaller circle at two points; let A be the
one outside the larger circle. Suppose B is a point on the smaller circle and C a point on
the larger circle such that B is the midpoint of AC. Compute the distance AC.

33. [8] The expression bxc denotes the greatest integer less than or equal to x. Find the
value of  
2002!
.
2001! + 2000! + 1999! + · · · + 1!


34. [7] Points P and Q are 3 units apart. A circle centered at P with a radius of 3 units
intersects a circle centered at Q with a radius of 3 units at points A and B. Find the area
of quadrilateral APBQ.

35. [±7] Suppose a, b, c, d are real numbers such that

|a − b| + |c − d| = 99; |a − c| + |b − d| = 1.

Determine all possible values of |a − d| + |b − c|.


2 3
36. [±6] Find the set consisting of all real values of x such that the three numbers 2x , 2x , 2x
form a non-constant arithmetic progression (in that order).

6
37. [8] Call a positive integer “mild” if its base-3 representation never contains the digit 2.
How many values of n (1 ≤ n ≤ 1000) have the property that n and n2 are both mild?

38. [6] Massachusetts Avenue is ten blocks long. One boy and one girl live on each block.
They want to form friendships such that each boy is friends with exactly one girl and vice-
versa. Nobody wants a friend living more than one block away (but they may be on the
same block). How many pairings are possible?

39. [7] In the x-y plane, draw a circle of radius 2 centered at (0, 0). Color the circle red
above the line y = 1, color the circle blue below the line y = −1, and color the rest of the
circle white. Now consider an arbitrary straight line at distance 1 from the circle. We color
each point P of the line with the color of the closest point to P on the circle. If we pick such
an arbitrary line, randomly oriented, what is the probability that it contains red, white, and
blue points?

p
40. [9] Find the volume of the three-dimensional solid given by the inequality x2 + y 2 +
|z| ≤ 1.

41. [9] For any integer n, define bnc as the greatest integer less than or equal to n. For any
positive integer n, let jnk jnk jnk
f (n) = bnc + + + ··· + .
2 3 n
For how many values of n, 1 ≤ n ≤ 100, is f (n) odd?

42. [±10] Find all the integers n > 1 with the following property: the numbers 1, 2, . . . , n
can be arranged in a line so that, of any two adjacent numbers, one is divisible by the other.

7
43. [9] Given that a, b, c are positive integers satisfying

a + b + c = gcd(a, b) + gcd(b, c) + gcd(c, a) + 120,

determine the maximum possible value of a.

44. [5] The unknown real numbers x, y, z satisfy the equations

x+y 1 − z + z2 x−y 9 + 3z + z 2
= 2 ; = 2 .
1+z x − xy + y 2 3−z x + xy + y 2
Find x.

45. [9] Find the number of sequences a1 , a2 , . . . , a10 of positive integers with the property
that an+2 = an+1 + an for n = 1, 2, . . . , 8, and a10 = 2002.

46. [±6] Points A, B, C in the plane satisfy AB = 2002, AC = 9999. The circles with
diameters AB and AC intersect at A and D. If AD = 37, what is the shortest distance from
point A to line BC?

47. [9] The real function f has the property that, whenever a, b, n are positive integers such
that a + b = 2n , the equation f (a) + f (b) = n2 holds. What is f (2002)?

48. [9] A permutation of a finite set is a one-to-one function from the set to itself; for
instance, one permutation of {1, 2, 3, 4} is the function π defined such that π(1) = 1, π(2) = 3,
π(3) = 4, and π(4) = 2. How many permutations π of the set {1, 2, . . . , 10} have the property
that π(i) 6= i for each i = 1, 2, . . . , 10, but π(π(i)) = i for each i?

49. [7] Two integers are relatively prime if they don’t share any common factors, i.e. if
their greatest common divisor is 1. Define ϕ(n) as the number of positive integers that are
less than n and relatively prime to n. Define ϕd (n) as the number of positive integers that
are less than dn and relatively prime to n.
What is the least n such that ϕx (n) = 64000, where x = ϕy (n), where y = ϕ(n)?

50. [6] Give the set of all positive integers n such that ϕ(n) = 20022 − 1.

51. [10] Define ϕk (n) as the number of positive integers that are less than or equal to n/k
and relatively prime to n. Find φ2001 (20022 − 1). (Hint: φ(2003) = 2002.)

8
52. [±8] Let ABCD be a quadrilateral, and let E, F, G, H be the respective midpoints
of AB, BC, CD, DA. If EG = 12 and F H = 15, what is the maximum possible area of
ABCD?

53. [10] ABC is a triangle with points E, F on sides AC, AB, respectively. Suppose that
BE, CF intersect at X. It is given that AF/F B = (AE/EC)2 and that X is the midpoint
of BE. Find the ratio CX/XF .

54. [10] How many pairs of integers (a, b), with 1 ≤ a ≤ b ≤ 60, have the property that b
is divisible by a and b + 1 is divisible by a + 1?

55. [10] A sequence of positive integers is given by a1 = 1 and an = gcd(an−1 , n) + 1 for


n > 1. Calculate a2002 .

56. [±6] x, y are positive real numbers such that x + y 2 = xy. What is the smallest possible
value of x?

57. [9] How many ways, without taking order into consideration, can 2002 be expressed
as the sum of 3 positive integers (for instance, 1000 + 1000 + 2 and 1000 + 2 + 1000 are
considered to be the same way)?

58. [8] A sequence is defined by a0 = 1 and an = 2an−1 for n ≥ 1. What is the last digit (in
base 10) of a15 ?

59. [7] Determine the value of

1 · 2 − 2 · 3 + 3 · 4 − 4 · 5 + · · · + 2001 · 2002.

60. [10] A 5 × 5 square grid has the number −3 written in the upper-left square and the
number 3 written in the lower-right square. In how many ways can the remaining squares
be filled in with integers so that any two adjacent numbers differ by 1, where two squares
are adjacent if they share a common edge (but not if they share only a corner)?

9
61. Bob Barker went back to school for a PhD in math, and decided to raise the intellectual
level of The Price is Right by having contestants guess how many objects exist of a certain
type, without going over. The number of points you will get is the percentage of the correct
answer, divided by 10, with no points for going over (i.e. a maximum of 10 points).
Let’s see the first object for our contestants...a table of shape (5, 4, 3, 2, 1) is an arrange-
ment of the integers 1 through 15 with five numbers in the top row, four in the next, three
in the next, two in the next, and one in the last, such that each row and each column is
increasing (from left to right, and top to bottom, respectively). For instance:
1 2 3 4 5
6 7 8 9
10 11 12
13 14
15
is one table. How many tables are there?

62. Our next object up for bid is an arithmetic progression of primes. For example, the
primes 3, 5, and 7 form an arithmetic progression of length 3. What is the largest possi-
ble length of an arithmetic progression formed of positive primes less than 1,000,000? Be
prepared to justify your answer.

63. Our third and final item comes to us from Germany, I mean Geometry. It is known
that a regular n-gon can be constructed with straightedge and compass if n is a prime
that is 1 plus a power of 2. It is also possible to construct a 2n-gon whenever an n-gon is
constructible, or a p1 p2 · · · pm -gon where the pi ’s are distinct primes of the above form. What
is really interesting is that these conditions, together with the fact that we can construct a
square, is that they give us all constructible regular n-gons. What is the largest n less than
4,300,000,000 such that a regular n-gon is constructible?

Help control the pet population. Have your pets spayed or neutered. Bye-bye.

10
Team Event
HMMT 2002

Palindromes. A palindrome is a positive integer n not divisible by 10 such that if you write the
decimal digits of n in reverse order, the number you get is n itself. For instance, the numbers 4
and 25752 are palindromes.

1. [15] Determine the number of palindromes that are less than 1000.

2. [30] Determine the number of four-digit integers n such that n and 2n are both palindromes.

3. [40] Suppose that a positive integer n has the property that n, 2n, 3n, . . ., 9n are all palindromes.
Prove that the decimal digits of n are all zeros or ones.

Floor functions. The notation bxc stands for the largest integer less than or equal to x.

4. [15] Let n be an integer. Prove that


   
n n+1
+ = n.
2 2

5. [20] Prove for integers n that     2


n n+1 n
= .
2 2 4

In problems 6–7 you may use without proof the known summations
L
X L
X
n = n(n + 1)/2 and n3 = n2 (n + 1)2 /4 for positive integers L.
n=1 n=1

PL
6. [20] For positive integers L, let SL = n=1 bn/2c. Determine all L for which SL is a square
number.

PL
n3/9 for positive integers L. Determine all L for which TL is a square
 
7. [45] Let TL = n=1
number.

Luck of the dice. Problems 8–12 concern a two-player game played on a board consisting of fourteen
spaces in a row. The leftmost space is labeled START, and the rightmost space is labeled END.
Each of the twelve other squares, which we number 1 through 12 from left to right, may be blank
or may be labeled with an arrow pointing to the right. The term blank square will refer to one of
these twelve squares that is not labeled with an arrow. The set of blank squares on the board will
be called a board configuration; the board below uses the configuration {1, 2, 3, 4, 7, 8, 10, 11, 12}.

START ⇒ ⇒ ⇒ END
1 2 3 4 5 6 7 8 9 10 11 12
For i ∈ {1, 2}, player i has a die that produces each integer from 1 to si with probability 1/si . Here
s1 and s2 are positive integers fixed before the game begins. The game rules are as follows:

1. The players take turns alternately, and player 1 takes the first turn.

2. On each of his turns, player i rolls his die and moves his piece to the right by the number
of squares that he rolled. If his move ends on a square marked with an arrow, he moves his
piece forward another si squares. If that move ends on an arrow, he moves another si squares,
repeating until his piece comes to rest on a square without an arrow.

3. If a player’s move would take him past the END square, instead he lands on the END square.

4. Whichever player reaches the END square first wins.

As an example, suppose that s1 = 3 and the first player is on square 4 in the sample board shown
above. If the first player rolls a 2, he moves to square 6, then to square 9, finally coming to rest on
square 12. If the second player does not reach the END square on her next turn, the first player
will necessarily win on his next turn, as he must roll at least a 1.

8. [35] In this problem only, assume that s1 = 4 and that exactly one board square, say square
number n, is marked with an arrow. Determine all choices of n that maximize the average distance
in squares the first player will travel in his first two turns.

9. [30] In this problem suppose that s1 = s2 . Prove that for each board configuration, the first
player wins with probability strictly greater than 12 .

10. [30] Exhibit a configuration of the board and a choice of s1 and s2 so that s1 > s2 , yet the
second player wins with probability strictly greater than 21 .

11. [55] In this problem assume s1 = 3 and s2 = 2. Determine, with proof, the nonnegative integer
k with the following property:

1. For every board configuration with strictly fewer than k blank squares, the first player wins
with probability strictly greater than 12 ; but

2. there exists a board configuration with exactly k blank squares for which the second player
wins with probability strictly greater than 12 .

12. [65] Now suppose that before the game begins, the players choose the initial game state as
follows:

1. The first player chooses s1 subject to the constraint that 2 ≤ s1 ≤ 5; then

2. the second player chooses s2 subject to the constraint that 2 ≤ s2 ≤ 5 and then specifies the
board configuration.

Prove that the second player can always make her decisions so that she will win the game with
probability strictly greater than 21 .
Harvard-MIT Mathematics Tournament
March 15, 2003

Individual Round: Algebra Subject Test



1. Find the smallest value of x such that a ≥ 14 a − x for all nonnegative a.
tan2 (20◦ )−sin2 (20◦ )
2. Compute tan2 (20◦ ) sin2 (20◦ )
.

3. Find the smallest n such that n! ends in 290 zeroes.


q √ √
4. Simplify: 2 1.5 + 2 − (1.5 + 2).

5. Several positive integers are given, not necessarily all different. Their sum is 2003.
Suppose that n1 of the given numbers are equal to 1, n2 of them are equal to 2, . . .,
n2003 of them are equal to 2003. Find the largest possible value of

n2 + 2n3 + 3n4 + · · · + 2002n2003 .

6. Let a1 = 1, and let an = bn3 /an−1 c for n > 1. Determine the value of a999 .

7. Let a, b, c be the three roots of p(x) = x3 + x2 − 333x − 1001. Find a3 + b3 + c3 .


1 1 1
8. Find the value of 32 +1
+ 42 +2
+ 52 +3
+ · · ·.
³ ´
1 2n
9. For how many integers n, for 1 ≤ n ≤ 1000, is the number 2 n
even?

10. Suppose P (x) is a polynomial such that P (1) = 1 and

P (2x) 56
=8−
P (x + 1) x+7

for all real x for which both sides are defined. Find P (−1).

1
Harvard-MIT Mathematics Tournament
March 15, 2003

Individual Round: Calculus Subject Test

1. A point is chosen randomly with uniform distribution in the interior of a circle of radius
1. What is its expected distance from the center of the circle?

2. A particle moves along the x-axis in such a way that its velocity at position x is given
by the formula v(x) = 2 + sin x. What is its acceleration at x = π6 ?

3. What is the area of the region bounded by the curves y = x2003 and y = x1/2003 and
lying above the x-axis?

4. The sequence of real numbers x1 , x2 , x3 , . . . satisfies limn→∞ (x2n + x2n+1 ) = 315 and
limn→∞ (x2n + x2n−1 ) = 2003. Evaluate limn→∞ (x2n /x2n+1 ).

5. Find the minimum distance from the point (0, 5/2) to the graph of y = x4 /8.

6. For n an integer, evaluate


µ ¶
1 1 1
lim √ + √ + · · · + q .
n→∞ n2 − 02 n2 − 12 n2 − (n − 1)2

7. For what value of a > 1 is Z a2


1 x−1
log dx
a x 32
minimum?

8. A right circular cone with a height of 12 inches and a base radius of 3 inches is filled
with water and held with its vertex pointing downward. Water flows out through a
hole at the vertex at a rate in cubic inches per second numerically equal to the height
of the water in the cone. (For example, when the height of the water in the cone is 4
inches, water flows out at a rate of 4 cubic inches per second.) Determine how many
seconds it will take for all of the water to flow out of the cone.

9. Two differentiable real functions f (x) and g(x) satisfy

f 0 (x)
= ef (x)−g(x)
g 0 (x)

for all x, and f (0) = g(2003) = 1. Find the largest constant c such that f (2003) > c
for all such functions f, g.

10. Evaluate Z ∞
1 − x2
dx.
−∞ 1 + x4

1
Harvard-MIT Mathematics Tournament
March 15, 2003

Individual Round: Combinatorics Subject Test

1. You have 2003 switches, numbered from 1 to 2003, arranged in a circle. Initially, each
switch is either ON or OFF, and all configurations of switches are equally likely. You
perform the following operation: for each switch S, if the two switches next to S were
initially in the same position, then you set S to ON; otherwise, you set S to OFF.
What is the probability that all switches will now be ON?

2. You are given a 10 × 2 grid of unit squares. Two different squares are adjacent if they
share a side. How many ways can one mark exactly nine of the squares so that no two
marked squares are adjacent?

3. Daniel and Scott are playing a game where a player wins as soon as he has two points
more than his opponent. Both players start at par, and points are earned one at a
time. If Daniel has a 60% chance of winning each point, what is the probability that
he will win the game?

4. In a certain country, there are 100 senators, each of whom has 4 aides. These senators
and aides serve on various committees. A committee may consist either of 5 senators,
of 4 senators and 4 aides, or of 2 senators and 12 aides. Every senator serves on 5
committees, and every aide serves on 3 committees. How many committees are there
altogether?

5. We wish to color the integers 1, 2, 3, . . . , 10 in red, green, and blue, so that no two
numbers a and b, with a − b odd, have the same color. (We do not require that all
three colors be used.) In how many ways can this be done?

6. In a classroom, 34 students are seated in 5 rows of 7 chairs. The place at the center of
the room is unoccupied. A teacher decides to reassign the seats such that each student
will occupy a chair adjacent to his/her present one (i.e. move one desk forward, back,
left or right). In how many ways can this reassignment be made?

7. You have infinitely many boxes, and you randomly put 3 balls into them. The boxes
are labeled 1, 2, . . .. Each ball has probability 1/2n of being put into box n. The balls
are placed independently of each other. What is the probability that some box will
contain at least 2 balls?

8. For any subset S ⊆ {1, 2, . . . , 15}, a number n is called an “anchor” for S if n and
n + |S| are both members of S, where |S| denotes the number of members of S. Find
the average number of anchors over all possible subsets S ⊆ {1, 2, . . . , 15}.

9. At a certain college, there are 10 clubs and some number of students. For any two
different students, there is some club such that exactly one of the two belongs to that
club. For any three different students, there is some club such that either exactly one
or all three belong to that club. What is the largest possible number of students?

1
10. A calculator has a display, which shows a nonnegative integer N , and a button, which
replaces N by a random integer chosen uniformly from the set {0, 1, . . . , N − 1}, pro-
vided that N > 0. Initially, the display holds the number N = 2003. If the button is
pressed repeatedly until N = 0, what is the probability that the numbers 1, 10, 100,
and 1000 will each show up on the display at some point?

2
Harvard-MIT Mathematics Tournament
March 15, 2003

Individual Round: Geometry Subject Test


1. AD and BC are both perpendicular to AB, and CD is perpendicular to AC. If AB = 4
and BC = 3, find CD.
D

A B

2. As shown, U and C are points on the sides of triangle M N H such that M U = s,


U N = 6, N C = 20, CH = s, HM = 25. If triangle U N C and quadrilateral M U CH
have equal areas, what is s?
N

6
20
U
s
M C
s
25 H

3. A room is built in the shape of the region between two semicircles with the same center
and parallel diameters. The farthest distance between two points with a clear line of
sight is 12m. What is the area (in m2 ) of the room?

4. Farmer John is inside of an ellipse with reflective sides, given by the equation x2 /a2 +
y 2 /b2 = 1, with a > b > 0. He is standing at the point (3, 0), and he shines a laser
pointer in the y-direciton. The light reflects off the ellipse and proceeds directly toward
Farmer Brown, traveling a distance of 10 before reaching him. Farmer John then spins
around in a circle; wherever he points the laser, the light reflects off the wall and hits
Farmer Brown. What is the ordered pair (a, b)?

1
5. Consider a 2003-gon inscribed in a circle and a triangulation of it with diagonals
intersecting only at vertices. What is the smallest possible number of obtuse triangles
in the triangulation?

6. Take a clay sphere of radius 13, and drill a circular hole of radius 5 through its center.
Take the remaining “bead” and mold it into a new sphere. What is this sphere’s
radius?

7. Let RST U V be a regular pentagon. Construct an equilateral triangle P RS with point


P inside the pentagon. Find the measure (in degrees) of angle P T V .

8. Let ABC be an equilateral triangle of side length 2. Let ω be its circumcircle, and let
ωA , ωB , ωC be circles congruent to ω centered at each of its vertices. Let R be the set
of all points in the plane contained in exactly two of these four circles. What is the
area of R?

9. In triangle ABC, 6 ABC = 50◦ and 6 ACB = 70◦ . Let D be the midpoint of side
BC. A circle is tangent to BC at B and is also tangent to segment AD; this circle
instersects AB again at P . Another circle is tangent to BC at C and is also tangent
to segment AD; this circle intersects AC again at Q. Find 6 AP Q (in degrees).

10. Convex quadrilateral M AT H is given with HM/M T = 3/4, and 6 AT M = 6 M AT =


6 AHM = 60◦ . N is the midpoint of M A, and O is a point on T H such that lines

M T, AH, N O are concurrent. Find the ratio HO/OT .

2
Harvard-MIT Mathematics Tournament
March 15, 2003

Individual Round: General Test, Part 1


1. 10 people are playing musical chairs with n chairs in a circle. They can be seated
in 7! ways (assuming only one person fits on each chair, of course), where different
arrangements of the same people on chairs, even rotations, are considered different.
Find n.
2. OP EN is a square, and T is a point on side N O, such that triangle T OP has area 62
and triangle T EN has area 10. What is the length of a side of the square?
3. There are 16 members on the Height-Measurement Matching Team. Each member was
asked, “How many other people on the team — not counting yourself — are exactly
the same height as you?” The answers included six 1’s, six 2’s, and three 3’s. What
was the sixteenth answer? (Assume that everyone answered truthfully.)
4. How many 2-digit positive integers have an even number of positive divisors?
5. A room is built in the shape of the region between two semicircles with the same center
and parallel diameters. The farthest distance between two points with a clear line of
sight is 12m. What is the area (in m2 ) of the room?

6. In how many ways can 3 bottles of ketchup and 7 bottles of mustard be arranged in a
row so that no bottle of ketchup is immediately between two bottles of mustard? (The
bottles of ketchup are mutually indistinguishable, as are the bottles of mustard.)
7. Find the real value of x such that x3 + 3x2 + 3x + 7 = 0.
8. A broken calculator has the + and × keys switched. For how many ordered pairs (a, b)
of integers will it correctly calculate a + b using the labelled + key?
9. Consider a 2003-gon inscribed in a circle and a triangulation of it with diagonals
intersecting only at vertices. What is the smallest possible number of obtuse triangles
in the triangulation?
10. Bessie the cow is trying to navigate her way through a field. She can travel only
from lattice point to adjacent lattice point, can turn only at lattice points, and can
travel only to the east or north. (A lattice point is a point whose coordinates are both
integers.) (0, 0) is the southwest corner of the field. (5, 5) is the northeast corner of
the field. Due to large rocks, Bessie is unable to walk on the points (1, 1), (2, 3), or
(3, 2). How many ways are there for Bessie to travel from (0, 0) to (5, 5) under these
constraints?

1
Harvard-MIT Mathematics Tournament
March 15, 2003

Individual Round: General Test, Part 2

1. A compact disc has the shape of a circle of diameter 5 inches with a 1-inch-diameter
circular hole in the center. Assuming the capacity of the CD is proportional to its
area, how many inches would need to be added to the outer diameter to double the
capacity?

2. You have a list of real numbers, whose sum is 40. If you replace every number x on
the list by 1 − x, the sum of the new numbers will be 20. If instead you had replaced
every number x by 1 + x, what would the sum then be?

3. How many positive rational numbers less than π have denominator at most 7 when
written in lowest terms? (Integers have denominator 1.)

4. In triangle ABC with area 51, points D and E trisect AB and points F and G trisect
BC. Find the largest possible area of quadrilateral DEF G.

5. You are given a 10 × 2 grid of unit squares. Two different squares are adjacent if they
share a side. How many ways can one mark exactly nine of the squares so that no two
marked squares are adjacent?

6. The numbers 112, 121, 123, 153, 243, 313, and 322 are among the rows, columns, and
diagonals of a 3 × 3 square grid of digits (rows and diagonals read left-to-right, and
columns read top-to-bottom). What 3-digit number completes the list?

7. Daniel and Scott are playing a game where a player wins as soon as he has two points
more than his opponent. Both players start at par, and points are earned one at a
time. If Daniel has a 60% chance of winning each point, what is the probability that
he will win the game?

8. If x ≥ 0, y ≥ 0 are integers, randomly chosen with the constraint x + y ≤ 10, what is


the probability that x + y is even?

9. In a classroom, 34 students are seated in 5 rows of 7 chairs. The place at the center of
the room is unoccupied. A teacher decides to reassign the seats such that each student
will occupy a chair adjacent to his/her present one (i.e. move one desk forward, back,
left or right). In how many ways can this reassignment be made?

10. Several positive integers are given, not necessarily all different. Their sum is 2003.
Suppose that n1 of the given numbers are equal to 1, n2 of them are equal to 2, . . .,
n2003 of them are equal to 2003. Find the largest possible value of

n2 + 2n3 + 3n4 + · · · + 2002n2003 .

1
Harvard-MIT Mathematics Tournament
March 15, 2003

Guts Round

.......................................................................................

HARVARD-MIT MATHEMATICS TOURNAMENT, MARCH 15, 2003 — GUTS ROUND

q √ √ q √
2003 4006
1. [5] Simplify 2 11 − 3 5 · 89 + 12 55.

2. [5] The graph of x4 = x2 y 2 is a union of n different lines. What is the value of n?

3. [5] If a and b are positive integers that can each be written as a sum of two squares,
then ab is also a sum of two squares. Find the smallest positive integer c such that
c = ab, where a = x3 + y 3 and b = x3 + y 3 each have solutions in integers (x, y), but
c = x3 + y 3 does not.
.....................................................................................

HARVARD-MIT MATHEMATICS TOURNAMENT, MARCH 15, 2003 — GUTS ROUND

1 2 3
4. [6] Let z = 1 − 2i. Find z
+ z2
+ z3
+ · · ·.

5. [6] Compute the surface area of a cube inscribed in a sphere of surface area π.

6. [6] Define the Fibonacci numbers by F0 = 0, F1 = 1, Fn = Fn−1 + Fn−2 for n ≥ 2. For


how many n, 0 ≤ n ≤ 100, is Fn a multiple of 13?
.....................................................................................

HARVARD-MIT MATHEMATICS TOURNAMENT, MARCH 15, 2003 — GUTS ROUND

√ q √
7. [6] a and b are integers such that a + b= 15 + 216. Compute a/b.

8. [6] How many solutions in nonnegative integers (a, b, c) are there to the equation

2a + 2b = c! ?

9. [6] For x a real number, let f (x) = 0 if x < 1 and f (x) = 2x − 2 if x ≥ 1. How many
solutions are there to the equation

f (f (f (f (x)))) = x?

1
.....................................................................................

HARVARD-MIT MATHEMATICS TOURNAMENT, MARCH 15, 2003 — GUTS ROUND

10. [7] Suppose that A, B, C, D are four points in the plane, and let Q, R, S, T, U, V be the
respective midpoints of AB, AC, AD, BC, BD, CD. If QR = 2001, SU = 2002, T V =
2003, find the distance between the midpoints of QU and RV .

11. [7] Find the smallest positive integer n such that 12 + 22 + 32 + 42 + · · · + n2 is divisible
by 100.

12. [7] As shown in the figure, a circle of radius 1 has two equal circles whose diameters
cover a chosen diameter of the larger circle. In each of these smaller circles we similarly
draw three equal circles, then four in each of those, and so on. Compute the area of
the region enclosed by a positive even number of circles.

.....................................................................................

HARVARD-MIT MATHEMATICS TOURNAMENT, MARCH 15, 2003 — GUTS ROUND

13. [7] If xy = 5 and x2 + y 2 = 21, compute x4 + y 4 .

14. [7] A positive integer will be called “sparkly” if its smallest (positive) divisor, other
than 1, equals the total number of divisors (including 1). How many of the numbers
2, 3, . . . , 2003 are sparkly?

15. [7] The product of the digits of a 5-digit number is 180. How many such numbers
exist?

2
.....................................................................................

HARVARD-MIT MATHEMATICS TOURNAMENT, MARCH 15, 2003 — GUTS ROUND

16. [8] What fraction of the area of a regular hexagon of side length 1 is within distance
1
2
of at least one of the vertices?

17. [8] There are 10 cities in a state, and some pairs of cities are connected by roads.
There are 40 roads altogether. A city is called a “hub” if it is directly connected to
every other city. What is the largest possible number of hubs?

18. [8] Find the sum of the reciprocals of all the (positive) divisors of 144.
.....................................................................................

HARVARD-MIT MATHEMATICS TOURNAMENT, MARCH 15, 2003 — GUTS ROUND

19. [8] Let r, s, t be the solutions to the equation x3 + ax2 + bx + c = 0. What is the value
of (rs)2 + (st)2 + (rt)2 in terms of a, b, and c?

20. [8] What is the smallest number of regular hexagons of side length 1 needed to com-
pletely cover a disc of radius 1?

21. [8] r and s are integers such that

3r ≥ 2s − 3 and 4s ≥ r + 12.

What is the smallest possible value of r/s?


.....................................................................................

HARVARD-MIT MATHEMATICS TOURNAMENT, MARCH 15, 2003 — GUTS ROUND

22. [9] There are 100 houses in a row on a street. A painter comes and paints every house
red. Then, another painter comes and paints every third house (starting with house
number 3) blue. Another painter comes and paints every fifth house red (even if it
is already red), then another painter paints every seventh house blue, and so forth,
alternating between red and blue, until 50 painters have been by. After this is finished,
how many houses will be red?

23. [9] How many lattice points are enclosed by the triangle with vertices (0, 99), (5, 100),
and (2003, 500)? Don’t count boundary points.

24. [9] Compute the radius of the inscribed circle of a triangle with sides 15, 16, and 17.

3
.....................................................................................

HARVARD-MIT MATHEMATICS TOURNAMENT, MARCH 15, 2003 — GUTS ROUND

25. [9] Let ABC be an isosceles triangle with apex A. Let I be the incenter. If AI = 3
and the distance from I to BC is 2, then what is the length of BC?

26. [9] Find all integers x such that x2 + 6x + 28 is a perfect square.

27. [9] The rational numbers x and y, when written in lowest terms, have denominators
60 and 70, respectively. What is the smallest possible denominator of x + y?
.....................................................................................

HARVARD-MIT MATHEMATICS TOURNAMENT, MARCH 15, 2003 — GUTS ROUND

28. [10] A point in three-space has distances 2, 6, 7, 8, 9 from five of the vertices of a regular
octahedron. What is its distance from the sixth vertex?

29. [10] A palindrome is a positive integer that reads the same backwards as forwards,
such as 82328. What is the smallest 5-digit palindrome that is a multiple of 99?

30. [10] The sequence a1 , a2 , a3 , . . . of real numbers satisfies the recurrence

a2n − an−1 + 2an


an+1 = .
an−1 + 1
Given that a1 = 1 and a9 = 7, find a5 .
.....................................................................................

HARVARD-MIT MATHEMATICS TOURNAMENT, MARCH 15, 2003 — GUTS ROUND

31. [10] A cylinder of base radius 1 is cut into two equal parts along a plane passing
through the center of the cylinder and tangent to the two base circles. Suppose that
each piece’s surface area is m times its volume. Find the greatest lower bound for all
possible values of m as the height of the cylinder varies.

32. [10] If x, y, and z are real numbers such that 2x2 + y 2 + z 2 = 2x − 4y + 2xz − 5, find
the maximum possible value of x − y + z.

33. [10] We are given triangle ABC, with AB = 9, AC = 10, and BC = 12, and a point
D on BC. B and C are reflected in AD to B 0 and C 0 , respectively. Suppose that lines
BC 0 and B 0 C never meet (i.e., are parallel and distinct). Find BD.

4
.....................................................................................

HARVARD-MIT MATHEMATICS TOURNAMENT, MARCH 15, 2003 — GUTS ROUND

34. [12] OKRA is a trapezoid with OK parallel to RA. If OK = 12 and RA is a positive


integer, how many integer values can be taken on by the length of the segment in the
trapezoid, parallel to OK, through the intersection of the diagonals?

35. [12] A certain lottery has tickets labeled with the numbers 1, 2, 3, . . . , 1000. The lottery
is run as follows: First, a ticket is drawn at random. If the number on the ticket is odd,
the drawing ends; if it is even, another ticket is randomly drawn (without replacement).
If this new ticket has an odd number, the drawing ends; if it is even, another ticket
is randomly drawn (again without replacement), and so forth, until an odd number
is drawn. Then, every person whose ticket number was drawn (at any point in the
process) wins a prize.
You have ticket number 1000. What is the probability that you get a prize?

36. [12] A teacher must divide 221 apples evenly among 403 students. What is the minimal
number of pieces into which she must cut the apples? (A whole uncut apple counts as
one piece.)
.....................................................................................

HARVARD-MIT MATHEMATICS TOURNAMENT, MARCH 15, 2003 — GUTS ROUND

37. [15] A quagga is an extinct chess piece whose move is like a knight’s, but much longer:
it can move 6 squares in any direction (up, down, left, or right) and then 5 squares in
a perpendicular direction. Find the number of ways to place 51 quaggas on an 8 × 8
chessboard in such a way that no quagga attacks another. (Since quaggas are naturally
belligerent creatures, a quagga is considered to attack quaggas on any squares it can
move to, as well as any other quaggas on the same square.)

38. [15] Given are real numbers x, y. For any pair of real numbers a0 , a1 , define a sequence
by an+2 = xan+1 + yan for n ≥ 0. Suppose that there exists a fixed nonnegative integer
m such that, for every choice of a0 and a1 , the numbers am , am+1 , am+3 , in this order,
form an arithmetic progression. Find all possible values of y.

39. [15] In the figure, if AE = 3, CE = 1, BD = CD = 2, and AB = 5, find AG.


A

E
G
F
C

D
B

5
.....................................................................................

HARVARD-MIT MATHEMATICS TOURNAMENT, MARCH 15, 2003 — GUTS ROUND

40. [18] All the sequences consisting of five letters from the set {T, U, R, N, I, P } (with
repetitions allowed) are arranged in alphabetical order in a dictionary. Two sequences
are called “anagrams” of each other if one can be obtained by rearranging the letters of
the other. How many pairs of anagrams are there that have exactly 100 other sequences
between them in the dictionary?
41. [18] A hotel consists of a 2 × 8 square grid of rooms, each occupied by one guest. All
the guests are uncomfortable, so each guest would like to move to one of the adjoining
rooms (horizontally or vertically). Of course, they should do this simultaneously, in
such a way that each room will again have one guest. In how many different ways can
they collectively move?
42. [18] A tightrope walker stands in the center of a rope of length 32 meters. Every
minute she walks forward one meter with probability 3/4 and backward one meter
with probability 1/4. What is the probability that she reaches the end in front of her
before the end behind her?
.....................................................................................

HARVARD-MIT MATHEMATICS TOURNAMENT, MARCH 15, 2003 — GUTS ROUND

43. Write down an integer N between 0 and 10, inclusive. You will receive N points —
unless some other team writes down the same N , in which case you receive nothing.
44. A partition of a number n is a sequence of positive integers, arranged in nonincreasing
order, whose sum is n. For example, n = 4 has 5 partitions: 1 + 1 + 1 + 1 = 2 +
1 + 1 = 2 + 2 = 3 + 1 = 4. Given two different partitions of the same number,
n = a1 + a2 + · · · + ak = b1 + b2 + · · · + bl , where k ≤ l, the first partition is said to
dominate the second if all of the following inequalities hold:
a1 ≥ b1 ;
a1 + a2 ≥ b1 + b2 ;
a1 + a2 + a3 ≥ b1 + b2 + b3 ;
..
.
a1 + a2 + · · · + ak ≥ b1 + b2 + · · · + bk .
Find as many partitions of the number n = 20 as possible such that none of the
partitions dominates any other. Your score will be the number of partitions you find.
If you make a mistake and one of your partitions does dominate another, your score is
the largest m such that the first m partitions you list constitute a valid answer.
45. Find a set S of positive integers such that no two distinct subsets of S have the same
sum. Your score will be b20(2n /r − 2)c, where n is the number of elements in the set S,
and r is the largest element of S (assuming, of course, that this number is nonnegative).

Hej då!

6
Harvard-MIT Mathematics Tournament
March 15, 2003

Team Round

Completions and Configurations


Given a set A and a nonnegative integer k, the k-completion of A is the collection of all
k-element subsets of A, and a k-configuration of A is any subset of the k-completion of A
(including the empty set and the entire k-completion). For instance, the 2-completion of
A = {1, 2, 3} is {{1, 2}, {1, 3}, {2, 3}}, and the 2-configurations of A are
{} {{1, 2}}
{{1, 3}} {{2, 3}}
{{1, 2}, {1, 3}} {{1, 2}, {2, 3}}
{{1, 3}, {2, 3}} {{1, 2}, {1, 3}, {2, 3}}
The order of an element a of A with respect to a given k-configuration of A is the number
of subsets in the k-configuration that contain a. A k-configuration of a set A is consistent if
the order of every element of A is the same, and the order of a consistent k-configuration is
this common value.

1. (a) [10] How many k-configurations are there of a set that has n elements?
(b) [10] How many k-configurations that have m elements are there of a set that has
n elements?

2. [15] Suppose A is a set with n elements, and k is a divisor of n. Find the number of
consistent k-configurations of A of order 1.

3. (a) [15] Let An = {a1 , a2 , a3 , . . . , an , b}, for n ≥ 3, and let Cn be the 2-configuration
consisting of {ai , ai+1 } for all 1 ≤ i ≤ n − 1, {a1 , an }, and {ai , b} for 1 ≤ i ≤ n.
Let Se (n) be the number of subsets of Cn that are consistent of order e. Find
Se (101) for e = 1, 2, and 3.
(b) [20] Let A = {V, W, X, Y, Z, v, w, x, y, z}. Find the number of subsets of the
2-configuration

{ {V, W }, {W, X}, {X, Y }, {Y, Z}, {Z, V }, {v, x}, {v, y}, {w, y}, {w, z}, {x, z},

{V, v}, {W, w}, {X, x}, {Y, y}, {Z, z} }


that are consistent of order 1.
(c) [30] Let A = {a1 , b1 , a2 , b2 , . . . , a10 , b10 }, and consider the 2-configuration C con-
sisting of {ai , bi } for all 1 ≤ i ≤ 10, {ai , ai+1 } for all 1 ≤ i ≤ 9, and {bi , bi+1 } for
all 1 ≤ i ≤ 9. Find the number of subsets of C that are consistent of order 1.

Define a k-configuration of A to be m-separable if we can label each element of A with an


integer from 1 to m (inclusive) so that there is no element E of the k-configuration all of
whose elements are assigned the same integer. If C is any subset of A, then C is m-separable
if we can assign an integer from 1 to m to each element of C so that there is no element E
of the k-configuration such that E ⊆ C and all elements of E are assigned the same integer.

1
4. (a) [15] Suppose A has n elements, where n ≥ 2, and C is a 2-configuration of A that
is not m-separable for any m < n. What is (in terms of n) the smallest number
of elements that C can have?
(b) [15] Show that every 3-configuration of an n-element set A is m-separable for
every integer m ≥ n/2.
2
(c) [25] Fix k ≥ 2, and suppose
³ 2 ´ A has k elements. Show that any k-configuration
of A with fewer than kk−1
−1
elements is k-separable.

5. [30] Let Bk (n) be the largest number of elements in a 2-separable k-configuration of


a set with 2n elements (2 ≤ k ≤ n). Find a closed-form expression (i.e. an expression
not involving any sums or products with an variable number of terms) for Bk (n).

6. [40] Prove
q that any 2-configuration containing e elements is m-separable for some
m ≤ 2 + 2e + 14 .
1

A cell of a 2-configuration of a set A is a nonempty subset C of A such that


i. for any two distinct elements a, b of C, there exists a sequence c0 , c1 , . . . , cn of elements
of A with c0 = a, cn = b, and such that {c0 , c1 }, {c1 , c2 }, . . . , {cn−1 , cn } are all elements
of the 2-configuration, and
ii. if a is an element of C and b is an element of A but not of C, there does NOT
exist a sequence c0 , c1 , . . . , cn of elements of A with c0 = a, cn = b, and such that
{c0 , c1 }, {c1 , c2 }, . . ., {cn−1 , cn } are all elements of the 2-configuration.
Also, we define a 2-configuration of A to be barren if there is no subset {a0 , a1 , . . . , an } of A,
with n ≥ 2, such that {a0 , a1 }, {a1 , a2 }, . . . , {an−1 , an } and {an , a0 } are all elements of the
2-configuration.

7. [20] Show that, given any 2-configuration of a set A, every element of A belongs to
exactly one cell.

8. (a) [15] Given a set A with n ≥ 1 elements, find the number of consistent 2-
configurations of A of order 1 with exactly 1 cell.
(b) [25] Given a set A with 10 elements, find the number of consistent 2-configurations
of A of order 2 with exactly 1 cell.
(c) [25] Given a set A with 10 elements, find the number of consistent 2-configurations
of order 2 with exactly 2 cells.

9. (a) [15] Show that if every cell of a 2-configuration of a finite set A is m-separable,
then the whole 2-configuration is m-separable.
(b) [30] Show that any barren 2-configuration of a finite set A is 2-separable.

10. [45] Show that every consistent 2-configuration of order 4 on a finite set A has a subset
that is a consistent 2-configuration of order 2.

2
Harvard-MIT Mathematics Tournament
February 28, 2004

Individual Round: Algebra Subject Test

1. How many ordered pairs of integers (a,b) satisfy all of the following inequalities?

a2 + b2 < 16
a2 + b2 < 8a
a2 + b2 < 8b

2. Find the largest number n such that (2004!)! is divisible by ((n!)!)!.

3. Compute: $ %
20053 20033
− .
2003 · 2004 2004 · 2005

4. Evaluate the sum


1 1 1 1
√ + √ + √ + ··· + √ .
2b 1c + 1 2b 2c + 1 2b 3c + 1 2b 100c + 1

5. There exists a positive real number x such that cos(tan−1 (x)) = x. Find the value of
x2 .

6. Find all real solutions to x4 + (2 − x)4 = 34.

7. If x, y, k are positive reals such that


à ! à !
2 x2 y 2 x y
3=k + +k + ,
y 2 x2 y x

find the maximum possible value of k.

8. Let x be a real number such that x3 + 4x = 8. Determine the value of x7 + 64x2 .

9. A sequence of positive integers is defined by a0 = 1 and an+1 = a2n + 1 for each n ≥ 0.


Find gcd(a999 , a2004 ).

10. There exists a polynomial P of degree 5 with the following property: if z is a complex
number such that z 5 +2004z = 1, then P (z 2 ) = 0. Calculate the quotient P (1)/P (−1).

1
Harvard-MIT Mathematics Tournament
February 28, 2004

Individual Round: Calculus Subject Test


f (x+h)−f (h)
1. Let f (x) = sin(sin x). Evaluate limh→0 x
at x = π.
2. Suppose the function f (x) − f (2x) has derivative 5 at x = 1 and derivative 7 at x = 2.
Find the derivative of f (x) − f (4x) at x = 1.
√ √
3. Find limx→∞ ( 3 x3 + x2 − 3 x3 − x2 ).
4. Let f (x) = cos(cos(cos(cos(cos(cos(cos(cos x))))))), and suppose that the number a
satisfies the equation a = cos a. Express f 0 (a) as a polynomial in a.
5. A mouse is sitting in a toy car on a negligibly small turntable. The car cannot turn on
its own, but the mouse can control when the car is launched and when the car stops
(the car has brakes). When the mouse chooses to launch, the car will immediately
leave the turntable on a straight trajectory at 1 meter per second.
Suddenly someone turns on the turntable; it spins at 30 rpm. Consider the set S of
points the mouse can reach in his car within 1 second after the turntable is set in
motion. (For example, the arrows in the figure below represent two possible paths the
mouse can take.) What is the area of S, in square meters?
.5 m

30 rpm

1m

6. For x > 0, let f (x) = xx . Find all values of x for which f (x) = f 0 (x).
7. Find the area of the region in the xy-plane satisfying x6 − x2 + y 2 ≤ 0.
8. If x and y are real numbers with (x + y)4 = x − y, what is the maximum possible value
of y?
9. Find the positive constant c0 such that the series

X n!
n
n=0 (cn)

converges for c > c0 and diverges for 0 < c < c0 .


10. Let P (x) = x3 − 23 x2 + x + 14 . Let P [1] (x) = P (x), and for n ≥ 1, let P [n+1] (x) =
R
P [n] (P (x)). Evaluate 01 P [2004] (x) dx.

1
Harvard-MIT Mathematics Tournament
February 28, 2004

Individual Round: Combinatorics Subject Test

1. There are 1000 rooms in a row along a long corridor. Initially the first room contains
1000 people and the remaining rooms are empty. Each minute, the following happens:
for each room containing more than one person, someone in that room decides it is
too crowded and moves to the next room. All these movements are simultaneous (so
nobody moves more than once within a minute). After one hour, how many different
rooms will have people in them?

2. How many ways can you mark 8 squares of an 8 × 8 chessboard so that no two marked
squares are in the same row or column, and none of the four corner squares is marked?
(Rotations and reflections are considered different.)

3. A class of 10 students took a math test. Each problem was solved by exactly 7 of the
students. If the first nine students each solved 4 problems, how many problems did
the tenth student solve?

4. Andrea flips a fair coin repeatedly, continuing until she either flips two heads in a row
(the sequence HH) or flips tails followed by heads (the sequence T H). What is the
probability that she will stop after flipping HH?

5. A best-of-9 series is to be played between two teams; that is, the first team to win 5
games is the winner. The Mathletes have a chance of 2/3 of winning any given game.
What is the probability that exactly 7 games will need to be played to determine a
winner?

6. A committee of 5 is to be chosen from a group of 9 people. How many ways can it be


chosen, if Bill and Karl must serve together or not at all, and Alice and Jane refuse to
serve with each other?

7. We have a polyhedron such that an ant can walk from one vertex to another, traveling
only along edges, and traversing every edge exactly once. What is the smallest possible
total number of vertices, edges, and faces of this polyhedron?

8. Urn A contains 4 white balls and 2 red balls. Urn B contains 3 red balls and 3 black
balls. An urn is randomly selected, and then a ball inside of that urn is removed. We
then repeat the process of selecting an urn and drawing out a ball, without returning
the first ball. What is the probability that the first ball drawn was red, given that the
second ball drawn was black?

9. A classroom consists of a 5 × 5 array of desks, to be filled by anywhere from 0 to 25


students, inclusive. No student will sit at a desk unless either all other desks in its row
or all others in its column are filled (or both). Considering only the set of desks that are
occupied (and not which student sits at each desk), how many possible arrangements
are there?

1
10. In a game similar to three card monte, the dealer places three cards on the table: the
queen of spades and two red cards. The cards are placed in a row, and the queen
starts in the center; the card configuration is thus RQR. The dealer proceeds to move.
With each move, the dealer randomly switches the center card with one of the two
edge cards (so the configuration after the first move is either RRQ or QRR). What is
the probability that, after 2004 moves, the center card is the queen?

2
Harvard-MIT Mathematics Tournament
February 28, 2004

Individual Round: Geometry Subject Test


1. In trapezoid ABCD, AD is parallel to BC. 6 A = 6 D = 45◦ , while 6 B = 6 C = 135◦ .
If AB = 6 and the area of ABCD is 30, find BC.
B ? C
135 135
6

45 45
A D
Area = 30

2. A parallelogram has 3 of its vertices at (1, 2), (3,8), and (4, 1). Compute the sum of
the possible x-coordinates for the 4th vertex.
3. A swimming pool is in the shape of a circle with diameter 60 ft. The depth varies
linearly along the east-west direction from 3 ft at the shallow end in the east to 15 ft
at the diving end in the west (this is so that divers look impressive against the sunset)
but does not vary at all along the north-south direction. What is the volume of the
pool, in ft3 ?
4. P is inside rectangle ABCD. P A = 2, P B = 3, and P C = 10. Find P D.
5. Find the area of the region of the xy-plane defined by the inequality |x|+|y|+|x+y| ≤ 1.
6. In trapezoid ABCD shown, AD is parallel to BC, and AB = 6, BC = 7, CD =
8, AD = 17. If sides AB and CD are extended to meet at E, find the resulting angle
at E (in degrees).
E

B 7 C

6 8

A D
17

7. Yet another trapezoid ABCD has AD parallel to BC. AC and BD intersect at P .


If [ADP ]/[BCP ] = 1/2, find [ADP ]/[ABCD]. (Here the notation [P1 · · · Pn ] denotes
the area of the polygon P1 · · · Pn .)
8. A triangle has side lengths 18, 24, and 30. Find the area of the triangle whose vertices
are the incenter, circumcenter, and centroid of the original triangle.
9. Given is a regular tetrahedron of volume 1. We obtain a second regular tetrahedron by
reflecting the given one through its center. What is the volume of their intersection?
10. Right triangle XY Z has right angle at Y and XY = 228, Y Z = 2004. Angle Y is
trisected, and the angle trisectors intersect XZ at P and Q so that X, P, Q, Z lie on
XZ in that order. Find the value of (P Y + Y Z)(QY + XY ).

1
Harvard-MIT Mathematics Tournament
February 28, 2004

Individual Round: General Test, Part 1

1. There are 1000 rooms in a row along a long corridor. Initially the first room contains
1000 people and the remaining rooms are empty. Each minute, the following happens:
for each room containing more than one person, someone in that room decides it is
too crowded and moves to the next room. All these movements are simultaneous (so
nobody moves more than once within a minute). After one hour, how many different
rooms will have people in them?

2. What is the largest whole number that is equal to the product of its digits?

3. Suppose f is a function that assigns to each real number x a value f (x), and suppose
the equation

f (x1 + x2 + x3 + x4 + x5 ) = f (x1 ) + f (x2 ) + f (x3 ) + f (x4 ) + f (x5 ) − 8

holds for all real numbers x1 , x2 , x3 , x4 , x5 . What is f (0)?

4. How many ways can you mark 8 squares of an 8 × 8 chessboard so that no two marked
squares are in the same row or column, and none of the four corner squares is marked?
(Rotations and reflections are considered different.)

5. A rectangle has perimeter 10 and diagonal 15. What is its area?

6. Find the ordered quadruple of digits (A, B, C, D), with A > B > C > D, such that

ABCD
− DCBA
= BDAC.

7. Let ACE be a triangle with a point B on segment AC and a point D on segment CE


such that BD is parallel to AE. A point Y is chosen on segment AE, and segment
CY is drawn. Let X be the intersection of CY and BD. If CX = 5, XY = 3, what is
the ratio of the area of trapezoid ABDE to the area of triangle BCD?

X D
A

Y
E

1
8. You have a 10 × 10 grid of squares. You write a number in each square as follows: you
write 1, 2, 3, . . . , 10 from left to right across the top row, then 11, 12, . . . , 20 across the
second row, and so on, ending with a 100 in the bottom right square. You then write
a second number in each square, writing 1, 2, . . . , 10 in the first column (from top to
bottom), then 11, 12, . . . , 20 in the second column, and so forth.
When this process is finished, how many squares will have the property that their two
numbers sum to 101?

9. Urn A contains 4 white balls and 2 red balls. Urn B contains 3 red balls and 3 black
balls. An urn is randomly selected, and then a ball inside of that urn is removed. We
then repeat the process of selecting an urn and drawing out a ball, without returning
the first ball. What is the probability that the first ball drawn was red, given that the
second ball drawn was black?

10. A floor is tiled with equilateral triangles of side length 1, as shown. If you drop a
needle of length 2 somewhere on the floor, what is the largest number of triangles it
could end up intersecting? (Only count the triangles whose interiors are met by the
needle — touching along edges or at corners doesn’t qualify.)

2
Harvard-MIT Mathematics Tournament
February 28, 2004

Individual Round: General Test, Part 2


1. Find the largest number n such that (2004!)! is divisible by ((n!)!)!.
2. Andrea flips a fair coin repeatedly, continuing until she either flips two heads in a row
(the sequence HH) or flips tails followed by heads (the sequence T H). What is the
probability that she will stop after flipping HH?
3. How many ordered pairs of integers (a, b) satisfy all of the following inequalities?
a2 + b2 < 16
a2 + b2 < 8a
a2 + b2 < 8b

4. A horse stands at the corner of a chessboard, a white square. With each jump, the
horse can move either two squares horizontally and one vertically or two vertically and
one horizontally (like a knight moves). The horse earns two carrots every time it lands
on a black square, but it must pay a carrot in rent to rabbit who owns the chessboard
for every move it makes. When the horse reaches the square on which it began, it can
leave. What is the maximum number of carrots the horse can earn without touching
any square more than twice?

5. Eight strangers are preparing to play bridge. How many ways can they be grouped
into two bridge games — that is, into unordered pairs of unordered pairs of people?
6. a and b are positive integers. When written in binary, a has 2004 1’s, and b has 2005 1’s
(not necessarily consecutive). What is the smallest number of 1’s a + b could possibly
have?
7. Farmer John is grazing his cows at the origin. There is a river that runs east to west
50 feet north of the origin. The barn is 100 feet to the south and 80 feet to the east
of the origin. Farmer John leads his cows to the river to take a swim, then the cows
leave the river from the same place they entered and Farmer John leads them to the
barn. He does this using the shortest path possible, and the total distance he travels
is d feet. Find the value of d.

1
8. A freight train leaves the town of Jenkinsville at 1:00 PM traveling due east at constant
speed. Jim, a hobo, sneaks onto the train and falls asleep. At the same time, Julie
leaves Jenkinsville on her bicycle, traveling along a straight road in a northeasterly
direction (but not due northeast) at 10 miles per hour. At 1:12 PM, Jim rolls over
in his sleep and falls from the train onto the side of the tracks. He wakes up and
immediately begins walking at 3.5 miles per hour directly towards the road on which
Julie is riding. Jim reaches the road at 2:12 PM, just as Julie is riding by. What is the
speed of the train in miles per hour?

9. Given is a regular tetrahedron of volume 1. We obtain a second regular tetrahedron by


reflecting the given one through its center. What is the volume of their intersection?

10. A lattice point is a point whose coordinates are both integers. Suppose Johann walks
in a line from the point (0, 2004) to a random lattice point in the interior (not on
the boundary) of the square with vertices (0, 0), (0, 99), (99, 99), (99, 0). What is the
probability that his path, including the endpoints, contains an even number of lattice
points?

2
Harvard-MIT Mathematics Tournament
February 28, 2004

Guts Round

.......................................................................................

HARVARD-MIT MATHEMATICS TOURNAMENT, FEBRUARY 28, 2004 — GUTS ROUND

1. [5] Find the value of


µ ¶ µ ¶ µ ¶ µ ¶ µ ¶ µ ¶
6 1 6 2 6 3 6 4 6 5 6 6
2 + 2 + 2 + 2 + 2 + 2.
1 2 3 4 5 6

2. [5] If the three points


(1, a, b)
(a, 2, b)
(a, b, 3)
are collinear (in 3-space), what is the value of a + b?

3. [5] If the system of equations

|x + y| = 99
|x − y| = c

has exactly two real solutions (x, y), find the value of c.
.....................................................................................

HARVARD-MIT MATHEMATICS TOURNAMENT, FEBRUARY 28, 2004 — GUTS ROUND

4. [6] A tree grows in a rather peculiar manner. Lateral cross-sections of the trunk,
leaves, branches, twigs, and so forth are circles. The trunk is 1 meter in diameter to
a height of 1 meter, at which point it splits into two sections, each with diameter .5
meter. These sections are each one meter long, at which point they each split into two
sections, each with diameter .25 meter. This continues indefinitely: every section of
tree is 1 meter long and splits into two smaller sections, each with half the diameter of
the previous.
What is the total volume of the tree?

5. [6] Augustin has six 1 × 2 × π bricks. He stacks them, one on top of another, to form
a tower six bricks high. Each brick can be in any orientation so long as it rests flat on
top of the next brick below it (or on the floor). How many distinct heights of towers
can he make?
√ √
6. [6] Find the smallest integer n such that n + 99 − n < 1.

1
.....................................................................................

HARVARD-MIT MATHEMATICS TOURNAMENT, FEBRUARY 28, 2004 — GUTS ROUND

7. [6] Find the shortest distance from the line 3x +4y = 25 to the circle x2 + y 2 = 6x − 8y.

8. [6] I have chosen five of the numbers {1, 2, 3, 4, 5, 6, 7}. If I told you what their product
was, that would not be enough information for you to figure out whether their sum
was even or odd. What is their product?

9. [6] A positive integer n is picante if n! ends in the same number of zeroes whether
written in base 7 or in base 8. How many of the numbers 1, 2, . . . , 2004 are picante?
.....................................................................................

HARVARD-MIT MATHEMATICS TOURNAMENT, FEBRUARY 28, 2004 — GUTS ROUND

10. [7] Let f (x) = x2 + x4 + x6 + x8 + · · · , for all real x such that the sum converges. For
how many real numbers x does f (x) = x?

11. [7] Find all numbers n with the following property: there is exactly one set of 8 different
positive integers whose sum is n.

12. [7] A convex quadrilateral is drawn in the coordinate plane such that each of its
vertices (x, y) satisfies the equations x2 + y 2 = 73 and xy = 24. What is the area of
this quadrilateral?
.....................................................................................

HARVARD-MIT MATHEMATICS TOURNAMENT, FEBRUARY 28, 2004 — GUTS ROUND

13. [7] Find all positive integer solutions (m, n) to the following equation:

m2 = 1! + 2! + · · · + n!.

an+2
14. [7] If a1 = 1, a2 = 0, and an+1 = an + 2
for all n ≥ 1, compute a2004 .

15. [7] A regular decagon A0 A1 A2 · · · A9 is given in the plane. Compute ∠A0 A3 A7 in


degrees.

2
.....................................................................................

HARVARD-MIT MATHEMATICS TOURNAMENT, FEBRUARY 28, 2004 — GUTS ROUND

16. [8] An n-string is a string of digits formed by writing the numbers 1, 2, . . . , n in some
order (in base ten). For example, one possible 10-string is

35728910461

What is the smallest n > 1 such that there exists a palindromic n-string?

17. [8] Kate has four red socks and four blue socks. If she randomly divides these eight
socks into four pairs, what is the probability that none of the pairs will be mismatched?
That is, what is the probability that each pair will consist either of two red socks or
of two blue socks?

18. [8] On a spherical planet with diameter 10, 000 km, powerful explosives are placed at
the north and south poles. The explosives are designed to vaporize all matter within
5, 000 km of ground zero and leave anything beyond 5, 000 km untouched. After the
explosives are set off, what is the new surface area of the planet, in square kilometers?
.....................................................................................

HARVARD-MIT MATHEMATICS TOURNAMENT, FEBRUARY 28, 2004 — GUTS ROUND

19. [8] The Fibonacci numbers are defined by F1 = F2 = 1, and Fn = Fn−1 + Fn−2 for
n ≥ 3. If the number
F2003 F2004

F2002 F2003
is written as a fraction in lowest terms, what is the numerator?

20. [8] Two positive rational numbers x and y, when written in lowest terms, have the
property that the sum of their numerators is 9 and the sum of their denominators is
10. What is the largest possible value of x + y?

21. [8] Find all ordered pairs of integers (x, y) such that 3x 4y = 2x+y + 22(x+y)−1 .

3
.....................................................................................

HARVARD-MIT MATHEMATICS TOURNAMENT, FEBRUARY 28, 2004 — GUTS ROUND

22. [9] I have written a strictly increasing sequence of six positive integers, such that each
number (besides the first) is a multiple of the one before it, and the sum of all six
numbers is 79. What is the largest number in my sequence?

23. [9] Find the largest integer n such that 3512 − 1 is divisible by 2n .

24. [9] We say a point is contained in a square if it is in its interior or on its boundary.
Three unit squares are given in the plane such that there is a point contained in all
three. Furthermore, three points A, B, C, are given, each contained in at least one of
the squares. Find the maximum area of triangle ABC.
.....................................................................................

HARVARD-MIT MATHEMATICS TOURNAMENT, FEBRUARY 28, 2004 — GUTS ROUND

25. [9] Suppose x3 − ax2 + bx − 48 is a polynomial with three positive roots p, q, and r
such that p < q < r. What is the minimum possible value of 1/p + 2/q + 3/r?

26. [9] How many of the integers 1, 2, . . . , 2004 can be represented as (mn + 1)/(m + n)
for positive integers m and n?

27. [9] A regular hexagon has one side along the diameter of a semicircle, and the two
opposite vertices on the semicircle. Find the area of the hexagon if the diameter of the
semicircle is 1.
.....................................................................................

HARVARD-MIT MATHEMATICS TOURNAMENT, FEBRUARY 28, 2004 — GUTS ROUND

28. [10] Find the value of


µ ¶ µ ¶ µ ¶ µ ¶
2003 2003 2003 2003
+ + + ··· + .
1 4 7 2002

29. [10] A regular dodecahedron is projected orthogonally onto a plane, and its image is
an n-sided polygon. What is the smallest possible value of n?

30. [10] We have an n-gon, and each of its vertices is labeled with a number from the
set {1, . . . , 10}. We know that for any pair of distinct numbers from this set there is
at least one side of the polygon whose endpoints have these two numbers. Find the
smallest possible value of n.

4
.....................................................................................

HARVARD-MIT MATHEMATICS TOURNAMENT, FEBRUARY 28, 2004 — GUTS ROUND

31. [10] P is a point inside triangle ABC, and lines AP, BP, CP intersect the opposite
sides BC, CA, AB in points D, E, F , respectively. It is given that ∠AP B = 90◦ , and
that AC = BC and AB = BD. We also know that BF = 1, and that BC = 999.
Find AF .

32. [10] Define the sequence b0 , b1 , . . . , b59 by


(
1 if i is a multiple of 3
bi =
0 otherwise.

Let {ai } be a sequence of elements of {0, 1} such that

bn ≡ an−1 + an + an+1 (mod 2)

for 0 ≤ n ≤ 59 (a0 = a60 and a−1 = a59 ). Find all possible values of 4a0 + 2a1 + a2 .

33. [10] A plane P slices through a cube of volume 1 with a cross-section in the shape of
a regular hexagon. This cube also has an inscribed sphere, whose intersection with P
is a circle. What is the area of the region inside the regular hexagon but outside the
circle?
.....................................................................................

HARVARD-MIT MATHEMATICS TOURNAMENT, FEBRUARY 28, 2004 — GUTS ROUND

34. [12] Find the number of 20-tuples of integers x1 , . . . , x10 , y1 , . . . , y10 with the following
properties:

• 1 ≤ xi ≤ 10 and 1 ≤ yi ≤ 10 for each i;


• xi ≤ xi+1 for i = 1, . . . , 9;
• if xi = xi+1 , then yi ≤ yi+1 .

35. [12] There are eleven positive integers n such that there exists a convex polygon with n
sides whose angles, in degrees, are unequal integers that are in arithmetic progression.
Find the sum of these values of n.

36. [12] For a string of P ’s and Q’s, the value is defined to be the product of the positions
of the P ’s. For example, the string P P QP QQ has value 1 · 2 · 4 = 8.
Also, a string is called antipalindromic if writing it backwards, then turning all the
P ’s into Q’s and vice versa, produces the original string. For example, P P QP QQ is
antipalindromic.
There are 21002 antipalindromic strings of length 2004. Find the sum of the reciprocals
of their values.

5
.....................................................................................

HARVARD-MIT MATHEMATICS TOURNAMENT, FEBRUARY 28, 2004 — GUTS ROUND

Q2004
37. [15] Simplify k=1 sin(2πk/4009).

38. [15] Let S = {p1 p2 · · · pn | p1 , p2 , . . . , pn are distinct primes and p1 , . . . , pn < 30}. As-
sume 1 is in S. Let a1 be an element of S. We define, for all positive integers n:

an+1 = an /(n + 1) if an is divisible by n + 1;

an+1 = (n + 2)an if an is not divisible by n + 1.


How many distinct possible values of a1 are there such that aj = a1 for infinitely many
j’s?

39. [15] You want to arrange the numbers 1, 2, 3, . . . , 25 in a sequence with the following
property: if n is divisible by m, then the nth number is divisible by the mth number.
How many such sequences are there?
.....................................................................................

HARVARD-MIT MATHEMATICS TOURNAMENT, FEBRUARY 28, 2004 — GUTS ROUND

40. [18] You would like to provide airline service to the 10 cities in the nation of Schizophre-
nia, by instituting a certain number of two-way routes between cities. Unfortunately,
the government is about to divide Schizophrenia into two warring countries of five cities
each, and you don’t know which cities will be in each new country. All airplane service
between the two new countries will be discontinued. However, you want to make sure
that you set up your routes so that, for any two cities in the same new country, it will
be possible to get from one city to the other (without leaving the country).
What is the minimum number of routes you must set up to be assured of doing this,
no matter how the government divides up the country?

41. [18] A tetrahedron has all its faces triangles with sides 13, 14, 15. What is its volume?

42. [18] S is a set of complex numbers such that if u, v ∈ S, then uv ∈ S and u2 + v 2 ∈ S.


Suppose that the number N of elements of S with absolute value at most 1 is finite.
What is the largest possible value of N ?

6
.....................................................................................

HARVARD-MIT MATHEMATICS TOURNAMENT, FEBRUARY 28, 2004 — GUTS ROUND

43. Write down an integer from 0 to 20 inclusive. This problem will be scored as follows: if
N is the second-largest number from among the responses submitted, then each team
that submits N gets N points, and everyone else gets zero. (If every team picks the
same number then nobody gets any points.)

44. Shown on your answer sheet is a 20 × 20 grid. Place as many queens as you can so
that each of them attacks at most one other queen. (A queen is a chess piece that can
move any number of squares horizontally, vertically, or diagonally.) It’s not very hard
to get 20 queens, so you get no points for that, but you get 5 points for each further
queen beyond 20. You can mark the grid by placing a dot in each square that contains
a queen.

45. A binary string of length n is a sequence of n digits, each of which is 0 or 1. The


distance between two binary strings of the same length is the number of positions
in which they disagree; for example, the distance between the strings 01101011 and
00101110 is 3 since they differ in the second, sixth, and eighth positions.
Find as many binary strings of length 8 as you can, such that the distance between
any two of them is at least 3. You get one point per string.

7
Harvard-MIT Mathematics Tournament
February 28, 2004

Team Round

A Build-It-Yourself Table [150 points]

An infinite table of nonnegative integers is constructed as follows: in the top row, some
number is 1 and all other numbers are 0’s; in each subsequent row, every number is the sum
of some two of the three closest numbers in the preceding row. An example of such a table
is shown below.

··· 0 0 0 0 1 0 0 0 0 ···
··· 0 0 0 0 1 1 0 0 0 ···
··· 0 0 0 1 1 2 1 0 0 ···
··· 0 0 1 1 3 3 2 0 0 ···
··· 0 1 2 4 4 6 3 2 0 ···
. .. .. .. .. .. .. .. .. .. ..
.. . . . . . . . . . .

The top row (with the one 1) is called row 0; the next row is row 1; the next row is row
2, and so forth.
Note that the following problems require you to prove the statements for every table that
can be constructed by the process described above, not just for the example shown.

1. [10] Show that any number in row n (for n > 0) is at most 2n−1 .

2. [20] What is the earliest row in which the number 2004 may appear?

3. [35] Let µ ¶ µ ¶ µ ¶ µ ¶
n−1 n−1 n−1 n−1
S(n, r) = + + + ··· +
r−1 r r+1 n−1
for all n, r > 0, and in particular S(n, r) = 0 if r > n > 0. Prove that the number in
row n of the table, r columns to the left of the 1 in the top row, is at most S(n, r).
(Hint: First prove that S(n − 1, r − 1) + S(n − 1, r) = S(n, r).)

4. [25] Show that the sum of all the numbers in row n is at most (n + 2)2n−1 .

A pair of successive numbers in the same row is called a switch pair if one number in the
pair is even and the other is odd.

5. [15] Prove that the number of switch pairs in row n is at most twice the number of
odd numbers in row n.

6. [20] Prove that the number of odd numbers in row n is at most twice the number of
switch pairs in row n − 1.

7. [25] Prove that the number of switch pairs in row n is at most twice the number of
switch pairs in row n − 1.

1
Written In The Stars [125 points]

Suppose S is a finite set with a binary operation ? — that is, for any elements a, b of S,
there is defined an element a ? b of S. It is given that (a ? b) ? (a ? b) = b ? a for all a, b ∈ S.

8. [20] Prove that a ? b = b ? a for all a, b ∈ S.

Let T be the set of elements of the form a ? a for a ∈ S.

9. [15] If b is any element of T , prove that b ? b = b.

Now suppose further that (a ? b) ? c = a ? (b ? c) for all a, b, c ∈ S. (Thus we can write an


expression like a ? b ? c ? d without ambiguity.)

10. [25] Let a be an element of T . Let the image of a be the set of all elements of T that
can be represented as a ? b for some b ∈ T . Prove that if c is in the image of a, then
a ? c = c.

11. [40] Prove that there exists an element a ∈ T such that the equation a ? b = a holds
for all b ∈ T .

12. [25] Prove that there exists an element a ∈ S such that the equation a ? b = a holds
for all b ∈ S.

Sigma City [125 points]

13. [25] Let n be a positive odd integer. Prove that

blog2 nc + blog2 (n/3)c + blog2 (n/5)c + blog2 (n/7)c + · · · + blog2 (n/n)c = (n − 1)/2.

Let σ(n) denote the sum of the (positive) divisors of n, including 1 and n itself.

14. [30] Prove that


σ(1) + σ(2) + σ(3) + · · · + σ(n) ≤ n2
for every positive integer n.

15. [30] Prove that


σ(1) σ(2) σ(3) σ(n)
+ + + ··· + ≤ 2n
1 2 3 n
for every positive integer n.

16. [40] Now suppose again that n is odd. Prove that

σ(1)blog2 nc + σ(3)blog2 (n/3)c + σ(5)blog2 (n/5)c + · · · + σ(n)blog2 (n/n)c < n2/8.

2
Harvard-MIT Mathematics Tournament
February 19, 2005

Individual Round: Algebra Subject Test

1. How many real numbers x are solutions to the following equation?

|x − 1| = |x − 2| + |x − 3|

2. How many real numbers x are solutions to the following equation?

2003x + 2004x = 2005x

3. Let x, y, and z be distinct real numbers that sum to 0. Find the maximum possible
value of
xy + yz + zx
.
x2 + y 2 + z 2
j k j k j k
4. If a, b, c > 0, what is the smallest possible value of a+b
c
+ b+c
a
+ c+a
b
? (Note that
bxc denotes the greatest integer less than or equal to x.)

5. Ten positive integers are arranged around a circle. Each number is one more than the
greatest common divisor of its two neighbors. What is the sum of the ten numbers?

6. Find the sum of the x-coordinates of the distinct points of intersection of the plane
curves given by x2 = x + y + 4 and y 2 = y − 15x + 36.

7. Let x be a positive real number. Find the maximum possible value of



x2 + 2 − x4 + 4
.
x

8. Compute

X n
4 2
.
n=0 n + n + 1

9. The number 27,000,001 has exactly four prime factors. Find their sum.

10. Find the sum of the absolute values of the roots of x4 − 4x3 − 4x2 + 16x − 8 = 0.

1
Harvard-MIT Mathematics Tournament
February 19, 2005

Individual Round: Calculus Subject Test

1. Let f (x) = x3 + ax + b, with a 6= b, and suppose the tangent lines to the graph of f at
x = a and x = b are parallel. Find f (1).
R ∞ cos u R
2. A plane curve is parameterized by x(t) = t u
du and y(t) = t∞ sinu u du for 1 ≤ t ≤
2. What is the length of the curve?
R1 R1
3. Let f : R → RR be a continuous function with 0 f (x)f 0 (x)dx = 0 and 0 f (x)2 f 0 (x)dx =
18. What is 01 f (x)4 f 0 (x)dx?

4. Let f : R → R be a smooth function such that f 0 (x)2 = f (x)f 00 (x) for all x. Suppose
f (0) = 1 and f (4) (0) = 9. Find all possible values of f 0 (0).

5. Calculate ³ ´
x
lim+ xx − xx .
x→0

6. The graph of r = 2 + cos 2θ and its reflection over the line y = x bound five regions in
the plane. Find the area of the region containing the origin.

7. Two ants, one starting at (−1, 1), the other at (1, 1), walk to the right along the
parabola y = x2 such that their midpoint moves along the line y = 1 with constant
speed 1. When the left ant first hits the line y = 12 , what is its speed?

8. If f is a continuous real function such that


R 2005
f (x − 1) + f (x + 1) ≥ x + f (x) for all x,
what is the minimum possible value of 1 f (x)dx?

9. Compute

X 4
.
k=0 (4k)!

10. Let f : R → R be a smooth function such that f 0 (x) = f (1 − x) for all x and f (0) = 1.
Find f (1).

1
Harvard-MIT Mathematics Tournament
February 19, 2005

Individual Round: Combinatorics Subject Test


1. A true-false test has ten questions. If you answer five questions “true” and five “false,”
your score is guaranteed to be at least four. How many answer keys are there for which
this is true?
2. How many nonempty subsets of {1, 2, 3, . . . , 12} have the property that the sum of the
largest element and the smallest element is 13?
3. The Red Sox play the Yankees in a best-of-seven series that ends as soon as one team
wins four games. Suppose that the probability that the Red Sox win Game n is n−1 6
.
What is the probability that the Red Sox will win the series?
4. In how many ways can 4 purple balls and 4 green balls be placed into a 4 × 4 grid such
that every row and column contains one purple ball and one green ball? Only one ball
may be placed in each box, and rotations and reflections of a single configuration are
considered different.
5. Doug and Ryan are competing in the 2005 Wiffle Ball Home Run Derby. In each round,
each player takes a series of swings. Each swing results in either a home run or an out,
and an out ends the series. When Doug swings, the probability that he will hit a home
run is 1/3. When Ryan swings, the probability that he will hit a home run is 1/2. In
one round, what is the probability that Doug will hit more home runs than Ryan hits?
6. Three fair six-sided dice, each numbered 1 through 6, are rolled. What is the probability
that the three numbers that come up can form the sides of a triangle?
7. What is the maximum number of bishops that can be placed on an 8 × 8 chessboard
such that at most three bishops lie on any diagonal?
8. Every second, Andrea writes down a random digit uniformly chosen from the set
{1, 2, 3, 4}. She stops when the last two numbers she has written sum to a prime
number. What is the probability that the last number she writes down is 1?
9. Eight coins are arranged in a circle heads up. A move consists of flipping over two
adjacent coins. How many different sequences of six moves leave the coins alternating
heads up and tails up?
10. You start out with a big pile of 32004 cards, with the numbers 1, 2, 3, . . . , 32004 written
on them. You arrange the cards into groups of three any way you like; from each group,
you keep the card with the largest number and discard the other two. You now again
arrange these 32003 remaining cards into groups of three any way you like, and in each
group, keep the card with the smallest number and discard the other two. You now
have 32002 cards, and you again arrange these into groups of three and keep the largest
number in each group. You proceed in this manner, alternating between keeping the
largest number and keeping the smallest number in each group, until you have just one
card left.
How many different values are possible for the number on this final card?

1
Harvard-MIT Mathematics Tournament
February 19, 2005

Individual Round: Geometry Subject Test

1. The volume of a cube (in cubic inches) plus three times the total length of its edges
(in inches) is equal to twice its surface area (in square inches). How many inches long
is its long diagonal?

2. Let ABCD be a regular tetrahedron with side length 2. The plane parallel to edges
AB and CD and lying halfway between them cuts ABCD into two pieces. Find the
surface area of one of these pieces.

3. Let ABCD be a rectangle with area 1, and let E lie on side CD. What is the area of
the triangle formed by the centroids of triangles ABE, BCE, and ADE?

4. Let XY Z be a triangle with 6 X = 60◦ and 6 Y = 45◦ . A circle with center P passes
through points A and B on side XY , C and D on side Y Z, and E and F on side ZX.
Suppose AB = CD = EF . Find 6 XP Y in degrees.

5. A cube with side length 2 is inscribed in a sphere. A second cube, with faces parallel
to the first, is inscribed between the sphere and one face of the first cube. What is the
length of a side of the smaller cube?

6. A triangular piece of paper of area 1 is folded along a line parallel to one of the sides
and pressed flat. What is the minimum possible area of the resulting figure?

7. Let ABCD be a tetrahedron such that edges AB, AC, and AD are mutually perpen-
dicular. Let the areas of triangles ABC, ACD, and ADB be denoted by x, y, and z,
respectively. In terms of x, y, and z, find the area of triangle BCD.

8. Let T be a triangle with side lengths 26, 51, and 73. Let S be the set of points inside
T which do not lie within a distance of 5 of any side of T . Find the area of S.

9. Let AC be a diameter of a circle ω of radius 1, and let D be the point on AC such


that CD = 1/5. Let B be the point on ω such that DB is perpendicular to AC, and
let E be the midpoint of DB. The line tangent to ω at B intersects line CE at the
point X. Compute AX.

10. Let AB be the diameter of a semicircle Γ. Two circles, ω1 and ω2 , externally tangent
to each other and internally tangent to Γ, are tangent to the line AB at P and Q,
respectively, and to semicircular arc AB at C and D, respectively, with AP < AQ.
Suppose F lies on Γ such that 6 F QB = 6 CQA and that 6 ABF = 80◦ . Find 6 P DQ
in degrees.

1
Harvard-MIT Mathematics Tournament
February 19, 2005

Individual Round: General Test, Part 1

1. How many real numbers x are solutions to the following equation?

|x − 1| = |x − 2| + |x − 3|

2. A true-false test has ten questions. If you answer five questions “true” and five “false,”
your score is guaranteed to be at least four. How many answer keys are there for which
this is true?

3. Let ABCD be a regular tetrahedron with side length 2. The plane parallel to edges
AB and CD and lying halfway between them cuts ABCD into two pieces. Find the
surface area of one of these pieces.

4. Find all real solutions to x3 + (x + 1)3 + (x + 2)3 = (x + 3)3 .

5. In how many ways can 4 purple balls and 4 green balls be placed into a 4 × 4 grid such
that every row and column contains one purple ball and one green ball? Only one ball
may be placed in each box, and rotations and reflections of a single configuration are
considered different.

6. In an election, there are two candidates, A and B, who each have 5 supporters. Each
supporter, independent of other supporters, has a 21 probability of voting for his or her
candidate and a 21 probability of being lazy and not voting. What is the probability of
a tie (which includes the case in which no one votes)?
¥ ¦ ¥ b+c ¦ ¥ c+a ¦
7. If a, b, c > 0, what is the smallest possible value of a+b
c
+ a + b ? (Note that
bxc denotes the greatest integer less than or equal to x.)

8. Ten positive integers are arranged around a circle. Each number is one more than the
greatest common divisor of its two neighbors. What is the sum of the ten numbers?

9. A triangular piece of paper of area 1 is folded along a line parallel to one of the sides
and pressed flat. What is the minimum possible area of the resulting figure?

10. What is the smallest integer x larger than 1 such that x2 ends in the same three digits
as x does?

1
Harvard-MIT Mathematics Tournament
February 19, 2005

Individual Round: General Test, Part 2

1. The volume of a cube (in cubic inches) plus three times the total length of its edges
(in inches) is equal to twice its surface area (in square inches). How many inches long
is its long diagonal?

2. Find three real numbers a < b < c satisfying:

a + b + c = 21/4
1/a + 1/b + 1/c = 21/4
abc = 1.

3. Working together, Jack and Jill can paint a house in 3 days; Jill and Joe can paint the
same house in 4 days; or Joe and Jack can paint the house in 6 days. If Jill, Joe, and
Jack all work together, how many days will it take them?

4. In how many ways can 8 people be arranged in a line if Alice and Bob must be next
to each other, and Carol must be somewhere behind Dan?

5. You and I play the following game on an 8 × 8 square grid of boxes: Initially, every
box is empty. On your turn, you choose an empty box and draw an X in it; if any
of the four adjacent boxes are empty, you mark them with an X as well. (Two boxes
are adjacent if they share an edge.) We alternate turns, with you moving first, and
whoever draws the last X wins. How many choices do you have for a first move that
will enable you to guarantee a win no matter how I play?

6. A cube with side length 2 is inscribed in a sphere. A second cube, with faces parallel
to the first, is inscribed between the sphere and one face of the first cube. What is the
length of a side of the smaller cube?

7. Three distinct lines are drawn in the plane. Suppose there exist exactly n circles in
the plane tangent to all three lines. Find all possible values of n.

8. What is the maximum number of bishops that can be placed on an 8 × 8 chessboard


such that at most three bishops lie on any diagonal?

9. In how many ways can the cells of a 4 × 4 table be filled in with the digits 1, 2, . . . , 9
so that each of the 4-digit numbers formed by the columns is divisible by each of the
4-digit numbers formed by the rows?

10. Let bxc denote the greatest integer less than or equal to x. How many positive integers
less than 2005 can be expressed in the form bx bxcc for some positive real x?

1
Harvard-MIT Mathematics Tournament
February 19, 2005

Guts Round

..........................................................................................
HARVARD-MIT MATHEMATICS TOURNAMENT, FEBRUARY 19, 2005 — GUTS ROUND

1. [5] Find the largest positive integer n such that 1 + 2 + 3 + · · · + n2 is divisible by


1 + 2 + 3 + · · · + n.
2. [5] Let x, y, and z be positive real numbers such that (x · y) + z = (x + z) · (y + z).
What is the maximum possible value of xyz?

3. [5] Find the sum


21 22 24 28
+ + + +··· .
41 − 1 42 − 1 44 − 1 48 − 1
.....................................................................................

HARVARD-MIT MATHEMATICS TOURNAMENT, FEBRUARY 19, 2005 — GUTS ROUND

4. [6] What is the probability that in a randomly chosen arrangement of the numbers and
letters in “HMMT2005,” one can read either “HMMT” or “2005” from left to right?
(For example, in “5HM0M20T,” one can read “HMMT.”)
5. [6] For how many integers n between 1 and 2005, inclusive, is 2 · 6 · 10 · · · (4n − 2)
divisible by n!?
6. [6] Let m ◦ n = (m + n)/(mn + 4). Compute ((· · · ((2005 ◦ 2004) ◦ 2003) ◦ · · · ◦ 1) ◦ 0).
.....................................................................................

HARVARD-MIT MATHEMATICS TOURNAMENT, FEBRUARY 19, 2005 — GUTS ROUND

7. [6] Five people of different heights are standing in line from shortest to tallest. As it
happens, the tops of their heads are all collinear; also, for any two successive people,
the horizontal distance between them equals the height of the shorter person. If the
shortest person is 3 feet tall and the tallest person is 7 feet tall, how tall is the middle
person, in feet?
8. [6] Let ABCD be a convex quadrilateral inscribed in a circle with shortest side AB.
The ratio [BCD]/[ABD] is an integer (where [XY Z] denotes the area of triangle
XY Z.) If the lengths of AB, BC, CD, and DA are distinct integers no greater than
10, find the largest possible value of AB.
9. [6] Farmer Bill’s 1000 animals — ducks, cows, and rabbits — are standing in a circle.
In order to feel safe, every duck must either be standing next to at least one cow or
between two rabbits. If there are 600 ducks, what is the least number of cows there
can be for this to be possible?
.....................................................................................

1
.....................................................................................

HARVARD-MIT MATHEMATICS TOURNAMENT, FEBRUARY 19, 2005 — GUTS ROUND

10. [7] You are given a set of cards labeled from 1 to 100. You wish to make piles of
three cards such that in any pile, the number on one of the cards is the product of
the numbers on the other two cards. However, no card can be in more than one pile.
What is the maximum number of piles you can form at once?

11. [7] The Dingoberry Farm is a 10 mile by 10 mile square, broken up into 1 mile by 1 mile
patches. Each patch is farmed either by Farmer Keith or by Farmer Ann. Whenever
Ann farms a patch, she also farms all the patches due west of it and all the patches
due south of it. Ann puts up a scarecrow on each of her patches that is adjacent to
exactly two of Keith’s patches (and nowhere else). If Ann farms a total of 30 patches,
what is the largest number of scarecrows she could put up?

12. [7] Two vertices of a cube are given in space. The locus of points that could be a third
vertex of the cube is the union of n circles. Find n.
.....................................................................................

HARVARD-MIT MATHEMATICS TOURNAMENT, FEBRUARY 19, 2005 — GUTS ROUND

√ √
13. [7] Triangle ABC has AB = 1, BC = 7, and CA = 3. Let ℓ1 be the line through
A perpendicular to AB, ℓ2 the line through B perpendicular to AC, and P the point
of intersection of ℓ1 and ℓ2 . Find P C.

14. [7] Three noncollinear points and a line ℓ are given in the plane. Suppose no two of the
points lie on a line parallel to ℓ (or ℓ itself). There are exactly n lines perpendicular
to ℓ with the following property: the three circles with centers at the given points and
tangent to the line all concur at some point. Find all possible values of n.

15. [7] Let S be the set of lattice points inside the circle x2 + y 2 = 11. Let M be the
greatest area of any triangle with vertices in S. How many triangles with vertices in
S have area M?
.....................................................................................

HARVARD-MIT MATHEMATICS TOURNAMENT, FEBRUARY 19, 2005 — GUTS ROUND

16. [8] A regular octahedron has a side length of 1. What is the distance between two
opposite faces?

17. [8] Compute s r



q
3 4 5
2 2 2 2 2 · · ·.

18. [8] If a, b, and c are random real numbers from 0 to 1, independently and uniformly
chosen, what is the average (expected) value of the smallest of a, b, and c?
.....................................................................................

2
.....................................................................................

HARVARD-MIT MATHEMATICS TOURNAMENT, FEBRUARY 19, 2005 — GUTS ROUND

19. [8] Regular tetrahedron ABCD is projected onto a plane sending A, B, C, and D
to A′ , B ′ , C ′ , and D ′ respectively. Suppose A′ B ′ C ′ D ′ is a convex quadrilateral with
A′ B ′ = B ′ C’ and C ′ D ′ = D ′ A′ , and suppose that the area of A′ B ′ C ′ D ′ = 4. Given
these conditions, the set of possible lengths of AB consists of all real numbers in the
interval [a, b). Compute b.

20. [8] If n is a positive integer, let s(n) denote the sum of the digits of n. We say that
n is zesty if there exist positive integers x and y greater than 1 such that xy = n and
s(x)s(y) = s(n). How many zesty two-digit numbers are there?

21. [8] In triangle ABC with altitude AD, ∠BAC = 45◦ , DB = 3, and CD = 2. Find the
area of triangle ABC.
.....................................................................................

HARVARD-MIT MATHEMATICS TOURNAMENT, FEBRUARY 19, 2005 — GUTS ROUND

22. [9] Find

{ln(1 + e)} + {ln(1 + e2 )} + {ln(1 + e4 )} + {ln(1 + e8 )} + · · · ,

where {x} = x − ⌊x⌋ denotes the fractional part of x.

23. [9] The sides of a regular hexagon are trisected, resulting in 18 points, including ver-
tices. These points, starting with a vertex, are numbered clockwise as A1 , A2 , . . . , A18 .
The line segment Ak Ak+4 is drawn for k = 1, 4, 7, 10, 13, 16, where indices are taken
modulo 18. These segments define a region containing the center of the hexagon. Find
the ratio of the area of this region to the area of the large hexagon.

24. [9] In the base 10 arithmetic problem HMMT + GUT S = ROUND, each distinct
letter represents a different digit, and leading zeroes are not allowed. What is the
maximum possible value of ROUND?
.....................................................................................

3
.....................................................................................

HARVARD-MIT MATHEMATICS TOURNAMENT, FEBRUARY 19, 2005 — GUTS ROUND

25. [9] An ant starts at one vertex of a tetrahedron. Each minute it walks along a random
edge to an adjacent vertex. What is the probability that after one hour the ant winds
up at the same vertex it started at?

26. [9] In triangle ABC, AC = 3AB. Let AD bisect angle A with D lying on BC, and
let E be the foot of the perpendicular from C to AD. Find [ABD]/[CDE]. (Here,
[XY Z] denotes the area of triangle XY Z).

27. [9] In a chess-playing club, some of the players take lessons from other players. It is
possible (but not necessary) for two players both to take lessons from each other. It
so happens that for any three distinct members of the club, A, B, and C, exactly one
of the following three statements is true: A takes lessons from B; B takes lessons from
C; C takes lessons from A. What is the largest number of players there can be?
.....................................................................................

HARVARD-MIT MATHEMATICS TOURNAMENT, FEBRUARY 19, 2005 — GUTS ROUND

28. [10] There are three pairs of real numbers (x1 , y1), (x2 ,y2 ), and 
(x3 , y3) that
 satisfy
x1 x2
3 2 3 2
both x − 3xy = 2005 and y − 3x y = 2004. Compute 1 − y1 1 − y2 1 − xy33 .

29. [10] Let n > 0 be an integer. Each face of a regular tetrahedron is painted in one
of n colors (the faces are not necessarily painted different colors.) Suppose there are
n3 possible colorings, where rotations, but not reflections, of the same coloring are
considered the same. Find all possible values of n.

30. [10] A cuboctahedron is a polyhedron whose faces are squares and equilateral triangles
such that two squares and two triangles alternate around each vertex, as shown.

What is the volume of a cuboctahedron of side length 1?


.....................................................................................

4
.....................................................................................

HARVARD-MIT MATHEMATICS TOURNAMENT, FEBRUARY 19, 2005 — GUTS ROUND

31. [10] The L shape made by adjoining three congruent squares can be subdivided into
four smaller L shapes.

Each of these can in turn be subdivided, and so forth. If we perform 2005 successive
subdivisions, how many of the 42005 L’s left at the end will be in the same orientation
as the original one?

32. [10] Let a1 = 3, and for n ≥ 1, let an+1 = (n + 1)an − n. Find the smallest m ≥ 2005
such that am+1 − 1 | a2m − 1.

33. [10] Triangle ABC has incircle ω which touches AB at C1 , BC at A1 , and CA at B1 .


Let A2 be the reflection of A1 over the midpoint of BC, and define B2 and C2 similarly.
Let A3 be the intersection of AA2 with ω that is closer to A, and define B3 and C3
similarly. If AB = 9, BC = 10, and CA = 13, find [A3 B3 C3 ]/[ABC]. (Here [XY Z]
denotes the area of triangle XY Z.)
.....................................................................................

HARVARD-MIT MATHEMATICS TOURNAMENT, FEBRUARY 19, 2005 — GUTS ROUND

34. [12] A regular octahedron ABCDEF is given such that AD, BE, and CF are per-
pendicular. Let G, H, and I lie on edges AB, BC, and CA respectively such that
AG
GB
= BH
HC
= CI
IA
= ρ. For some choice of ρ > 1, GH, HI, and IG are three edges of a
regular icosahedron, eight of whose faces are inscribed in the faces of ABCDEF . Find
ρ.

35. [12] Let p = 224036583 − 1, the largest prime currently known. For how many positive
integers c do each of the quadratics ±x2 ± px ± c have rational roots?

36. [12] One hundred people are in line to see a movie. Each person wants to sit in
the front row, which contains one hundred seats, and each has a favorite seat, chosen
randomly. They enter the row one at a time from the far right. As they walk, if they
reach their favorite seat, they sit, but to avoid stepping over people, if they encounter
a person already seated, they sit to that person’s right. If the seat furthest to the right
is already taken, they sit in a different row. What is the most likely number of people
that will get to sit in the first row?
.....................................................................................

5
.....................................................................................

HARVARD-MIT MATHEMATICS TOURNAMENT, FEBRUARY 19, 2005 — GUTS ROUND

37. [15] Let a1 , a2 , . . . , a2005 be real numbers such that

a1 · 1 + a2 · 2 + a3 · 3 + · · · + a2005 · 2005 = 0
a1 · 12 + a2 · 22 + a3 · 32 + · · · + a2005 · 20052 = 0
a1 · 13 + a2 · 23 + a3 · 33 + · · · + a2005 · 20053 = 0
..
.
a1 · 12004 + a2 · 22004 + a3 · 32004 + · · · + a2005 · 20052004 = 0

and

a1 · 12005 + a2 · 22005 + a3 · 32005 + · · · + a2005 · 20052005 = 1.

What is the value of a1 ?

38. [15] In how many ways can the set of ordered pairs of integers be colored red and blue
such that for all a and b, the points (a, b), (−1 − b, a + 1), and (1 − b, a − 1) are all the
same color?

39. [15] How many regions of the plane are bounded by the graph of

x6 − x5 + 3x4 y 2 + 10x3 y 2 + 3x2 y 4 − 5xy 4 + y 6 = 0?

.....................................................................................

HARVARD-MIT MATHEMATICS TOURNAMENT, FEBRUARY 19, 2005 — GUTS ROUND

40. [18] In a town of n people, a governing council is elected as follows: each person casts
one vote for some person in the town, and anyone that receives at least five votes is
elected to council. Let c(n) denote the expected number of people elected to council if
everyone votes randomly. Find limn→∞ c(n)/n.

41. [18] There are 42 stepping stones in a pond, arranged along a circle. You are standing
on one of the stones. You would like to jump among the stones so that you move
counterclockwise by either 1 stone or 7 stones at each jump. Moreover, you would like
to do this in such a way that you visit each stone (except for the starting spot) exactly
once before returning to your initial stone for the first time. In how many ways can
you do this?

42. [18] In how many ways can 6 purple balls and 6 green balls be placed into a 4 × 4 grid
such that every row and column contains two balls of one color and one ball of the
other color? Only one ball may be placed in each box, and rotations and reflections of
a single configuration are considered different.
.....................................................................................

6
.....................................................................................

HARVARD-MIT MATHEMATICS TOURNAMENT, FEBRUARY 19, 2005 — GUTS ROUND

43. Write down an integer N between 0 and 20 inclusive. If at least N teams write down
N, your score is N; otherwise it is 0.

44. Write down a set S of positive integers, all greater than 1, whose product is P , such
that for each x ∈ S, x is a proper divisor of (P/x) + 1. Your score is 2n, where n = |S|.

45. A binary word is a finite sequence of 0’s and 1’s. A square subword is a subsequence
consisting of two identical chunks next to each other. For example, the word 100101011
contains the square subwords 00, 0101 (twice), 1010, and 11.
Find a long binary word containing a small number of square subwords. Specifically,
write down a binary word of any length n ≤ 50. Your score will be max{0, n − s},
where s is the number of occurrences of square subwords. (That is, each different
square subword will be counted according to the number of times it appears.)

7
Harvard-MIT Mathematics Tournament
February 19, 2005

Team Round A

Disconnected Domino Rally [175]

On an infinite checkerboard, the union of any two distinct unit squares is called a (dis-
connected) domino. A domino is said to be of type (a, b), with a ≤ b integers not both zero,
if the centers of the two squares are separated by a distance of a in one orthogonal direction
and b in the other. (For instance, an ordinary connected domino is of type (0, 1), and a
domino of type (1, 2) contains two squares separated by a knight’s move.)

Each of the three pairs of squares above forms a domino of type (1, 2).

Two dominoes are said to be congruent if they are of the same type. A rectangle is said to
be (a, b)-tileable if it can be partitioned into dominoes of type (a, b).

1. [15] Prove that for any two types of dominoes, there exists a rectangle that can be
tiled by dominoes of either type.

2. [25] Suppose 0 < a ≤ b and 4 - mn. Prove that the number of ways in which an m × n
rectangle can be partitioned into dominoes of type (a, b) is even.

3. [10] Show that no rectangle of the form 1 × k or 2 × n, where 4 - n, is (1, 2)-tileable.

4. [35] Show that all other rectangles of even area are (1, 2)-tileable.

5. [25] Show that for b even, there exists some M such that for every n > M , a 2b × n
rectangle is (1, b)-tileable.

6. [40] Show that for b even, there exists some M such that for every m, n > M with mn
even, an m × n rectangle is (1, b)-tileable.

7. [25] Prove that neither of the previous two problems holds if b is odd.

An Interlude — Discovering One’s Roots [100]

A kth root of unity is any complex number ω such that ω k = 1. You may use the following
facts: if ω 6= 1, then
1 + ω + ω 2 + · · · + ω k−1 = 0,
and if 1, ω, . . . , ω k−1 are distinct, then

(xk − 1) = (x − 1)(x − ω)(x − ω 2 ) · · · (x − ω k−1 ).

1
8. [25] Suppose x is a fifth root of unity. Find, in radical form, all possible values of

1 x x2 x3
2x + + + + .
1 + x 1 + x2 1 + x3 1 + x4

9. [25] Let A1 A2 . . . Ak be a regular k-gon inscribed in a circle of radius 1, and let P


be a point lying on or inside the circumcircle. Find the maximum possible value of
(P A1 )(P A2 ) · · · (P Ak ).

10. [25] Let P be a regular k-gon inscribed in a circle of radius 1. Find the sum of the
squares of the lengths of all the sides and diagonals of P .

11. [25] Let P (x) = an xn + an−1 xn−1 + · · · + a0 be a polynomial with real coefficients,
an 6= 0. Suppose every root of P is a root of unity, but P (1) 6= 0. Show that the
coefficients of P are symmetric; that is, show that an = a0 , an−1 = a1 , . . .

Early Re-tile-ment [125]

Let S = {s0 , . . . , sn } be a finite set of integers, and define S + k = {s0 + k, . . . , sn + k}. We


say that two sets S and T are equivalent, written S ∼ T , if T = S + k for some k. Given a
(possibly infinite) set of integers A, we say that S tiles A if A can be partitioned into subsets
equivalent to S. Such a partition is called a tiling of A by S.

12. [20] Suppose the elements of A are either bounded below or bounded above. Show
that if S tiles A, then it does so uniquely, i.e., there is a unique tiling of A by S.

13. [35] Let B be a set of integers either bounded below or bounded above. Then show
that if S tiles all other integers Z\B, then S tiles all integers Z.

14. [35] Suppose S tiles the natural numbers N. Show that S tiles the set {1, 2, . . . , k} for
some positive integer k.

15. [35] Suppose S tiles N. Show that S is symmetric; that is, if −S = {−sn , . . . , −s0 },
show that S ∼ −S.

2
Harvard-MIT Mathematics Tournament
February 19, 2005

Team Round B

Disconnected Domino Rally [150]

On an infinite checkerboard, the union of any two distinct unit squares is called a (dis-
connected) domino. A domino is said to be of type (a, b), with a ≤ b integers not both zero,
if the centers of the two squares are separated by a distance of a in one orthogonal direction
and b in the other. (For instance, an ordinary connected domino is of type (0, 1), and a
domino of type (1, 2) contains two squares separated by a knight’s move.)

Each of the three pairs of squares above forms a domino of type (1, 2).

Two dominoes are said to be congruent if they are of the same type. A rectangle is said to
be (a, b)-tileable if it can be partitioned into dominoes of type (a, b).

1. [15] Let 0 < m ≤ n be integers. How many different (i.e., noncongruent) dominoes
can be formed by choosing two squares of an m × n array?

2. [10] What are the dimensions of the rectangle of smallest area that is (a, b)-tileable?

3. [20] Prove that every (a, b)-tileable rectangle contains a rectangle of these dimensions.

4. [30] Prove that an m × n rectangle is (b, b)-tileable if and only if 2b | m and 2b | n.

5. [35] Prove that an m × n rectangle is (0, b)-tileable if and only if 2b | m or 2b | n.

6. [40] Let k be an integer such that k | a and k | b. Prove that if an m × n rectangle is


(a, b)-tileable, then 2k | m or 2k | n.

An Interlude — Discovering One’s Roots [100]

A kth root of unity is any complex number ω such that ω k = 1.

7. [15] Find a real, irreducible quartic polynomial with leading coefficient 1 whose roots
are all twelfth roots of unity.

8. [25] Let x and y be two kth roots of unity. Prove that (x + y)k is real.

9. [30] Let x and y be two distinct roots of unity. Prove that x + y is also a root of unity
if and only if xy is a cube root of unity.

10. [30] Let x, y, and z be three roots of unity. Prove that x + y + z is also a root of unity
if and only if x + y = 0, y + z = 0, or z + x = 0.

1
Early Re-tile-ment [150]

Let S = {s0 , . . . , sn } be a finite set of integers, and define S + k = {s0 + k, . . . , sn + k}.


We say that S and T are equivalent, written S ∼ T , if T = S + k for some k. Given a
(possibly infinite) set of integers A, we say that S tiles A if A can be partitioned into subsets
equivalent to S. Such a partition is called a tiling of A by S.

11. [20] Find all sets S with minimum element 1 that tile A = {1, . . . , 12}.

12. [35] Let A be a finite set with more than one element. Prove that the number of
nonequivalent sets S which tile A is always even.

13. [25] Exhibit a set S which tiles the integers Z but not the natural numbers N.

14. [30] Suppose that S tiles the set of all integer cubes. Prove that S has only one
element.

15. [40] Suppose that S tiles the set of odd prime numbers. Prove that S has only one
element.

2
IXth Annual Harvard-MIT Mathematics Tournament
Saturday 25 February 2006

Individual Round: Algebra Test

1. Larry can swim from Harvard to MIT (with the current of the Charles River) in 40 minutes,
or back (against the current) in 45 minutes. How long does it take him to row from Harvard
to MIT, if he rows the return trip in 15 minutes? (Assume that the speed of the current and
Larry’s swimming and rowing speeds relative to the current are all constant.) Express your
answer in the format mm:ss.

2. Find all real solutions (x, y) of the system x2 + y = 12 = y 2 + x.

3. The train schedule in Hummut is hopelessly unreliable. Train A will enter Intersection X
from the west at a random time between 9:00 am and 2:30 pm; each moment in that interval
is equally likely. Train B will enter the same intersection from the north at a random time
between 9:30 am and 12:30 pm, independent of Train A; again, each moment in the interval is
equally likely. If each train takes 45 minutes to clear the intersection, what is the probability
of a collision today?

4. Let a1 , a2 , . . . be a sequence defined by a1 = a2 = 1 and an+2 = an+1 + an for n ≥ 1. Find



X an
.
4n+1
n=1

5. Tim has a working analog 12-hour clock with two hands that run continuously (instead of,
say, jumping on the minute). He also has a clock that runs really slow—at half the correct
rate, to be exact. At noon one day, both clocks happen to show the exact time. At any given
instant, the hands on each clock form an angle between 0◦ and 180◦ inclusive. At how many
times during that day are the angles on the two clocks equal?
√ √ √
6. Let a, b, c be the roots of x3 −9x2 +11x−1 = 0, and let s = a+ b+ c. Find s4 −18s2 −8s.

7. Let
f (x) = x4 − 6x3 + 26x2 − 46x + 65.
Let the roots of f (x) be ak + ibk for k = 1, 2, 3, 4. Given that the ak , bk are all integers, find
|b1 | + |b2 | + |b3 | + |b4 |.

8. Solve for all complex numbers z such that z 4 + 4z 2 + 6 = z.

9. Compute the value of the infinite series



X n4 + 3n2 + 10n + 10
2n · (n4 + 4)
n=2

10. Determine the maximum value attained by

x4 − x2
x6 + 2x3 − 1
over real numbers x > 1.
IXth Annual Harvard-MIT Mathematics Tournament
Saturday 25 February 2006

Individual Round: Calculus Test

1. A nonzero polynomial f (x) with real coefficients has the property that f (x) = f 0 (x)f 00 (x).
What is the leading coefficient of f (x)?
ex cos x − 1 − x
2. Compute lim .
x→0 sin(x2 )
3. At time 0, an ant is at (1, 0) and a spider is at (−1, 0). The ant starts walking
counterclockwise along the unit circle, and the spider starts creeping to the right along
the x-axis. It so happens that the ant’s horizontal speed is always half the spider’s.
What will the shortest distance ever between the ant and the spider be?

X k4
4. Compute .
k=1
k!
Z 1
dx
5. Compute √ √ .
0 x+ 3x
6. A triangle with vertices at (1003, 0), (1004, 3), and (1005, 1) in the xy-plane is revolved
all the way around the y-axis. Find the volume of the solid thus obtained.

7. Find all positive real numbers c such that the graph of f : R → R given by f (x) =
x3 − cx has the property that the circle of curvature at any local extremum is centered
at a point on the x-axis.
Z π/3
8. Compute x tan2 (x)dx.
0

9. Compute the sum of all real numbers x such that

2x6 − 3x5 + 3x4 + x3 − 3x2 + 3x − 1 = 0.

10. Suppose f and g are differentiable functions such that

xg(f (x))f 0 (g(x))g 0 (x) = f (g(x))g 0 (f (x))f 0 (x)

for all real x. Moreover, f is nonnegative and g is positive. Furthermore,


Z a
e−2a
f (g(x))dx = 1 −
0 2

for all reals a. Given that g(f (0)) = 1, compute the value of g(f (4)).
IXth Annual Harvard-MIT Mathematics Tournament
Saturday 25 February 2006

Individual Round: Combinatorics Test

1. Vernonia High School has 85 seniors, each of whom plays on at least one of the school’s three varsity
sports teams: football, baseball, and lacrosse. It so happens that 74 are on the football team; 26 are
on the baseball team; 17 are on both the football and lacrosse teams; 18 are on both the baseball and
football teams; and 13 are on both the baseball and lacrosse teams. Compute the number of seniors
playing all three sports, given that twice this number are members of the lacrosse team.
2. Compute
2
X n60
X n3
X n2
X n1
X
··· 1.
n60 =0 n59 =0 n2 =0 n1 =0 n0 =0

3. A moth starts at vertex A of a certain cube and is trying to get to vertex B, which is opposite A, in
five or fewer “steps,” where a step consists in traveling along an edge from one vertex to another. The
moth will stop as soon as it reaches B. How many ways can the moth achieve its objective?
4. A dot is marked at each vertex of a triangle ABC. Then, 2, 3, and 7 more dots are marked on the
sides AB, BC, and CA, respectively. How many triangles have their vertices at these dots?
5. Fifteen freshmen are sitting in a circle around a table, but the course assistant (who remains standing)
has made only six copies of today’s handout. No freshman should get more than one handout, and any
freshman who does not get one should be able to read a neighbor’s. If the freshmen are distinguishable
but the handouts are not, how many ways are there to distribute the six handouts subject to the above
conditions?
6. For how many ordered triplets (a, b, c) of positive integers less than 10 is the product a × b × c divisible
by 20?
7. Let n be a positive integer, and let Pushover be a game played by two players, standing squarely
facing each other, pushing each other, where the first person to lose balance loses. At the HMPT,
2n+1 competitors, numbered 1 through 2n+1 clockwise, stand in a circle. They are equals in Pushover:
whenever two of them face off, each has a 50% probability of victory. The tournament unfolds in n + 1
rounds. In each round, the referee randomly chooses one of the surviving players, and the players pair
off going clockwise, starting from the chosen one. Each pair faces off in Pushover, and the losers leave
the circle. What is the probability that players 1 and 2n face each other in the last round? Express
your answer in terms of n.

8. In how many ways can we enter numbers from the set {1, 2, 3, 4} into a 4 × 4 array so that all of the
following conditions hold?
(a) Each row contains all four numbers.
(b) Each column contains all four numbers.
(c) Each “quadrant” contains all four numbers. (The quadrants are the four corner 2 × 2 squares.)

9. Eight celebrities meet at a party. It so happens that each celebrity shakes hands with exactly two
others. A fan makes a list of all unordered pairs of celebrities who shook hands with each other. If
order does not matter, how many different lists are possible?
10. Somewhere in the universe, n students are taking a 10-question math competition. Their collective
performance is called laughable if, for some pair of questions, there exist 57 students such that either
all of them answered both questions correctly or none of them answered both questions correctly.
Compute the smallest n such that the performance is necessarily laughable.
IXth Annual Harvard-MIT Mathematics Tournament
Saturday 25 February 2006

Individual Round: Geometry Test

1. Octagon ABCDEF GH is equiangular. Given that AB = 1, BC = 2, CD = 3,


DE = 4, and EF = F G = 2, compute the perimeter of the octagon.

2. Suppose ABC is a scalene right triangle, and P is the point on hypotenuse AC such
that ∠ABP = 45◦ . Given that AP = 1 and CP = 2, compute the area of ABC.

3. Let A, B, C, and D be points on a circle such that AB = 11 and CD = 19. Point


P is on segment AB with AP = 6, and Q is on segment CD with CQ = 7. The line
through P and Q intersects the circle at X and Y . If P Q = 27, find XY .

4. Let ABC be a triangle such that AB = 2, CA = 3, and BC = 4. A semicircle with its


diameter on BC is tangent to AB and AC. Compute the area of the semicircle.

5. Triangle ABC has side lengths AB = 2 5, BC = 1, and CA = 5. Point D is on side
AC such that CD = 1, and F is a point such that BF = 2 and CF = 3. Let E be the
intersection of lines AB and DF . Find the area of CDEB.

6. A circle of radius t is tangent to the hypotenuse, the incircle, and one leg of an isosceles
right triangle with inradius r = 1 + sin π8 . Find rt.

7. Suppose ABCD is an isosceles trapezoid in which AB k CD. Two mutually externally


tangent circles ω1 and ω2 are inscribed in ABCD such that ω1 is tangent to AB, BC,
and CD while ω2 is tangent to AB, DA, and CD. Given that AB = 1, CD = 6,
compute the radius of either circle.

8. Triangle ABC has a right angle at B. Point D lies on side BC such that 3∠BAD =
∠BAC. Given AC = 2 and CD = 1, compute BD.

9. Four spheres, each of radius r, lie inside a regular tetrahedron with side length 1 such
that each sphere is tangent to three faces of the tetrahedron and to the other three
spheres. Find r.

10. Triangle ABC has side lengths AB = 65, BC = 33, and AC = 56. Find the radius of
the circle tangent to sides AC and BC and to the circumcircle of triangle ABC.
IXth Annual Harvard-MIT Mathematics Tournament
Saturday 25 February 2006

Individual Round: General Test, Part 1

1. How many positive integers x are there such that 3x has 3 digits and 4x has four digits?

2. What is the probability that two cards randomly selected (without replacement) from
a standard 52-card deck are neither of the same value nor the same suit?

3. A square and an equaliteral triangle together have the property that the area of each
is the perimeter of the other. Find the square’s area.

4. Find

q p
31 + 31 + 31 + . . .
.

q p
1 + 1 + 1 + ...

5. In the plane, what is the length of the shortest path from (−2, 0) to (2, 0) that avoids
the interior of the unit circle (i.e., circle of radius 1) centered at the origin?

6. Six celebrities meet at a party. It so happens that each celebrity shakes hands with
exactly two others. A fan makes a list of all unordered pairs of celebrities who shook
hands with each other. If order does not matter, how many different lists are possible?

7. The train schedule in Hummut is hopelessly unreliable. Train A will enter Intersection
X from the west at a random time between 9:00 am and 2:30 pm; each moment in that
interval is equally likely. Train B will enter the same intersection from the north at
a random time between 9:30 am and 12:30 pm, independent of Train A; again, each
moment in the interval is equally likely. If each train takes 45 minutes to clear the
intersection, what is the probability of a collision today?

8. A dot is marked at each vertex of a triangle ABC. Then, 2, 3, and 7 more dots are
marked on the sides AB, BC, and CA, respectively. How many triangles have their
vertices at these dots?

9. Take a unit sphere S, i.e., a sphere with radius 1. Circumscribe a cube C about S,
and inscribe a cube D in S, so that every edge of cube C is parallel to some edge of
cube D. What is the shortest possible distance from a point on a face of C to a point
on a face of D?

10. A positive integer n is called “flippant” if n does not end in 0 (when written in decimal
notation) and, moreover, n and the number obtained by reversing the digits of n are
both divisible by 7. How many flippant integers are there between 10 and 1000?
IXth Annual Harvard-MIT Mathematics Tournament
Saturday 25 February 2006

Individual Round: General Test, Part 2

1. Larry can swim from Harvard to MIT (with the current of the Charles River) in 40 minutes, or back
(against the current) in 45 minutes. How long does it take him to row from Harvard to MIT, if he
rows the return trip in 15 minutes? (Assume that the speed of the current and Larry’s swimming and
rowing speeds relative to the current are all constant.) Express your answer in the format mm:ss.
2. Find
22 32 42 20062
· 2 · 2 · ··· · .
22 −1 3 −1 4 −1 20062 − 1
3. Let C be the unit circle. Four distinct, smaller congruent circles C1 , C2 , C3 , C4 are internally tangent
to C such that Ci is externally tangent to Ci−1 and Ci+1 for i = 1, . . . , 4 where C5 denotes C1 and C0
represents C4 . Compute the radius of C1 .
4. Vernonia High School has 85 seniors, each of whom plays on at least one of the school’s three varsity
sports teams: football, baseball, and lacrosse. It so happens that 74 are on the football team; 26 are
on the baseball team; 17 are on both the football and lacrosse teams; 18 are on both the baseball and
football teams; and 13 are on both the baseball and lacrosse teams. Compute the number of seniors
playing all three sports, given that twice this number are members of the lacrosse team.

2 2
a + b
5. If a, b are nonzero real numbers such that a + b = 8ab, find the value of .
a − b
6. Octagon ABCDEF GH is equiangular. Given that AB = 1, BC = 2, CD = 3, DE = 4, and
EF = F G = 2, compute the perimeter of the octagon.
7. What is the smallest positive integer n such that n2 and (n + 1)2 both contain the digit 7 but (n + 2)2
does not?
8. Six people, all of different weights, are trying to build a human pyramid: that is, they get into the
formation

A
BC
DEF

We say that someone not in the bottom row is “supported by” each of the two closest people beneath
her or him. How many different pyramids are possible, if nobody can be supported by anybody of
lower weight?
9. Tim has a working analog 12-hour clock with two hands that run continuously (instead of, say, jumping
on the minute). He also has a clock that runs really slow—at half the correct rate, to be exact. At
noon one day, both clocks happen to show the exact time. At any given instant, the hands on each
clock form an angle between 0◦ and 180◦ inclusive. At how many times during that day are the angles
on the two clocks equal?
10. Fifteen freshmen are sitting in a circle around a table, but the course assistant (who remains standing)
has made only six copies of today’s handout. No freshman should get more than one handout, and any
freshman who does not get one should be able to read a neighbor’s. If the freshmen are distinguishable
but the handouts are not, how many ways are there to distribute the six handouts subject to the above
conditions?
IXth Annual Harvard-MIT Mathematics Tournament
Saturday 25 February 2006

Guts Round

.............................................................................................

IXth HARVARD-MIT MATHEMATICS TOURNAMENT, 25 FEBRUARY 2006 — GUTS ROUND

1. [5] A bear walks one mile south, one mile east, and one mile north, only to find itself where
it started. Another bear, more energetic than the first, walks two miles south, two miles east,
and two miles north, only to find itself where it started. However, the bears are not white
and did not start at the north pole. At most how many miles apart, to the nearest .001 mile,
are the two bears’ starting points?

2. [5] Compute the positive integer less than 1000 which has exactly 29 positive proper divisors.
(Here we refer to positive integer divisors other than the number itself.)

3. [5] At a nursey, 2006 babies sit in a circle. Suddenly each baby pokes the baby immediately
to either its left or its right, with equal probability. What is the expected number of unpoked
babies?
...........................................................................................

IXth HARVARD-MIT MATHEMATICS TOURNAMENT, 25 FEBRUARY 2006 — GUTS ROUND

4. [6] Ann and Anne are in bumper cars starting 50 meters apart. Each one approaches the
other at a constant ground speed of 10 km/hr. A fly starts at Ann, flies to Anne, then back
to Ann, and so on, back and forth until it gets crushed when the two bumper cars collide.
When going from Ann to Anne, the fly flies at 20 km/hr; when going in the opposite direction
the fly flies at 30 km/hr (thanks to a breeze). How many meters does the fly fly?

5. [6] Find the number of solutions in positive integers (k; a1 , a2 , . . . , ak ; b1 , b2 , . . . , bk ) to the


equation a1 (b1 ) + a2 (b1 + b2 ) + · · · + ak (b1 + b2 + · · · + bk ) = 7.

6. [6] Suppose ABC is a triangle such that AB = 13, BC = 15, and CA = 14. Say D is the
midpoint of BC, E is the midpoint of AD, F is the midpoint of BE, and G is the midpoint
of DF . Compute the area of triangle EF G.
...........................................................................................

IXth HARVARD-MIT MATHEMATICS TOURNAMENT, 25 FEBRUARY 2006 — GUTS ROUND

jxk jxk
7. [6] Find all real numbers x such that x2 + + = 10.
2 3
8. [6] How many ways are there to label the faces of a regular octahedron with the integers
1–8, using each exactly once, so that any two faces that share an edge have numbers that
are relatively prime? Physically realizable rotations are considered indistinguishable, but
physically unrealizable reflections are considered different.

9. [6] Four unit circles are centered at the vertices of a unit square, one circle at each vertex.
What is the area of the region common to all four circles?
...........................................................................................
...........................................................................................

IXth HARVARD-MIT MATHEMATICS TOURNAMENT, 25 FEBRUARY 2006 — GUTS ROUND

10. [7] Let f (x) = x2 − 2x. How many distinct real numbers c satisfy f (f (f (f (c)))) = 3?
n2 +7n+136
11. [7] Find all positive integers n > 1 for which n−1 is the square of a positive integer.

12. [7] For each positive integer n let Sn denote the set {1, 2, 3, . . . , n}. Compute the number
of triples of subsets A, B, C of S2006 (not necessarily nonempty or proper) such that A is a
subset of B and S2006 − A is a subset of C.
...........................................................................................

IXth HARVARD-MIT MATHEMATICS TOURNAMENT, 25 FEBRUARY 2006 — GUTS ROUND

The problems in this batch all depend on each other. If you solve them correctly, you will
produce a triple of mutually consistent answers. There is only one such triple. Your score
will be determined by how many of your answers match that triple.

13. [7] Let Z be as in problem 15. Let X be the greatest integer such that |XZ| ≤ 5. Find X.

14. [7] Let X be as in problem 13. Let Y be the number of ways to order X crimson flowers, X
scarlet flowers, and X vermillion flowers in a row so that no two flowers of the same hue are
adjacent. (Flowers of the same hue are mutually indistinguishable.) Find Y .

15. [7] Let Y be as in problem 14. Find the maximum Z such that three circles of radius Z can
simultaneously fit inside an equilateral triangle of area Y without overlapping each other.
...........................................................................................

IXth HARVARD-MIT MATHEMATICS TOURNAMENT, 25 FEBRUARY 2006 — GUTS ROUND

q
1+an
16. [8] A sequence a1 , a2 , a3 , . . . of positive reals satisfies an+1 = 2 . Determine all a1 such
√ √
6+ 2
that ai = 4 for some positive integer i.

17. [8] Begining at a vertex, an ant is crawls between the vertices of a regular octahedron. After
reaching a vertex, it randomly picks a neighboring vertex (sharing an edge) and walks to that
vertex along the adjoining edge (with all possibilities equally likely.) What is the probability
that after walking along 2006 edges, the ant returns to the vertex where it began?

18. [8] Cyclic quadrilateral ABCD has side lengths AB = 1, BC = 2, CD = 3 and DA = 4.


Points P and Q are the midpoints of BC and DA. Compute P Q2 .
...........................................................................................
...........................................................................................

IXth HARVARD-MIT MATHEMATICS TOURNAMENT, 25 FEBRUARY 2006 — GUTS ROUND

19. [8] Let ABC be a triangle with AB = 2, CA = 3, BC = 4. Let D be the point diametrically
opposite A on the circumcircle of ABC, and let E lie on line AD such that D is the midpoint
of AE. Line l passes through E perpendicular to AE, and F and G are the intersections of
the extensions of AB and AC with l. Compute F G.

20. [8] Compute the number of real solutions (x, y, z, w) to the system of equations:

x = z + w + zwx z = x + y + xyz
y = w + x + wxy w = y + z + yzw

21. [8] Find the smallest positive integer k such that z 10 + z 9 + z 6 + z 5 + z 4 + z + 1 divides z k − 1.
...........................................................................................

IXth HARVARD-MIT MATHEMATICS TOURNAMENT, 25 FEBRUARY 2006 — GUTS ROUND

22. [9] Let f (x) be a degree 2006 polynomial with complex roots c1 , c2 , . . . , c2006 , such that the set
{|c1 |, |c2 |, . . . , |c2006 |} consists of exactly 1006 distinct values. What is the minimum number
of real roots of f (x)?

23. [9] Let a0 , a1 , a2 , . . . be a sequence of real numbers defined by a0 = 21, a1 = 35, and an+2 =
4an+1 − 4an + n2 for n ≥ 2. Compute the remainder obtained when a2006 is divided by 100.

24. [9] Two 18-24-30 triangles in the plane share the same circumcircle as well as the same incircle.
What’s the area of the region common to both the triangles?
...........................................................................................

IXth HARVARD-MIT MATHEMATICS TOURNAMENT, 25 FEBRUARY 2006 — GUTS ROUND

25. [9] Points A, C, and B lie on a line in that order such that AC = 4 and BC = 2. Circles
ω1 , ω2 , and ω3 have BC, AC, and AB as diameters. Circle Γ is externally tangent to ω1 and
ω2 at D and E respectively, and is internally tangent to ω3 . Compute the circumradius of
triangle CDE.

26. [9] Let a ≥ b ≥ c be real numbers such that

a2 bc + ab2 c + abc2 + 8 = a + b + c
a2 b + a2 c + b2 c + b2 a + c2 a + c2 b + 3abc = −4
a2 b2 c + ab2 c2 + a2 bc2 = 2 + ab + bc + ca

If a + b + c > 0, then compute the integer nearest to a5 .

27. [9] Let N denote the number of subsets of {1, 2, 3, . . . , 100} that contain more prime numbers
than multiples of 4. Compute the largest integer k such that 2k divides N .
...........................................................................................
...........................................................................................

IXth HARVARD-MIT MATHEMATICS TOURNAMENT, 25 FEBRUARY 2006 — GUTS ROUND

28. [10] A pebble is shaped as the intersection of a cube of side length 1 with the solid sphere
tangent to all of the cube’s edges. What is the surface area of this pebble?

29. [10] Find the area in the first quadrant bounded by the hyperbola x2 − y 2 = 1, the x-axis,
and the line 3x = 4y.

30. [10] ABC is an acute triangle with incircle ω. ω is tangent to sides BC, CA, and AB at
D, E, and F respectively. P is a point on the altitude from A such that Γ, the circle with
diameter AP , is tangent to ω. Γ intersects AC and AB at X and Y respectively. Given
XY = 8, AE = 15, and that the radius of Γ is 5, compute BD · DC.
...........................................................................................

IXth HARVARD-MIT MATHEMATICS TOURNAMENT, 25 FEBRUARY 2006 — GUTS ROUND

The problems in this batch all depend on each other. If you solve them correctly, you will
produce a triple of mutually consistent answers. There is only one such triple. Your score
will be determined by how many of your answers match that triple.

31. [10] Let A be as in problem 33. Let W be the sum of all positive integers that divide A.
Find W .

32. [10] In the alphametic W E × EY E = SCEN E, each different letter stands for a different
digit, and no word begins with a 0. The W in this problem has the same value as the W in
problem 31. Find S.

33. [10] Let W , S be as in problem 32. Let A be the least positive integer such that an acute
triangle with side lengths S, A, and W exists. Find A.
...........................................................................................

IXth HARVARD-MIT MATHEMATICS TOURNAMENT, 25 FEBRUARY 2006 — GUTS ROUND

34. [12] In bridge, a standard 52-card deck is dealt in the usual way to 4 players. By convention,
each hand is assigned a number of “points” based on the formula

4 × (# A’s) + 3 × (# K’s) + 2 × (# Q’s) + 1 × (# J’s).

Given that a particular hand has exactly 4 cards that are A, K, Q, or J, find the probability
that its point value is 13 or higher.

35. [12] A sequence is defined by A0 = 0, A1 = 1, A2 = 2, and, for integers n ≥ 3,


An−1 + An−2 + An−3 1
An = + 4
3 n − n2
Compute limN →∞ AN .

36. [12] Four points are independently chosen uniformly at random from the interior of a regular
dodecahedron. What is the probability that they form a tetrahedron whose interior contains
the dodecahedron’s center?
...........................................................................................
...........................................................................................

IXth HARVARD-MIT MATHEMATICS TOURNAMENT, 25 FEBRUARY 2006 — GUTS ROUND


X 2n + 5
37. [15] Compute
2n · (n3 + 7n2 + 14n + 8)
n=1

38. [15] Suppose ABC is a triangle with incircle ω, and ω is tangent to BC and CA at D and
E respectively. The bisectors of ∠A and ∠B intersect line DE at F and G respectively, such
that BF = 1 and F G = GA = 6. Compute the radius of ω.

39. [15] A fat coin is one which, when tossed, has a 2/5 probability of being heads, 2/5 of being
tails, and 1/5 of landing on its edge. Mr. Fat starts at 0 on the real line. Every minute, he
tosses a fat coin. If it’s heads, he moves left, decreasing his coordinate by 1; if it’s tails, he
moves right, increasing his coordinate by 1. If the coin lands on its edge, he moves back to
0. If Mr. Fat does this ad infinitum, what fraction of his time will he spend at 0?
...........................................................................................

IXth HARVARD-MIT MATHEMATICS TOURNAMENT, 25 FEBRUARY 2006 — GUTS ROUND


X 3k + 1
40. [18] Compute · (−1)k+1 .
2k 3 + k 2
k=1

41. [18] Let Γ denote the circumcircle of triangle ABC. Point D is on AB such that CD bisects
∠ACB. Points P and Q are on Γ such that P Q passes through D and is perpendicular to
CD. Compute P Q, given that BC = 20, CA = 80, AB = 65.

42. [18] Suppose hypothetically that a certain, very corrupt political entity in a universe holds
an election with two candidates, say A and B. A total of 5,825,043 votes are cast, but, in a
sudden rainstorm, all the ballots get soaked. Undaunted, the election officials decide to guess
what the ballots say. Each ballot has a 51% chance of being deemed a vote for A, and a 49%
chance of being deemed a vote for B. The probability that B will win is 10−X . What is X
rounded to the nearest 10?
...........................................................................................
...........................................................................................

IXth HARVARD-MIT MATHEMATICS TOURNAMENT, 25 FEBRUARY 2006 — GUTS ROUND

43. Write down at least one, and up to ten, different 3-digit prime numbers. If you somehow fail
to do this, we will ignore your submission for this problem. Otherwise, you’re entered into a
game with other teams. In this game, you start with 10 points, and each number you write
down is like a bet: if no one else writes that number, you gain 1 point, but if anyone else
writes that number, you lose 1 point. Thus, your score on this problem can be anything from
0 to 20.

44. On the Euclidean plane are given 14 points:

A = (0, 428) B = (9, 85) C = (42, 865) D = (192, 875)


E = (193, 219) F = (204, 108) G = (292, 219) H = (316, 378)
I = (375, 688) J = (597, 498) K = (679, 766) L = (739, 641)
M = (772, 307) N = (793, 0)

A fly starts at A, visits all the other points, and comes back to A in such a way as to minimize
the total distance covered. What path did the fly take? Give the names of the points it visits
in order. Your score will be

20 + bthe optimal distancec − byour distancec

or 0, whichever is greater.

45. On your answer sheet, clearly mark at least seven points, as long as

(i) No three are collinear.


(ii) No seven form a convex heptagon.

Please do not cross out any points; erase if you can do so neatly. If the graders deem that
your paper is too messy, or if they determine that you violated one of those conditions, your
submission for this problem will be disqualified. Otherwise, your score will be the number of
points you marked minus 6, even if you actually violated one of the conditions but were able
to fool the graders.
IXth Annual Harvard-MIT Mathematics Tournament
Saturday 25 February 2006

Team Round A

Mobotics [120]
Spring is finally here in Cambridge, and it’s time to mow our lawn. For the purpose of these problems,
our lawn consists of little clumps of grass arranged at the points of a certain grid (to be specified later).
Our machinery consists of a fleet of identical mowbots (or “mobots” for short). A mobot is a lawn-mowing
machine. To mow our lawn, we begin by choosing a formation: we place as many mobots as we want at
various clumps of grass and orient each mobot’s head in a certain direction. At the blow of a whistle, each
mobot starts moving in the direction we’ve chosen, mowing every clump of grass in its path (including the
clump it starts on) until it goes off the lawn.
Because the spring is so young, our lawn is rather delicate. Consequently, we want to make sure that
every clump of grass is mowed once and only once. We will not consider formations that do not meet this
criterion.
One more thing: two formations are considered “different” if there exists a clump of grass for which either
(1) for exactly one of the formations does a mobot starts on that clump, or (2) there are mobots starting on
this clump for both the formations, but they’re oriented in different directions.
1. [15] For this problem, our lawn consists of a row of n clumps of grass. This row runs in an east-west
direction. In our formation, each mobot may be oriented toward the north, south, east, or west. One
example of an allowable formation if n = 6 is symbolized below:
↑ ↑
· ·← ↓

(The mobot on the third clump will move westward, mowing the first three clumps. Each of the last
three clumps is mowed by a different mobot.) Here’s another allowable formation for n = 6, considered
different from the first:
· ·← ↑ ↑ →
Compute the number of different allowable formations for any given n.
2. [25] For this problem, our lawn is an m × n rectangular grid of clumps, that is, with m rows running
east-west and n columns running north-south. To be even more explicit, we might say our clumps are
at the lattice points
{(x, y) ∈ Z2 | 0 ≤ x < n and 0 ≤ y < m}.
However, mobots are now allowed to be oriented to go either north or east only. So one allowable
formation for m = 2, n = 3 might be as follows:

· → ·

→ ·

(m + n)!
Prove that the number of allowable formations for given m and n is .
m! n!
3. [40] In this problem, we stipulate that m ≥ n, and the lawn is shaped differently. The clumps are now
at the lattice points in a trapezoid:
{(x, y) ∈ Z2 | 0 ≤ x < n and 0 ≤ y < m + 1 − n + x},
As in problem 2, mobots can be set to move either north or east. For given m and n, determine with
proof the number of allowable formations.
4. [15] In this problem and the next, the lawn consists of points in a triangular grid of size n, so that for
n = 3 the lawn looks like
·
· ·
· · ·

1
Mobots are allowed to be oriented to the east, 30◦ west of north, or 30◦ west of south. Under these
conditions, for any given n, what is the minimum number of mobots needed to now the lawn?
5. [25] With the same lawn and the same allowable mobot orientations as in the previous problem, let
us call a formation “happy” if it is invariant under 120◦ rotations. (A rotation applies both to the
positions of the mobots and to their orientations.) An example of a happy formation for n = 2 might
be

-
. →

Find the number of happy formations for a given n.

Polygons [110]
6. [15] Let n be an integer at least 5. At most how many diagonals of a regular n-gon can be simultaneously
drawn so that no two are parallel? Prove your answer.
7. [25] Given a convex n-gon, n ≥ 4, at most how many diagonals can be drawn such that each drawn
diagonal intersects every other drawn diagonal either in the interior of the n-gon or at a vertex? Prove
your answer.
8. [15] Given a regular n-gon with sides of length 1, what is the smallest radius r such that there is
a non-empty intersection of n circles of radius r centered at the vertices of the n-gon? Give r as a
formula in terms of n. Be sure to prove your answer.
9. [40] Let n ≥ 3 be a positive integer. Prove that given any n angles 0 < θ1 , θ2 , . . . , θn < 180◦ , such that
their sum is 180(n − 2) degrees, there exists a convex n-gon having exactly those angles, in that order.
10. [15] Suppose we have an n-gon such that each interior angle, measured in degrees, is a positive integer.
Suppose further that all angles are less than 180◦ , and that all angles are different sizes. What is the
maximum possible value of n? Prove your answer.

What do the following problems have in common? [170]


11. [15] The lottery cards of a certain lottery contain all nine-digit numbers that can be formed with the
digits 1, 2 and 3. There is exactly one number on each lottery card. There are only red, yellow and
blue lottery cards. Two lottery numbers that differ from each other in all nine digits always appear on
cards of different color. Someone draws a red card and a yellow card. The red card has the number
122 222 222 and the yellow card has the number 222 222 222. The first prize goes to the lottery card
with the number 123 123 123. What color(s) can it possibly have? Prove your answer.
12. [25] A 3 × 3 × 3 cube is built from 27 unit cubes. Suddenly five of those cubes mysteriously teleport
away. What is the minimum possible surface area of the remaining solid? Prove your answer.
13. [40] Having lost a game of checkers and my temper, I dash all the pieces to the ground but one. This
last checker, which is perfectly circular in shape, remains completely on the board, and happens to
cover equal areas of red and black squares. Prove that the center of this piece must lie on a boundary
between two squares (or at a junction of four).
14. [40] A number n is called bummed out if there is exactly one ordered pair of positive integers (x, y)
such that
bx2 /yc + by 2 /xc = n.
Find all bummed out numbers.
15. [50] Find, with proof, all positive integer palindromes whose square is also a palindrome.

2
IXth Annual Harvard-MIT Mathematics Tournament
Saturday 25 February 2006

Team Round B

Mobotics [135]
Spring is finally here in Cambridge, and it’s time to mow our lawn. For the purpose of these problems, our
lawn consists of little clumps of grass arranged in an m × n rectangular grid, that is, with m rows running
east-west and n columns running north-south. To be even more explicit, we might say our clumps are at the
lattice points
{(x, y) ∈ Z2 | 0 ≤ x < n and 0 ≤ y < m}.
Our machinery consists of a fleet of identical mowbots (or “mobots” for short). A mobot is a lawn-mowing
machine. To mow our lawn, we begin by choosing a formation: we place as many mobots as we want at
various clumps of grass and orient each mobot’s head in a certain direction, either north or east (not south
or west). At the blow of a whistle, each mobot starts moving in the direction we’ve chosen, mowing every
clump of grass in its path (including the clump it starts on) until it goes off the lawn.
Because the spring is so young, our lawn is rather delicate. Consequently, we want to make sure that
every clump of grass is mowed once and only once. We will not consider formations that do not meet this
criterion.
One more thing: two formations are considered “different” if there exists a clump of grass for which either
(1) for exactly one of the formations does a mobot starts on that clump, or (2) there are mobots starting on
this clump for both the formations, but they’re oriented in different directions.
As an example, one allowable formation for m = 2, n = 3 might be as follows:
· → ·

→ ·

1. [25] Prove that the maximum number of mobots you need to mow your lawn is m + n − 1.
2. [40] Prove that the minimum number of mobots you need to mow your lawn is min{m, n}.
3. [15] Prove that, given any formation, each mobot may be colored in one of three colors — say, white,
black, and blue — such that no two adjacent clumps of grass are mowed by different mobots of the
same color. Two clumps of grass are adjacent if the distance between them is 1. In your proof, you
may use the Four-Color Theorem if you’re familiar with it.
4. [15] For n = m = 4, find a formation with 6 mobots for which there are exactly 12 ways to color the
mobots in three colors as in problem 3. (No proof is necessary.)
5. [40] For n, m ≥ 3, prove that a formation has exactly six possible colorings satisfying the conditions
in problem 3 if and only if there is a mobot that starts at (1, 1).

Polygons [130]
6. [15] Suppose we have a regular hexagon and draw all its sides and diagonals. Into how many regions
do the segments divide the hexagon? (No proof is necessary.)
7. [25] Suppose
√ we have an octagon with all angles of 135◦ , and consecutive sides of alternating length
1 and 2. We draw all its sides and diagonals. Into how many regions do the segments divide the
octagon? (No proof is necessary.)
8. [25] A regular 12-sided polygon is inscribed in a circle of radius 1. How many chords of the circle that
join two of the vertices of the 12-gon have lengths whose squares are rational? (No proof is necessary.)
9. [25] Show a way to construct an equiangular hexagon with side lengths 1, 2, 3, 4, 5, and 6 (not
necessarily in that order).
10. [40] Given a convex n-gon, n ≥ 4, at most how many diagonals can be drawn such that each drawn
diagonal intersects every other drawn diagonal strictly in the interior of the n-gon? Prove that your
answer is correct.

1
What do the following problems have in common? [135]
11. [15] Find the largest positive integer n such that 1! + 2! + 3! + · · · + n! is a perfect square. Prove that
your answer is correct.
12. [15] Find all ordered triples (x, y, z) of positive reals such that x + y + z = 27 and x2 + y 2 + z 2 − xy −
yz − zx = 0. Prove that your answer is correct.
13. [25] Four circles with radii 1, 2, 3, and r are externally tangent to one another. Compute r. (No proof
is necessary.)
14. [40] Find the prime factorization of

20062 · 2262 − 6692 · 3599 + 15932 · 1337.

(No proof is necessary.)


15. [40] Let a, b, c, d be real numbers so that c, d are not both 0. Define the function

ax + b
m(x) =
cx + d
on all real numbers x except possibly −d/c, in the event that c 6= 0. Suppose that the equation
x = m(m(x)) has at least one solution that is not a solution of x = m(x). Find all possible values of
a + d. Prove that your answer is correct.

2
HMMT 1998: Algebra Solutions

1. Problem: The cost of 3 hamburgers, 5 milk shakes, and 1 order of fries at a certain fast food restaurant
is $23.50. At the same restaurant, the cost of 5 hamburgers, 9 milk shakes, and 1 order of fries is $39.50.
What is the cost of 2 hamburgers, 2 milk shakes ,and 2 orders of fries at this restaurant?

Solution: Let H = hamburger, M = milk shake, and F = order of fries. Then 3H + 5M + F = $23.50.
Multiplying the equation by 2 yields 6H + 10M + 2F = $47. Also, it is given that 5H + 9M + F = $39.50.
Then subtracting the following equations
6H +10M +2F = $47.00
5H +9M +F = $39.50

yields H + M + F = $7.50. Multiplying the equation by 2 yields 2H + 2M + 2F = $15 .

2. Problem: Bobbo starts swimming at 2 feet/s across a 100 foot wide river with a current of 5 feet/s.
Bobbo doesn’t know that there is a waterfall 175 feet from where he entered the river. He realizes his
predicament midway across the river. What is the minimum speed that Bobbo must increase to make it to
the other side of the river safely?

Solution: When Bobbo is midway across the river, he has travelled 50 feet. Going at a speed of 2
feet/s, this means that Bobbo has already been in the river for 2050feet/sfeet
= 25 s. Then he has traveled
5 feet/s · 25s = 125 feet down the river. Then he has 175 feet-125 feet = 50 feet left to travel downstream
before he hits the waterfall.
Bobbo travels at a rate of 5 feet/s downstream. Thus there are 550feet/s
feet
= 10s before he hits the waterfall.
He still has to travel 50 feet horizontally across the river. Thus he must travel at a speed of 5010s feet
= 5
feet/s. This is a 3 feet/s difference from Bobbo’s original speed of 2 feet/s.

3. Problem: Find the sum of every even positive integer less than 233 not divisible by 10.

Solution: We find the sum of all positive even integers less than 233 and then subtract all the positive
integers less than 233 that are divisible by 10.
2+4+. . .+232 = 2(1+2+. . .+116) = 116·117 = 13572. The sum of all positive integers less than 233 that
are divisible by 10 is 10+20+. . .+230 = 10(1+2+. . .+23) = 2760. Then our answer is 13572-2760 = 10812 .
√ √
r 2( 2+ 10)
4. Problem: Given that r and s are relatively prime positive integers such that s = √ √ , find r
5( 3+ 5)
and s.
√ √
r2 4(12+4 5) 16(3+ 5) 16
Solution: Squaring both sides of the given equation yields s2 = √
25(3+ 5)
= √
25(3+ 5)
= 25 . Because r
and s are positive and relatively prime, then by inspection, r = 4 and s = 5 .

5. Problem: A man named Juan has three rectangular solids, each having volume 128. Two of the faces
of one solid have areas 4 and 32. Two faces of another solid have areas 64 and 16. Finally, two faces of the
last solid have areas 8 and 32. What is the minimum possible exposed surface area of the tallest tower Juan
can construct by stacking his solids one on top of the other, face to face? (Assume that the base of the tower
is not exposed).

Solution: Suppose that x, y, z are the sides of the following solids. Then Volume = xyz = 128. For the
first solid, without loss of generality (with respect to assigning lengths to x, y, z), xy = 4 and yz = 32. Then
xy 2 z = 128. Then y = 1. Solving the remaining equations yields x = 4 and z = 32. Then the first solid has
dimensions 4 × 1 × 32.
For the second solid, without loss of generality, xy = 64 and yz = 16. Then xy 2 z = 1024. Then y = 8.
Solving the remaining equations yields x = 8 and z = 2. Then the second solid has dimensions 8 × 8 × 2.
For the third solid, without loss of generality, xy = 8 and yz = 32. Then y = 2. Solving the remaining
equations yields x = 4 and z = 16. Then the third solid has dimensions 4 × 2 × 16.
To obtain the tallest structure, Juan must stack the boxes such that the longest side of each solid is
oriented vertically. Then for the first solid, the base must be 1 × 4, so that the side of length 32 can

1
contribute to the height of the structure. Similarly, for the second solid, the base must be 8 × 2, so that the
side of length 8 can contribute to the height. Finally, for the third solid, the base must be 4 × 2. Thus the
structure is stacked, from bottom to top: second solid, third solid, first solid. This order is necessary, so
that the base of each solid will fit entirely on the top of the solid directly beneath it.
All the side faces of the solids contribute to the surface area of the final solid. The side faces of the bottom
solid have area 8 · (8 + 2 + 8 + 2) = 160. The side faces of the middle solid have area 16 · (4 + 2 + 4 + 2) = 192.
The sides faces of the top solid have area 32 · (4 + 1 + 4 + 1) = 320.
Furthermore, the top faces of each of the solids are exposed. The top face of the bottom solid is partially
obscured by the middle solid. Thus the total exposed area of the top face of the bottom solid is 8·2−4·2 = 8.
The top face of the middle solid is partially obscured by the top solid. Thus the total exposed area of the
top face of the middle solid is 4 · 2 − 4 · 1 = 4. The top face of the top solid is fully exposed. Thus the total
exposed area of the top face of the top solid is 4 · 1 = 4.
Then the total surface area of the entire structure is 160 + 192 + 320 + 8 + 4 + 4 = 688 .

1 1
6. Problem: How many pairs of positive integers (a, b) with ≤ b satisfy a + b = 16 ?

Solution: a1 + 1b = 61 ⇒ a+b 1
ab = 6 ⇒ ab = 6a+6b ⇒ ab−6a−6b = 0. Factoring yields (a−b)(b−6)−36 = 0.
Then (a − 6)(b − 6) = 36. Because a and b are positive integers, only the factor pairs of 36 are possible values
of a − 6 and b − 6. The possible pairs are:

a − 6 = 1, b − 6 = 36
a − 6 = 2, b − 6 = 18
a − 6 = 3, b − 6 = 12
a − 6 = 4, b − 6 = 9
a − 6 = 6, b − 6 = 6

Because a ≤ b, the symmetric cases, such as a − 6 = 12, b − 6 = 3 are not applicable. Then there are 5
possible pairs.

7. Problem: Given that three roots of f (x) = x4 +ax2 +bx+c are 2, -3, and 5, what is the value of a+b+c?

Solution: By definition, the coefficient of x3 is negative the sum of the roots. In f (x), the coefficient of x3
is 0. Thus the sum of the roots of f (x) is 0. Then the fourth root is -4. Then f (x) = (x−2)(x+3)(x−5)(x+4).
Notice that f (1) is 1 + a + b + c. Thus our answer is f (1) − 1 = (1 − 2)(1 + 3)(1 − 5)(1 + 4) − 1 = 79 .

x+1 3x+4
8. Problem: Find the set of solutions for x in the inequality x+2 > 2x+9 when x 6= −2, x 6= 92 .

Solution: There are 3 possible cases of x: 1) − 29 < x, 2) 92 ≤ x ≤ −2, 3) −2 < x. For the cases (1) and
(3), x + 2 and 2x + 9 are both positive or negative, so the following operation can be carried out without
changing the inequality sign:

x+1 3x + 4
>
x+2 2x + 9
⇒ 2x2 + 11x + 9 > 3x2 + 10x + 8
⇒ 0 > x2 − x − 1

√ √
The inequality holds for all 1−2 5 < x < 1+2 5 . The initial conditions were − 29 < x or −2 < x. The
√ √
intersection of these three conditions occurs when 1−2 5 < x < 1+2 5 .
Case (2) is 92 ≤ x ≤ −2. For all x satisfying these conditions, x + 2 < 0 and 2x + 9 > 0. Then the
following operations will change the direction of the inequality:

2
x+1 3x + 4
>
x+2 2x + 9
⇒ 2x2 + 11x + 9 < 3x2 + 10x + 8
⇒ 0 < x2 − x − 1

√ √
The inequality holds for all x < 1−2 5 and 1+2 5 < x. The initial condition was −92 ≤ x ≤ −2. Hence
the intersection of these conditions yields all x such that −9
2 ≤ x ≤ −2. Then all possible cases of x are
√ √
−9 1− 5 1+ 5
2 ≤ x ≤ −2 ∪ 2 <x< 2 .
2f (x)
9. Problem: Suppose f (x) is a rational function such that 3f ( x1 ) + x = x2 for x 6= 0. Find f (−2).

−1
Solution: Let x = 2 . Then

2f ( −1
1 2 )
3f (−2) + −1 =
2
4
−1 1
⇒ 3f (−2) − 4f ( )= (1)
2 4

Let x = −2. Then

−1 2f (−2)
3f ( )+ =4
2 −2
−1
⇒ 3f ( ) − f (−2) = 4 (2)
2

67
Solving this system of equations {(1), (2)} for f (−2) yields f (−2) = 20 .

10. Problem: G.H. Hardy once went to visit Srinivasa Ramanujan in the hospital, and he started the
conversation with: “I came here in taxi-cab number 1729. That number seems dull to me, which I hope
isn’t a bad omen.” “Nonsense,” said Ramanujan. “The number isn’t dull at all. It’s quite interesting. It’s
the smallest number that can be expressed as the sum of two cubes in two different ways.” Ramanujan had
immediately seen that 1729 = 123 + 13 = 103 + 93 . What is the smallest positive integer representable as
the sum of the cubes of three positive integers in two different ways?

Solution: Let this smallest positive integer be represented as a3 + b3 + c3 = d3 + e3 + f 3 . By inspection,


a solution is not possible with the first 4 cubes. We prove that it is impossible to write the same number
as two different sums of the first 5 cubes. Because we necessarily need to use the 5th cube (otherwise, this
proof would be for the first 4 cubes), we have 53 + b3 + c3 = d3 + e3 + f 3 . Without loss of generality, suppose
d = 5. By inspection, there is no solution to b3 + c3 = e3 + f 3 , such that b, c, e, f ≤ 5 and b, c and e, f are
unique.
Then none of d, e, f are 5. Then at least two must be 4, otherwise the RHS would be too small. Without
loss of generality, suppose d = e = 4. Then b3 + c3 = 3 + f 3 . By inspection, there are no possible solutions
if b, c, f ≤ 4.
Thus if a = 5, there are no solutions.
Suppose that there is a solution within the first 6 cubes. Then a = 6. By the same analysis as above,
d = e = 5, otherwise the RHS would be too small. Then b3 + c3 = 34 + f 3 . By inspection, we see that a
possible solution is b = 3, c = 2, f = 1. Then the desired integer is 63 + 33 + 23 = 251 .

3
HMMT 1998: Calculus Solutions

1. Problem: Farmer Tim is lost in the densely-forested Cartesian plane. Starting from the origin he walks
a sinusoidal path in search of home; that is, after t minutes he is at position (t, sin t).
Five minutes after he sets out, Alex enters the forest at the origin and sets out in search of Tim. He
walks in such a way that after he has been in the forest for m minutes, his position is (m, cos t).
What is the greatest distance between Alex and Farmer Tim while they are walking in these paths?

Solution: At arbitrary time t, Farmer Tim is at position (t, psin t) and Alex is at position (t − 5, cos t).
Hence at time t, the distance, d, between Tim and Alex is d = (sin t − cos t)2 + 25. To find the maximum
value of d, we solve for t such that dd dt = 0.
dd (sin t−cos t)(cos t+sin t) 2 2
dt =
√ 2
. Then dd 2 2
dt = 0 ⇒ sin t − cos t = 0 ⇒ sin t = cos t. Equality happens if t is
(sin t−cos t) +25
any constant multiple of π4 .
Notice that to maximize d, we need to maximize (sin t − cos t)2 . This is achieved when cos t = − sin t.
Because we determined earlier that t is a constant multiple of π4 , then with this new condition, we see that
t must be a constant multiple of 3π
4 .
2

Then (sin t − cos t) = 2 ⇒ d = 29 .

2. Problem: A cube with sides 1m in length is filled with water, and has a tiny hole through which the
water drains into a cylinder of radius 1m. If the water level in the cube is falling at a rate of 1 cm/s, at what
rate is the water level in the cylinder rising?

Solution: The magnitude of the change in volume per unit time of the two solids is the same. The change
in volume per unit time of the cube is 1 cm·m2 /s. The change in volume per unit time of the cylinder is
π · dh 2 dh
dt · m , where dt is the rate at which the water level in the cylinder is rising.
dh 1
Solving the equation π · dt · m2 = 1 cm · m2 /s yields π cm/s .

3. Problem: Find the area of the region bounded by the graphs of y = x2 , y = x, and x = 2.

Solution: There are two regions to consider. First, there is the region bounded by y = x2 and y = x, in
the interval [0, 1]. In this interval, the values of y = x are greater than the values of y = x2 , thus the area is
Z 1
x − x2 dx.

calculated by
0
Second, there is the region bounded by y = x2 and y = x and x = 2, in the interval [1, 2]. In this interval,
Z 2
x2 − x dx.

the values of y = x2 are greater than the values of y = x, thus the area is calculated by
Z 1 Z 2 1
2 2
 
Then the total area of the region bounded by the three graphs is x − x dx + x − x dx = 1 .
0 1
s Z
1

2 3
f (x)dx
x x x
4. Problem: Let f (x) = 1 + 2 + 4 + 8 + . . ., for −1 ≤ x ≤ 1. Find e 0 .

1 2
Solution: Observe that f (x) is merely an infinite geometric series. Thus f (x) = 1− x = 2−x . Then
2
Z 1
2 √ √
= 2 ln 2. Then e2 ln 2 = 22 = 2 .
0 2 − x
x
5. Problem: Evaluate lim x sin(1−x) .
x→1
x x
sin(1−x) sin(1−x)
Solution: Rewrite the expression to evaluate as eln x . Then we must evaluate lim eln x .
  x→1
x x
lim ln x sin(1−x) = lim ln x . Because direct calculation of the limit results in indeterminate
x→1 x→1 sin(1 − x)
 
x
form ( 10 · 0), we can use L’Hopital’s rule to evaluate the limit. By L’Hopital’s rule, lim ln x =
x→1 sin(1 − x)
ln x + 1
lim . This limit is simply -1.
x→1 − cos(1 − x)

1
x
sin(1−x)
Hence lim eln x = e−1 = 1
e .
x→1

6. Problem: Edward, the author of this test, had to escape from prison to work in the grading room
today. He stopped to rest at a place 1,875 feet from the prison and was spotted by a guard with a crossbow.
The guard fired an arrow with an initial velocity of 100 ft/s. At the same time, Edward started running
away with an acceleration of 1 ft/s2 . Assuming that air resistance causes the arrow to decelerate at 1 ft/s2
and that it does hit Edward, how fast was the arrow moving at the moment of impact (in ft/s)?

Solution: We use the formula for distance, d = 21 at2 + vt + d0 . Then after t seconds, Edward is at
location 1875 + 12 (1)(t2 ) from the prison. After t seconds, the arrow is at location 21 (−1)(t2 ) + 100t from the
prison. When the arrow hits Edward, both objects are at the same distance away from the tower. Hence
1875 + 21 (1)(t2 ) = 12 (−1)(t2 ) + 100t. Solving for t yields t2 − 100t + 1875 = 0 ⇒ t = 25 or t = 75. Then it
must be t = 25, because after the arrow hits Edward, he will stop running.
After 25 seconds, the arrow is moving at a velocity of 100-25(1) = 75 ft/s .

7. Problem: A parabola is inscribed in equilateral triangle ABC of side length 1 in the sense that AC
and BC are tangent to the parabola at A and B, respectively. Find the area between AB and the parabola.

Solution: Suppose A = (0, 0), B = (1, 0), and C = ( 12 , 23 ). Then the parabola in question goes through
√ √
(0, 0) and (1, 0) and has tangents with slopes of 3 and − 3, respectively, at these points. Suppose the
dy
parabola has equation y = ax2 + bx + c. Then dx = 2ax + b.
dy
√ √
At point (0, 0), dx = b. Also the slope at (0, 0), as we determined earlier, is 3. Hence b = 3. Similarly,
dy
√ √
at point (1, 0), dx = 2a + b. The slope at (1, 0), as we determined earlier, is − 3. Then a = − 3.
√ √
Since the parabola goes through (0, 0), c = 0. Hence the equation of the parabola is y = − 3x2 + 3x.
The desired area is simply the area under the parabolic curve in the interval [0, 1].
Z 1 √ √  √
Hence − 3x2 + 3x dx = 63 .
0

8. Problem: Find the slopes of all lines passing through the origin and tangent to the curve
y 2 = x3 + 39x − 35.

Solution: Any line passing throug the origin has equation y = mx, where m is the slope of the line. If a
dy
line is tangent to the given curve, then at the point of tangency, (x, y), dx = m.
dy dy 2
2 3x +39
First, we calculate dx of the curve: 2ydy = 3x dx + 39dx ⇒ dx = 2y . Substituting mx for y, we get
the following system of equations:

m2 x2 = x3 + 39x − 35
3x2 + 39
m=
2mx

Solving for x yields the equation x3 − 39x + 70 = 0 ⇒ (x − 2)(x + 7)(x − 5) = 0 ⇒ x = 2 or x = −7 or


x = 5. These solutions indicate the x-coordinate of the points

at which the desired lines are tangent to the
51
curve. Solving for the slopes of these lines, we get m = ± 2 for x = 2, no real solutions for x = −7, and
√ √ √
285 51 285
m=± 5 for x = 5. Thus m = ± 2 ,± 5 .

X 1
9. Problem: Evaluate n−1
.
n=1
n · 2
2
x
Solution: Note that if we take the integral of f (x) in problem 4, we get the function F (x) = x + 2·2 +
3
x 1 1
3·22 + . . .. ZEvaluating this integral in the interval [0, 1], we get 1 + 2·2 + 3·22 + . . ., which is the desired sum.
1
2
Hence dx = 2 ln 2.
0 2 − x

2
x y
10. Problem: Let S be the locus of all points (x, y) in the first quadrant such that t + 1−t = 1 for some
t with 0 < t < 1. Find the area of S.

Solution: Solving for t in √ the given equation, we get t2 + (y − x − 1)t + x = 0. Using the quadratic
(x+1−y)± (y−x−1)2 −4x
equation, we get t = 2 . For all valid combinations of (x, y), t is positive and less than 1
(this is easy to see by inspection). All valid combinations of (x, y) are those that make (y − x − 1)2 − 4x ≥ √ 0.
Solving for y in the equation (y −x−1)2 −4x = 0 yields y 2 −(2x+2)y +(x−1)2 ≥ 0 ⇒ y = (x+1)±2 x.
y
In the original equation, it is given that xt + 1−t = 1, and 0 < t < 1. This implies that x, y < 1. Then the
2

only possible y < 1 that satisfies (y − x − 1) − 4x = 0 is y = x + 1 − 2 x. √
Then to satisfy the inequality (y − x − 1)2 − 4x ≥ 0, we must have y ≤ x + 1 − 2 x. Recall that this is

when 0 < y < 1. Hence we integrate in the interval [0, 1]: ∈10 x + 1 − 2 x = 16 .

3
Geometry Solutions
Harvard-MIT Math Tournament
Written by Anders Kaseorg
1. m6 EDX = 180◦ − m6 LAX − m6 ELA = 180◦ − m6 LAX − (180◦ − m6 AXE) = 80◦ .
2. If the angle in radians is θ, then Anne travels Rθ and Lisa travels (R − r) + rθ + (R − r).
Setting these equal yields R(θ − 2) = r(θ − 2), so θ = 2 radians .
_
3. m6 M LD = 12 AB = 45◦ .
1
4. The cylinder has a cross-sectional area π times greater than the cube, so the water raises π
1 cm
times as quickly in the cylinder as it lowers in the cube; that is, at π s .

5. If the new circle has radius r, then the distance from its center to E can be computed either
√ √ √
as 1 + r or (2 − r) 2. Setting these equal yields r = 2√2+1
2−1
= 5−3 2 .
1
2 π
6. The central circle has area π √
2 3
= 12 ,
and each of the three small triangles are copies of
1 π
2
the entire figure dilated by 3. Therefore, the total area is given by K = 12 + 3 · 13 K ⇐⇒
2 π π
3 K = 12 ⇐⇒ K = 8 .

7. Let E = (a, b, 0), A = (−c, b, 0), R = (−c, −d, 0), L = (a, −d, 0), Y = (0, 0, h), and observe
that EY 2 + RY 2 = a2 + b2 + c2 + d2 + 2h2 = AY 2 + LY 2 , which can only be satisfied by
EY = 1, AY = 4, RY = 8, LY = 7 (or the symmetric configurations). Since EA is an
integral side of a triangle whose other sides are 1 and 4, we must have EA = 4; similarly,
EL = 7. Therefore, the area of rectangle EARL is 28 . (Such a pyramid may be constructed

3071
by taking a = 18 , b = 14
1
, c = 31 97
8 , d = 14 , h = 56 .)

8. Since OD ⊥ AC and 4AOC is equilateral, we have 6 AOD = 30◦ . So AE = √1 ,


3
and
√ q
1 2
 q
40

2 2 2
BH = AB + AH = 2 + 3 − 3 = 3 − 2 3 ≈ 3.141533339.

9. The dilation of ratio − 32 about T sends C2 to C1 , O2 to O1 , and S to the other intersection


3 3 2
of s with C1 , which we shall call U . We can
 2  now compute
2
TR· T S = 2 T R · T U = 2 T P =
3 3 O1 O2
2 2
2 (O1 T − O1 P ) = 2 1+3/2 − O1 P 2 = 32 20
5/2 − 42 = 72 .

10. Suppose that when the ball hits a side of the table, instead of reflecting the ball’s path, we
reflect the entire table over this side so that the path remains straight. If we repeatedly
reflect the table over its sides in all possible ways, we get a triangular grid that tiles the
plane. Whenever the path crosses n lines in this grid parallel to CT , it will cross 78 n lines
parallel to CH and 15 8 n lines parallel to HT . After crosssing 8 + 7 + 15 = 30 grid lines it will
have crossed three lines simultaneously again, which means that the ball will have landed in
a pocket after bouncing 27 times. By picturing the grid it is easy to see that the pocket
in question is H . The distance the ball travels during the 18 of its trip described in the
problem
√ is the third side of a triangle with an 120◦ angle between two sides 16 and 14, which
is 16 + 142 − 2 · 16 · 14 cos 120◦ = 26, so length of the entire trip is 208 .
2

1
Advanced Topics
1. Evaluate
sin(1998◦ + 237◦ ) sin(1998◦ − 1653◦ ).
1
Answer: − . We have sin(1998◦ +237◦ ) sin(1998◦ −1653◦ ) = sin(2235◦ ) sin(345◦ ) = sin(75◦ ) sin(−15◦ ) =
4

− sin(75 ) sin(15◦ ) = − sin(15◦ ) cos(15◦ ) = − sin(30

2
)
= − 41 .
2. How many values of x, −19 < x < 98, satisfy
cos2 x + 2 sin2 x = 1?
Answer: 38. For any x, sin2 x+cos2 x = 1. Subtracting this from the given equation gives sin2 x = 0,
or sin x = 0. Thus x must be a multiple of π, so −19 < kπ < 98 for some integer k, or approximately
−6.1 < k < 31.2. There are 32 values of k that satisfy this, so there are 38 values of x that satisfy
cos2 x + 2 sin2 x = 1.
3. Find the sum of the infinite series
1 1 2 1 3
1 + 2( ) + 3( ) + 4( ) + ....
1998 1998 1998
1998 2 3992004

Answer: 1997 or 3988009 . We can rewrite the sum as
 2 !  2  3 !  2  3 !
1 1 1 1 1 1 1
1+ + + ... + + + + ... + + + . . . +. . . .
1998 1998 1998 1998 1998 1998 1998
Evaluating each of the infinite sums gives
2 !
1
( 1 )2
  
1 1998 1998 1 1 1998 1 1 2
1 + + 1998 1 +. . . =
1 · 1+ + + ... = · 1+ +( ) + ... ,
1 − 1998 1 − 1998
1 − 1998 1997 1998 1998 1997 1998 1998
1998 2
, or 3992004

which is equal to 1997 3988009 , as desired.

4. Find the range of


(sin A) 3 cos2 A + cos4 A + 3 sin2 A + (sin2 A)(cos2 A)

f (A) =
(tan A) (sec A − (sin A)(tan A))

if A 6= 2 .

Answer: (3, 4). We factor the numerator and write the denominator in term of fractions to get

(sin A)(3 + cos2 A)(sin2 A + cos2 A) (sin A)(3 + cos2 A)(sin2 A + cos2 A)
  = (sin A)(1−sin2 A)
.
sin A 1 sin2 A

cos A cos A − cos A cos2 A

Because sin2 A + cos2 A = 1, 1 − sin2 A = cos2 A, so the expression is simply equal to 3 + cos2 A. The
range of cos2 A is (0, 1) (0 and 1 are not included because A 6= nπ 2
2 , so the range of 3 + cos A is (3, 4).

5. How many positive integers less than 1998 are relatively prime to 1547? (Two integers are relatively
prime if they have no common factors besides 1.)
Answer: 1487. The factorization of 1547 is 7 · 13 · 17, so we wish to find the number of positive
integers less than 1998 that are not divisible by 7, 13, or 17. By the Principle of Inclusion-Exclusion, we
first subtract the numbers that are divisible by one of 7, 13, and 17, add back those that are divisible
by two of 7, 13, and 17, then subtract those divisible by three of them. That is,
             
1997 1997 1997 1997 1997 1997 1997
1997 − − − + + + − ,
7 13 17 7 · 13 7 · 17 13 · 17 7 · 13 · 17
or 1487.

1
6. In the diagram below, how many distinct paths are there from January 1 to December 31, mov-
ing from one adjacent dot to the next either to the right, down, or diagonally down to the right?

Answer: 372. For each dot in the diagram, we can count the number of paths from January 1
to it by adding the number of ways to get to the dots to the left of it, above it, and above and to
the left of it, starting from the topmost leftmost dot. This yields the following numbers of paths:

So the number of paths from January 1 to December 31 is 372.

7. The Houson Association of Mathematics Educators decides to hold a grand forum on mathematics
education and invites a number of politicians from the United States to participate. Around lunch
time the politicians decide to play a game. In this game, players can score 19 points for pegging
the coordinator of the gathering with a spit ball, 9 points for downing an entire cup of the forum’s
interpretation of coffee, or 8 points for quoting more than three consecutive words from the speech
Senator Bobbo delivered before lunch. What is the product of the two greatest scores that a player
cannot score in this game?
Answer: 1209. Attainable scores are positive integers that can be written in the form 8a + 9b + 19c,
where a, b, and c are nonnegative integers. Consider attainable number of points modulo 8.
Scores that are 0 (mod 8) can be obtained with 8a for positive a.
Scores that are 1 (mod 8) greater than or equal to 9 can be obtained with 9 + 8a for nonnegative a.
Scores that are 2 (mod 8) greater than or equal to 18 can be obtained with 9 · 2 + 8a.
Scores that are 3 (mod 8) greater than or equal to 19 can be obtained with 19 + 8a.
Scores that are 4 (mod 8) greater than or equal to 19 + 9 = 28 can be obtained with 19 + 9 + 8a.
Scores that are 5 (mod 8) greater than or equal to 19 + 9 · 2 = 37 can be obtained with 19 + 9 · 2 + 8a.
Scores that are 6 (mod 8) greater than or equal to 19 · 2 = 38 can be obtained with 19 · 2 + 8a.
Scores that are 7 (mod 8) greater than or equal to 19 · 2 + 9 = 47 can be obtained with 19 · 2 + 9 + 8a.
So the largest two unachievable values are 39 and 31. Multiplying them gives 1209.

2
8. Given any two positive real numbers x and y, then x  y is a positive real number defined in terms of
x and y by some fixed rule. Suppose the operation x  y satisfies the equations (x · y)  y = x(y  y)
and (x  1)  x = x  1 for all x, y > 0. Given that 1  1 = 1, find 19  98.
Answer: 19. Note first that x1 = (x·1)1 = x·(11) = x·1 = x. Also, xx = (x1)x = x1 = x.
Now, we have (x · y)  y = x · (y  y) = x · y. So 19  98 = ( 19 19 19
98 · 98)  98 = 98 · (98  98) = 98 · 98 = 19.

9. Bob’s Rice ID number has six digits, each a number from 1 to 9, and any digit can be used any number
of times. The ID number satisfies the following property: the first two digits is a number divisible by
2, the first three digits is a number divisible by 3, etc. so that the ID number itself is divisible by 6.
One ID number that satisfies this condition is 123252. How many different possibilities are there for
Bob’s ID number?
Answer: 324. We will count the number of possibilities for each digit in Bob’s ID number, then
multiply them to find the total number of possibilities for Bob’s ID number. There are 3 possibilities
for the first digit given any last 5 digits, because the entire number must be divisible by 3, so the
sum of the digits must be divisible by 3. Because the first two digits are a number divisible by 2, the
second digit must be 2, 4, 6, or 8, which is 4 possibilities. Because the first five digits are a number
divisible by 5, the fifth digit must be a 5. Now, if the fourth digit is a 2, then the last digit has two
choices, 2, 8, and the third digit has 5 choices, 1, 3, 5, 7, 9. If the fourth digit is a 4, then the last
digit must be a 6, and the third digit has 4 choices, 2, 4, 6, 8. If the fourth digit is a 6, then the last
digit must be a 4, and the third digit has 5 choices, 1, 3, 5, 7, 9. If the fourth digit is an 8, then the
last digit has two choices, 2, 8, and the third digit has 4 choices, 2, 4, 6, 8. So there are a total of
3 · 4 (2 · 5 + 4 + 5 + 2 · 4) = 3 · 4 · 27 = 324 possibilities for Bob’s ID number.
10. In the fourth annual Swirled Series, the Oakland Alphas are playing the San Francisco Gammas. The
first game is played in San Francisco and succeeding games alternate in location. San Francisco has
a 50% chance of winning their home games, while Oakland has a probability of 60% of winning at
home. Normally, the serios will stretch on forever until one team gets a three-game lead, in which case
they are declared the winners. However, after each game in San Francisco there is a 50% chance of an
earthquake, which will cause the series to end with the team that has won more games declared the
winner. What is the probability that the Gammas will win?
34
Answer: 73 . Let F (x) be the probability that the Gammas will win the series if they are ahead by
x games and are about to play in San Francisco, and let A(x) be the probability that the Gammas will
win the series if they are ahead by x games and are about to play in Oakland. Then we have

3 A(1)
F (2) = +
4 4
6F (0) 4F (2)
A(1) = +
10 10
1 A(1) A(−1)
F (0) = + +
4 4 4
6F (−2) 4F (0)
A(−1) = +
10 10
A(−1)
F (−2) =
4
6F (0) 4F (2) 3 A(1)
Plugging A(1) = 10 + 10 into F (2) = 4 + 4 , we get
 
3 1 6F (0) 4F (2)
F (2) = + +
4 4 10 10

9F (2) 3 6F (0) 5 F (0)


= + ⇔ F (2) = +
10 4 40 6 6

3
6F (−2) 4F (0) A(−1)
Plugging A(−1) = 10 + 10 into F (−2) = 4 , we get

34A(−1) 4F (0) 2F (0)


= ⇔ F (−2) =
40 10 17
Now,    
1 1 6F (0) 4F (2) 1 6F (−2) 4F (0)
F (0) = + + + +
4 4 10 10 4 10 10
1 F (0) F (2) 6F (−2)
This simplifies to F (0) = 4 + 4 + 10 + .
Then, plugging our formulas in, we get
40
 
1 F (0) 1 5 F (0) 3F (0)
F (0) = + + + + .
4 4 10 6 6 170

73F (0) 1 34
= ⇔ F (0) = .
102 3 73
34
Since F (0) is the situation before the Series has started, the probability that the Gammas will win is 73 .

4
1998 Power Question Solutions

I. Graphs, total of 20 points


a. completely correct gets 1 point, total of 6 points
i. yes. vertices A,B,C,D, edges AB,AC,AD,BD
ii. no. A and B are connected twice
iii. yes. vertices A,B,C, edges ABC
iv. yes. vertices A,B,C,D,E, edges AB,AD,AE,BE,CD,CE
v. no. the edge from B does not connect to another vertex
vi. no. E is connected to itself

b. completely correct gets 1 point, total of 2 points


i. BL,BE,BU,EL,EU
ii. A↔B, B↔L, C↔U, D↔E

c. 1 point each for i-iii, 3 for iv-vi, total of 12 points


i. no
ii. no
iii. no
iv. yes
v. no
vi. yes
note that the second graph in v has more vertices of degree 3 than the first

II. Planar Graphs, total of 60 points


a. 1 point each for v, e, f, total of 6 points
i. v=9, e=16, f=9
ii. v=7, e=14, f=9
note that v-e+f=2 by Euler’s formula

b. 3 points each, total of 12 points note that this is a planar drawing of ii:
i. yes
ii. yes
iii. no
iv. yes
for iii, note that if we remove the two vertices of degree two and make the vertices they
were adjacent to adjacent to each other then we get a graph isomorphic to the one in d.ii.

c. total of 15 points
partial credit: up to 5 for effort, 5 for insight, otherwise as described below
[1 point] If G has 3 vertices and two edges then the assertion is easy since f=1.
[6 points] Otherwise each face is bounded by 3 or more edges, hence counting the number
of edges bounding each faces and summing, then using the fact that we at most double
counted [-2 for asserting we exactly double counted] each face we get 3f ≤ 2e, thus
proving the left inequality.
[8 points] Now f ≤ (2/3)e and we want to eliminate f, so we can add v-e to both sides to
get v-e+f ≤ v-e+(2/3)e, thus by Euler’s formula 2 ≤ v-e/3, and rearranging yields the right
inequality.

d. 5 points for i, 10 for ii, total of 15 points


partial credit: if Jordan curve thm is used then 3 points for i, 5 for ii.
on part ii: 3 for effort, 3 for insight, otherwise as described
i. Assume it is planar. v=5, e=10, 3v-6=9<e, contradicting c, thus nonplanar
ii. [1 point] Note that v=6, e=9, so 3v-6=12>e is not a contradiction
[2 points] There are no triangles in this graph, thus every face is bordered by at least 4
edges. This will allow us to prove a better inequality as in c with the assumption the
graph is planar, and that will provide the contradiction.
[2 points] Thus as in the proof of c we get 4f ≤ 2e.
[4 points] Hence f ≤ e/2, and again we want to eliminate f so adding v-e to both sides and
applying Euler’s formula yields 2 ≤ v-e/2. (or e ≤ 2v-4)
[1 point] 6-9/2<2, contradicting the inequality just proved, thus the graph is not planar.

e. total of 12 points
partial credit: 4 for effort, 4 for insight, otherwise as described
[2 points] Without loss of generality we may assume the graph is connected.
[2 points] If the graph has fewer than 3 vertices then the assertion is obvious.
[5 points] Otherwise we can apply the inequality in c. First assume every vertex has
degree at least 6, then adding the degrees of each vertex double counts the edges so 6v ≤
2e.
[3 points] Now by c. e ≤ 3v-6, and combining these inequalities we get 6v ≤ 6v-12, a
contradiction, thus the graph must have at least one vertex of degree 5 or less.

III. Coloring, total of 56 points


a. 3 points each, total of 12 points
i. 4
ii. 5
iii. 4
iv. 3

b. total of 14 points
partial credit: 5 for effort, 5 for insight, otherwise as described
[8 points] Number the colors used in G 1, 2, ..., χ. Let v1 be the number of vertices
colored with color 1. Then since no pair of them are adjacent, the are all adjacent in the
new graph and thus χ ≥ v1. Repeating this procedure for v2, ..., vχ and adding we get χχ
≥ v.
[6 points] By the AM-GM inequality χ+ χ ≥ 2 χχ ≥ 2 v .

c. 5 points for i and iii, 20 for ii, total of 30 points


partial credit: as described below
i. By II.e. G must have a vertex of degree 5 or less
ii. [2 points] We want to remove the vertex V without decreasing the number of
colors needed. In the cases where V has degree < 5 this is obviously possible.
[10 points] In the case where V has degree 5, color the adjacent vertices with colors 1, 2,
3, 4, 5 in clockwise order. If the vertices colored 1 and 3 are not connected by a walk with
all vertices colored 1 or 3 then change the vertex colored 3 to 1, change any 1 vertices
adjacent to it to 3, then change any 3 vertices adjacent to those to 1, and so on. This
recoloring does not affect any of the other vertices adjacent to V and thus allows us to
make V color 3, i.e. if a sixth color is needed for G then it is still needed after we remove
V.
[8 points] If the vertices colored 1 and 3 are connected by a walk of vertices colored 1 or
3 then look at the vertices colored 2 and 4. If they are connected by a walk then by the
Jordan Curve Theorem [-4 for not using this] this walk must cross the closed curve
formed by the 1-3 walk and the edges from V to the vertices colored 1 and 3. The graph is
planar, so we can assume it is drawn without edge crossings, thus the walk from the
vertex colored 2 to the one colored 4 must cross this curve at a vertex, hence they are not
connected by a walk with all vertices colored 2 or 4, and by the previous argument we can
change the vertex colored 2 to 4 and make V color 2, so it can be removed without
removing the need for a sixth color.
iii. [4 points] We have shown that for any graph with χ>5 we can find one with fewer
vertices, and since the proof did not depend on the number of vertices we can keep doing
this until we get a graph with 5 vertices, which can clearly be colored with only 5 colors,
thus G could not require more than 5 colors.
[1 points] To make this rigorous some mention should be made of the well-ordering
principal or infinite descent.

Historical notes and inspiration for further study of the subject:


The Four Color theorem was first conjectured in 1852. Many “proofs” were given,
including one in 1879 by A. B. Kempe that was believed to be correct until P. J. Heawood
found a flaw in 1890. The proof of the Four Color theorem, given in 1976 by Haken and
Appel, uses a similar technique to what we just used, but instead of using 6 possible
subgraphs it uses 1482. The way these theorems relate to coloring maps is that we can
consider the dual graph, formed by putting a vertex in every face and connecting the
vertices whose faces share an edge. Coloring the dual graph is the same as coloring the
faces of the original graph in such a way that no two faces sharing an edge have the same
color, which is what one does when coloring a map.
While it may seem that we used many definitions in this power question, there are many
more that had to be omitted. Some of the other interesting definitions used in graph
theory are expansion, which is what you get if you add vertices along an edge of a graph,
and supergraph, which is what you get if you add new vertices anywhere and connect
them arbitrarily to the other vertices. Kuratowski’s Theorem states that every nonplanar
graph is a supergraph of an expansion of one of the two graphs in II.d. A corollary of this
is that a graph is nonplanar iff it is a supergraph of an expansion of one of those two
graphs, thus we can classify planar and nonplanar graphs in terms of just two particular
graphs.
A planar graph can alo be described as one that can be drawn on a sphere with no edge
crossings. A torus is essentially the surface of a solid sphere with a hole drilled all the
way through it. The genus g of a graph is the minimum number of holes that must be
drilled in a sphere in order to be able to draw it without edge crossings. In other words a
graph of genus g can be drawn on a g holed torus without edge crossings, but not on a g-1
holed torus. The Heawood Coloring Theorem states that if G has genus g > 0 then G can
7 + 1 + 48g 
be colored with colors. Note that plugging in g=0 yields 4, but no proof
 2 
of this theorem is known that does not depend on the condition g > 0 (of course we can
combine the proof for g>0 with the four color theorem to conclude that the result holds
for all g, but it would be nice to have just one proof that works equally well for all g).
Another thing we can study about graphs is what kind of walks exist in them. An
eulerian walk, named after the great Swiss mathematician Leonhard Euler (1707-1783,
pronounced Oiler), is one that uses every edge exactly once. A hamiltonian walk, named
after the Irish mathematician Sir William Rowan Hamilton (1805-1865), is one that uses
every vertex exactly once. Try to classify which graphs have such walks, with or without
the condition that the starting and ending points must be the same (in the case of a
hamiltonian walk the end vertex is thus counted twice and it is called a closed
hamiltonian walk to distinguish from the open walk).
This power question was barely an introduction to graph theory. It is a very broad field of
mathematics, closely related to topology and knot theory. I hope you enjoyed this test,
learned something from it, and that you will continue your studies of mathematics for
many years.

Edward Early
2/14/98
Algebra Solutions
Harvard-MIT Math Tournament
February 27, 1999

Problem A1 [3 points]

a3 −b3
If a@b = a−b , for how many real values of a does a@1 = 0?

3
Solution: If aa−1
−1
= 0, then a3 − 1 = 0, or (a − 1)(a2 + a + 1) = 0. Thus a = 1, which is an
extraneous solution since that makes the denominator of the original expression 0,√ or a is a root
of a2 + a + 1. But this quadratic has no real roots, in particular its roots are −1±2 −3 . Therefore
there are no such real values of a, so the answer is 0.

Problem A2 [3 points]

For what single digit n does 91 divide the 9-digit number 12345n789?

Solution 1: 123450789 leaves a remainder of 7 when divided by 91, and 1000 leaves a remainder of
90, or -1, so adding 7 multiples of 1000 will give us a multiple of 91.

Solution 2: For those who don’t like long division, there is a quicker way. First notice that 91 = 7·13,
and 7·11·13 = 1001. Observe that 12345n789 = 123·1001000+45n·1001−123·1001+123−45n+789
It follows that 91 will divide 12345n789 iff 91 divides 123 − 45n + 789 = 462 − n. The number 462
is divisible by 7 and leaves a remainder of 7 when divided by 13.

Problem A3 [4 points]

Alex is stuck on a platform floating over an abyss at 1 ft/s. An evil physicist has arranged for the
platform to fall in (taking Alex with it) after traveling 100ft. One minute after the platform was
launched, Edward arrives with a second platform capable of floating all the way across the abyss.
He calculates for 5 seconds, then launches the second platform in such a way as to maximize the
time that one end of Alex’s platform is between the two ends of the new platform, thus giving Alex
as much time as possible to switch. If both platforms are 5 ft long and move with constant velocity
once launched, what is the speed of the second platform (in ft/s)?

Solution: The slower the second platform is moving, the longer it will stay next to the first platform.
However, it needs to be moving fast enough to reach the first platform before it’s too late. Let v
be the velocity of the second platform. It starts 65 feet behind the first platform, so it reaches the
60 70
back of the first platform at v−1 seconds, and passes the front at v−1 seconds, so the time to switch
10
is v−1 . Hence we want v to be as small as possible while still allowing the switch before the first
platform falls. Therefore the time to switch will be maximized if the back of the second platform
lines up with the front of the first platform at the instant that the first platform has travelled 100ft,
which occurs after 100 seconds. Since the second platform is launched 65 seconds later and has to
travel 105 feet, its speed is 105/35 = 3ft/s.

1
Problem A4 [4 points]

d
Find all possible values of a where a2 − 6ad + 8d2 = 0, a 6= 0.

Solution: Dividing a2 − 6ad + 8d2 = 0 by a2 , we get 1 − 6 ad + 8( ad )2 = 0. The roots of this quadratic


are 21 , 14 .

Problem A5 [5 points]

You are trapped in a room with only one exit, a long hallway with a series of doors and land mines.
To get out you must open all the doors and disarm all the mines. In the room is a panel with
3 buttons, which conveniently contains an instruction manual. The red button arms a mine, the
yellow button disarms two mines and closes a door, and the green button opens two doors. Initially
3 doors are closed and 3 mines are armed. The manual warns that attempting to disarm two mines
or open two doors when only one is armed/closed will reset the system to its initial state. What is
the minimum number of buttons you must push to get out?

Solution: Clearly we do not want to reset the system at any time. After pressing the red button
r times, the yellow button y times, and the green button g times, there will be 3 + r − 2y armed
mines and 3 + y − 2g closed doors, so we want the values of r, y, and g that make both of these
quantities 0 while minimizing r + y + g. From the number of doors we see that y must be odd,
from the number of mines we see y = (3 + r)/2 ≥ 3/2, so y ≥ 3. Then g = (3 + y)/2 ≥ 3, and
r = 2y − 3 ≥ 3, so r + y + g ≥ 9. Call the red, yellow, and green buttons 1, 2, and 3 respectively for
notational convenience, then a sequence of buttons that would get you out is 123123123. Another
possibility is 111222333, and of course there are others. Therefore the answer is 9.

Problem A6 [5 points]

Carl and Bob can demolish a building in 6 days, Anne and Bob can do it in 3, Anne and Carl in
5. How many days does it take all of them working together if Carl gets injured at the end of the
first day and can’t come back? Express your answer as a fraction in lowest terms.

Solution: Let a be the portion of the work that Anne does in one day, similarly b for Bob and c for
Carl. Then what we are given is the system of equations b + c = 1/6, a + b = 1/3, and a + c = 1/5.
Thus in the first day they complete a + b + c = 12 (1/6 + 1/3 + 1/5) = 7/20, leaving 13/20 for Anne
and Bob to complete. This takes 13/20 59
1/3 = 39/20 days, for a total of 20 .

Problem A7 [5 points]

Matt has somewhere between 1000 and 2000 pieces of paper he’s trying to divide into piles of the
same size (but not all in one pile or piles of one sheet each). He tries 2, 3, 4, 5, 6, 7, and 8 piles
but ends up with one sheet left over each time. How many piles does he need?

Solution: The number of sheets will leave a remainder of 1 when divided by the least common
multiple of 2, 3, 4, 5, 6, 7, and 8, which is 8 · 3 · 5 · 7 = 840. Since the number of sheets is between
1000 and 2000, the only possibility is 1681. The number of piles must be a divisor of 1681 = 412 ,
hence it must be 41.

2
Problem A8 [6 points]

If f (x) is a monic quartic polynomial such that f (−1) = −1, f (2) = −4, f (−3) = −9, and
f (4) = −16, find f (1).

Solution: The given data tells us that the roots of f (x) + x2 are -1, 2, -3, and 4. Combining with
the fact that f is monic and quartic we get f (x) + x2 = (x + 1)(x − 2)(x + 3)(x − 4). Hence
f (1) = (2)(−1)(4)(−3) − 1 = 23.

Problem A9 [7 points]

How many ways are there to cover a 3 × 8 rectangle with 12 identical dominoes?

Solution: Trivially there is 1 way to tile a 3 × 0 rectangle, and it is not hard to see there are 3 ways
to tile a 3 × 2. Let Tn be the number of tilings of a 3 × n rectangle, where n is even. From the
diagram below we see the recursion Tn = 3Tn−2 + 2(Tn−4 + Tn−6 + . . . + T2 + T0 ). Given that, we
can just calculate T4 = 11, T6 = 41, and T8 is 153.

... ... etc...

Problem A10 [8 points]

Pyramid EARLY is placed in (x, y, z) coordinates so that E = (10, 10, 0), A = (10, −10, 0), R =
(−10, −10, 0), L = (−10, 10, 0), and Y = (0, 0, 10). Tunnels are drilled through the pyramid in
such a way that one can move from (x, y, z) to any of the 9 points (x, y, z − 1), (x ± 1, y, z − 1),
(x, y ± 1, z − 1),(x ± 1, y ± 1, z − 1). Sean starts at Y and moves randomly down to the base of the
pyramid, choosing each of the possible paths with probability 19 each time. What is the probability
that he ends up at the point (8, 9, 0)?

Solution 1: Start by figuring out the probabilities of ending up at each point on the way down the
pyramid. Obviously we start at the top vertex with probability 1, and each point on the next level
down with probability 1/9. Since each probability after n steps will be some integer over 9n , we
will look only at those numerators. The third level down has probabilities as shown below. Think
of this as what you would see if you looked at the pyramid from above, and peeled off the top two
layers.

1 2 3 2 1
2 4 6 4 2
3 6 9 6 3
2 4 6 4 2
1 2 3 2 1

3
What we can observe here is not only the symmetry along vertical, horizontal, and diagonal axes,
but also that each number is the product of the numbers at the ends of its row and column (e.g.
6 = 2 · 3). This comes from the notion of independence of events, i.e. that if we east and then south,
we end up in the same place as if we had moved south and then east. Since we are only looking for
the probability of ending up at (8, 9, 0), we need only know that this is true for the top two rows
of the square of probabilities, which depend only on the top two rows of the previous layer. This
will follow from the calculation of the top row of each square, which we can do via an algorithm
similar to Pascal’s triangle. In the diagram below, each element is the sum of the 3 above it.

1 1 1

1 2 3 2 1

1 3 6 7 6 3 1

1 4 10 16 19 16 10 4 1

1 5 15 30 45 51 45 30 15 5 1

Now observe that the first 3 numbers in row n, where the top is row 0, are 1, n, n(n+1)
2 . This fact
is easily proved by induction on n, so the details are left to the reader. Now we can calculate the
top two rows of each square via another induction argument, or by independence, to establish that
the second row is always n times the first row. Therefore the probability of ending up at the point
(8,9,0) is 550
910
.

Solution 2: At each move, the x and y coordinates can each increase by 1, decrease by 1, or stay
the same. The y coordinate must increase 9 times and stay the same 1 times, the x coordinate can
either increase 8 times and stay the same 1 time or decrease 1 time and increase 9 times. Now we
consider every possible case. First consider the cases where the x coordinate decreases once. If the
x coordinate decreases while the y coordinate increases, then we have 8 moves that are the same
and 2 that are different, which can be done in 10!8! = 90 ways. If the x coordinate decreases while
the y coordinate stays the same, then we have 9 moves that are the same and 1 other, which can be
done in 10!
9! = 10 ways. Now consider the cases where the x coordinate stays the same twice. If the
y coordinate stays the same while the x coordinate increases, then we have 7 moves that are the
10!
same, 2 that are the same, and 1 other, which can be done in 7!2! = 360 ways. If the y coordinate
stays the same while the x coordinate stays the same, then we have 8 moves that are the same and 2
that are different, which can be done in 10!
8! = 90 ways. Therefore there are 360 + 90 + 90 + 10 = 550
paths to (8,9,0), out of 91 0 possible paths to the bottom, so the probability of ending up at the
point (8,9,0) is 550
910
.

4
Calculus Solutions
Harvard-MIT Math Tournament
February 27, 1999

Problem C1 [3 points]

Find all twice differentiable functions f (x) such that f ′′ (x) = 0, f (0) = 19, and f (1) = 99.

Solution: Since f ′′ (x) = 0 we must have f (x) = ax + b for some real numbers a, b. Thus f (0) =
b = 19 and f (1) = a + 19 = 99, so a = 80. Therefore f (x) = 80x + 19.

Problem C2 [3 points]

A rectangle has sides of length sin x and cos x for some x. What is the largest possible area of such
a rectangle?

1
Solution: We wish to maximize sin x · cos x = 2 sin 2x. But sin 2x ≤ 1, with equality holding for
x = π/4, so the maximum is 21 .

Problem C3 [4 points]

Find √
Z 4π 2 sin x
√ ( + 1)dx.
−4π 2 1 + x4

sin x
Solution: The function 1+x 4 is odd, so its integral over this interval is 0. Thus we get the same

answer if we just integrate dx, namely, 8π 2.

Problem C4 [4 points]

f is a continuous real-valued function such that f (x + y) = f (x)f (y) for all real x, y. If f (2) = 5,
find f (5).

Solution 1: Since
√ f (nx) = f (x)n for all integers n, f (5) = f (1)5 and f (2) = f (1)2 , so f (5) =
5/2
f (2) = 25 5.

Solution 2: More generally, since f (nx) = f (x)n for all integers n, f (1) = c = f (1/n)n for some
constant c and all integers n. Thus f (k/n) = f (1/n)k = f (1)k/n = ck/n for all rational numbers

k/n. By continuity,
√ it follows that f (x) = cx for all real numbers x. Since f (2) = 5, c = 5, so
f (5) = 25 5.

1
Problem C5 [5 points]

1
Let f (x) = x + 2x+ 1 for x > 0. Find f (99)f ′ (99).
2x+ 1
..
2x+ .

Solution: Assume that the continued fraction converges (it √ does) so that f (x) is well defined. Notice
1 2 2
that f (x) − x = x+f (x) , so f (x) − x = 1, or f (x) = 1 + x2 (we need the positive square root
x
since x > 0). Thus f ′ (x) = √1+x 2
, so f (x)f ′ (x) = x. In particular, f (99)f ′ (99) = 99.

Problem C6 [5 points]

d
Evaluate dx (sin x − 43 sin3 x) when x = 15.

Solution: Of course this problem can be done by brute force, differentiating and then using the half
angle formula to find sin and cos of 15, but there is a quicker way. eix = cos x + i sin x, so sin(3x)
is the imaginary part of (cos x + i sin x)3 , which is 3 cos2 x sin x − sin3 x = 3 sin x − 4 sin3 x, so √
the
1
expression we are differentiating is just 3 sin(3x). Hence the derivative is cos(3x), and cos 45 = 22 .

Problem C7 [5 points]

If a right triangle is drawn in a semicircle of radius 1/2 with one leg (not the hypotenuse) along
the diameter, what is the triangle’s maximum possible area?

Solution: It is easy to see that we will want one vertex of the triangle to be where the diameter
meets the semicircle, so the diameter is divided into segments of length x and 1 − x, where x is the
length of the leg on the diameter. The other leg of the triangle will
√ be the geometric mean of these
p x x(1−x)
two numbers, x(1 − x). Therefore the area of the triangle is 2 , so it will be maximized

d 3
when dx (x − x4 ) = 3x2 − 4x3 = 0, or x = 3/4. Therefore the maximum area is 3 3
32 .

Problem C8 [6 points]

A circle is randomly chosen in a circle of radius 1 in the sense that a point is randomly chosen
for its center, then a radius is chosen at random so that the new circle is contained in the original
circle. What is the probability that the new circle contains the center of the original circle?

Solution: If the center of the new circle is more than 1/2 away from the center of the original
circle then the new circle cannot possibly contain the center of the original one. Let x be the
distance between the centers (by symmetry this is all we need to consider), then for 0 ≤ x ≤ 1/2
x
the probability of the new circle containing the center of the original one is 1 − 1−x . Hence we need
R 1/2 x 1 R 1/2 x
to compute 0 (1 − 1−x )dx = 2 − 0 1−x dx. To evaluate the integral, we can integrate by parts
to get
Z 1/2 1 1 1
1/2 1/2
−x ln(1 − x)|0 − − ln(1 − x)dx = − ln( ) − [(1 − x) ln(1 − x) − (1 − x)]0 = ln 2 − .
0 2 2 2

2
x 1 R 1/2 x
Alternatively, we can use polynomial division to find that 1−x = −1 + 1−x , so 0 1−x dx =
R 1/2 1
0 (−1 + 1−x )dx = ln 2 − 12 . Therefore the probability is 1
2 − (ln 2 − 21 ) = 1 − ln 2.

Problem C9 [7 points]

What fraction of the Earth’s volume lies above the 45 degrees north parallel? You may assume
the Earth is a perfect sphere. The volume in question is the smaller piece that we would get if the
sphere were sliced into two pieces by a plane.

Solution 1: Without loss of generality, look at a sphere of radius 1 centered at the origin. If you like
cartesian coordinates, then you can slice the sphere into discs with the same z coordinate, which √

have radius 1 − z 2 , so the region we are considering has volume √1 2/2 π(1 − z 2 )dz = π( 23 − 5122 ),
R

8−5 2
and dividing by 4π/3 we get 16 .

Solution 2: For those who prefer spherical coordinates, we can find the volume of the spherical cap
plus a cone whose vertex is the center of the sphere. This region is where r ranges from 0 to 1, θ
ranges from 0 to 2π, and φ ranges from 0 to π/4. Remembering we need to subtract off the volume of
the cone, which has height √12 and a circular base of radius √12 , then divide by 34 π to get the fraction
R 1 R 2π R π/4
r 2 sin φdφdθdr− 31 π( √1 )2 √1
of the volume of the sphere, we find that we need to evaluate 0 0 0
4π/3
2 2
. The

4π−2π 2
integral is just 2π 13 (− cos π/4 + cos 0) = 6 . Putting this back in the answer and simplifying

8−5 2
yields 16 .

Solution 3: Cavalieri’s Principle states that if two solids have the same cross-sectional areas at every
height, then they have the same volume. This principle is very familiar in the plane, where we know
that the area of a triangle depends only on the base and height, not the precise position of the apex.
To apply it to a sphere, consider a cylinder with radius 1 and height 1. Now cut out a cone whose
base is the upper circle of the cylinder and whose apex is the center of the lower circle. Then at a
height z the area is π(12 − z 2 ), exactly the same as for the upper hemisphere! The portion lying
above the the 45 degrees north parallel is that which ranges from height √12 to 1. The volume of
the cylinder in this range is π · 12 (1 − √12 ). The volume of the cone in this range is the volume of the
entire cone minus the portion from height 0 to √1 , i.e., 31 π(12 · 1 − ( √12 )2 √12 ). Therefore the fraction
2
π(1− √1 )− 13 π(1− 1
√ ) √
8−5 2
of the Earth’s volume that lies above the 45 degrees north parallel is 2
4π/3
2 2
= 16 .

p
Solution 4: Another way to approach this problem is to integrate the function 1 − x2 − y 2 over
the region x2 + y 2 ≤ √12 , subtract off a cylinder of radius and height √12 , then divide by the volume
of the sphere. One could also use the nontrivial fact that the surface area of a portion of a sphere of
radius r between two parallel planes separated by a distance z is 2πr2 z, so in particular the surface
area of this cap is 2π(1 − √12 ). Now, the ratio of the surface area of the cap to the surface area of
the sphere is the same as the ratio of the volume of the cap plus the cone considered in Solution 2
to the volume of the whole sphere, so this allows us to avoid integration entirely.

3
Problem C10 [8 points]

Let An be the area outside a regular n-gon of side length 1 but inside its circumscribed circle, let
Bn be the area inside the n-gon but outside its inscribed circle. Find the limit as n tends to infinity
An
of B n
.

Solution: The radius of the inscribed circle is 21 cot πn , the radius of the circumscribed circle is
1 π n π
2 csc n , and the area of the n-gon is 4 cot n . The diagram below should help you verify that these
are correct.

r 1/2

π/n

π 2 π
An π(csc n ) −n cot n
Then An = π( 12 csc πn )2 − n
4 cot πn , and Bn = n
4 cot πn − π( 12 cot πn )2 , so Bn = π π 2 . Let s
n cot n −π(cot n )
An
denote sin πn and c denote cos πn . Multiply numerator and denominator by s2 to get B n
π−ncs
= ncs−πc 2.
( π )3 ( π )2
Now use Taylor series to replace s by πn − n6 + . . . and c by 1 − n2 + . . .. By l’Hôpital’s rule
it will suffice to take just enough terms so that the highest power of n in the numerator and
denominator is determined, and that turns out to be n−2 in each case. In particular, we get the
π π 3 2 π3
An π−n n +n 23 ( n ) +... 3 n2
+...
limit Bn = π 2 π 3
n n −n 3 ( n ) −π+π( n π 2
) +...
= 1 π3
→ 2.
+...
3 n2

4
Geometry Solutions
Harvard-MIT Math Tournament
February 27, 1999

Problem G1 [3]

Two 10 × 24 rectangles are inscribed in a circle as shown. Find the shaded area.

Solution: The rectangles are 10 × 24, so their diagonals, which are diameters of the circle, have
length 26. Therefore the area of the circle is π132 , and the overlap is a 10 × 10 square, so the
shaded area is π132 − 2 · 10 · 24 + 102 = 169π − 380.

Problem G2 [3]

A semicircle is inscribed in a semicircle of radius 2 as shown. Find the radius of the smaller
semicircle.

Solution: Draw a line from the center of the smaller semicircle to the center of the larger one, and
a line from the center of the larger semicircle to one of the other points of intersection of the two
semicircles. We now have a right triangle whose legs are both the radius of√the smaller semicircle
and whose hypotenuse is 2, therefore the radius of the smaller semicircle is 2.

Problem G3 [4]

In a cube with side length 6, what is the volume of the tetrahedron formed by any vertex and the
three vertices connected to that vertex by edges of the cube?

Solution: We have a tetrahedron whose base is half a face of the cube and whose height is the side
length of the cube, so its volume is 31 · ( 12 · 62 ) · 6 = 36.

1
Problem G4 [4]

A cross-section of a river is a trapezoid with bases 10 and 16 and slanted sides of length 5. At this
section the water is flowing at π mph. A little ways downstream is a dam where the water flows
through 4 identical circular holes at 16 mph. What is the radius of the holes?

Solution: The volume of water going through any cross-section of the river in an hour (assuming
the cross-sections are parallel) is the area times the velocity. The trapezoid has height 4, hence
area 52, so the volume of water going through at any hour is 52π. Let r be the radius of the holes,

then the total area is 4πr2 , so the volume of water is 64πr2 . Therefore 64πr2 = 52π, so r = 413 .

Problem G5 [5]

In triangle BEN shown below with its altitudes intersecting at X, N A = 7, EA = 3, AX = 4, and


N S = 8. Find the area of BEN .
B H E

S A

Solution: The idea is to try to find a base and height√ for the triangle so that we can find the area.
By the Pythagorean theorem, EX = 5, N X = 65, and SX = 1. Triangles AXE and BXS are
similar since they have the same angles. The ratio of their side lengths is 4:1, so BS = 3/4 and
BX = 5/4. Now using either N E or N B as a base, we get that the area of BEN is 21 · (8 + 34 ) · 6
or 12 · (4 + 45 ) · 10, both of which simplify to 105
4 .

Problem G6 [5]

A sphere of radius 1 is covered in ink and rolling around between concentric spheres of radii 3 and
5. If this process traces a region of area 1 on the larger sphere, what is the area of the region traced
on the smaller sphere?

Solution: The figure drawn on the smaller sphere is just a scaled down version of what was drawn
on the larger sphere, so the ratio of the areas is the ratio of the surface area of the spheres. This
9
is the same as the ratio of the squares of the radii, which is 25 .

Problem G7 [5]

A dart is thrown at a square dartboard of side length 2 so that it hits completely randomly. What
is the probability that it hits closer to the center than any corner, but within a distance 1 of a
corner?

2
Solution: By symmetry it will suffice to consider one quarter of the dartboard, which is a square
of side length 1. Therefore the probability is the area of the desired region in this square. The
desired region is the part of the circle of radius 1 centered at a corner that is closer to the opposite
corner. The points closer to the opposite corner are those that are on the other side of the diagonal
through the other two corners, so the desired region is a quarter of a circle of radius 1 minus a right
triangle with legs of length 1. Therefore the area (and hence the probability) is π−2 4 .

Problem G8 [6]

Squares ABKL, BCM N, CAOP are drawn externally on the sides of a triangle ABC. The line
segments KL, M N, OP , when extended, form a triangle A′ B ′ C ′ . Find the area of A′ B ′ C ′ if ABC
is an equilateral triangle of side length 2.

Solution: Triangle ABC has area 3, and each of the three squares has area 4. The three remaining
regions are congruent, so just consider the one that includes vertex B. Triangle KBN has two sides
of length 2 and an angle of 120◦ between them, to bisecting √ that angle we get two halves of an
equilateral triangle of side
√ length 2, so the area is
√ 2√ again 3. The
√ remaining region is an equilateral
triangle
√ of side length
√ 2 √3, so its area√is (2 3) 3/4 = 3 3. Therefore the area of A′ B ′ C ′ is
3 + 3 · 4 + 3 · 3 + 3 · 3 3 = 12 + 13 3.

Note that this problem is still solvable, but much harder, if the first triangle is not equilateral.

Problem G9 [7]

A regular tetrahedron has two vertices on the body diagonal of a cube with side length 12. The
other two vertices lie on one of the face diagonals not intersecting that body diagonal. Find the
side length of the tetrahedron.

Solution: Let ABCD be a tetrahedron of side s. We want to find the distance between two of
its opposite
√ sides. Let E be the midpoint of AD, F the midpoint of BC. Then AE = s/2,
AF = s 3/2, and angle AEF = 90◦ . So the distance between the two opposite sides is EF =
√ p √
AF 2 − AE 2 = 3s2 /4 − s2 /4 = s/ 2.

Now we find the distance between a body diagonal and a face diagonal of a cube of side a. Let O
be the center of the cube and P be the midpoint of the face diagonal. Then the plane containing
P and the body diagonal is perpendicular to the face diagonal. So the distance between
q the body
a 2
and face diagonals is the distance between P and the body diagonal, which is 2 3 (the altitude
q
from P of right triangle OP Q, where Q is the appropriate vertex of the cube). So now √s = a 2
2 2 3,
√ √ √
thus s = a/ 3 = 12/ 3 = 4 3.

Problem G10 [8]

In the figure below, AB = 15, BD = 18, AF = 15, DF = 12, BE = 24, and CF = 17. Find
BG : F G.

Solution: Our goal is to find the lengths BG and F G. There are several ways to go about doing
this, but we will show only one here. We will make several uses of Stewart’s theorem, which can

3
B C

A
D
F

be proved using the law of cosines twice. By Stewart’s theorem on triangle ABD and line BF ,
152 · 12 + 182 · 15 = BF 2 · 27 + 15 · 12 · 27, so BF = 10 and EF = 14. By Stewart’s √ theorem
2 2 2
on triangle ABE and line AF , AE · 10 + 15 · 14 = 15 · 24 + 14 · 10 · 24, so AE = 561. By
Stewart’s theorem
√ on triangle AED and line EF , ED2 · 15 + 561 · 12 = 142 · 27 + 12 · 15 √ · 27,
so ED = 2 57. By Stewart’s theorem on triangle CF E and line F D, 142 · CD + 172 · 2 57 =
√ √ √ √ √
122 · (CD + 2 57) + 2 57 · CD · (CD + 2 57), so CD = 57 and CE = 3 57. Note that
DG = 18 − BG and apply Menelaus’ theorem to triangle BED and the line through C, G, and F
to get 3 · 18−BG 10 135
BG · 14 = 1, so BG = 11 . Similarly CG = 17 − F G, so applying Menelaus’ theorem to
triangle CF E and the line through B, G, and D we get 24 FG 1 85
10 · 17−F G · 2 = 1, so F G = 11 . Therefore
BG : F G = 27 : 17.

4
Advanced Topics Solutions
Harvard-MIT Math Tournament
February 27, 1999

Problem AT1 [3 points]

One of the receipts for a math tournament showed that 72 identical trophies were purchased for
$-99.9-, where the first and last digits were illegible. How much did each trophy cost?

Solution: The price must be divisible by 8 and 9. Thus the last 3 digits must be divisible by
8, so the price ends with 992, and the first digit must be 7 to make the total divisible by 9.
$799.92/72 = $11.11.

Problem AT2 [3 points]

Stacy has d dollars. She enters a mall with 10 shops and a lottery stall. First she goes to the lottery
and her money is doubled, then she goes into the first shop and spends 1024 dollars. After that
she alternates playing the lottery and getting her money doubled (Stacy always wins) then going
into a new shop and spending $1024. When she comes out of the last shop she has no money left.
What is the minimum possible value of d?

Solution: Work backwards. Before going into the last shop she had $1024, before the lottery she
had $512, then $1536, $768, . . . . We can easily prove by induction that if she ran out of money
after n shops, 0 ≤ n ≤ 10, she must have started with 1024 − 210−n dollars. Therefore d is 1023.

Problem AT3 [4 points]

An unfair coin has the property that when flipped four times, it has the same probability of turning
up 2 heads and 2 tails (in any order) as 3 heads and 1 tail (in any order). What is the probability
of getting a head in any one flip?

Solution: Let p be the probability of getting a head in one flip. There are 6 ways to get 2 heads and
2 tails, each with probability p2 (1 − p)2 , and 4 ways to get 3 heads and 1 tail, each with probability
p3 (1 − p). We are given that 6p2 (1 − p)2 = 4p3 (1 − p). Clearly p is not 0 or 1, so we can divide by
p2 (1 − p) to get 6(1 − p) = 4p. Therefore p is 35 .

Problem AT4 [4 points]

You are given 16 pieces of paper numbered 16, 15, ..., 2, 1 in that order. You want to put them
in the order 1, 2, ..., 15, 16 switching only two adjacent pieces of paper at a time. What is the
minimum number of switches necessary?

Solution: Piece 16 has to move to the back 15 times, piece 15 has to move to the back 14 times,
. . . , piece 2 has to move to the back 1 time, piece 1 has to move to the back 0 times. Since only
one piece can move back in each switch, we must have at least 15 + 14 + . . . + 1 = 120 switches.

1
Problem AT5 [5 points]

For any finite set S, let f (S) be the sum of the elements of S (if S is empty then f (S) = 0). Find
the sum over all subsets E of S of ff (E)
(S) for S = {1, 2, ..., 1999}.

Solution: An n element set has 2n subsets, so each element of S appears in 21998 subsets E, so our
1+2+...+1999
sum is 21998 · 1+2+...+1999 = 21998 .

Problem AT6 [5 points]

Matt has somewhere between 1000 and 2000 pieces of paper he’s trying to divide into piles of the
same size (but not all in one pile or piles of one sheet each). He tries 2, 3, 4, 5, 6, 7, and 8 piles
but ends up with one sheet left over each time. How many piles does he need?

Solution: The number of sheets will leave a remainder of 1 when divided by the least common
multiple of 2, 3, 4, 5, 6, 7, and 8, which is 8 · 3 · 5 · 7 = 840. Since the number of sheets is between
1000 and 2000, the only possibility is 1681. The number of piles must be a divisor of 1681 = 412 ,
hence it must be 41.

Problem AT7 [5 points]

Find an ordered pair (a, b) of real numbers for which x2 + ax + b has a non-real root whose cube
is 343.

Solution: The cube roots of 343 are the roots of x3 − 343, which is (x − 7)(x2 + 7x + 49). Therefore
the ordered pair we want is (7,49).

Problem AT8 [6 points]

Let C be a circle with two diameters intersecting at an angle of 30 degrees. A circle S is tangent
to both diameters and to C, and has radius 1. Find the largest possible radius of C.

Solution: For C to be as large as possible we want S to be as small as possible. It is not hard


√ to√see
that this happens in the situation shown below. Then the radius of C is 1 + csc 15 = 1 + 2 + 6.
The computation of sin 15 can be done via the half angle formula.

1
15

2
Problem AT9 [7 points]

As part of his effort to take over the world, Edward starts producing his own currency. As part of
an effort to stop Edward, Alex works in the mint and produces 1 counterfeit coin for every 99 real
ones. Alex isn’t very good at this, so none of the counterfeit coins are the right weight. Since the
mint is not perfect, each coin is weighed before leaving. If the coin is not the right weight, then it
is sent to a lab for testing. The scale is accurate 95% of the time, 5% of all the coins minted are
sent to the lab, and the lab’s test is accurate 90% of the time. If the lab says a coin is counterfeit,
what is the probability that it really is?

Solution: 5% of the coins are sent to the lab, and only .95% of the coins are sent to the lab and
counterfeit, so there is a 19% chance that a coin sent to the lab is counterfeit and an 81% chance
that it is real. The lab could correctly detect a counterfeit coin or falsely accuse a real one of
being counterfeit, so the probability that a coin the lab says is counterfeit really is counterfeit is
19/100·9/10 19
19/100·9/10+81/100·1/10 = 28 .

Problem AT10 [8 points]

Find the minimum possible value of the largest of xy, 1−x−y +xy, and x+y −2xy if 0 ≤ x ≤ y ≤ 1.

s± s2 −4p
Solution: I claim the answer is 4/9. Let s = x + y, p = xy, so x and y are 2 . Since x
2
and y are real, s − 4p ≥ 0. If one of the three quantities is less than or equal to 1/9, then at
least one of the others is at least 4/9 by the pigeonhole principle since they add up to 1. Assume
that s − 2p < 4/9, then s2 − 4p < (4/9 + 2p)2 − 4p, and since the left side is non-negative we get
0 ≤ p2 − 59 p + 81
4
= (p − 91 )(p − 49 ). This implies that either p ≤ 91 or p ≥ 49 , and either way we’re
done. This minimum is achieved if x and y are both 1/3, so the answer is 94 , as claimed.

3
Oral Solutions
Harvard-MIT Math Tournament
February 27, 1999

Problem O1 [25 points]

Start with an angle of 60◦ and bisect it, then bisect the lower 30◦ angle, then the upper 15◦ angle,
and so on, always alternating between the upper and lower of the previous two angles constructed.
This process approaches a limiting line that divides the original 60◦ angle into two angles. Find
the measure (degrees) of the smaller angle.

Solution: The fraction of the original angle is 12 − 14 + 18 − + · · ·. This is just a geometric series with
first term 1/2 and ratio -1/2, so the sum is 1/3. Therefore the smaller angle is 20◦ .

Score 20 points for the correct answer, 5 points for a correct justification. Just setting up the sum
is worth 6 points, getting 1/3 is worth 9 more.

Problem O2 [25 points]

Alex, Pei-Hsin, and Edward got together before the contest to send a mailing to all the invited
schools. Pei-Hsin usually just stuffs the envelopes, but if Alex leaves the room she has to lick them
as well and has a 25% chance of dying from an allergic reaction before he gets back. Licking the
glue makes Edward a bit psychotic, so if Alex leaves the room there is a 20% chance that Edward
will kill Pei-Hsin before she can start licking envelopes. Alex leaves the room and comes back to
find Pei-Hsin dead. What is the probability that Edward was responsible?

Solution: There are two possibilities: either Edward killed Pei-Hsin or the envelopes did. The
envelope could only be responsible if Edward was not, so the chances of that would be 4/5·1/4 = 1/5.
This is the same as the probability that Edward killed her, so the events are equally likely and the
answer is 50%, or 1/2.

Score 20 points for the correct answer, 5 points for a correct justification. Not many places to give
partial credit.

Problem O3 [30 points]

If x, y, and z are distinct positive integers such that x2 + y 2 = z 3 , what is the smallest possible
value of x + y + z.

Solution 1: Without loss of generality let x > y. We must have z 3 expressible as the sum of two
squares, and this first happens when z = 5. Then x and y can be 10 and 5 or 11 and 2. If z > 5
then z ≥ 10 for z 3 to be a sum of two distinct squares, so x2 > 500, x > 22, so x + y + z > 32.
Thus the smallest possible value of x + y + z is 11 + 2 + 5 = 18.

Solution 2: If z > 5, then z ≥ 6, so z 3 ≥ 216. Now x2 + y 2 ≥ 216, so x ≥ 11 and y ≥ 1, thus


x + y + z ≥ 18. Since x = 11, y = 1, z = 6 does not work, we must have x + y + z > 18, and the
solution given is the best possible.

1
Score 20 points for the correct answer, 5 points for justifying that we can’t do better with z ≤ 5, 5
points for justifying that we can’t do better with z > 5.

Problem O4 [35 points]

P∞ cos nθ
Evaluate n=0 2n , where cos θ = 15 .

P∞ einθ
Solution: cos nθ is the real part of einθ , so the sum is the real part of n=0 2n . This is a geometric


series with initial term 1 and ratio e2 , so its sum is 1−e1iθ /2 . We are given cos θ = 15 , so sin θ = ± 2 5 6 .

10 √ 90±20i 6
Thus the sum is 10−1∓2i 6
= 105 , and the real part is 67 .

Score 20 points for the correct answer, 15 points for a correct justification.

Problem O5 [45 points]

Let r be the radius of the inscribed circle of triangle ABC. Take a point D on side BC, and let r1
and r2 be the inradii of triangles ABD and ACD. Prove that r, r1 , and r2 can always be the side
lengths of a triangle.

Solution: We must show that r, r1 , and r2 satisfy the triangle inequality, i.e. that the sum of any
two of them exceeds the third. Clearly r is the largest of the three, so we need only verify that
r1 + r2 > r. Let K and s be the area and semiperimeter of triangle ABC. Similarly define K1 ,
K2 , s1 , and s2 . Observe that s is larger than s1 or s2 and that K1 + K2 = K. While these facts
are almost trivial to verify, they must be stated. Then r = K/s, r1 = K1 /s1 , and r2 = K2 /s2 , so
r1 + r2 = K1 /s1 + K2 /s2 > K1 /s + K2 /s = K/s = r.

The correct use of areas and semiperimeters is worth 25 points, each of the critical facts is worth 10
points. I don’t know of any other way to do this problem, so attempts at alternate proofs should
get at most 15 points for effort unless they really on the right track to another solution.

Problem O6 [45 points]

You want to sort the numbers 5 4 3 2 1 using block moves. In other words, you can take any set
of numbers that appear consecutively and put them back in at any spot as a block. For example,
6 5 3 4 2 1 → 4 2 6 5 3 1 is a valid block move for 6 numbers. What is the minimum number of
block moves necessary to get 1 2 3 4 5?

Solution 1: Here is a sequence of 3 moves that works: 54321→32541→34125→12345. But how do


we know we can’t do it in 2 moves? From any position there are 20 possible permutations via block
moves, 16 from moving a block of size 1 and 4 from moving a block of size 2. One could simply
write the 20 permutations of 54321 and the 20 permutations of 12345 and try to see that they have
nothing in common, which would suffice since the inverse of a block move is also a block move. A
more clever method is to notice that if we could sort 54321 in 2 moves then we could sort 4321
in 2 moves as well by simply deleting the 5 from each step. But 4321 has only 10 permutations
from block moves, namely 3421, 3241, 3214, 4231, 4213, 2431, 4312, 1432, 4132, and 2143. The 10

2
permutations of 1234 are 2134, 2314, 2341, 1324, 1342, 3124, 1243, 4123, 1423, and 3412. These
two sets of permutations have nothing in common, thus it takes at least 3 moves to sort 4321, and
hence at least 3 moves to sort 54321.

Solution 2: There is a more elegant way to show we need at least 3 moves. Given a permutation
of {1, 2, 3, 4, 5} (or any ordered set), define a descent to be an adjacent pair of numbers in the
permutation such that the left number is greater than the right one. For example, 12345, 34215,
and 54321 have 0, 2, and 4 descents, respectively. Any permutation obtained from 12345 by
one block move has (at most) one descent, at the left edge of the moved block. Similarly, any
permutation obtained from 54321 by one block move has (at least) three descents, so that we can’t
get from 54321 to 12345 by two block moves.

Score 20 points for the correct answer, 10 points for a numerical example proving that is attainable,
and 15 points for proving it can’t be done in 2 or fewer moves.

Problem O7 [55 points]

P∞ n5
Evaluate n=1 n! .

n P∞ P∞ 1 P∞ 1
Solution: We start by noticing that n=1 n! = n=1 (n−1)! = n=0 n! = e. Next we see that
P∞ n2 P∞ n P∞ 1+n P∞ 1 P ∞ n P∞ nk
n=1 n! = n=1 (n−1)! = n=0 n! = n=0 n! + n=0 n! = e + e = 2e. Let f (k) = n=1 n! , then
P∞ nk P∞ nk−1 P∞ (1+n)k−1 Pk−1 k−1
n=1 n! = n=1 (n−1)! = n=0 n! , so by the binomial theorem f (k) = j=0 j · f (j).
Armed with this formula, we can easily compute f (3) = f (0) + 2f (1) + f (2) = e + 2e + 2e = 5e,
f (4) = 1 · e + 3 · e + 3 · 2e + 1 · 5e = 15e, and f (5) = 1 · e + 4 · e + 6 · 2e + 4 · 5e + 1 · 15e = 52e.

Score 30 points for the correct answer, 25 points for a correct justification. If on the right track
with a good justification, but an arithmetic error is made along the way, score 5 points for each
f (j), j = 0, 1, 2, 3, 4, correctly computed.

Problem O8 [55 points]

What is the smallest square-free composite number that can divide a number of the form
4242 . . . 42 ± 1?

Solution: It is easy to see that such a number can never be divisible by 2, 3, 5, or 7. They can
be divisible by 11, the smallest example being 4242424241 = 11 · 547 · 705073. What makes this
problem hard is finding the next prime that can divide such a number. Let Tn = ni=0 42 · 102i .
P

Then the numbers Tn modulo a prime p will always be periodic, since Tn = 100Tn1 + 42, so we just
need to compute one period and see if it contains ±1. Thus we find that modulo 13 we get 3, 4, 0,
3, . . . , modulo 17 we get 8, 9, 7, 11, 3, 2, 4, 0, 8, . . . , modulo 19 we get 4, 5, 10, 16, 8, 6, 15, 3, 0, 4,
. . . , and modulo 23 we get 19, 10, 7, 6, 21, 3, 20, 18, 2, 12, 0, 19, . . . , so none of these primes can
ever divide Tn ± 1. But 424241 = 29 · 14629, so 29 can also divide numbers of this form. Therefore
the smallest composite number that can divide Tn ± 1 for some n is 319, and the smallest such n
is 83.

Score 30 points for the correct answer, 15 points for showing it is the smallest, 10 points for showing
it does work. Just seeing that 11 is the smallest prime divisor is worth 5 points, finding for exactly
which n is worth 5 more.

3
Problem O9 [60 points]

You are somewhere on a ladder with 5 rungs. You have a fair coin and an envelope that contains
either a double-headed coin or a double-tailed coin, each with probability 1/2. Every minute you
flip a coin. If it lands heads you go up a rung, if it lands tails you go down a rung. If you move
up from the top rung you win, if you move down from the bottom rung you lose. You can open
the envelope at any time, but if you do then you must immediately flip that coin once, after which
you can use it or the fair coin whenever you want. What is the best strategy (i.e. on what rung(s)
should you open the envelope)?

Solution: First consider the probability of winning if you never open the envelope. Let q(n) be the
probability of winning from the nth rung with just the fair coin, then q(n) = q(n−1)+q(n+1)
2 , so it
is not hard to calculate that q(n) = n/6. If we open the envelope, then there’s a 1/2 chance that
it is heads and we win, and a 1/2 chance that it is tails and we end up one rung down with just
the fair coin (obviously we keep using the double sided coin iff it is double headed). Let us start
by analyzing rung 1. If we don’t open the envelope, then we have a 1/2 chance of losing and a 1/2
chance of ending up on rung 2 with the envelope. If we do open the envelope, then we have a 1/2
chance of losing and a 1/2 chance of winning, which is a better outcome, so we should open the
envelope on rung 1. Next we look at rung 5. If we don’t open the envelope, then we have a 1/2
chance of winning and a 1/2 chance of moving down to rung 4 with the envelope. If we do open
the envelope, then we we have a 1/2 chance of winning and a 1/2 chance of moving down to rung
4 without the envelope. Let p(n) be the probability of winning from rung n if we are there with
the envelope still unopened. Then clearly p(n) ≥ q(n) for all n if we’re using optimal strategy, so
we should not open the envelope on rung 5. Next we look at rung 4. If we open the envelope,
then our chance of winning is 1/2 + q(3)/2 = 3/4. If we don’t, then our chance of winning is
p(5)/2 + p(3)/2. We do know that p(5) = 1/2 + p(4)/2, but this is not enough to tell us what to
do on rung 4. Looking at rung 3, we can open the envelope for a probability 1/2 + q(2)/2 = 2/3 of
winning, and we can not open the envelope for a probability p(4)/2 + p(2)/2 of winning. On rung
2, we can open the envelope for a probability 1/2 + q(1)/2 = 7/12 of winning, and we can not open
the envelope for a probability p(3)/2 + p(1)/2 = p(3)/2 + 1/4 of winning.
Now we can use all this information together for the complete answer. We know p(2) ≥ 7/12,
therefore p(3) ≥ p(4)/2 + 7/24, and we know p(4) ≥ p(5)/2 + p(3)/2 ≥ 1/2+p(4)/2 2 + p(3)/2 ≥
1/2+p(4)/2 p(4)/2+7/24
2 + 2 . Isolating p(4) in this inequality, we get p(4) ≥ 1/2+7/24 > 3/4, therefore we
should not open the envelope on rung 4. Now from p(4) = p(5)/2+p(3)/2 and p(5) = 1/2+p(4)/2
we have p(4) = 1/2+p(4)/2
2 + p(3)/2, so p(3) = 3p(4)/2 − 1/2 ≥ 11/16 > 2/3, so we should not open
the envelope on rung 3. Now p(2) ≥ p(3)/2 + 1/4 ≥ 19/32 > 7/12, so we should not open the
envelope on rung 2. Therefore the best strategy is to open the envelope iff we are on the bottom
rung.

For each rung, score 4 points for the correct answer and 8 more for a correct justification.

Problem O10 [75 points]

A, B, C, D, and E are relatively prime integers (i.e., have no single common factor) such that the
polynomials 5Ax4 + 4Bx3 + 3Cx2 + 2Dx + E and 10Ax3 + 6Bx2 + 3Cx + D together have 7 distinct
integer roots. What are all possible values of A? Your team has been given a sealed envelope that
contains a hint for this problem. If you open the envelope, the value of this problem decreases by
20 points. To get full credit, give the sealed envelope to the judge before presenting your solution.

4
Hint: Consider A = 1, B = D = 0, C = 750, and E = 19845.

Solution: Call the negatives of the roots of the first polynomial a, b, c, d, and the negatives of the
roots of the second polynomial e, f, g (using the negatives avoids negative signs for the rest of the
proof, thus preventing the possibility of dropping a sign). Then 5Ax4 + 4Bx3 + 3Cx2 + 2Dx + E =
5A(x + a)(x + b)(x + c)(x + d) and 10Ax3 + 6Bx2 + 3Cx + D = 10A(x + e)(x + f )(x + g). Thus
B = 45 A(a + b + c + d) = 53 A(e + f + g), C = 53 A(ab + ac + ad + bc + bd + cd) = 10 3 A(ef + eg + f g),
D = 52 A(abc + abd + acd + bcd) = 10A(ef g), and E = 5A(abcd). From these equations we see that
since all the variables are integers, it must be the case that A|B, A|3C, A|D, and A|E, therefore
A, B, C, D, and E can only be relatively prime if A is ±1 or ±3. Now we need numerical examples
to show that both√ of these are possible. Without loss of generality let g = 0, so D = 0. Then e
−3B± 9B 2 −30AC
and f are 10A , so 9B 2 − 30AC must be a perfect square. Let B = 0 in the hope that
solutions will still exist to this simplified problem. First let us try to find an example with A = −1,
so we need 30C to be a perfect square. This first happens for C = 30, and in that case e and f
are ±3. We need a + b + c + d = 0, so let’s try to look for a = −b, c = −d. This doesn’t work
for C = 30 since 3C/5 = 18 is not the sum of two distinct squares. For that we will need to try
C = 52 · 30 = 750, for which we get e, f = ±15, a, b = ±3, and c, d = ±21. Thus for A = ±1 we can
use B = D = 0, C = ∓750, and E = ±19845. Similarly we can find A = ±3, B = D = 0, C = ∓250,
and E = ±735, for which (a, b, c, d, e, f, g) = (1, −1, 7, −7, 5, −5, 0).

Note that these give us quintic polynomials with integer coefficients possessing 4 relative extrema
and 3 points of inflection at lattice points, such as 3x5 − 250x3 + 735x.

Score 20 points for the correct answer, 15 points for a correct justification, 20 points for a numerical
example for ±1, 20 points for a numerical example for ±3. If the sealed envelope isn’t presented
at the beginning of the solution, no credit is given for the ±1 example. If the ± is forgotten, give
10 points for the answer, 10 for the justification, and 10 for each numerical example. There are
infinitely many possible examples, so anything other than the two given above must be checked for
accuracy.

5
Team Solutions
Harvard-MIT Math Tournament
February 27, 1999

Problem T1 [15]

A combination lock has a 3 number combination, with each number an integer between 0 and 39
inclusive. Call the numbers n1 , n2 , and n3 . If you know that n1 and n3 leave the same remainder
when divided by 4, and n2 and n1 + 2 leave the same remainder when divided by 4, how many
possible combinations are there?

Solution: There are 40 choices for the last number, and for each of these we have 10 choices for
each of the first two numbers, thus giving us a total of 4000 possible combinations. It is interesting
to note that these restrictions are actually true for Master locks.

Problem T2 [15]

A ladder is leaning against a house with its lower end 15 feet from the house. When the lower end
is pulled 9 feet farther from the house, the upper end slides 13 feet down. How long is the ladder
(in feet)?

Solution: Of course the house makes a right angle with the ground, so we can use the Pythagorean
theorem. Let x be the length of the ladder and y be the original height at which it touched the
house. Then we are given x2 = 152 + y 2 = 242 + (y − 13)2 . Isolating y in the second equation we
get y = 20, thus x is 25.

Problem T3 [20]

How many non-empty subsets of {1, 2, 3, 4, 5, 6, 7, 8} have exactly k elements and do not contain
the element k for some k = 1, 2, ..., 8.

Solution: Probably the easiest way to do this problem is to count how many non-empty subsets of
{1, 2, . . . , n} have k elements and do contain the element k for some k. The element k must have
k − 1 other elements with it to be in a subset of k elements, so there are n−1

k−1 such subsets. Now
Pn n−1 n−1 n−1
k=1 k−1 = (1 + 1) =2 , so that is how many non-empty sets contain some k and have
k elements. The set {1, 2, . . . , n} has 2n subsets (each element either is or is not in a particular
subset), one of which is the empty set, so the number of non-empty subsets of {1, 2, 3, 4, 5, 6, 7, 8}
have exactly k elements and do not contain the element k for some k is 2n − 2n−1 − 1 = 2n−1 − 1.
In the case n = 8, this yields 127.

Problem T4 [20]

Consider the equation F ORT Y + T EN + T EN = SIXT Y , where each of the ten letters represents
a distinct digit from 0 to 9. Find all possible values of SIXT Y .

1
Solution: Since Y + N + N ends in Y , N must be 0 or 5. But if N = 5 then T + E + E + 1 ends in
T, which is impossible, so N = 0 and E = 5. Since F 6= S we must have O = 9, R + T + T + 1 > 10,
and S = F + 1. Now I 6= 0, so it must be that I = 1 and R + T + T + 1 > 20. Thus R and T are
6 and 7, 6 and 8, or 7 and 8 in some order. But X can’t be 0 or 1 since those are taken, and X
cannot be 3 since F and S have to be consecutive, so it must be that R + T + T + 1 is 21 or 23.
This is satisfied only for R = 7, T = 8, so F = 2, S = 3, and Y = 6. This SIXT Y = 31486.

Problem T5 [30]

If a and b are randomly selected real numbers between 0 and 1, find the probability that the nearest
a−b
integer to a+b is odd.

Solution: The only reasonable way I know of to do this problem is geometrically (yes, you can use
integrals to find the areas of the triangles involved, but I don’t consider that reasonable). First let
1 1
us find the points (a, b) in the plane for which the nearest integer to a−b a−b
a+b is 0, i.e. − 2 ≤ a+b ≤ 2 .
Taking the inequalities one at a time, − 21 ≤ a+b
a−b
implies that a + b ≥ 2(b − a), or b ≤ 3a, so these
1
points must lie below the line y = 3x. Similarly, a−b a+b ≤ 2 implies that (a, b) must lie above the
line y = 13 x. Now we can look for the points (a, b) for which the nearest integer to a−b a+b is 1, i.e.
1 a−b 3
2 ≤ a+b ≤ 2 , and we find that all points in the first quadrant that lie above the line y = 3x satisfy
this inequality. Similarly, the closest integer to a−b
a+b is -1 for all points in the first quadrant below
1
the line y = 3 x. For a and b between 0 and 1, the locus of points (a, b) for which the nearest integer
a−b
to a+b is odd is two right triangles with legs of length 1 and 31 , so together they have area 13 . The
locus of all points (a, b) with a and b between 0 and 1 is a square of side length 1, and thus has
a−b
area 1. Therefore the probability that the nearest integer to a+b is odd is 13 .

Problem T6 [30]

q
3
√ q
3

Reduce the number 2+ 5+ 2− 5.
q √ q √ √ q √ q √ √
Solution: Observe that ( 2 + 5+ 2 − 5)3 = (2+ 5)−3( 2 + 5)−3( 2 − 5)+(2− 5) =
3 3 3 3
q √ q √ q √ q √
4 − 3( 2 + 5 + 2 − 5). Hence 2 + 5 + 2 − 5 is a root of the cubic x3 + 3x − 4 =
3 3 3 3
q √ q √
(x − 1)(x2 + x + 4). The roots of x2 + x + 4 are imaginary, so 2 + 5 + 2 − 5 = 1.
3 3

Problem T7 [30]

1 n, for what positive integers n does an−1 = n2 ?


P∞
Let 1−x−x2 −x3
= i=0 an x

Solution: Multiplying both sides by 1−x−x2 −x3 the right hand side becomes a0 +(a1 −a0 )x+(a2 −
a1 −a0 )x2 +. . ., and setting coefficients of xn equal to each other we find that a0 = 1, a1 = 1, a2 = 2,
and an = an−1 + an−2 + an−3 for n ≥ 3. Thus the sequence of an ’s starts 1, 1, 2, 4, 7, 13, 24, 44, 81,
149, . . . . So we now see that a0 = 12 and a8 = 92 . What makes it impossible for this to happen again
is that the sequence is growing exponentially. It will suffice to show that an > 1.5n for n > 2, since
n2 /(n − 1)2 < 1.5 for n ≥ 6, thus when an−1 exceeds n2 at n = 10 there can be no more solutions
to an−1 = n2 . Observe that an > 1.5an−1 for n = 3, 4, 5. By way of induction, assume it for n − 2,

2
n−1, and n, then an+1 = an +an−1 +an−2 > 1.5n +1.5n−1 +1.5n−2 = 1.5n−2 (1+1.5+1.52 ) > 1.5n+1 .
Thus, by induction, an > 1.5n for n > 2, so the only solutions are 1, 9.

Problem T8 [35]

Find all the roots of (x2 + 3x + 2)(x2 − 7x + 12)(x2 − 2x − 1) + 24 = 0.

Solution: We re-factor as (x + 1)(x − 3)(x + 2)(x − 4)(x2 − 2x − 1) + 24, or (x2 − 2x − 3)(x2 −


2x − 8)(x2 − 2x − 1) + 24, and this becomes (y − 4)(y − 9)(y − 2) + 24 where y = (x − 1)2 . Now,
(y − 4)(y − 9)(y − 2) + 24
√= (y − 8)(y
√ − 6)(y − 1), so y is 1, 6, or 8. Thus the roots of the original
polynomial are 0, 2, 1 ± 6, 1 ± 2 2.

Problem T9 [40]

P17 n2 +n+1
Evaluate n=2 n4 +2n3 −n2 −2n .

Solution: Observe that the denominator n4 + 2n3 − n2 − 2n = n(n − 1)(n + 1)(n + 2). Thus we can
n2 −n+1 a b c d
rewrite the fraction as n4 +2n 3 −n2 −2n = n−1 + n + n+1 + n+2 for some real numbers a, b, c, and d. This

method is called partial fractions. Condensing the right hand side as a fraction over n4 +2n3 −n2 −2n
we get n2 − n + 1 = a(n3 + 3n2 + 2n) + b(n3 + 2n2 − n − 2) + c(n3 + n2 − 2n) + d(n3 − n). Comparing
coefficients of each power of n we get a+b+c+d = 0, 3a+2b+c = 2, 2a−b−2c−d = 2, and −2b = 2.
This is a system of 4 equations in 4 variables, and its solution is a = 1/2, b = −1/2, c = 1/2, and d =
−1/2. Thus the summation becomes 21 (1− 21 + 31 − 41 + 21 − 31 + 14 − 51 + 31 − 41 + 51 − 61 +· · ·+ 16
1 1
− 17 1
+ 18 1
− 19 ).
1 1 1 1 592
Notice that almost everything cancels to leave us with 2 (1 + 3 − 17 − 19 ) = 969 .

Problem T10 [45]

If 5 points are placed in the plane at lattice points (i.e. points (x, y) where x and y are both
integers) such that no three are collinear, then there are 10 triangles whose vertices are among
these points. What is the minimum possible number of these triangles that have area greater than
1/2?

Solution: By the pigeonhole principle, the 5 points cannot all be distinct modulo 2, so two of them
must have a midpoint that is also a lattice point. This midpoint is not one of the 5 since no 3
are collinear. Pick’s theorem states that the area of a polygon whose vertices are lattice points is
B/2 + I − 1 where B is the number of lattice points on the boundary and I is the number in the
interior. Thus those two points form the base of 3 triangles whose area will be greater than 1/2 by
Pick’s theorem since there are 4 lattice points on the boundary. Now it also turns out that at least
one of the triangles must contain a lattice point, thus giving us a fourth triangle with area greater
than 1/2. This is actually pretty easy to show with the aid of a picture or some visualization.
Suppose we have 4 points and we’re trying to find a 5th one so that no triangle will contain an
interior lattice point. The 4 lattice points must form a quadrilateral of area 1, so in fact it is a
parallelogram (think deeply about it). Draw the four sides, extending them throughout the plain.
Each vertex is now the tip of an infinite triangular region of the plane, and if the 5th lattice point
is chosen in that region then the triangle formed by the 5th point and the two vertices of the
parallelogram adjacent to the one we are considering will form a triangle containing the vertex we

3
are considering. But the part of the plane that isn’t in one of these 4 regions contains no lattice
points or else we could draw a parallelogram congruent to the first one with lattice point vertices
and containing that lattice point, but that would violate Pick’s theorem since the parallelogram has
area 1. Therefore we must have a fourth triangle with area greater than 1/2 (one must justify that
this really is in addition to the 3 triangles we already knew we’d get). An example that achieves
this minimum is the points (0,0), (1,0), (1,1), (2,1), and (2,-1). Therefore the minumum possible
number of these triangles that have area greater than 1/2 is 4.
A less trivial example that achieves the minimum is (0,0), (1,1), (2,1), (3,2), and (7,5).

Problem T11 [55]

Circles C1 , C2 , C3 have radius 1 and centers O, P, Q respectively. C1 and C2 intersect at A, C2 and


C3 intersect at B, C3 and C1 intersect at C, in such a way that 6 AP B = 60◦ , 6 BQC = 36◦ , and
6 COA = 72◦ . Find angle ABC (degrees).

Solution: Using a little trig, we have BC = 2 sin 18, AC = 2 sin 36, and AB = 2 sin 30 (see left
diagram). Call these a, b, and c, respectively. By the law of cosines, b2 = a2 + c2 − 2ac cos ABC,
2 2 30−sin2 36
therefore cos ABC = sin 18+sin
2 sin 18 sin 30 . In the right diagram below we let x = 2 sin 18 and see

−1+ 5
that x + x2 = 1, hence sin 18 = 4 . Using whatever trig identities you prefer you can find that

2 5− 5 1
sin 36 = 4 , and of course sin 30 = Now simplification yields sin2 18 + sin2 30 − sin2 36 = 0,
2.
so 6 ABC = 90◦ .
Note that this means that if a regular pentagon, hexagon, and decagon are inscribed in a circle,
then we can take one side from each and form a right triangle.

36

x
1

x 72 2
x
36
36 72
1
x

Problem T12 [65]

A fair coin is flipped every second and the results are recorded with 1 meaning heads and 0
meaning tails. What is the probability that the sequence 10101 occurs before the first occurance
of the sequence 010101?

Solution: Call it a win if we reach 10101, a loss if we reach 010101. Let x be the probability of
winning if the first flip is a 1, let y be the probability of winning if the first flip is a 0. Then the
probability of winning is (x + y)/2 since the first flip is 1 or 0, each with probability 1/2. If we ever
get two 1’s in a row, that is the same as starting with a 1 as far as the probability is concerned.
Similarly, if we get two 0’s in a row, then we might as well have started with a single 0.
start
1 0
11 10... 01 00...
101 100... 011...010
1010 1011... 0101 0100...
10101 10100... 01011... 01010
010101 010100...

4
From the tree of all possible sequences, shown above, in which the probability of moving along any
particular line is 1/2, we see that x = x(1/2 + 1/8) + y(1/4 + 1/16) + 1/16, and y = x(1/4 + 1/16) +
y(1/2 + 1/8 + 1/32). Solving these two equations in two unknowns we get x = 11/16 and y = 5/8.
Therefore the probablity that the sequence 10101 occurs before the first occurance of the sequence
010101 is 21/32.

5
Algebra Solutions
Ri e Mathemati s Tournament 2000

1. The only integers that satisfy jxj + 5 < 7 are the ones that satisfy jxj < 2 - namely,
1; 0; 1. The integers that satisfy jx 3j > 2 are 6; 7; 8; : : : and 0; 1; 2; : : :. So, the
integers that satisfy both are 0; 1, and there are 2 of them.
2. 20003 1999  20002 19992  2000 + 19993 an be fa tored into (2000 1999)20002 +
19992( 2000 + 1999), whi h redu es to 20002 19992 . This fa tors into (2000 + 1999)(2000 1999)
whi h is equal to 3999.
3. Let the s ores be a,b, ,d,e, where 0  a  b   d  e  100. So, the mean is
5 (a + b + + d + e); and the median is . So, we want to maximize 5 (a + b + + d + e)
1 1

. To do this, we must maximize d and e and minimize or maximize . One way to

do this is to let a = b = = 0 and d = e = 100, so the di eren e between the mean


and the median is 51 (0 + 0 + 0 + 100 + 100) 0) = 200 5 = 40. If we maximize , then
= d = e = 100, and then the mean is
5 (0 + 0 + 100 + 100 + 100) = 60, and the
1

median is 60, with a di eren e of 40 as well.


4. The pri e starts at $100. Clearly, the order of pri e hanges does not matter. It is
redu ed by 10% three times ( $100 ! $90 ! $81 ! $72:90), and the new pri e is
$72.90. It is in reased by 10% four times ($72:90 ! $80:19 ! $88:209 ! $97:0299 !
$106:73289) , and the new pri e is $106.73289. Rounded to the nearest ent, this is
$106:73.
5. Every day Edward works, he gets 19 of the test done. Similarly, every day Barbara
works, she gets 101 of the test done, every day Abhinav works, he gets 111 of the test
done, and every day Alex works, he gets 121 of the test done. So, after 4 days (after
everyone has worked on the test one day, they have ompleted 19 + 101 + 111 + 121 = 38:535%
of the test. After 8 days, they have ompleted twi e that, or 77:0707% of the test. After
Edward, Barbara, and Abhinav ea h work one more day, the test will be omplete in
the minimum amount of time, so the test will take 11 days to omplete. If the least
eÆ ient workers work after the 8th day, the test still takes 11 days to omplete.
6. A shortest path is ! 2 ! 4 ! 8 ! 12 ! 24 !
x x x x x x x
25
! x
50
! x
100
! x
200
!
x
400
! 800 ! 1600 ! 2000 , using 13 multipli ations.
x x x

7. All multipli atively perfe t numbers have exa tly 4 distin t positive divisors, or 1. So,
we must look for numbers that are either
1
 a produ t of two distin t primes
 a ube of a prime
Numbers satisfying one of these onditions less than 100 are: 1, 6, 8, 10, 14, 15, 21, 22,
26, 27, 33, 34, 35, 38, 39, 46, 51, 55, 57, 58, 62, 65, 69, 74, 77, 82, 85, 86, 87, 91, 93, 94,
95. There are 33 of these.
8. 168 = 23  3  7. There are only 2 ombinations of these whose sums allow indistinguisha-
bility of the ages. If there are 27 trees, 2, 4, 21 and 1, 12, 14 years are possible. If
there are 21 trees, 2, 7, 12 and 3, 4, 14 are possible. So, the possible ages of the oldest
daughter are 12, 14, 21.
9. We an view these onditions as a geometry diagram as seen pebelow. So, we know that
e
= 3
(sin e e = a b =
3 3
d =
3
f and we know that
2 + f 2 = 15 (sin e this is
pf 2 4 2 p 2 2 4
4 4
a + b + d ). Also, note that a + bd ad b = (a b)( d) = ef . So, solving

for e and f , we nd that e2 + f 2 = 225, so 16e2 + 16f 2 = 3600, so (4e)2 + (4f )2 = 3600,
so (3f )2 +(4f )2 = 3600, so f 2 (32 + 42 ) = 3600, so 25f 2 = 3600, so f 2 = 144 and f = 12.
Thus, e = 43 12 = 9. Therefore, ef = 9  12 = 108.

e
f
a
b

10. a = 1 learly does not work, sin e if x = 1, then x4 + a2 = 2, whi h is prime. a = 2


learly does not work, sin e if x = 1, then x4 + a2 = 5, whi h is also prime. Here is a
table for a's and values of x that show they do not work:
a + x
4 2
a x

3 10 10009
4 1 17
5 2 41
6 1 37
7 20 160049
So, let us onsider a = 8 - i.e. the sum x4 + 64. This is the same as (x2 + 8)2 16x2 =
(x2 + 4x + 8)(x2 4x + 8) by the di eren e of squares. This is learly not prime for
any integer x. So, the answer is a = 8.
Cal ulus Solutions
Ri e Mathemati s Tournament 2000

1. y = x3 3x2 + 6x + 2000, so y 0 = 3x2 6x + 6 and y 00 = 6x 6, so the point of in e tion


is the solution to 6x 6 = 0, or x = 1. At x = 1, the slope is f 0 jx=1 = 3(1)2 6(1) + 6 =
3.

2. The hange in Karen's position is x x3 . The optimal length to limb is at a p riti al


point. The only realisti riti al point is at the solution to 1 3x = 0 or x = 33 .
2

3.
R 1

sin x + C = 0, from the
4
statement of the problem. So, [ os x + Cx℄j04 = 0. p

Thus,
0

p 2 2 4
 
os 4 + 4 C + 1 = 0. So, 2 + 4 C + 1 = 0, and solving for C , we nd that C =  .
2

4. Let y = tan x. So, we want to nd the minimum of y + y1 , where 0  y  1. Taking the


derivative and minimizing, we nd that the minimum o urs at y = 1, so the minimum
of the given fun tion o urs at ar tan 1 = 4 .
P i P
5. f (x) = 1 x . So, f 0 (x) = 1 xi = 1 . Thus, f (x) = ln(1 x).
i=1 i i=0 1 x
6. Assume the pipe barely ts around the orner (i.e. it is in p onta t with the orner).
The lower orner is at (0; 0) and the upper orner is at (6; 6 5). Call x0 the point on
p r longestpa pipep ould
the lower wall it hits at the tightest spot. Given an x0 , the
2
be with
one end at x0 and leaning against the (6; 6 5) orner is x20 + (6 5 + 36 x0 6 ) . We want
5

the minimum of all of these "longest pipes", be ause the pipe needs to t at all angles
around the orner. Taking the derivative (without the square root for simpli ity) and
setting it equal to 0, we need to solve x30 6x20 + 36x0 1296 = 0. Wep an qui kly nd
that x0 = 12 is the only good solution, so the maximum length is 12 6.
dy
7. Using the hint, take dx . Set this equal to 0 and solve for x relative to x0 . Plug this in
for x0 in the given family of lines to obtain the envelope y = x + x1 ; x > 0.
8. Let I denote the given integral. Under the transformation  !  , I transforms to
R 2 ln( os())d. So, 2

0
R
2I R= 02 ln(sin  os )d
= 0 (ln(sin 2R) ln 2)d(2)=2
= 2 ln 2 + 21 0 sin( )d giving I  ln 2.
pi R 
= 2  ln 2 + 0 sin( )d
2
= 2

= pi2  ln 2 + I
9. Note rst that ([f (x)℄2 x)2 = f (x), so if f (a) = 4, then (16 a)2 = 4, so a = 14.
f 0 (x)
0
Now, f (x) =
1+
2([f (x)℄2 x)
, so f 0 (14) = 4 .
2f (x) 31
10. Solution: Throughout this solution we will use the fa t that when light boun es o a
mirror, the angle of in iden e is equal to the angle of re e tion. First the beam hits
the point (8,-1), then (6,1), (4,-1), (2,1), and then is travelling
p5 1 p5 along the line p y = x 1.
Thus the beam hits the parabola at the ppoint (1 + 2 ; 2 ).p To estimate 5, noti e
1
p
that 222 = 484 and 232 = 529, so 5 = 10 500
= 2:2 : : :. Thus 1 2 5 = :6 : : :, so the light
hits the parabola at approximately (.4,-.6). The slope of the tangent to the parabola
at this point is 21 (:4) 1=2 , whi h is about -.8, so we need to nd the slope of the beam
after it re e ts o of this tangent. For purposes of nding this slope, hange oordinates
so that the point of interse tion is the origin. The beam is oming in along y = x, and
y = 1:2x is perpendi ular to the tangent. The diagram below should larify the setup.

perpendicular to
tangent
reflected
beam

y=x

tangent

We will nd the new path of the light by nding the re e tion about the line y = 1:2x
of a point on its in oming path. We know the point (1,1.2) is on the line y = 1:2x, so a
perpendi ular through this point is y 1:2 = :8(x 1), whi h interse ts y = x at the
point (1.1,1.1). Thus the new path goes through the point (.9,1.3), so it has slope 1.4
(all values rounded to one de imal pla e). Going ba k to our original oordinate system,
the light is now travelling along the line y + :6 = 1:4(x :4), so it next hits the mirror
at (1.5,1). After that the x oordinate in reases by 2=1:4 = 1:4 between boun es, so it
hits (2.9,-1), (4.3,1), (5.7,-1), (7.1,1), (8.5,-1), and nally (9.9,1). A loser examination
of the approximations made (e.g. by re ning them to two de imal pla es) reveals that
the last boun e is a tually further to the left (at (9.21,1), to be more pre ise), so indeed
the light does boun e 12 times.
Geometry Solutions
Ri e Mathemati s Tournament 2000

1. Consider the board labeled as below, with labels for olumns and rows. To hoose any
re tangle on the board, it is suÆ ient to hoose some number (1-8) of adja ent olumns,
and some number (1-8) of adja ent rows, sin e the re tangle an be reated by forming
the interse tion of the olumns and rows. For instan e, the interse tion of olumns 2,3
and rows 3,4,5 is the re tangle shaded below. So, there are 8 ways to hoose 1 adja ent
olumn, 7 ways to hoose 2 adja ent olumns, : : :, 1 way to hoose 8 adja ent olumns,
so there are 8 + 7 + 6 + 5 + 4 + 3 + 2 + 1 = 36 total ways to hoose the olumns, and
36 ways to hoose the rows. Thus, the total number of ways to hoose a re tangle (i.e.
the total number of re tangles) is 362 = 1296.
8

1 2 3 4 5 6 7 8

2. The maximum o urs in an equilateral triangle,


2 2 2 2 p in whi h ase the sides aap=3 b are
given by a + b + = 3a = 96, so a = 4 2. Thus, the medians are 3  ( 2 ) =6 6.
p
3. Let s be the side length of ABCD. Sin e ABCD is a square, we an write 2s2 =
(P A)2 + (P C )2 = (P B )2 + (P C )2 . So, we an substitute in for P A,P C , and P D to get
that P B =24.
4. Noti e that in general, when there is a re tangle of side length x and y , the area of
the non-triangle regions ( reated by drawing a line onne ting the midpoint of two
opposite lines and a line onne ting two opposite orners - see diagram for examples)
is simply 43 of the original area of the box, sin e the area of the ex luded triangles are
2  2  2 + 2  2  2 = 2  2 = 4 , so the desired area is simply ab 4 = 4 ab, as desired.
1 y x 1 y x xy xy ab 3
So, if we onsider the four re tangles making up the large re tangle that are divided
this way (the one with dimensions a4 x 3b , et .), we an say that the total shaded area is
3 3
4 of the total area of the re tangle - that is, 4 ab.
5. The area of 4ABF = 12 bh = 12  3h = 23 (3 sin 60 Æ) = 29 sin 60 Æ. The area of 4F CDE =
area of 4ABF - area of 4F GC = 92 sin 60 Æ 12 sin 60 Æ = 4 sin 60 Æ. Therefore, areapof
ABCDE = area of 4ABF + area of F CDE = 92 sin 60 Æ + 4 sin 60 Æ = 172 sin 60 Æ = 174 3 .
p
6. Theparea of an equilateralptriangle with side length s is 43 s2 . Therefore, the side length
is 4 2 and height h = 2 6. Now, if r is the radius of the ins ribed ir le, then r = h3 ,
sin e we have an equilateral triangle. Thus the area is r2 = 8 . 3
7. Let's "resize" the oordinates to be x = x; y = 23y . This keeps the origin at (0,0), but
0 0

turns our ellipse into a ir le of radius 2. Thus now, the triangle is equilateral, and we
A

h1
G
h2
F
b C
2
B

E D

p
an see it now has area 3 3. On e we p expand ba k, we an see we are just multiplying
3
the area by 2 and so the answer is 2 .9 3

8. By standard formula, we have that the radius of the ins ibed ir le, r, is r = ab2asin +b
A

2 2 2
q 2 verti ally
(iso eles triangle that is formed by utting the pyramid in half ( uts the base
into 2 equal re tangles)). h + ( 2 ) = a gives a = h + 4 . Also sin A = ha . Therefore,
b b 2

r = q bh 2 . Note that the diameter of the ube is the diameter of the sphere. Let l
2 h2 + b4 +b
p
be the length of the side of the ube, so the diameter of the ube is l 3 = 2r, so l = p2r3 .
p
So, the volume of the pyramid is 13 b2 h and the ube volume if l3 . So, the ratio is 256 3 .
9. A hexagon an be formed by removing any vertex, removing all verti es onne ted to
that vertex, and then removing any edges onne ted to any of the removed verti es,
and these are the only hexagons in the diagram. Thus, sin e there are 10 verti es, there
are 10 hexagons in the gure below.

10. Consider the attened version of the situation. Then let O be the enter fo the spheres,
A be the rst re e tion point,pB be the point of C1 su h that OB is perpendi ular to OP .
Then sin e OB = 1,OP = 3, 6 P BO = 60 Æ and P; B; A are ollinear, 6 BAO = 60 Æ
implies that 6 P OA = 30 Æ. Therefore, ea h re e tion takes the ray 121 around the ir le,
so there are 11 re e tions.
General Solutions
Ri e Mathemati s Tournament 2000

1. Sin e d = a , substitute into the equation b = 2 + d to get b = 2 + a = a + .


Also, substitute into 2 = d + a 1 to get 2 = a + a 1, or 3 = 2a 1. Now, sin e
b = a + , we an substitute into a = 2b + to get a = 2a +2 + , or a = 3 . Sin e we
know 3 = 2a 1 from above, we substitute in to get 3 = 6 1, or = . Thus, 1

we nd that a = , d = , and b = .
9
1 4 2
3 9 9

2. Let x be the temperature we are looking for, so x = + 32. So, x = 32, so


9 4

x=  32 = 40.
5 5
5
4

3. Let x be the length of Henry's shadow in feet. Using similar triangles, we nd that
: = x , so x = :  5 =
55
12 5
55
12
11
24
5= . 55
24

4. Let x be the number of students and y be the number of non-students. We then


have the equations x + y = 3000 and 10x + 15y = 36250. Substituting, we nd that
10(3000 y) + 15y = 36250, or 30000 10y + 15y = 36250, so 30000 + 5y = 36250. So,
5y = 6250, and y = 1250, so x =1750.
5. The total number of degrees in an o tagon is (8 2)  180 = 1080. Sin e the degrees are
evenly distributed among the angles, the measure of one interior angle is =135 Æ.
1080
8

6. Pi k any ard rst, then pi k the other fa e-down ard.


(a) 1
3

(b) 2

p
3

7. 19992000  4471:241, and [4471:241℄ =4471.


8. The time that Bobo an juggle is the number of ows times se onds. So, we get the
following table:
ows started juggling 1 2 3 4 5 6 7 8 9 10 11
total time 64 110 141 160 165 162 154 144 126 130 132
ows started juggling 12 13 14 15 16 17 18 19 20 21 22
total time 132 130 126 120 112 102 90 76 60 42 22
Thus, we see that the maximum o urs with 5 ows, and the total time is 165 se onds
= 2 minutes.
3
4

9. Let p be the pri e of HMMT. So p = k  xy , where k is a onstant to be determined. We


know that when x = 8 and y = 4 that p = 12, so, solving for k, we nd that k = 6. So,
when x = 4 and y = 8, we nd that p = 6  =3.
4
8

10. On the 12 foot sides, he needs 7 posts, and on the 20 foot sides, he needs 9 posts, so he
needs 7 + 9 + 7 + 9 = 32 total posts. Let x be the number of normal fen eposts and y
be the number of strong fen eposts, so x + y = 32. To spend $70, we have the equation
2x +3y = 70. Substituting, we nd that 2(32 y)+3y = 70, so 64 2y +3y = 70, and
64 + y = 70, so y =6.
11. Substituting, we nd that nn = 3, so 3 + n = 9 3n, thus 4n = 6. So, n = .
3+
3
3
2
12. Yes - one possible path is 4 ! 1 ! 2 ! 4 ! 5 ! 2 ! 3 ! 5, so the di eren e between
the start island and end island is 1.
13. The number of rearrangements keeping 1 number in its spot and rearranging the other
5 su h that none are in the right spot is 44. There are 6 numbers to x, this gives us
an answer of 44  6 =264.
14. Feburary 26, 2000 is a Saturday. April 24, 2000 is 23  365+1+1+1+1+1+1 = 8401
days away from April 24, 1977 (in luding the leap years). So, Feburary 26, 2000 is
8401 3 31 24 = 8343 days after April 24, 1977. Now, = 1191 with a remainder 8343

of 6. So, 6 days before Saturday is Sunday.


7

15. Noti e that 3 = 729 while 5 = 625, and sin e ln5 > ln3, it follows that 5 ln3 < 3 ln5,
6 4 4 6

so 5  5 ln3 < 3  3 ln5, and so by laws of logarithms, 5 ln3 < 3 ln5 . Again applying
3 5 3 5 5 3

laws of logarithms, it follows that ln(3 ) < ln(5 ) . So, sin e ln x is5 an in reasing
5 (53 ) 3 (35 )

fun tion, it follows that (3 ) < (5 ) , and so it follows that (5 ) is greater.


3
5 (5 )
5
3 (3 ) 3 (3 )

One an also use the laws of exponents to redu e the values to 3 and 5 . The (54 ) (36 )

se ond is learly larger.


16. Expressing the total time Joe has biked in hours leads to the equation x + x + x = 1. +2

So, x = 5. Thus, we an onstru t the diagram below, and nd the total time that it
20 20 14

takes to get ba k: 13  = hours, or 10 minutes.


1
78
1
6

5 mi. Friend’s house

5 mi.
13 mi.

shore 7 mi. Gma’s house


12 mi.

17. There are 7! ways to arrange those letters. However, for every distin t arrangement,
there are 2!  2! = 4 total arrangements of the 2 T's and 2 N's. Therefore, the total
number of distin t ways to arrange the letters is = 1260. 7!
2!2!

18. The only digits possible are 4, 6, 8, and 9. The only groups of numbers allowed keeping
the average at 5 are 8444 are 6464. There are 4 ways to arrange 8444 and 6 ways to
arrange 6464 so there are only 10 ombinations to try.
19. Let y be the number of oins in the hest. From the problem, we know that y  5
(mod 11), y  3 (mod 10), and y  0 (mod 9). Combining these gives us that
y  423 (mod 990), so the answer is 423.
20. The area of the big ir le is ( )  = . The area of the ir le with diameter AB is
5 2 25

 , and the area of the ir le with diameter BC is  . Thus, the per entage of the big
2 4
9

= = 48%.
4

ir le that is shaded is    =
25 9
4 4 25 4 9 12
25 25 25
4

21. It is lear that we an split the gure into 12 equilateral triangles,


p all of whi h havepside
length s. So, sin e the area of one of these triangles is , the total area is 3s 3.
s 2
4
3 2
p p p
22. Aiso = A sopA = Aeq Aiso = Aeq . Also, Aeq =  = 4 3, so A = 2 3. Thus,
eq 1 42 3

the area is 2 3.
2 2 4

23. Noti e that 7 k  3 (mod 4) for k 2 Z (k is an integer). Also, 7  7 (mod 100),


2 +1 1

7  49 (mod 100), 7  43 (mod 100), 7  1 (mod 100), with the y le repeating


2 3 4

afterwards. So, learly 7 7  3 (mod 4), and sin e the y le has period 4 (mod 100),
7
77

we an on lude that 7 7  43 (mod 100).


7

24. If there are no empty boxes, there are 126 ways of distributing the identi al andy.
If there is one empty box, there are 84 ways of distributing the andy and 5 ways of
hoosing the empty box, so there are 84  5 = 420 ways. If there are two empty boxes,
there are 36 ways of distributing the andy and 6 ombinations of assigning the empty
boxes so that they are not adja ent, so there are 36  6 = 216 ways. If there are three
empty boxes, there are 9 ways of distributing the andy, and only 1 way of arranging
the empty boxes. Having four or ve empty boxes is impossible, sin e some two would
have to be adja ent. So, the total number of ways is 126 + 420 + 216 + 9 = 771.
25. We an pla e 9 points as shown, all at least unit apart, but the next point must be
1

less than , so 10 points must be pla ed. There is no arrangement of 10 points with
2
1

distan e at least . The proof of this is a simple appli ation of the Pigeonhole Prin iple.
2
1
2

p
26. Janet is at (5; 5) and Tim is at (7; 4). They are 85 miles apart.
Advan ed Topi s Solutions
Ri e Mathemati s Tournament 2000

1. Assume we have 4 olors - 1, 2, 3, and 4. Fix the bottom as olor 1. On the remaining
sides you an have olors 2, 3, 4 (in that order), or 2, 4, 3, whi h are not rotationally
identi al. So, there are 2 ways to olor it.
p
2. Sin e os 23 = 21 and sin 23 = p23 , we an write the rst term as ( os 23 + i sin 23 )6 .
Sin e os 43 = 12 and sin 43 = 23 , we an write the se ond term as ( os 43 + i sin 43 )6 .
Now, we apply DeMoivre's Theorem to simplify the rst expression to ( os 6  23 + sin 6  23 ) =
( os 4 + sin 4 ) = 1+0 = 1. Similarly, we simplify the se ond expression to ( os 6  43 + sin 6  43 ) =
( os 8 + sin 8 ) = 1 + 0 = 1. Thus, the total sum is 1 + 1 = 2.
1 1
3. We know that n2 +2 n n+2 . So, if we sum this from 1 to 1, all terms ex ept
n = n(n+2) =
1 1
2
1 1
for 21 + 22 will an el out (a "teles oping" series). Therefore, the sum will be 43 .
4. The possibilities for the numbers are:
 all ve are divisible by 3
 three are divisible by 3, one is  1 (mod 3) and one is  2 (mod 3)
 two are divisible by 3, and the other three are either  1 (mod 3) or  2 (mod 3)
 one is divisible by 3, two are  1 (mod 3) and two are  2 (mod 3)
 four are  1 (mod 3) and one is  2 (mod 3)
 four are  2 (mod 3) and one is  1 (mod 3)
 
This gives us 1001 possible ombinations out of 15
5
or 3003. So, the probability is
1001
3003
= 13 .
5. 153,370,371,407
6. There are 6 people that ould get their hat ba k, so we must multiply 6 by the number
of ways that the other 5 people an arrange their hats su h that no one gets his/her hat
ba k. So, the number of ways this will happen is (6  derangement of 5), or 6  44 = 264.
Sin e there are 6! = 720 possible arrangements of hats, the probability of exa tly one
person getting their hat ba k is 720
264
= 11
30 .
7. We an view these onditions as a geometry diagram as seen pebelow. So, we know that
e = 3 (sin e e = a b = 3
3
d = 3
f and we know that + f = 15 (sin e this is
pf 2 4 2 p 2 2
2 2
4 4 4

a + b + d ). Also, note that a + bd ad b = (a b)( d) = ef . So, solving


for e and f , we nd that e2 + f 2 = 225, so 16e2 + 16f 2 = 3600, so (4e)2 + (4f )2 = 3600,
so (3f )2 +(4f )2 = 3600, so f 2 (32 + 42 ) = 3600, so 25f 2 = 3600, so f 2 = 144 and f = 12.
Thus, e = 43  12 = 9. Therefore, ef = 9  12 = 108.
e
f
a
b

8. It suÆ es to onsider the omplements of the graphs, so we are looking for graphs with
9 verti es, where ea h vertex is onne ted to 2 others. There are 4 di erent graphs -
see below.

9. The probability of the  Reals hitting 0 singles is ( 23 )3 . The probability of the Reals hitting
exa tly 1 single is 32 ( 32 )3  13 , sin e there are 3 spots to put the two outs (the last spot
m ust be an out, sin e the inning  has to end on an out). The probability of the Reals
hitting exa tly 2 singles is 2 ( 3 )  ( 31 )3 . The probability of the Reals hitting exa tly
4 2 3

3 singles is 52 ( 23 )3  ( 31 )3 . If any of these happen, the Alphas win right away. Adding
these
 gives us a 729 han e of this happening. If exa tly 4 singles o ur (with probability
656

6
2
( 3 )  ( 3 ) ), then there is a 52 han e that the Alphas win. The probability of this
2 3 1 4

happening is 25  729 40
. Thus, the total probability of the Alphas winning is the sum of
224
these two probabilities, or 656 729
+ 729
16
= 243 .
10. A will say yes when B says no to n 1 or n, as A will then know B's number is one
greater than A's number. Thus, A responds rst, after n 2 1 "no" responses if n is
odd, after n2 "no" responses if n is even.
Guts Solutions
HMMT 2000

1. π e /3
XY
2. X+Y −1

3. 57

4. (1,2)

5. 1, 512, 4913, 5832

6. 3
2
7. ( x 2+1 )m .

8. 14

9. 15
√ q
R2
10. πR R2 + H 2 + 2πr2 (1 + 1 − r2
)

11. 58

12. 274

13. 3

14. 51.

15. 249 .

16. (-1,3) and (3,-1)

17. 1 = 30

18. −π/4

19. 1
√ √ √
20. 3 + 2 2 + 3 + 6

21. 546

22. 2008

23. 224640
24. 6

25. 1113122113111221

26. 313

27. 325

28. 1/6

29. 3

30. 1

31. D is harmonic conjugate of C, standard construction.

32.

33. Of the form 4n + 2.

34.

35. 2/3

36. 90 degrees

37.
8
88
33
38. 3

39. 254.50000... (more than 100 zeros).

40. n any odd number, p = 2


q
1
41. h = 4b
(a + b)(3b − a)

42. n(n2 + 1)/2

43. 120/1147

44. f(0) = -1, f(1) = 1

45. 1999000, 2000

46. 1,2

47.
Oral Round Solutions
HMMT 2000

1. The equation involves only even powers, so we may assume m, n are positive (negative
solutions may be generated by switching signs). n6 ≤ n6 + 1 ≤ n6 + 2n3 + 1, so n6 + 1
lies between two squares, with strict inequality unless n = 0. So the only solutions are
m = ±1, n = 0.
2. Dividing by x6 , we get the equation x4 + 7x3 + 14x2 + 1729x − 1379 = 0. This is a
monotonically increasing function of x on [0, ∞). From f (0) < 0 and f (x) → ∞ as
x → ∞ we see that there’s only one positive solution. In fact, f (1) > 0, so there’s no
positive integer solution.
3. Assume w.lo.g that c > b ≥ a. Then cn − bn = (c − b)(cn−1 + ... + bn−1 ) > 1(nbn−1 ) ≥
nan−1 . So an > nan−1 , hence a > n.
4. Let m be the smallest number, written on some square S. Then clearly all of the
adjacent squares to S must have m on them (they can’t have anything smaller, since m
is the smallest, and can’t have anything larger since m is the average of those adjacent
numbers). Since this applies to every square on which m is written, all of the squares
must have m on them. Otherwise there will be some “boundary” square with m on it,
which will not have m written on one of its neighbours, a contradiction.
5. Suppose there
q is such a triangle with vertices at (m1 , n1 ), (m2 , n2 ), (m3 , n3 ). Then we
are given (m1 − m2 )2 + (n1 − n2 )2 is an odd integer, so that (m1 − m2 )2 + (n1 − n2 )2
is 1 (mod 4). Hence exactly one of m1 − m2 and n1 − n2 must be odd and the other
must be even. Suppose w.lo.g m1 − m2 is odd and n1 − n2 is even. Similarly, exactly
one of m1 − m3 and n1 − n3 must be odd and the other must be even. If m1 − m3 is odd
and n1 − n3 even, then m2 − m3 and n2 − n3 are both even, which is a contradiction
sice we want (m2 − m3 )2 + (n2 − n3 )2 to be 1 (mod 4). Similarly if m1 − m3 is even and
n1 − n3 is odd then m2 − m3 and n2 − n3 are both odd, and (m2 − m3 )2 + (n2 − n3 )2 is
2 (mod 4), a contradiction.
Alternate solution: By Pick’s theorem (A = I + 21 B − 1), the area is an integer or a

half-integer. But by Hero’s formula, the area is 14 product of 4 odd integers. this is a
contradiction.
6. Any multiple of 3 can be written as 6k or 6k + 15. Now, 6k = (k + 1)3 + (k − 1)3 +
(−k)3 + (−k)3 . 6k + 15 = (k + 2)3 + (−k + 2)3 + (2k − 1)3 + (−2k)3 .

1
7. If the tetrahedron is oriented so that two opposite edges of the tetrahedron are parallel
to the ground, it is clear from symmetry that the plane parallel to the ground and
passing through the center of the tetrahedron splits it into equal spaces of volume
1/2. Let’s call this position 1. If, instead, it is oriented with the point down and the
top face parallel to the ground, the plane through the center splits it into regions of
volume 27/64 and 37/64. To see this, note that the center of a regular tetrahedron is
3/4 of the way down the altitude from each vertex. Hence the tetrahedral section has
volume (3/4)3 = 27/64. Let’s call this position position 2. If the tetrahedron is turned
continuously from position 1 to position 2, because 27/64 < 7/16 < 1/2, it must pass
through an orientation s.t. the fraction of volume below the center is 7/16.
The cube has 180◦ rotational symmetry, so any plane through the center splits it into
sections of equal volume. Hence it is impossible to get a volume of 7/16.
8. Let f be one such polynomial, for example, the obvious one: f (x) = x2 + 1. Then all
such polynomials must be of the form f (x) + g(x) where g(x) = 0 at x = 1, 2, ..., n. g(x)
must of the form C(x−1)(x−2)...(x−n) and since f +g has to have integer coefficients,
C must be an integer. At 0 this gives us (f + g)(0) = f (0) + g(0) = 1 + C(−1)n n!. Since
C can be any positive or negative integer, the answer is that f(0) can be any integer
congruent to 1(mod n!), i.e. of the form 1 + Cn!, C an integer.
9. This is a special case of the problem with n + 2 vectors in n dimensions. First it is
clear that we can take all the vectors to be of length 1. Then we induct on n. The first
case is n=1. Here the statement is that given a, b, c real numbers, the at least one of
ab, bc, and ac is nonnegative. Without loss of generality, we can assume that a and b
are of the same sign, but then ab ≥ 0. Now assume the statement is false for the n-
dimensional case. Choose some vector, say − v− →
n+2 , and project the other vectors onto the


space perpendicular to − v− → 0 →
− → −−→ −−→

n+2 to get vi = vi − ( vi · vn+2 )vn+2 . This is essentially taking
out the space parallel to − v−→
n+2 and reducing the problem by one dimension. The only
→ −
− →
thing left to check is that if − →
vi · −

vj < 0 for all i,j then vi0 · vj0 < 0 for all i,j. This is just a
→ −
− → → −
calculation: vi0 · vj0 = −
vi · →
vj −2(− →
vj · −
v−→ − → −−→ −−→ −−→ − → −−→ − → −−→
n+2 )( vi · vn+2 )+(vn+2 · vn+2 )( vj · vn+2 )( vi · vn+2 ) and
since (−
v−→ −−→ → −
− → − → −−→ − → −−→ − → −−→ → −−→

n+2 · vn+2 ) = 1, this is just vi · vj − ( vj · vn+2 )( vi · vn+2 ). ( vj · vn+2 ) and ( vi · vn+2 )
→0 −
− →0
are both negative by assumption so their product is positive and v · v < − i j

vi · −→
vj < 0.
10. Let ω be a 23rd root of unity, a solution to the equation x23 = 1. We can encode the
state of the game as 23 rational numbers a0 , ..., a22 representing the amount of water
each person has, with Alex having the frction a0 of the water and continuing to his
right. Then we can look at the polynomial f (x) = a22 x22 + ... + a0 which still encodes
the state of the game. In this schema the original state is the constant 1, and we have
at all times the condition f (1) = 1 (conservation of water). However, we want a cyclic
representation in order to encode the play of the game, so we let the state of the game
be f (ω) = a22 ω 22 + ... + a0 . In this representation, a step in the game can be encoded
as multiplication by another polynomial in ω, g(ω) = b22 ω 22 + ... + b0 where the bi
are the fraction of water that Alex initially gives to the i-th person on his right. The
example in the problem is represented by 61 ω + 13 + 12 ω 22 . So we would like to know
which g are solutions to g(ω)23 = 1. We can take 23rd roots of both sides and find
that g(ω) = b22 ω 22 + ... + b0 = ω k for some integer k. To simplify this we need a small
lemma.

2
Lemma: If a22 ω 22 + ... + a0 is some rational linear combination of the powers of ω and
is equal to 0, then a22 = a21 = ... = a0 .
Proof: Assume this is false. So there exists some polynomial A(x) = a22 x22 + ... + a0
with ω as a root that is not a constant multiple of B(x) = x22 + x21 + ... + 1. Now apply
Euclid’s algorithm to find the G.C.D. of A(x) and B(x) and call it D(x). But D(x)
is a rational polynomial that divides B(x), which has no non-trivial rational divisors.
Hence D(x) must be either a constant or of degree 22. Since A(ω) = B(ω) = 0 all
the terms in the algorithm have a root at ω and so does D(x). So since 0 is not an
acceptable value for a G.C.D., D(x) must have degree 22. Since the degree of each
successive term in Euclid’s algorithm decreases by at least one, this means that D(x)
must be A(x) and hence that B(x) is a constant multiple of A(x). This contradicts the
original assumption and the Lemma is proved.
So the Lemma tells us that b22 = b21 = ... = bk − 1 = ... = b0 . Since we already know
that g(1) = 1 and hence b22 + b21 + ... + b0 = 1, and that all the bi are nonnegative,
we get that bi = 0 for i 6= k and bk = 1. Since this is a solution for all k, the possible
schemes for Alex to use are just those involving giving all his water to any one other
person.

3
Team Test Solutions
Ri e Mathemati s Tournament 2000

1. .6445 rounds to .645 to .65 to .7. Otherwise .6444... rounds to .644. So the smallest
number is .6445.
2. Let =pri e, p=purity, d =diameter,
p h=depth of gold4mine, ki= onstant. We are given
2
= k p d , p = k h 2 , and d = k 3 h. So, = k k h 4 k h = k h . Thus, k = h,
1
2 3
2 3 1
2
2
3
3
3
4
5 1
4
4

1
and = k h 4 . Thus, p varies as h 4 .
1
5

3. The sum of the numbers from 700 to 799 is   = 74950. The sum of 799 800 699 700

the numbers from 70 to 79 is   = 745. So, all numbers that end from 2 2
79 80 69 70
2 2

70 to 79 (ex luding those starting with 7, sin e we ounted those from 700 to 799) is
745  9+10(100 + 200 + : : : + 600 + 800 + 900) = 44705. The sum of all numbers ending
in 7 is 9(7+17+27+37+47+57+67+87+97)+9(100+200+: : : +600+800+900) = 38187.
So, the total sum of numbers ontaining a 7 is 74950 + 44705 + 38186 = 157842.
4. 738,826. This an be arrived at by stepping down, starting with nding how many
ombinations are there that begin with a letter other than V or W, and so forth. The
answer is  +  + 4  6! + 4  4! + 3! + 2! + 2! = 738826.
8 9!
2 2
4 7!
2

5. 0 = os ( + ) + sin ( ) = os  os sin  sin + sin  os sin os =


( os + sin )( os sin ). So os +sin = 0 or os sin = 0. Then tan = 1
or tan = 1. Sin e tan is given as , tan = 1. 1
2000

6. Sin e 1 = 0, then = + . So, we an redu e our expression to


3 10 8 7

3 8
3 3 + 4 + 2
6 5
4 6 17. Also, 3
4
3 3 = 0, so our
3 2 8 6 5

expression redu es to 4 + 2 4 6 17. Also, 4 4


4 4 , so our
3 2 4 2 0

expression redu es to 2 2 17. Now, 2 2 2 = 0, so our expression redu es


3 3

to -15, whi h is our answer.


7. Another 4-digit number that satis es this property is 9801, sin e 9801=9*1089.
8. If she has a silver dollar, then she would have too many other oins, as 0 half dollars,
2 quarters, 3 dimes, et . would be greater than the total. So she has no silver dollars,
and at least one of every other denomination. Continuing, it turns out the only feasible
solution is 0 silver dollars, 1 half dollar, 2 quarters, 3 dimes, 4 ni kels, 8 pennies, for a
total of 18 oins.
9. It suÆ es to onsider x  1, sin e 4( x) + 1 = 4(x) + 1, and 4(0) + 1 = 1 is not prime.
4 4

So, 4x +1 = (4x + 4x + 1) 4x = (2x + 1) (2x) = (2x + 1 2x)(2x + 1 + 2x).


4 4 2 2 2 2 2 2 2

For integers x, both 2x 2x + 1 and 2x + 2x + 1 are integers, so this fa tors 4x + 1


2 2 4

unless 2x 2x + 1 = 1 or 2x + 2x + 1 = 1. Sin e x > 0, then 2x + 2x + 1 > 1, so


2 2 2

we must have 2x 2x + 1 = 1. 2x 2x + 1 = 1 is absurd (4x + 1; 2x + 2x + 1 > 0,


2 2 4 2

4
so 2x 2
2x + 1 = x2x x > 0), so we solve 2x
2
4
2x + 1 = 1, or 2x
+1
+2 +1
2x = 0, so 2 2

x(x 1) = 0, and x = 0 or x = 1. We have already reje ted x = 0, so the only ase left
is x = 1, or 4(1) + 1 = 5. 4

10. The se ond hand rosses the minute hand 59 times an hour. The se ond hand rosses the
hour hand 60 times an hour, ex ept for 2 of the hours, due to the movement of the hour
hand. The minute hand and the hour hand ross 22 times total, be ause the hour hand
ompletes 2 rotations in a day, and the minute hand ompletes 24. The se ond, hour,
and minute hand all oin ide only at noon and midnight, but we've ounted ea h of
these 12:00's 3 times instead of on e. Therefore, the answer is 59  60+60  24 2+22 2  2,
giving us 2872 rossings.
11. f (x) is either 0 or something of the form xm, where m  0.
12. A's position is (aq Va t; 0) and P 's position is (0; b Vb t). So, at time t, the distan e
between them is (a Va t) + (b Vb t) . Noti e that this distan e is the same as the
2 2

distan e between the point (a; b) and the line (Va t; Vb t), whi h is the same as the line
Vb x Va y = 0. The distan e from a line Ax + By + C = 0 and (x ; y ) is jAxp0A2ByB0 2 C j , + +

pVa +V
0 0

so the answer is jbVa 2 aVb2j


+

b
13. Given any 4 verti es, there
n is exa tly one interse tion of all the diagonals onne ting
them. So, the answer is 4 .
14. x if n  0 (mod 4),
0
1+
1
x0
x0 if n  1 (mod 4), 1
x0 if n  2 (mod 4), x0 1
x0 +1 if n  3 (mod
4).
15. Consider a regular n-gon with radius r. Let x be the side length of the n-gon. So,
sin e the entral angle is n (see diagram below), use the Law of Cosines q
p to nd that
2

x = r + r 2r  r os n , so x = 2r (1 os n ).q Thus, x = r 2 1 os n .
2 2 2  2 2 2 2 2

p
So, the total perimeter of the n-gon is nx = nr 2 1 os n . Now, if we take 2

limn!1 of thep perimeter,


q the result will be 2n, sin eqthe n-gon approa hes a irle, p
so limn!1 nr 2 1 os n = 2r, and so limn!1 nr 1 os 2n = r 2.
 2

r r

n
Power Test Solutions
Ri e Mathemati s Tournament 2000

1. (a) D4 = 9; D5 = 44
(b) This is D =1854 7

( ) Dn = (n 1)(Dn 1 + Dn 2 )
(d) Dn = n  Dn 1 + ( 1)n
2. 10  9  8 give the number of all di erent 3 letter words. One sixth of these are in
alphabeti al order.   = 120. 10 9 8
6

3. A < B < C . More spe i ally, A = :26; B = :37; C = :56.


4. This is just derangements of 23541. D5 =44.
5. We want 1 365  364  363 : : : (365 n + 1)=365n > 50%. Just guestimating gives the
answer of 23 people.
6. Sin e we want to hoose k times from n distin t elements, this is equivalent to hoos-
ing where to put n 1 "dividers" that separate the hoi es. For example, if we wanted
to hoose 3 s oops of i e ream from the avors ho olate, vanilla, strawberry, and o ee,
we an represent the hoi e of 2 vanilla and 1 o ee by (divider), hoi e, hoi e,(divider),(divider), ho
Noti e that the hoi es of positions for the dividers ompletely determines whi h ele-
ments we hoose. Therefore, we have n + k  1 spa es to ll with n 1 dividers, so the
number of ways of doing this is n 1 or
n+k 1 n+k 1
k

7. We an use the same method as the above argument, ex ept that we know every element
o urs at least on e. So, this is the
 same
 k as hoosing
 k 1 k n times from the n distin t
obje ts, so the answer is k n n
n
, or n 1 or k n .
1
1
+1

8. FIX????????? 1 971
1033
= 1033
62

     
9. Let S be the sum desired. Then 101  S = + + +:::+101 100 101 100 101 100 101

    1 0 2 1 3 2 101

. Now, onsider a general term in this expression - i.e. i


100
. This is equal to 101 100
100
  i +1

i +1
 i i =  i  i  i  =  i i = i . So, we an simplify the terms
101 100!
(100 )! ! (101
101 100!
( +1))! ! ( +1) (101
101!
( +1))! +1!
101
+1

to get 101  S = + + 101


1
+:::+ 101
2
+ . Thus, 101  S = 2
101
3
1, 101
100
101
101
101

2101 1
so S = 101 .
n+m   
10. m
1
or n+nm1 1 . This an be seen from the fa t that ea h distribution an be
des ribed by a ombination of letters, where ea h letter represents a di erent box. The
number of times ea h letter o urs in the ombination is determined by the number of
balls in the orresponding box.
   
11. m!
[m1 !m2 !m3 !:::mn !℄
. This is also equal to m
m1
m
m2
m1 m m1
m3
m2
:::

12. This is very similiar to 11. A) 7!


2!3!2!
=210 B) 7!=(3!3!) =-140
13. This is the sum of all the distribution numbers in whi h the numbers Pm1 +m : : : mn run 1

through all possible sequen es of n positive integers adding up to m: mi>=1


:::mn =m m!
m1 !m2 !:::mn !
14. The boxes an be in any order, so we have a fa tor of 6. Seven anh be partitioned into i
three distin t parts as 0,1,6; 0,2,5; 0,3,4; or 1,2,4. So, the answer is 6 +
7!
0!1!6!
+ 7!
+
0!2!5!
7!
0!3!4!
7!
1!2!4!
=
1008.

15. This is partitioning 10 into 3 partitions, 2 of 1 type and one of the other. Thus, the
answer is =2100.
10!
3!3!4!2!

16. This is equivalent to nding the number of sequen es of length 10 omposed of 0's and
1's. (0 in a spot orresponds to that spot's number (0-9) not being in the subset.)
However, we an't have two onse utive 1's. If we try to generalize, let 0 = A, 1 = B
and we are doing an n-letter "word" instead of ten. Set wn = number of n-letter words
(satisfying the onditions); set an = number of words ounted by wn that begin with A;
set bn = number of words ounted by wn that begin with B. wn = an + bn . an = bn . 1

bn = wn . Combining these we get the re ursive relationship wn = wn + wn . Then


1 1 2

we an build up to nd that w10 = 144.


17. T (m; n) = n(T (m 1; n 1) + T (m for 1 < n < m. To prove this,
1; n))
look at T (5; 3) and think of it as the number of 5-letter words from fA; B; C g with no
missing letters. There are 3 hoi es for the rst letter. After this, the remaining four
letters must be lled in, and the rst letter ( all it X ) does not have to be used again.
There are two ases:
 If X does not o ur again, then the word an be ompleted in T (4; 2) ways.
 If X does o ur again, then the number of ways to omplete the word is T (4; 3).
As we have n hoi es for the letter X ( rst letter), we get that T (5; 3) = 3  (T (4; 2) +
T (4; 3)), or the above, in general.

18. an = an 1 + an 2 + an 5 . Every way to make n ents ends in either a 1 ent, 2 ents,


or 5 ents (sin e order matters, there is a distin t last stamp). There are an ways to
1

make n ents ending in a 1 ent stamp, sin e this is the number of ways to make n 1
ents. Similarly, there are an ways to make n ents ending in a 2 ents stamp, and
2

an ways to make n ents ending in a 5 ents stamp. Sin e every way to make n ents
5

ends in one of these stamps, and there is no overlap, the total number of ways to make
n ents is the sum of these, or an + an + an .
1 2 5

19. (a) rn = rn 1 + n
 
(b) n+1
2
+ 1

20. First, we will show that a positive integer x is not a di eren e of squares if and only if
x  2 (mod 4).
First, suppose x = a b for some integers a; b, so x = (a b)(a + b). Now, if a b = 1,
2 2

then a + b = 2n + 1 for some integer n, so x = (a b)(a + b) = 2n + 1, and x is odd.


Conversely, if x is odd, then x = 2n + 1 for some integer n, so x = 1(2n + 1) =
(n + 1 n)(n + 1 + n) = (n + 1) n . 2 2

Now, if a b = 2, then a + b = 2n + 2 for some integer n, so x = (a b)(a + b) =


2(2n + 2) = 4n + 4
Conversely, if x  0 (mod 4), then x = 4n +4 for some integer n, so x = (n + 2) n . 2 2

So, for all other ases, a b  2. If a b is even, then we know a b = 2n for some integer
n, and a + b = 2n + 2m for some integer m, so x = (a b)(a + b) = 2n  (2n + 2m) =

4nm + 4n , so x  0 (mod 4), and we have already overed this ase.


2
If a b is odd, then we know a b = 2n + 1 for some integer n, and a + b = 2n + 2m + 1,
where m is some integer, so x = (a b)(a + b) = (2n + 1)  (2n + 2m + 1) = 4(n + nm +
2

n) + 2n + 2m + 1  1 (mod 4), whi h is a ase that we have already overed.


Therefore, x is not a di eren e of two squares if and only if x  2 (mod 4). So, the
2000th number is 4  1999 + 2 =7998.
Algebra Test Solutions
Harvard-MIT Math Tournament
March 3, 2001

1. Find x − y, given that x4 = y 4 + 24, x2 + y 2 = 6, and x + y = 3.


24 x4 −y 4 (x2 +y 2 )(x+y)(x−y)
Solution: 6·3 = (x2 +y 2 )(x+y) = (x2 +y 2 )(x+y)
= x − y = 43 .

2. Find (x + 1)(x2 + 1)(x4 + 1)(x8 + 1) · · · , where |x| < 1.


Solution: Let S = (x + 1)(x2 + 1)(x4 + 1)(x8 + 1) · · · = 1 + x + x2 + x3 + · · · . Since
1
xS = x + x2 + x3 + x4 + · · · , we have (1 − x)S = 1, so S = 1−x .

3. How many times does 24 divide into 100! (factorial)?


Solution: We first determine the number of times 2 and 3 divide into 100! = 1·2·3 · · · 100.
Let hN in be the number of times n divides into N (i.e. we want to find h100!i24 ). Since 2
only divides into even integers, h100!i2 = h2 · 4 · 6 · · · 100i. Factoring out 2 once from each
of these multiples, we get that h100!i2 = h250 · 1 · 2 · 3 · · · 50i2 . Repeating this process, we
find that h100!i2 = h2050+25+12+6+3+1 · 1i2 = 97. Similarly, h100!i3 = h333+11+3+1 i3 = 48.
Now 24 = 23 · 3, so for each factor of 24 in 100! there needs to be three multiples of 2 and
one multiple of 3 in 100!. Thus h100!i24 = ([h100!i2 /3] + h100!i3 ) = 32 , where [N ] is the
greatest integer less than or equal to N .

4. Given that 7, 999, 999, 999 has at most two prime factors, find its largest prime factor.
Solution: 7, 999, 999, 999 = 8 · 109 − 1 = 20003 − 1 = (2000 − 1)(20002 + 2000 + 1), so
(20002 + 2000 + 1) = 4,002,001 is its largest prime factor.

5. Find the 6-digit number beginning and ending in the digit 2 that is the product of
three consecutive even integers.
Solution: Because the last digit of the product is 2, none of the three consecutive even
integers end in 0. Thus they must end in 2, 4, 6 or 4, 6, 8, so they must end in 4, 6, 8 since 2·4·6
does not√end in 2. Call
√ the middle integer n. Then the product
√ − 2)n(n + 2) = n3 − 4n,
is (n √
3 3 3 3
so n > 200000 = 200 · 103 ≈ 60, but clearly n < 300000 = 300 · 103 < 70. Thus
n = 66, and the product is 663 − 4 · 66 = 287232 .

6. What is the last digit of 11 + 22 + 33 + · · · + 100100 ?


Solution: Let L(d, n) be the last digit of a number ending in d to the nth power. For
n ≥ 1, we know that L(0, n) = 0, L(1, n) = 1, L(5, n) = 5, L(6, n) = 6. All numbers
ending in odd digits in this series are raised to odd powers; for odd n, L(3, n) = 3 or 7,
L(7, n) = 3 or 7, L(9, n) = 9. All numbers ending in even digits are raised to even powers;
for even n, L(2, n) = 4 or 6, L(4, n) = L(6, n) = 6, L(8, n) = 6 or 4. Further, for each
last digit that has two possible values, the possible values will be present equally as often.
Now define S(d) such that S(0)=0 and for 1 ≤ d ≤ 9, S(d) = L(d, d) + L(d, d + 10) +
L(d, d + 20) + L(d, d + 30) + · · · + L(d, d + 90), so that the sum we want to calculate becomes
S(0) + S(1) + S(2) + · · · + S(9). But by the above calculations all S(d) are divisible by 10,
so their sum is divisible by 10, which means its last digit is 0 .

7. A polynomial P has four roots, 14 , 12 , 2, 4. The product of the roots is 1, and P (1) = 1.
Find P (0).
Solution: A polynomial Q with n roots, x1 , . . . , xn , and Q(x0 ) = 1 is given by Q(x) =
(x−x1 )(x−x2 )···(x−xn ) 1
(x0 −x1 )(x0 −x2 )···(x0 −x4 )
, so P (0) = 3 · 1 ·(−1)·(−3) = 89 .
4 2

8. How many integers between 1 and 2000 inclusive share no common factors with 2001?
Solution: Two integers are said to be relatively prime if they share no common factors,
that is if there is no integer greater than 1 that divides evenly into both of them. Note that
1 is relatively prime to all integers. Let ϕ(n) be the number of integers less than n that
are relatively prime to n. Since ϕ(mn) = ϕ(m)ϕ(n) for m and n relatively prime , we have
ϕ(2001) = ϕ(3 · 23 · 29) = (3 − 1)(23 − 1)(29 − 1) = 1232 .

9. Find the number of positive integer solutions to nx + ny = nz with nz < 2001.


Solution: If n = 1, the relation cannot hold, so assume otherwise. If x > y, the left
hand side factors as ny (nx−y + 1) so nx−y + 1 is a power of n. But it leaves a remainder of 1
when divided by n and is greater than 1, a contradiction. We reach a similar contradiction
if y > x. So y = x and 2nx = nz , so 2 is a power of n and n = 2. So all solutions are of the
form 2x + 2x = 2x+1 , which holds for all x. 2x+1 < 2001 implies x < 11, so there are 10
solutions.

10. Find the real solutions of (2x + 1)(3x + 1)(5x + 1)(30x + 1) = 10.
Solution: (2x + 1)(3x + 1)(5x + 1)(30x + 1) = [(2x + 1)(30x + 1)][(3x + 1)(5x + 1)] =
(60x2 + 32x + 1)(15x2 + 8x + 1) = (4y + 1)(y + 1) = 10, where y = 15x2 + 8x. The quadratic √
equation in y yields y = 1 and y = − 94 . For y = 1, we have 15x2 + 8x − 1 = 0, so x = −4±15 31 .
For y = − 94 , we have 15x2 + 8x + 94 , which yields only complex solutions for x. Thus the real

−4± 31
solutions are 15
.
Calculus Test Solutions
Harvard-MIT Math Tournament
March 3, 2001

1. A sequence of ants walk from (0, 0) to (1, 0) in the plane. The nth ant walks along
n semicircles of radius n1 with diameters lying along the line from (0, 0) to (1, 0). Let Ln be
the length of the path walked by the nth ant. Compute lim Ln .
n→∞
Solution: A semicircle of radius n1 has length 12 π n2 = πn , so n such semicircles have


total length π .

2. The polynomial 3x5 −250x3 +735x is interesting because it has the maximum possible
number of relative extrema and points of inflection at integer lattice points for a quintic
polynomial. What is the sum of the x-coordinates of these points?
Solution: The first derivative is 15x4 − 750x2 + 735, whose roots (which give the relative
extrema) sum to 750/15 = 50. The second derivative is 60x3 − 1500x, whose roots (which
give the points of inflection) sum to 1500/60 = 25, for a grand total of 75 .

3. A balloon that blows up in the shape of a perfect cube is being blown up at a rate such
that at time t fortnights, it has surface area 6t square furlongs. At how many cubic furlongs
per fortnight is the air being pumped in when the surface area is 144 square furlongs?
Solution: The surface area at time t is 6t, so the volume is t3/2 . Hence the air is being
√ √
pumped in at a rate of 32 t. When the surface area is 144, t = 24, so the answer is 3 6 .

4. What is the size of the largest rectangle that can be drawn inside of a 3-4-5 right
triangle with one of the rectangle’s sides along one of the legs of the triangle?
Solution: Clearly one vertex of the rectangle will be at the right angle. Position the
triangle with the leg of length 4 along the x-axis and the leg of length 3 along the y-axis.
Then the hypotenuse is along the line y = 3 − (3/4)x.
Suppose the rectangle has a side of length y along the leg of length 3. Then the area is
y(4/3)(3 − y) = 4y − (4/3)y 2 . The derivative of this is 0 when 4 − (8/3)y = 0, or y = 3/2,
giving an area of 3.
Or, if you prefer, suppose the rectangle has a side of length x along the leg of length 4. Then
the area is x(3 − (3/4)x) = 3x − (3/4)x2 . The derivative of this is 0 when 3 − (3/2)x = 0,
or x = 2, again giving an area of 3 .

5. Same as question 4, but now we want one of the rectangle’s sides to be along the
hypotenuse.
Solution: Put the hypotenuse along the x-axis, with the short leg starting at the origin
so that the right angle is at the point (9/5, 12/5). For notational convenience, let’s just scale
everything by a factor of 5 and then remember to divide the final area by 25, so now the top
point is at (9, 12).
Let (a, 0) be the point where the edge of the rectangle along the hypotenuse starts. Then
the height is h = (4/3)a since the leg of length 3 is along the line y = (4/3)x. The leg of
length 4 is along the line x = 25 − (4/3)y, so the horizontal edge of the rectangle ends at
b = 25 − (4/3)h = 25 − (16/9)a. The area of the rectangle is (b − a)h = (25 − (16/9)a −
a)(4/3)a = 1003
a − 10027
a2 . The derivative of this is 0 when 100
3
= 200
27
a, or a = 9/2. Thus the
(100/3)(9/2)−(100/27)(81/4)
maximum area is 25
=3.

6. The graph of x2 − (y − 1)2 = 1 has one tangent line with positive slope that passes
through (x, y) = (0, 0). If the point of tangency is (a, b), find sin−1 ( ab ) in radians.
dy
Solution: Differentiating both sides of the equation, we find that 2x − 2(y − 1) dx = 0,
dy x a b b a
and so dx = y−1 = b−1 . The line passing through (0, 0) and (a, b) has slope a , so a = b−1 .
2 2 2 2
Solving simultaneously with a − (b − 1) = 1, we get b − b − [(b − 1) + 1] = 0, and so
b = 2, a = (2). Finally, sin−1 ( ab ) = π4 .
p

7. Find the coefficient of x12 in the Maclaurin series (i.e. Taylor series around x = 0) for
1
1−3x+2x2
.
Solution: If you know formal power series, then this is not such a hard question, but
1 1/2 1
since this is a calculus test... Use partial fractions to get 1−3x+2x 2 = 1−2x − 1−x . Now each

of these can be expanded as a geometric series (or take derivatives and get the same result)
to get 12 (1 + 2x + 4x2 + 8x3 + · · · ) − (1 + x + x2 + x3 + · · · ), so the coefficient of xn is 2n−1 − 1.
When n = 12, that’s 2047 .


cot−1 (n2 + n + 1).
P
8. Evaluate
n=0
∞ ∞ ∞
1
cot(n2 + n + 1) =
P P P
Solution: arctan( n2 +n+1 ) = arctan(n + 1) − arctan(n)
n=0 n=0 n=0
by the sum/difference formula for tangent. This sum, taken out to n = N , telescopes to
− arctan(0) + arctan(N + 1). So as N goes to infinity, the sum goes to π/2 .

9. On the planet Lemniscate, the people use the elliptic table of elements, a far more
advanced version of our periodic table. They’re not very good at calculus, though, so they’ve
asked for your help. They know that Kr is somewhat radioactive and deteriorates into Pl, a
very unstable element that deteriorates to form the stable element As. They started with a
block of Kr of size 10 and nothing else. (Their units don’t translate into English, sorry.) and
nothing else. At time t, they let x(t) be the amount of Kr, y(t) the amount of Pl, and z(t)
the amount of As. They know that x0 (t) = −x, and that, in the absence of Kr, y 0 (t) = −2y.
Your job is to find at what time t the quantity of Pl will be largest. You should assume that
the entire amount of Kr that deteriorates has turned into Pl.
Solution: This problem is long-winded since it’s giving an autonomous linear system of
differential equations without using any such language (and it includes a number of subtle
references). The system we have is x0 = −x, y 0 = x − 2y. It’s not hard to see that x = 10e−t
satisfies the first equation and the initial condition. Plugging this into the second equation
and using the integrating factor e2t (or using eigenvalues and eigenvectors to solve the system
directly, though I don’t want to begin to explain what that means) lets us solve for y. More
precisely, we want to solve y 0 + 2y = 10e−t . Multiply by e2t and simplify the left hand side
to get (ye2t )0 = 10et . Integrating both sides with respect to t then yields ye2t = 10et + C, or
y = 10e−t + Ce−2t . Since y(0) = 0, we find C = −10. Now to maximize y, we solve y 0 (t) = 0,
or −10e−t + 20e−2t = 0, or t = ln 2 .

+1
2u332 +u998 +4u1664 sin u691
R
10. Evaluate the definite integral 1+u666
du.
−1
4u1664 sin u691
Solution: The term 1+u666
is odd in u, so its integral is 0. Now make the sub-
+1
R 2u332 +u998 +1
R 2+v2
1 −332/333 1
stitution u = v 1/333 ⇒ du = 333 v dv to find that 1+u666
du = 333 1+v 2
dv =
−1 −1
+1 R1 R1 1
 
1 1 2 1 2 2
(1 + tan−1 1) = 333 2
1 + π4 .
R   
333
1 + 1+v2 dv = 333 1 + 1+v2 dv = 333 1 + 1+v2 dv = 333
−1 0 0
Geometry Test Solutions
Harvard-MIT Math Tournament
March 3, 2001

1. A circle of radius 3 crosses the center of a square of side length 2. Find the positive
difference between the areas of the nonoverlapping portions of the figures.
Solution: Call the area of the square s, the area of the circle c, and the area of the
overlapping portion x. The area of the circle not overlapped by the square is c − x and the
are of the square not overlapped by the circle is s − x, so the difference between these two
is (c − x) − (s − x) = c − s = 9π 2 − 4 .

2. Call three sides of an opaque cube adjacent if someone can see them all at once. Draw
a plane through the centers of each triple of adjacent sides of a cube with edge length 1.
Find the volume of the closed figure bounded by the resulting planes.
Solution: The volume of the figure is half the volume of the cube (which can be seen
by cutting the cube into 8 equal cubes and realizing that the planes cut each of these cubes
in half), namely 12 .

3. Square ABCD is drawn. Isosceles Triangle CDE is drawn with E a right angle.
Square DEF G is drawn. Isosceles triangle F GH is drawn with H a right angle. This
process is repeated infinitely so that no two figures overlap each other. If square ABCD has
area 1, compute the area of the entire figure.
Solution: Let the area of the nth square drawn be Sn and √ the area of the nth triangle
be Tqn . Since the hypotenuse of the nth triangle is of length Sn , its legs are of length
Sn l2
l= 2
, so Sn+1 = l2 = S2n and Tn = 2
= Sn
4
.
Using the recursion relations, Sn = 2n−11
and
1 1 1 1
 1 5 1
Tn = 2n+1 , so Sn + Tn = 2n−1 + 2n+1 = 2
+ 2 2n = 2 2n . Thus the total area of the figure is
∞ ∞
Sn + Tn = 52 1
= 52 .
P P
2n
n=1 n=1

4. A circle has two parallel chords of length x that are x units apart. If the part of the
circle included between the chords has area 2 + π, find x.
Solution: Let C be the area of the circle, S be the area of the square two of whose
edges are the chords, and A be√ the area of the part of the circle included between the chords.
The radius of the circle is 22 x, so C = π2 x2 , and S = x2 . Then the area A is the area
of the square plus one half of the difference between the areas of the circle and square:
1+ π2 2
q
C−S C+S 2A
A = 2 + S = 2 = 2 x , so x = 1+ π = 2 .
2

5. Find the volume of the tetrahedron with vertices (5, 8, 10), (10, 10, 17), (4, 45, 46), (2, 5, 4).
Solution: Each vertex (x, y, z) obeys x + y = z + 3, so all the vertices are coplanar and
the volume of the tetrahedron is 0 .

6. A point on a circle inscribed in a square is 1 and 2 units from the two closest sides of
the square. Find the area of the square.
Solution: Call the point in question A, the center of the circle O, and its radius r.
Consider a right triangle BOA with hypotenuse OA: OA has length r, and BO and BA
have lengths r − 1 and r − 2. By the Pythagorean theorem, (r − 1)2 + (r − 2)2 = r2 ⇒
r2 − 6r + 5 = 0 ⇒ r = 5 since r > 4. The area of the square is (2r)2 = 100 .

7. Equilateral triangle ABC with side length 1 is drawn. A square is drawn such that its
vertex at A is opposite to its vertex at the midpoint of BC. Find the area √ √
enclosed within
2( 3+1)
the intersection of the insides of the triangle and square. Hint: sin 75 = 4
.
Solution: Let D be the midpoint of BC, F 6= A be the point of intersection of the
square and triangle lying on AC, b be the length of F C, x be the side length of the triangle,
and y be the length

of AD. By the law of sines on triangle CDF , we have 2 sinx 75 = sinb45 , so
b = x2 sin
sin 45
75
= 4 sin2x75 . The area of the desired figure can easily be seen to be 12 
(x − b)y since
 it

can be seen as two triangles of width y and height This reduces to 12 1 − 4 sin275 xy.
x−b
2
.

x 2 3
2
 2
Then by the Pythagorean theorem on triangle ABD, x = 2
+ y , so y = 2
x, and the
√  √  √  √ 
3 2 3 2 3
area becomes 4 1 − 4 sin 75 x2 = 4 1 − 4 sin 75 = 4(√3+1) .

8. Point D is drawn on side BC of equilateral triangle ABC, and AD is extended past


D to E such that angles EAC and EBC are equal. If BE = 5 and CE = 12, determine the
length of AE.
Solution:By construction, ABEC is a cyclic quadrilateral. Ptolemy’s theorem says that
for cyclic quadrilaterals, the sum of the products of the lengths of the opposite sides equals the
product of the lengths of the diagonals. This yields (BC)(AE) = (BA)(CE) + (BE)(AC).
Since ABC is equilateral, BC = AC = AB, so dividing out by this common value we get
AE = CE + BE = 17 .

9. Parallelogram AECF is inscribed in square ABCD. It is reflected across diagonal


AC to form another parallelogram AE 0 CF 0 . The region common to both parallelograms has
area m and perimeter n. Compute the value of nm2 if AF : AD = 1 : 4.
Solution: By symmetry, the region is a rhombus, AXCY, centered at the center of the
square, O. Consider isoceles right triangle ACD. By the technique of mass points, we find
that DO : Y O = 7 : 1.√Therefore, the rhombus is composed of√four triangles, whose sides
are in the ratio 1 : 7 : 5 2. The perimeter of the rhombus is 20 2N , and the area is 14N 2 .
7
The required ratio is thus 400 .
10. A is the center of a semicircle, with radius AD lying on the base. B lies on the base
between A and D, and E is on the circular portion of the semicircle such that EBA is a
right angle. Extend EA
√ through A√ to C, and√put√ F on line CD such √ √that EBF is a line.
Now EA = 1, AC = 2, BF = 2−4 2 , CF = 2 5+4 10 , and DF = 2 5−4 10 . Find DE.
Solution: Let θ = ∠AED and x = DE. By the law of cosines on triangle ADE, we have
1 = 1+x2 −2x cos θ ⇒ 2x cos θ = x2 . Then by the law of cosines on triangle CDE (note that
√ √ 2 √  √ 2 √ 
CD = 5), we have 5 = 1 + 2 + x2 − 2 1 + 2 x cos θ = 1 + 2 + x2 − 1 + 2 x2 .
p √
Solving the quadratic equation gives x = 2− 2.
General Test Solutions (First Half)
Harvard-MIT Math Tournament
March 3, 2001

1. What is the last digit of 17103 + 5?


Solution: Let hN i be the last digit of N .
h172 i = 9, h173 i = 3, h174 i = 1, andh175 i = 7.
Since this pattern keeps on repeating itself, 174N = 1 for any integer N . Thus h172·25 i =
h17100 i = 1, so h17103 i = 3, and h17103 + 5i = h3 + 5i = 8 .

2. Find x + y, given that x2 − y 2 = 10 and x − y = 2.


Solution: x2 − y 2 = (x − y)(x + y) = 2(x + y) = 10, so x + y = 5 .

3. There are some red and blue marbles in a box. We are told that there are twelve
more red marbles than blue marbles, and we experimentally determine that when we pick
a marble randomly we get a blue marble one quarter of the time. How many marbles are
there in the box?
Solution: Call the number of blue marbles x, so the number of red marbles is x + 12
and the total number of marbles is 2x + 12. The probability of picking a red marble is
x
2x+12
= 14 ⇒ x = 6, so 2x + 12 = 24 .

4. Find a + b + c + d + e if

3a + 2b + 4d = 10,
6a + 5b + 4c + 3d + 2e = 8,
a + b + 2c + 5e = 3,
2c + 3d + 3e = 4, and
a + 2b + 3c + d = 7.

Solution: Adding the first, third, and fifth equations, we get 5a + 5b + 5c + 5d + 5e =


10 + 3 + 7 ⇒ a + b + c + d + e = 4 .

5. What is the sum of the coefficients of the expansion (x + 2y − 1)6 ?


Solution: The sum of the coefficients of a polynomial is that polynomial evaluated at
1, which for the question at hand is (1 + 2 · 1 − 1)6 = 26 = 64 .

6. A right triangle has a hypotenuse of length 2, and one of its legs has length 1. The
altitude to its hypotenuse is drawn. What is the area of the rectangle whose diagonal is this
altitude?
Solution: Call√ the triangle ABC, with AC = 2 and BC = 1. By the Pythagorean
theorem, AB = 3. Call the point at which the altitude intersects the hypotenuse D. Let
E 6= B be the vertex of the rectangle on AB and F 6= B be√ the vertex of the rectangle on
BC. Triangle BDC is similar to triangle ABC, so BD = 23 . Triangle DBF is similar to
√ √ √
3 33 3 3
triangle ABC, so DF = 4
and BF = 34 . The area of the rectangle is thus 4 4
= 16
.

7. Find (x + 1)(x2 + 1)(x4 + 1)(x8 + 1) · · · , where |x| < 1.


Solution: Let S = (x + 1)(x2 + 1)(x4 + 1)(x8 + 1) · · · = 1 + x + x2 + x3 + · · · . Since
1
xS = x + x2 + x3 + x4 + · · · , we have (1 − x)S = 1, so S = 1−x .

8. How many times does 24 divide into 100!?


Solution: We first determine the number of times 2 and 3 divide into 100! = 1·2·3 · · · 100.
Let hN in be the number of times n divides into N (i.e. we want to find h100!i24 ). Since 2
only divides into even integers, h100!i2 = h2 · 4 · 6 · · · 100i. Factoring out 2 once from each
of these multiples, we get that h100!i2 = h250 · 1 · 2 · 3 · · · 50i2 . Repeating this process, we
find that h100!i2 = h2050+25+12+6+3+1 · 1i2 = 97. Similarly, h100!i3 = h333+11+3+1 i3 = 48.
Now 24 = 23 · 3, so for each factor of 24 in 100! there needs to be three multiples of 2 and
one multiple of 3 in 100!. Thus h100!i24 = ([h100!i2 /3] + h100!i3 ) = 32 , where [N ] is the
greatest integer less than or equal to N .

9. Boris was given a Connect Four game set for his birthday, but his color-blindness makes
it hard to play the game. Still, he enjoys the shapes he can make by dropping checkers into
the set. If the number of shapes possible modulo (horizontal) flips about the vertical axis of
symmetry is expressed as 9(1 + 2 + · · · + n), find n. (Note: the board is a vertical grid with
seven columns and eight rows. A checker is placed into the grid by dropping it from the top
of a column, and it falls until it hits either the bottom of the grid or another checker already
in that column. Also, 9(1 + 2 + · · · + n) is the number of shapes possible, with two shapes
that are horizontal flips of each other counted as one. In other words, the shape that consists
solely of 3 checkers in the rightmost row and the shape that consists solely of 3 checkers in
the leftmost row are to be considered the same shape.)
Solution: There are 97 total shapes possible, since each of the 7 columns can contain
anywhere from 0 to 8 checkers. The number of shapes symmetric with respect to a horizontal
flip is the number of shapes of the leftmost four columns, since the configuration of these
four columns uniquely determines the configuration of the remaining columns if it is known
the shape is symmetric: 94 . Now we know there are 97 − 94 non-symmetric shapes, so there
7 4
are 9 −92
non-symmetric
 shapes
 modulo flips.
 Thus the total number of shapes modulo flips
8 (36 −1) 6 (36 +1)
97 −94 93 −1 93 +1
is 94 + 2
= 94 1 + 2
= 94 2
= (3 2
= 93 2
= 9(1 + 2 + · · · + 36 ), so
n = 36 = 729 .

10. Find the 6-digit number beginning and ending in the digit 2 that is the product of
three consecutive even integers.
Solution: Because the last digit of the product is 2, none of the three consecutive even
integers end in 0. Thus they must end in 2, 4, 6 or 4, 6, 8, so they must end in 4, 6, 8 since 2·4·6
does not√end in 2. Call
√ the middle integer n. Then the product
√ − 2)n(n + 2) = n3 − 4n,
is (n √
3 3 3 3
so n > 200000 = 200 · 10 ≈ 60, but clearly n < 300000 = 300 · 103 < 70. Thus
3

n = 66, and the product is 663 − 4 · 66 = 287232 .


General Test Solutions (Second Half)
Harvard-MIT Math Tournament
March 3, 2001

1. A circle of radius 3 crosses the center of a square of side length 2. Find the difference
between the areas of the nonoverlapping portions of the figures.
Solution: Call the area of the square s, the area of the circle c, and the area of the
overlapping portion x. The area of the circle not overlapped by the square is c − x and the
are of the square not overlapped by the circle is s − x, so the difference between these two
is (c − x) − (s − x) = c − s = 9π 2 − 4 .

2. Call three sides of an opaque cube adjacent if someone can see them all at once. Draw
a plane through the centers of each triple of adjacent sides of a cube with edge length 1.
Find the volume of the closed figure bounded by the resulting planes.
Solution: The volume of the figure is half the volume of the cube (which can be seen
by cutting the cube into 8 equal cubes and realizing that the planes cut each of these cubes
in half), namely 12 .

.
..
xx
3. Find x if x √ =.. 2.
1 ..
√ √ 2 2. 1 2 2.
2 1.
.. √
Solution: 2 = 22 = 21 = 2, so x = 2 .

4. Some students are taking a math contest, in which each student takes one of four
tests. One third of the students take one test, one fourth take another test, one fifth take
the next test, and 26 students take the last test. How many students are taking the contest
in total?
Solution: Call the total number of students n. We know n = n3 + n4 + n5 +26, so n = 120 .

5. What is the area of a square inscribed in a semicircle of radius 1, with one of its sides
flush with the diameter of the semicircle?
Solution: Call the center of the semicircle O, a point of contact of the square and the
circular part of the semicircle A, the closer vertex of the square on the diameter B, and the
side length of the square x. We know OA = 1, AB = x, OB = x2 , and ∠ABO is right. By
4
the Pythagorean theorem, x2 = 5
.

6. You take a wrong turn on the way to MIT and end up in Transylvania, where 99% of
the inhabitants are vampires and the rest are regular humans. For obvious reasons, you want
to be able to figure out who’s who. On average, nine-tenths of the vampires are correctly
identified as vampires and nine-tenths of humans are correct identified as humans. What is
the probability that someone identified as a human is actually a human?
Solution: Consider a sample of 1000 inhabitants. On average, 990 are vampires and 10
are people. 99 vampires are identified as human and 9 humans are identified as human. So
1
out of the 108 who pass, only 12 are human.

7. A real numbers x is randomly chosen in the interval −15 12 , 15 12 . Find the probability
 

that the closest integer to x is odd.


Solution: By using a graphical method, we can see that, for real x on −n − 21 , n + 12 ,
 
n
n an even integer, the probability that the closest integer to x is odd is 2n+1 . The desired
15

probability is 31 .

8. A point on a circle inscribed in a square is 1 and 2 units from the two closest sides of
the square. Find the area of the square.
Solution: Call the point in question A, the center of the circle O, and its radius r.
Consider a right triangle BOA with hypotenuse OA: OA has length r, and BO and BA
have lengths r − 1 and r − 2. By the Pythagorean theorem, (r − 1)2 + (r − 2)2 = r2 ⇒
r2 − 6r + 5 = 0 ⇒ r = 5 since r > 4. The area of the square is (2r)2 = 100 .

9. Two circles are concentric. The area of the ring between them is A. In terms of A,
find the length of the longest chord contained entirely within the ring.
Solution: Let the radii of the circles be r and R > r, so A = π(R2 − r2 ). By the
√ q
Pythagorean theorem, the length of the chord is 2 R2 − r2 = 2 Aπ .

10. Find the volume of the tetrahedron with vertices (5, 8, 10), (10, 10, 17), (4, 45, 46), (2, 5, 4).
Solution: Each vertex (x, y, z) obeys x + y = z + 3, so all the vertices are coplanar and
the volume of the tetrahedron is 0 .
Advanced Topics Test Solutions
Harvard-MIT Math Tournament
March 3, 2001

1. Find x − y, given that x4 = y 4 + 24, x2 + y 2 = 6, and x + y = 3.


24 x4 −y 4 (x2 +y 2 )(x+y)(x−y)
Solution: 6·3 = (x2 +y 2 )(x+y) = (x2 +y 2 )(x+y)
= x − y = 43 .

2. Find logn 21 logn−1 13 · · · log2 n1 in terms of n.


  

Solution: Using log x1 = − log x and logb a = log a


log b
, we get that the product equals
(− log 2)(− log 3)···(− log n)
log n··· log 3 log 2
= (−1)n−1 .

3. Calculate the sum of the coefficients of P (x) if (20x27 + 2x2 + 1)P (x) = 2001x2001 .
Solution: The sum of coefficients of f (x) is the value of f (1) for any polynomial f .
Plugging in 1 to the above equation, P (1) = 2001
23
= 87 .

4. Boris was given a Connect Four game set for his birthday, but his color-blindness makes
it hard to play the game. Still, he enjoys the shapes he can make by dropping checkers into
the set. If the number of shapes possible modulo (horizontal) flips about the vertical axis of
symmetry is expressed as 9(1 + 2 + · · · + n), find n. (Note: the board is a vertical grid with
seven columns and eight rows. A checker is placed into the grid by dropping it from the top
of a column, and it falls until it hits either the bottom of the grid or another checker already
in that column. Also, 9(1 + 2 + · · · + n) is the number of shapes possible, with two shapes
that are horizontal flips of each other counted as one. In other words, the shape that consists
solely of 3 checkers in the rightmost row and the shape that consists solely of 3 checkers in
the leftmost row are to be considered the same shape.)
Solution: There are 97 total shapes possible, since each of the 7 columns can contain
anywhere from 0 to 8 checkers. The number of shapes symmetric with respect to a horizontal
flip is the number of shapes of the leftmost four columns, since the configuration of these
four columns uniquely determines the configuration of the remaining columns if it is known
the shape is symmetric: 94 . Now we know there are 97 − 94 non-symmetric shapes, so there
7 4
are 9 −92
non-symmetric
 shapes
 modulo flips.
 Thus the total number of shapes modulo flips
8 (36 −1) 6 (36 +1)
97 −94 93 −1 93 +1
is 94 + 2
= 94 1 + 2
= 94 2
= (3 2
= 93 2
= 9(1 + 2 + · · · + 36 ), so
n = 36 = 729 .

5. Find the 6-digit number beginning and ending in the digit 2 that is the product of
three consecutive even integers.
Solution: Because the last digit of the product is 2, none of the three consecutive even
integers end in 0. Thus they must end in 2, 4, 6 or 4, 6, 8, so they must end in 4, 6, 8 since 2·4·6
does not√end in 2. Call
√ the middle integer n. Then the product
√ − 2)n(n + 2) = n3 − 4n,
is (n √
3 3
so n > 3 200000 = 200 · 103 ≈ 60, but clearly n < 3 300000 = 300 · 103 < 70. Thus
n = 66, and the product is 663 − 4 · 66 = 287232 .

6. There are two red, two black, two white, and a positive but unknown number of blue
socks in a drawer. It is empirically determined that if two socks are taken from the drawer
without replacement, the probability they are of the same color is 15 . How many blue socks
are there in the drawer?
Solution: Let the number of blue socks be x > 0. Then the probability of drawing a
2
red sock from the drawer is 6+x and the probability of drawing a second red sock from the
1 1
drawer is 6+x−1 = 5+x , so the probability of drawing two red socks from the drawer without
2
replacement is (6+x)(5+x) . This is the same as the probability of drawing two black socks
from the drawer and the same as the probability of drawing two white socks from the drawer.
x(x−1)
The probability of drawing two blue socks from the drawer, similarly, is (6+x)(5+x) . Thus the
probability of drawing two socks of the same color is the sum of the probabilities of drawing
2 x(x−1)
two red, two black, two white, and two blue socks from the drawer: 3 (6+x)(5+x) + (6+x)(5+x) =
x2 −x+6
(6+x)(5+x)
= 15 . Cross-multiplying and distributing gives 5x2 − 5x + 30 = x2 + 11x + 30, so
2
4x − 16x = 0, and x = 0 or 4. But since x > 0, there are 4 blue socks.

7. Order these four numbers from least to greatest: 556 , 1051 , 1735 , 3128 .
Solution: 1051 > 951 = 3102 = 2734 > 1735 > 1635 = 3228 > 3128 > 2528 = 556 , so the
ordering is 556 , 3128 , 1735 , 1051 .

8. Find the number of positive integer solutions to nx + ny = nz with nz < 2001.


Solution: If n = 1, the relation can not hold, so assume otherwise. If x > y, the left
hand side factors as ny (nx−y + 1) so nx−y + 1 is a power of n. But it leaves a remainder of 1
when divided by n and is greater than 1, a contradiction. We reach a similar contradiction
if y > x. So y = x and 2nx = nz , so 2 is a power of n and n = 2. So all solutions are of the
form 2x + 2x = 2x+1 , which holds for all x. 2x+1 < 2001 implies x < 11, so there are 10
solutions.

9. Find the real solutions of (2x + 1)(3x + 1)(5x + 1)(30x + 1) = 10.


Solution: (2x + 1)(3x + 1)(5x + 1)(30x + 1) = [(2x + 1)(30x + 1)][(3x + 1)(5x + 1)] =
(60x2 + 32x + 1)(15x2 + 8x + 1) = (4y + 1)(y + 1) = 10, where y = 15x2 + 8x. The quadratic √
equation in y yields y = 1 and y = − 94 . For y = 1, we have 15x2 + 8x − 1 = 0, so x = −4±15 31 .
For y = − 94 , we have 15x2 + 8x + 94 , which yields only complex solutions for x. Thus the real

−4± 31
solutions are 15
.
10. Alex picks his favorite point (x, y) in the first quadrant on the unit circle x2 + y 2 = 1,
such that a ray from the origin through(x, y) is θ radians counterclockwise from the positive
x-axis. He then computes cos−1 4x+3y 5
and is surprised to get θ. What is tan(θ)?
Solution: x = cos(θ), y = sin(θ). By the trig identity you never thought you’d need,
4x+3y
5
= cos(θ − φ), where φ has sine 3/5 and cosine 4/5. Now θ − φ = θ is impossible, since
φ 6= 0, so we must have θ − φ = −θ, hence θ = φ/2. Now use the trusty half-angle identities
to get tan(θ) = 13 .
1. [5] January 3, 1911 was an odd date as its abbreviated representation, 1/3/1911, can
be written using only odd digits (note all four digits are written for the year). To the nearest
month, how many months will have elapsed between the most recent odd date and the next
odd date (today is 3/3/2001, an even date).
Solution: The most recent odd date was 11/19/1999 (November has 30 days, but the
assumption that it has 31 days does not change the answer), and the next odd date will
be 1/1/3111. From 11/19/1999 to 1/1/2000 is about 1 month. From 2000 to 3111 is 1111
years, or 12 · 1111 = 13332 months, so the total number of months is 13333 .

2. [4] Ken is the best sugar cube retailer in the nation. Trevor, who loves sugar, is
coming over to make an order. Ken knows Trevor cannot afford more than 127 sugar cubes,
but might ask for any number of cubes less than or equal to that. Ken prepares seven cups
of cubes, with which he can satisfy any order Trevor might make. How many cubes are in
the cup with the most sugar?
Solution: The only way to fill seven cups to satisfy the above condition is to use a binary
scheme, so the cups must contain 1, 2, 4, 8, 16, 32, and 64 cubes of sugar.

3. [7] Find the number of triangulations of a general convex 7-gon into 5 triangles by 4
diagonals that do not intersect in their interiors.
1 2n

Solution: Define the Catalan numbers by C(n) = n+1 n
. The current solution is the
C(number of triangles) = C(5) = 42 .


1
Q 
4. [7] Find 1− n2
.
n=2
∞ ∞ ∞
1 n2 −1 (n−1)(n+1) 1·3 2·4 3·5 4·6 5·7 1·2·3·3·4·4·5·5···
Q  Q Q
Solution: 1− n2
= n2
= n·n
= 2·2 3·3 4·4 5·5 6·6
··· = 2·2·3·3·4·4·5·5···
=
n=2 n=2 n=2
1
2
.

5. [± 6] Let ABC be a triangle with incenter I and circumcenter O. Let the circumradius
be R. What is the least upper bound of all possible values of IO?
Solution: I always lies inside the convex hull of ABC, which in turn always lies in the
circumcircle of ABC, so IO < R. On the other hand, if we first draw the circle Ω of radius
R about O and then pick A, B, and C very close together on it, we can force the convex hull
of ABC to lie outside the circle of radius R −  about O for any . Thus the answer is R .

6. [8] Six students taking a test sit in a row of seats with aisles only on the two sides of
the row. If they finish the test at random times, what is the probability that some student
will have to pass by another student to get to an aisle?

1
Solution: The probability p that no student will have to pass by another student to get
to an aisle is the probability that the first student to leave is one of the students on the end,
the next student to leave is on one of the ends of the remaining students, etc.: p = 26 · 25 · 24 · 23 ,
43
so the desired probability is 1 − p = 45
.

7. [5] Suppose a, b, c, d, and e are objects that we can multiply together, but the
multiplication doesn’t necessarily satisfy the associative law, i.e. (xy)z does not necessarily
equal x(yz). How many different ways are there to interpret the product abcde?
Solution: C(number of letters − 1) = C(4) = 14 .

8. [10] Compute 1 · 2 + 2 · 3 + · · · + (n − 1)n.


n n
n(n+1)
i2 =
P P
Solution: Let S = 1 · 2 + 2 · 3 + · · · + (n − 1)n. We know i= 2
and
i=1 i=1
n(n+1)(2n+1)
6
. So S = 1(1 + 1) + 2(2 + 1) + · · · + (n − 1)n = (12 + 22 + · · · + (n − 1)2 ) +
(n−1)(n)(2n−1) (n−1)(n) (n−1)n(n+1)
(1 + 2 + · · · + (n − 1)) = 6
+ 2
= 3
.
n n n n
2
i2 + i2 + i2 + · · · +
P P P P
We can also arrive at the solution by realizing that i =
1  i=1 i=1 i=2 i=3
n n 2 3 n−1  
n(n+1) (n−1)n
i2 = n i 2 − i2 + i2 + i2 + · · · i2 = n 2 − 1·2 2·3
P P P P P P
2
+ 2
+ · · · + 2
=
i=n i=1 i=1 i=1 i=1 n=1
n n(n+1)
2
− 21 S = n(n+1)(2n+1)
6
, so S = (n−1)n(n+1)
3
.

9. [5] Suppose x satisfies x3 + x2 + x + 1 = 0. What are all possible values of x4 + 2x3 +


2x2 + 2x + 1?
Solution: x4 + 2x3 + 2x2 + 2x + 1 = (x + 1)(x3 + x2 + x + 1) = 0 is the only possible
solution.

10. [8] Two concentric circles have radii r and R > r. Three new circles are drawn so
that they are each tangent to the big two circles and tangent to the other two new circles.
Find Rr .
Solution: The centers of the three new circles form a triangle. The diameter of the new
circles is R − r, so the side length of the triangle is R − r. Call the center of the concentric
circle O, two vertices of the triangle A and B, and AB’s midpoint D. OA is the average R
and r, namely R+r2
. Using the law of sines on triangle DAO, we get sin(30)
AD
= sin(90)
AO
⇒ R = 3r,
R
so r = 3 .

11. [8] 12 points are placed around the circumference of a circle. How many ways are
there to draw 6 non-intersecting chords joining these points in pairs?
Solution: C(number of chords) = C(6) = 132 .

2
12. [± 6] How many distinct sets of 8 positive odd integers sum to 20?
Solution: This is the same as the number of ways 8 nonnegative even integers sum to
12 (we subtract 1 from each integer in the above sum). All 11 possibilities are (leaving out
0s): 12, 10 + 2, 8 + 4, 8 + 2 + 2, 6 + 6, 6 + 4 + 2, 6 + 2 + 2 + 2 + 2, 4 + 4 + 4, 4 + 4 + 2 + 2,
4 + 2 + 2 + 2 + 2, 2 + 2 + 2 + 2 + 2 + 2.

13. [5] Find the number of real zeros of x3 − x2 − x + 2.


Solution: Let f (x) = x3 − x2 − x + 2, so f 0 (x) = 3x 2
− 2x − 1. The slope is zero when
1 1
2

3x − 2x − 1 = 0, where x = − 3 and x = 1. Now f 3 > 0 and f (1) > 0, so there are no
zeros between x = − 31 and x = 1. Since lim f (x) > 0, there are no zeros for x > 1. Since
x→+∞
lim f (x) < 0, there is one zero for x < − 13 , for a total of 1 zero.
x→−∞

1
14. [8] Find the exact value of 1 + 1+ 2 .
1+ 1
2
1+ 1+···
1 1
Solution: Let x be what we are trying to find. x − 1 = 1+ 2 ⇒ x−1
−1 =
1+ 1
2
1+ 1+···

2 2

1+ 1 ⇒ 1
−1
= x ⇒ x2 − 2 = 0, so x = 2 since x > 0.
2
1+ 1+··· x−1

15. [6] A beaver walks from (0, 0) to (4, 4) in the plane, walking one unit in the positive
x direction or one unit in the positive y direction at each step. Moreover, he never goes to
a point (x, y) with y > x. How many different paths can he walk?
Solution: C(4) = 14 .

16. [6] After walking so much that his feet get really tired, the beaver staggers so that,
at each step, his coordinates change by either (+1, +1) or (+1, −1). Now he walks from
(0, 0) to (8, 0) without ever going below the x-axis. How many such paths are there?
Solution: C(4) = 14 .

17. [4] Frank and Joe are playing ping pong. For each game, there is a 30% chance that
Frank wins and a 70% chance Joe wins. During a match, they play games until someone
wins a total of 21 games. What is the expected value of number of games played per match?
Solution: The expected value of the ratio of Frank’s to Joe’s score is 3:7, so Frank is
expected to win 9 games for each of Frank’s 21. Thus the expected number of games in a
match is 30 .

10 8 6 4 2
√ [5] Find the largest prime factor of −x − x − x − x − x − 1, where x = 2i,
18.
i = −1.

3
Solution: 13 .

2001
n3 .
P
19. [9] Calculate
n=1
2001
2001 2
2001·2002 2
P 3 P 
Solution: n = n = 2
= 4012013006001 .
n=1 n=1

20. [± 4] Karen has seven envelopes and seven letters of congratulations to various
HMMT coaches. If she places the letters in the envelopes at random with each possible
configuration having an equal probability, what is the probability that exactly six of the
letters are in the correct envelopes?
Solution: 0 , since if six letters are in their correct envelopes the seventh is as well.


P (i+1)(i+2)(i+3)
21. [10] Evaluate (−2)i
.
i=1
6
Solution: This is the power series of (1+x)4
expanded about x = 0 and evaluated at
x= − 12 , so the solution is 96 .

22. [6] A man is standing on a platform and sees his train move such that after t
seconds it is 2t2 + d0 feet from his original position, where d0 is some number. Call the
smallest (constant) speed at which the man have to run so that he catches the train v. In
terms of n, find the nth smallest value of d0 that makes v a perfect square.
Solution: The train’s distance from the man’s original position is t2 + d0 , and the man’s
distance from his original position if he runs at speed v is vt at time t. We need to find
where√t2 + d0 = vt has a solution. Note that this is a quadratic √ equation with discriminant
D = v 2 − 4d0 , so it has solutions for real D, i.e. where v ≥ 4d0 , so 4d0 must be a perfect
square. This happens when 4d0 is an even power of 2: the smallest value is 20 , the second
smallest is 22 , the third smallest is 24 , and in general the nth smallest is 22(n−1) , or 4n−1 .

23. [5] Alice, Bob, and Charlie each pick a 2-digit number at random. What is the
probability that all of their numbers’ tens’ digits are different from each others’ tens’ digits
and all of their numbers’ ones digits are different from each others’ ones’ digits?
9 8 87
Solution: 10 10 9 9
= 112
225
.

24. [6] Square ABCD has side length 1. A dilation is performed about point A, creating
square AB 0 C 0 D0 . If BC 0 = 29, determine the area of triangle BDC 0 .
1 29 1

Solution: 292 − 2 · 2 (29) 2 − 2 = 420 .

4
25. [± 10] What is the remainder when 100! is divided by 101?
Solution: Wilson’s theorem says that for p a prime, (p − 1)! ≡ −1(p), so the remainder
is 100 .

26. [6] A circle with center at O has radius 1. Points P and Q outside the circle are
placed such that P Q passes through O. Tangent lines to the circle through P hit the circle
at P1 and P2 , and tangent lines to the circle through Q hit the circle at Q1 and Q2 . If
∠P1 P P2 = 45◦ and angleQ1 QQ2 = 30◦ , find the minimum possible length of arc P2 Q2 .
Solution: (45 − 30)◦ = 12π
.

27. [5] Mona has 12 match sticks of length 1, and she has to use them to make regular
polygons, with each match being a side or a fraction of a side of a polygon, and no two
matches overlapping or crossing each other. What is the smallest total area of the polygons
Mona can make?√ √
Solution: 4 43 = 3 .

28. [4] How many different combinations of 4 marbles can be made from 5 indistinguish-
able red marbles, 4 indistinguishable blue marbles, and 2 indistinguishable black marbles?
Solution: 5 + 4 + 3 = 12 .

29. [10] Count the number of sequences a1 , a2 , a3 , a4 , a5 of integers such that ai ≤ 1 for
all i and all partial sums (a1 , a1 + a2 , etc.) are non-negative.
Solution: C(length + 1) = C(6) = 132 .

30. [± 4] How many roots does arctan x = x2 − 1.6 have, where the arctan function is
defined in the range − pi2 < arctan x < pi2 ?
Solution: 2 .

31. [5] If two fair dice are tossed, what is the probability that their sum is divisible by
5?
1
Solution: 4
.

32. [10] Count the number of permutations a1 a2 . . . a7 of 1234567 with longest decreasing
subsequence of length at most two (i.e. there does not exist i < j < k such that ai > aj > ak ).
Solution: C(7) = 429 .

33. [± 5] A line of soldiers 1 mile long is jogging. The drill sergeant, in a car, moving
at twice their speed, repeatedly drives from the back of the line to the front of the line and

5
back again. When each soldier has marched 15 miles, how much mileage has been added to
the car, to the nearest mile?
Solution: 30 .

34. [8] Find all the values of m for which the zeros of 2x2 − mx − 8 differ by m − 1.
Solution: 6, − 103
.

35. [7] Find the largest integer that divides m5 − 5m3 + 4m for all m ≥ 5.
. 5! = 120 .

36. [4] Count the number of sequences 1 ≤ a1 ≤ a2 ≤ · · · ≤ a5 of integers with ai ≤ i


for all i.
Solution: C(number of terms) = C(5) = 42 .

37. [5] Alex and Bob have 30 matches. Alex picks up somewhere between one and six
matches (inclusive), then Bob picks up somewhere between one and six matches, and so on.
The player who picks up the last match wins. How many matches should Alex pick up at
the beginning to guarantee that he will be able to win?
Solution: 2 .

38. [9] The cafeteria in a certain laboratory is open from noon until 2 in the afternoon
every Monday for lunch. Two professors eat 15 minute lunches sometime between noon
and 2. What is the probability that they are in the cafeteria simultaneously on any given
Monday?
Solution: 15
64
.

39. [9] What is the remainder when 22001 is divided by 27 − 1?


Solution: 22001(mod7) = 26 = 64 .

40. [5] A product of five primes is of the form ABC, ABC, where A, B, and C represent
digits. If one of the primes is 491, find the product ABC, ABC.
Solution: 491 · 1001 · 2 = 982,982 .

1 2 1 3
 
41. [4] If a + a
= 3, find a + a
in terms of a.
Solution: 0 .

6
q q
1 1
42. [10] Solve x = x− x
+ 1− x
for x.

1+ 5
Solution: 2
.

43. [4] When a single number is added to each member of the seqence 20, 50, 100, the
sequence becomes expressable as x, ax, a2 x. Find a.
Solution: 53 .

44. [7] Through a point in the interior of a triangle ABC, three lines are drawn, one
parallel to each side. These lines divide the sides of the triangle into three regions each.
Let a, b, and c be the lengths of the sides opposite ∠A, ∠B, and ∠C, respectively, and let
a0 , b0 , and c0 be the lengths of the middle regions of the sides opposite ∠A, ∠B, and ∠C,
respectively. Find the numerical value of a0 /a + b0 /b + c0 /c.
Solution: 1 .

45. [4] A stacking of circles in the plane consists of a base, or some number of unit
circles centered on the x-axis in a row without overlap or gaps, and circles above the x-axis
that must be tangent to two circles below them (so that if the ends of the base were secured
and gravity were applied from below, then nothing would move). How many stackings of
circles in the plane have 4 circles in the base?
Solution: C(4) = 14 .

46. [± 5] Draw a rectangle. Connect the midpoints of the opposite sides to get 4
congruent rectangles. Connect the midpoints of the lower right rectangle for a total of 7
rectangles. Repeat this process infinitely. Let n be the minimum number of colors we can
assign to the rectangles so that no two rectangles sharing an edge have the same color and m
be the minimum number of colors we can assign to the rectangles so that no two rectangles
sharing a corner have the same color. Find the ordered pair (n, m).
Solution: (3, 4) .

47. [7] For the sequence of numbers n1 , n2 , n3 , . . . , the relation ni = 2ni−1 + a holds for
all i > 1. If n2 = 5 and n8 = 257, what is n5 ?
Solution: 33 .

48. [8] What is the smallest positive integer x for which x2 + x + 41 is not a prime?
Solution: 40 .

1 1
49. [5] If 9
of 60 is 5, what is 20
of 80?

7
Solution: In base 15, 6 .

50. [9] The Fibonacci sequence F1 , F2 , F3 , . . . is defined by F1 = F2 = 1 and Fn+2 =


Fn+1 + Fn . Find the least positive integer t such that for all n > 0, Fn = Fn+t .
Solution: 60 .

51. [5] Some people like to write with larger pencils than others. Ed, for instance, likes
to write with the longest pencils he can find. However, the halls of MIT are of limited height
L and width L. What is the longest pencil Ed can bring through the halls so that he can
negotiate a square turn?
Solution: 3L .

52. [6] Find all ordered pairs (m, n) of integers such that 231m2 = 130n2 .
Solution: The unique solution is (0, 0) .

1 1 1 1 1 1 1 1 1
  
53. [7] Find the sum of the infinite series 32 −12 12
− 32
+ 52 −3 2 32
− 52
+ 72 −5 2 52
− 72
+
···.
Solution: 1 − 13 + 13 − 15 + · · · = 1 .

54. [10] The set of points (x1 , x2 , x3 , x4 ) in R4 such that x1 ≥ x2 ≥ x3 ≥ x4 is a cone


(or hypercone, if you insist). Into how many regions is this cone sliced by the hyperplanes
xi − xj = 1 for 1 ≤ i < j ≤ n?
Solution: C(4) = 14 .

6 9
55. [7] How
qmany multiples
q of 7 between 10 and 10 are perfect squares?
109 106
Solution: 72
− 72
= 4517 − 142 = 4375 .

56. [6] A triangle has sides of length 888, 925, and x > 0. Find the value of x that
minimizes the area of the circle circumscribed about the triangle.
Solution: 259 .

57. [5] Let x = 20011002 − 2001−1002 and y = 20011002 + 2001−1002 . Find x2 − y 2 .


Solution: -4 .

58. [9] Let (x, y) be a point in the cartesian plane, x, y > 0. Find a formula in terms of
x and y for the minimal area of a right triangle with hypotenuse passing through (x, y) and
legs contained in the x and y axes.

8
Solution: 2xy .

59. [10] Trevor and Edward play a game in which they take turns adding or removing
beans from a pile. On each turn, a player must either add or remove the largest perfect
square number of beans that is in the heap. The player who empties the pile wins. For
example, if Trevor goes first with a pile of 5 beans, he can either add 4 to make the total
9, or remove 4 to make the total 1, and either way Edward wins by removing all the beans.
There is no limit to how large the pile can grow; it just starts with some finite number of
beans in it, say fewer than 1000.
Before the game begins, Edward dispatches a spy to find out how many beans will be in
the opening pile, call this n, then “graciously” offers to let Trevor go first. Knowing that the
first player is more likely to win, but not knowing n, Trevor logically but unwisely accepts,
and Edward goes on to win the game. Find a number n less than 1000 that would prompt
this scenario, assuming both players are perfect logicians. A correct answer is worth the
nearest integer to log2 (n − 4) points.
Solution: The correct answers are 0 (worth imaginary points), 5 (worth 0 points), 20
(4 points), 29, 45 (5 points), 80 (6 points), 101, 116, 135, 145, 165, 173 (7 points), 236, 257
(8 points), 397, 404, 445, 477, 540, 565, 580, 629, 666 (9 points), 836, 845, 885, 909, 944,
949, 954, 975 (10 points). This game is called Epstein’s Put-or-Take-a-Square game. It is
unknown whether or not these numbers (or the first player’s win positions) have positive
density.

60. [∞] Find an n such that n! −n (n − 1)! + (n − 2)! − (n − 3)! + · · · ± 1! is prime. Be


n, n≤25
prepared to justify your answer for [ n+225 ], n>25 points, where [N ] is the greatest integer
10
less than N .
Solution: 3, 4, 5, 6, 7, 8, 10, 15, 19, 41 (26 points), 59, 61 (28 points), 105 (33 points),
160 (38 points) are the only ones less than or equal to 335. If anyone produces an answer
larger than 335, then we ask for justification to call their bluff. It is not known whether or
not there are infinitely many such n.

9
Team Round Solutions
Harvard-MIT Math Tournament
March 3, 2001

1. How many digits are in the base two representation of 10! (factorial)?
Solution: We write 10! = 28 · 34 · 52 · 7. The number of digits (base 2) of 10! is equal to
[log2 10!] = 8 + log2 (34 · 52 · 7). Since 21 3 < 32 · 52 · 7 < 21 4, the number of digits is 8 + 13 =
21 .

2. On a certain unidirectional highway, trucks move steadily at 60 miles per hour spaced
1/4 of a mile apart. Cars move steadily at 75 miles per hour spaced 3 seconds apart. A lone
sports car weaving through traffic at a steady forward speed passes two cars between each
truck it passes. How quickly is it moving in miles per hour?
Solution: The cars are 1/8 of a mile apart. Consider the reference frame in which the
trucks move at 0 velocity (and the cars move at 15). Call the speed of the sports car in this
reference frame v. The amount of time for the sports car to move from one truck to the
next is 1/4 vmiles , and the amount of time for two regular cars to pass the truck is 1/8 miles
15mph
.
Equating these, we get v = 30, and v + 60 = 90 mph.

3. What is the 18th digit after the decimal point of 10000


9899
?
10000
Solution: 9899 satisfies 100(x − 1) = 1.01x, so each pair of adjacent digits is generated
by adding the previous two pairs of digits. So the decimal is 1.01020305081321345590 . . . ,
and the 18th digit is 5 .

4. P is a polynomial. When P is divided by x − 1, the remainder is −4. When P is


divided by x − 2, the remainder is −1. When P is divided by x − 3, the remainder is 4.
Determine the remainder when P is divided by x3 − 6x2 + 11x − 6.
Solution: The remainder polynomial is simply the order two polynomial that goes
through the points (1, −4), (2, −1), and (3, 4): x2 − 5 .

5. Find all x between − π2 and π2 such that 1 − sin4 x − cos2 x = 16


1
.
4 4
2 1 2
Solution: 1 − sin x − cos x = 16 ⇒ (16 − 16 cos x) − sin x − 1 = 0 ⇒ 16 √
sin4 x −
16 sin2 x + 1 = 0. Use the quadratic formula in sin x to obtain sin2 x = 12 ± 43 . Since

cos 2x = 1 − 2 sin2 x = ± 2
3
, π
we get x = ± 12 , ± 5π
12
.

6. What is the radius of the smallest sphere in which 4 spheres of radius 1 will fit?
Solution: The centers of the smaller spheres lie on a tetrahedron. Let the points of the
−1, 1), (−1, 1, −1), and (1, −1, −1). These points have distance
tetrahedron be (1, 1, 1), (−1,p
p
(3) from the center, and p (2) from each p other,p so the radius of the smallest sphere in
which 4 spheres of radius (2) will fit is (2) + (3). Scale this to the correct answer by
p √
dividing by (2): 2+2 6 .

7. The Fibonacci numbers are defined by F1 = F2 = 1 and Fn+2 = Fn+1 + Fn for n ≥ 1.


The Lucas numbers are defined by L1 = 1, L2 = 2, and Ln+2 = Ln+1 + Ln for n ≥ 1.
15
Q F2n
Fn
n=1
Calculate 13
Q
.
Ln
n=1
F2n
Solution: It is easy to show that Ln = Fn
, so the product above is L1 4L1 5 = 843 ·
1364 = 1149852 .

8. Express sin 10+sin 20+sin 30+sin 40+sin 50+sin 60+sin 70+sin 80


cos 5 cos 10 cos 20
without using trigonometric func-
tions.
Solution: We will use the identities cos a + cos b = 2 cos a+b 2
cos a−b
2
and sin a + sin b =
a+b a+b
2 sin 2 cos 2 . The numerator is (sin 10+sin 80)+(sin 20+sin 70)+(sin 30+sin 60)+(sin 40+
sin 60) = 2 sin 45 (cos 35 + cos 25 + cos 15 + cos 35) = 2 sin 45 ((cos 35 + cos 5) + (cos 25 + cos 15)) =

4 sin 45 cos 20 (cos 15 + cos 5) = 8 sin 45 cos 20 cos 10 cos 5, so the fraction equals 8 sin 45 = 4 2 .


ai
P
9. Compute ai
for a > 1.
i=1
Solution: The sum S = a+ax+ax2 +ax3 +· · · for x < 1 can be determined by realizing

a ai
that xS = ax + ax2 + ax3 + · · · and (1 − x)S = a, so S = 1−x
P
. Using this, we have ai
=
i=1

i
P 1 2 3
  1 1 1
 1 1 1
 
a ai
= a a
+ a2
+ a3
+ ··· = a a
+ a2
+ a3
+ ··· + a2
+ a3
+ a4
+ ··· + ··· =
i=1
 1 1 1 1 1
 a
 1 1
 a
2
a 1−a
+ a 1−a
+ a2 1−a
+ ··· = 1−a
1+ a
+ a2
+ ··· = 1−a
.

10. Define a monic irreducible polynomial with integral coefficients to be a polynomial


with leading coefficient 1 that cannot be factored, and the prime factorization of a polynomial
with leading coefficient 1 as the factorization into monic irreducible polynomials. How many
not necessarily distinct monic irreducible polynomials are there in the prime factorization of
(x8 + x4 + 1)(x8 + x + 1) (for instance, (x + 1)2 has two prime factors)?
Solution: x8 +x4 +1 = (x8 +2x4 +1)−x4 = (x4 +1)2 −(x2 )2 = (x4 −x2 +1)(x4 +x2 +1) =
(x4 −x2 +1)(x2 +x+1)(x2 −x+1), and x8 +x+1 = (x2 +x+1)(x6 −x5 +x3 −x2 +1). If an integer
polynomial f (x) = an xn + · · · + a0 (modp), where p does not divide an , has no zeros, then f
has no rational roots. Taking p = 2, we find x6 − x5 + x3 − x2 + 1 is irreducible. The prime
factorization of our polynomial is thus (x4 −x2 +1)(x2 −x+1)(x2 +x+1)2 (x6 −x5 +x3 −x2 +1),
so the answer is 5 .

11. Define a? = (a − 1)/(a + 1) for a 6= −1. Determine all real values N for which
(N ?)? = tan 15.
Solution:Let x = N ?. Then (x − 1) cos 15 = (x + 1) sin 15. Squaring and rearranging
2

3

2
terms, and using the fact that cos √15 − sin 15 = cos 30 = 2 , we have 3x2 − 4 3x + 3 = 0.

Solving, we find that x = 3 or 33 . However, we may reject the second root because it
√ √
1+√3

yields a negative value for (N ?)?. Therefore x = 3 and N = 1+x 1−x
= 1− 3
= −2 − 3.

12. All subscripts in this problem are to be considered modulo 6, that means for example
that ω7 is the same as ω1 . Let ω1 , . . . ω6 be circles of radius r, whose centers lie on a regular
hexagon of side length 1. Let Pi be the intersection of ωi and ωi+1 that lies further from the
center of the hexagon, for i = 1, . . . 6. Let Qi , i = 1 . . . 6, lie on ωi such that Qi , Pi , Qi+1
are colinear. Find the number of possible values of r.
Solution: Consider two consecutive circles ωi and ωi+1 . Let Qi , Q0i be two points on ωi
and Qi+1 , Q0i+1 on ωi+1 such that Qi , Pi and Qi+1 are colinear and also Q0i , Pi and Q0i+1 .
Then Qi Q0i = 2∠Qi Pi Q0i = 2∠Qi+1 Pi Q0i+1 = ∠Qi+1 Q0i+1 . Refer to the center of ωi as Oi .
The previous result shows that the lines Oi Qi and Oi+1 Qi+1 meet at the same angle as the
lines Oi Q0i and Oi+1 Q0i+1 , call this angle ψi . ψi is a function solely of the circles ωi and ωi+1
and the distance between them (we have just showed that any two points Qi and Q0i on ωi
give the same value of ψi , so ψi can’t depend on this.) Now, the geometry of ωi and ωi+1
is the same for every i, so ψi is simply a constant ψ which depends only on r. We know
6ψ = 0 mod 2π because Q7 = Q1 .
We now compute ψ. It suffices to do the computaiton for some specific choice of Qi .
Take Qi to be the intersection of Oi Oi+1 and ωi which is further from Oi+1 . We are to
compute the angle between Oi Qi and Oi+1 Qi+1 which is the same as ∠Oi Oi+1 Qi+1 . Note the
triangle 4Oi Pi Oi+1 is isosceles, call the base angle ξ. We have ∠Oi Oi+1 Qi+1 = ∠Oi Oi+1 Pi +
∠Pi Oi+1 Qi+1 = ξ + (π − 2∠Oi+1 Pi Qi+1 ) = ξ + (π − 2(π − ∠Qi Oi+1 Pi − ∠Pi Qi Oi+1 )) =
ξ − π + 2(ξ + (1/2)∠Pi Oi Oi+1 ) = ξ − π + 2(ξ + (1/2)ξ) = 4ξ − π.
So we get 6(4ξ−π) = 0 mod 2π. Noting that ξ must be acute, ξ = π/12, π/6, π/4, π/3 or 5π/12.
r is uniquely determined as (1/2) sec ξ so there are 5 possible values of r.
Harvard-MIT Math Tournament
March 17, 2002
Individual Subject Test: Algebra

1. Nine nonnegative numbers have average 10. What is the greatest possible value for
their median?
Solution: 18 If the median is m, then the five highest numbers are all at least m, so
the sum of all the numbers is at least 5m. Thus 90 ≥ 5m ⇒ m ≤ 18. Conversely, we can
achieve m = 18 by taking four 0’s and five 18’s.

2. p and q are primes such that the numbers p + q and p + 7q are both squares. Find
the value of p.
Solution: 2 . Writing x2 = p + q, y 2 = p + 7q, we have 6q = y 2 − x2 = (y − x)(y + x).
Since 6q is even, one of the factors y − x, y + x is even, and then the other is as well; thus
6q is divisible by 4 ⇒ q is even ⇒ q = 2 and 6q = 12. We may assume x, y are both taken
to be positive; then we must have y − x = 2, y + x = 6 ⇒ x = 2, so p + 2 = 22 = 4 ⇒ p = 2
also.

3. Real numbers a, b, c satisfy the equations a + b + c = 26, 1/a + 1/b + 1/c = 28. Find
the value of
a b c a c b
+ + + + + .
b c a c b a
Solution: 725 Multiplying the two given equations gives

a a a b b b c c c
+ + + + + + + + = 26 · 28 = 728,
a b c a b c a b c
and subtracting 3 from both sides gives the answer, 725.

4. If a positive integer multiple of 864 is picked randomly, with each multiple having the
same probability of being picked, what is the probability that it is divisible by 1944?
Solution: The probability that a multiple of 864 = 25 33 is divisible by 2744 = 23 35 is
the same as the probability that a multiple of 22 is divisible by 32 , which since 4 and 9 are
relatively prime is 19 .

5. Find the greatest common divisor of the numbers 2002 + 2, 20022 + 2, 20023 + 2, . . ..
Solution: 6 . Notice that 2002 + 2 divides 20022 − 22 , so any common divisor of 2002 + 2
and 20022 + 2 must divide (20022 + 2) − (20022 − 22 ) = 6. On the other hand, every number
in the sequence is even, and the nth number is always congruent to 1n + 2 ≡ 0 modulo 3.
Thus, 6 divides every number in the sequence.

6. Find the sum of the even positive divisors of 1000.

1
Solution: 2184 . Notice that 2k is a divisor of 1000 iff k is a divisor of 500, so we need
only find the sum of the divisors of 500 and multiply by 2. This can be done by enumerating
the divisors individually, or simply by using the formula: σ(22 · 53 ) = (1 + 2 + 22 )(1 + 5 +
52 + 53 ) = 1092, and then doubling gives 2184. Alternate Solution: The sum of all the
divisors of 1000 is (1 + 2 + 22 + 23 )(1 + 5 + 52 + 53 ) = 2340. The odd divisors of 1000 are
simply the divisors of 125, whose sum is 1 + 5 + 52 + 53 = 156; subtracting this from 2340,
we are left with the sum of the even divisors of 1000, which is 2184.

7. The real numbers x, y, z, w satisfy

2x + y + z + w = 1
x + 3y + z + w = 2
x + y + 4z + w = 3
x + y + z + 5w = 25.

Find the value of w.


Solution: 11/2 . Multiplying the four equations by 12, 6, 4, 3 respectively, we get

24x + 12y + 12z + 12w = 12


6x + 18y + 6z + 6w = 12
4x + 4y + 16z + 4w = 12
3x + 3y + 3z + 15w = 75.

Adding these yields 37x + 37y + 37z + 37w = 111, or x + y + z + w = 3. Subtract this from
the fourth given equation to obtain 4w = 22, or w = 11/2.

8. Determine the value of the sum


3 5 7 29
+ 2 2 + 2 2 + ··· + 2 .
12 ·22 2 ·3 3 ·4 14 · 152

Solution: 224/225 The sum telescopes as


1 1 1 1  1 1  1 1 224
− + − + · · · + − = 2− 2 = .
12 22 22 32 14 2 152 1 15 225

9. For any positive integer n, let f (n) denote the number of 1’s in the base-2 represen-
tation of n. For how many values of n with 1 ≤ n ≤ 2002 do we have f (n) = f (n + 1)?
Solution: 501 . If n is even, then n + 1 is obtained from n in binary by changing the
final 0 to a 1; thus f (n + 1) = f (n) + 1. If n is odd, then n + 1 is obtained by changing
the last 0 to a 1, the ensuing string of 1s to 0s, and then changing the next rightmost 0 to
a 1. This produces no net change in the number of 1’s iff n ends in 01 in base 2. Thus,

2
f (n + 1) = f (n) if and only if n is congruent to 1 mod 4, and there are 501 such numbers in
the specified range.

10. Determine the value of


1 1 1 1
2002 + (2001 + (2000 + · · · + (3 + · 2)) · · · ).
2 2 2 2

Solution: 4002 . We can show by induction that n+ 12 ([n−1]+ 12 (· · ·+ 12 ·2) · · · ) = 2(n−1).


For n = 3 we have 3 + 21 · 2 = 4, giving the base case, and if the result holds for n, then
(n + 1) + 12 2(n − 1) = 2n = 2(n + 1) − 2. Thus the claim holds, and now plug in n = 2002.
Alternate Solution: Expand the given expression as 2002+2001/2+2000/22 +· · ·+2/22000 .
Letting S denote this sum, we have S/2 = 2002/2 + 2001/22 + · · · + 2/22001 , so S − S/2 =
2002 − (1/2 + 1/4 + · · · + 1/22000 ) − 2/22001 = 2002 − (1 − 1/22000 ) − 1/22000 = 2001, so
S = 4002.

3
Harvard-MIT Math Tournament
March 17, 2002
Individual Subject Test: Calculus


1. Two circles have centers that are d units apart, and each has diameter d. For any d,
let A(d) be the area of the smallest circle that contains both of these circles. Find lim A(d)
d2
.
d→∞
 √ 2
d+ d
π 2 π
Solution: This equals lim d2
= 4
.
d→∞

x2 −(x+h)2
2. Find lim h
.
h→0
x2 −x2 −h2 −2hx
Solution: This equals lim h
= lim −h − 2x = −2x . Alternate Solution: This
h→0 h→0
2
is the definition of the derivative of −x with respect to x, which is −2x.

3. We are given the values of the differentiable real functions f, g, h, as well as the
derivatives of their pairwise products, at x = 0:

f (0) = 1; g(0) = 2; h(0) = 3; (gh)0 (0) = 4; (hf )0 (0) = 5; (f g)0 (0) = 6.

Find the value of (f gh)0 (0).


Solution: 16 By the product rule, (f gh)0 = f 0 gh + f g 0 h + f gh0 = ((f g)0 h + (gh)0 f +
(hf )0 g)/2. Evaluated at 0, this gives 16.

4. Find the area of the region in the first quadrant x > 0, y > 0 bounded above the
graph of y = arcsin(x) and below the graph of the y = arccos(x).
π/4
R
Solution: We can integrate over y rather than x. In particular, the solution is sin y dy+
0
π/2   √
√1
R
cos y dy = 1 − 2
2=2− 2.
π/4

5. What is the minimum vertical distance between the graphs of 2 + sin(x) and cos(x)?
Solution: The derivative of 2 + sin(x) − cos(x) is cosx + sinx, which in the interval
0 ≤ x < 2π is zero at x = 3π
4
, 7π
4
. At 7π
4
, when sin(x) is negative and cos(x) is positive, the

distance reaches its minimal value of 2 − 2 .

6. Determine the positive value of a such that the parabola y = x2 + 1 bisects the area
of the rectangle with vertices (0, 0), (a, 0), (0, a2 + 1), and (a, a2 + 1).

Solution: 3 The area of the rectangle is a3 + a. The portion under the parabola has
Ra 2
area 0 x + 1dx = a3 /3 + a. Thus we wish to solve the equation a3 + a = 2(a3 /3 + a);

dividing by a and rearranging gives a2 /3 = 1, so a = 3.

1
7. Denote by hxi the fractional part of the real number x (for instance, h3.2i = 0.2).

87set {1,
A positive integer N is selected randomly from the 2, 3, . . . , M }, with each integer
having the same probability of being picked, and 303 N is calculated. This procedure is
repeated M times and the average value A(M ) is obtained. What is lim A(M )?
M →∞
Solution: This method of picking N is equivalent

87 to uniformly randomly selecting a
positive integer. Call this the average value of 303 N for N a positive integer. In lowest
87 29 0 1
terms, 303 = 101 , so the answer is the same as the average value of 101 , 101 , . . . , 100
101
, which is
1+2+···+100 100·101/2 50
101·101
= 101·101
= 101
.

( 2−1)/2
dx
R
8. Evaluate √
(2x+1) x2 +x
.
0√
Solution: Let u = x2 + x. Then du = 2√2x+1 du
R
2 dx. So the integral becomes 2 (4x2 +4x+1) ,
−1
x +x √
or 2 4udu (2u), yielding a final answer of tan−1 (2 x2 + x) + C for the in-
R
2 +1 . This is tan

definite integral. The definite integral becomes tan−1 (1) − tan−1 (0) = π4 .

9. Suppose f is a differentiable real function such that f (x) + f 0 (x) ≤ 1 for all x, and
f (0) = 0. What is the largest possible value of f (1)? (Hint: consider the function ex f (x).)
Solution: 1 − 1/e Let g(x) = ex f (x); then g 0 (x) = ex (f (x) + f 0 (x)) ≤ ex . Integrating
R1 R1
from 0 to 1, we have g(1) − g(0) = 0 g 0 (x)dx ≤ 0 ex dx = e − 1. But g(1) − g(0) = e · f (1),
so we get f (1) ≤ (e − 1)/e. This maximum is attained if we actually have g 0 (x) = ex
everywhere; this entails the requirement f (x) + f 0 (x) = 1, which is met by f (x) = 1 − e−x .

R 1 10. A continuous real R 2 function f satisfies the identity f (2x) = 3f (x) for all x. If
0
f (x) dx = 1, what is 1 f (x) dx?
R2 R1
Solution: 5 Let S = 1 f (x) dx. By setting u = 2x, we see that 1/2 f (x) dx =
R1 R2 R 1/2
1/2
f (2x)/3 dx = 1 f (u)/6 du = S/6. Similarly, 1/4 f (x) dx = S/36, and in general
R 1/2n−1 R1
1/2n
f (x) dx = S/6n . Adding finitely many of these, we have 1/2n f (x) dx = S/6 + S/36 +
R1
· · · + S/6n = S · (1 − 1/6n )/5. Taking the limit as n → ∞, we have 0 f (x) dx = S/5. Thus
S = 5, the answer.

2
Harvard-MIT Math Tournament
March 17, 2002
Individual Subject Test: Geometry

1. A man, standing on a lawn, is wearing a circular sombrero of radius 3 feet. Unfortu-


nately, the hat blocks the sunlight so effectively that the grass directly under it dies instantly.
If the man walks in a circle of radius 5 feet, what area of dead grass will result?
Solution: 60π ft2 Let O be the center of the man’s circular trajectory. The sombrero
kills all the grass that is within 3 feet of any point that is 5 feet away from O — i.e. all
the grass at points P with 2 ≤ OP ≤ 8. The area of this annulus is then π(82 − 22 ) = 60π
square feet.

2. Dan is holding one end of a 26 inch long piece of light string that has a heavy bead
on it with each hand (so that the string lies along straight lines). If he starts with his hands
together at the start and leaves his hands at the same height, how far does he need to pull
his hands apart so that the bead moves upward by 8 inches?
Solution: After he pulls the bead is 5 inches below his hands, and it is 13 inches from
each hand. Using the Pythagorean theorem, his hands must be 2 · 12 = 24 inches apart.

3. A square
√ and a regular hexagon are drawn with the same side length. If the area of
the square is 3, what is the area of the hexagon? √
Solution: The hexagon is √composed of six equilateral triangles each of side length 4 3

(with base b = 4 3 and height 23 b), so the total area is 92 .

4. We call a set of professors and committees on which they serve a university if


(1) given two distinct professors there is one and only one committee on which they both
serve,
(2) given any committee, C, and any professor, P , not on that committee, there is exactly
one committee on which P serves and no professors on committee C serve, and
(3) there are at least two professors on each committee; there are at least two committees.
What is the smallest number of committees a university can have?
Solution: Let C be any committee. Then there exists a professor P not on C (or else
there would be no other committees). By axiom 2, P serves on a committee D having no
common members with C. Each of these committees has at least two members, and for
each Q ∈ C, R ∈ D, there exists (by axiom 1) a committee containing Q and R, which
(again by axiom 1) has no other common members with C or D. Thus we have at least
2 + 2 · 2 = 6 committees. This minimum is attainable - just take four professors and let any
two professors form a committee.

5. Consider a square of side length 1. Draw four lines that each connect a midpoint of a
side with a corner not on that side, such that each midpoint and each corner is touched by
only one line. Find the area of the region completely bounded by these lines.

1
Solution: In unit square ABCD, denote by E, F, G, H the respective midpoints of sides
AB, BC, CD, DA. Let I be the intersection of AF and DE, let J be the intersection of BG
and AF , let K be the intersection of CH and BG, and let L be the intersection of DE and
CH. We want√ to find the area of square IJKL. The area of ABF is 14 , which is equal to
1
2
AF · BJ = 45 BJ, so BJ = √15 . Using similar triangles, GK = JF = 12 BJ. Thus the

5 √1 1 √1 √1 , 1
length of a side of IJKL is JK = 2
− 5
− 2 5
= 5
and the area of IJKL is 5
.

6. If we pick (uniformly) a random square of area 1 with sides parallel to the x− and
y−axes that lies entirely within the 5-by-5 square bounded by the lines x = 0, x = 5, y =
0, y = 5 (the corners of the square need not have integer coordinates), what is the probability
that the point (x, y) = (4.5, 0.5) lies within the square of area 1?
Solution: The upper-left corner of the unit square is picked uniformly from the square
0 ≤ x ≤ 4; 1 ≤ y ≤ 5, and for it to contain the desired point it must lie in the square
3.5 ≤ x ≤ 4; 1 ≤ y ≤ 1.5. The answer is the ratio of the squares’ areas, 41 /16 = 641
.

7. Equilateral triangle ABC of side length 2 is drawn. Three squares external to the
triangle, ABDE, BCF G, and CAHI, are drawn. What is the area of the smallest triangle
that contains these squares?
Solution: The equilateral triangle with sides lying on lines DG, EH, and F I has minimal
area. (The only other reasonable candidate is the triangle with sides along DE, F G, HI, but
a quick sketch shows that it is larger.) Let J, K, and L be the vertices of this triangle closest
to D, H, and F , respectively. Clearly, KI = F L = √ 2. Triangle F CI is a 30◦ − 30◦ − 120◦
triangle, so we can calculate the length of F I as 2 3, making the side length of 4JKL
√ √
4 + 2 3, and its area 12 + 7 3 .

8. Equilateral triangle ABC of side length 2 is drawn. Three squares containing the
triangle, ABDE, BCF G, and CAHI, are drawn. What is the area of the smallest triangle
that contains these squares?
Solution: The equilateral triangle with sides lying on lines DE, F G, and HI has minimal
area. Let J, K, and L be the vertices of this triangle closest to D, H, and F , respectively.
◦ ◦ ◦
Clearly, DE = 2. Denote by M the √ intersection of CI and BD. √ Using the 30 − 30 − 120
triangle BCM , we get BM = 2/ 3, and thus M D = 2 − 2/ 3. By symmetry, M J √ bisects
angle DM I, from which we see that 4JM D is a 30◦ −60◦ −90◦ . We then get JD = 2 3−2,
√ √
making the side length of JKL 4 3 − 2, and its area 13 3 − 12 .

9. A and B are two points on a circle with center O, and C lies outside the circle, on
ray AB. Given that AB = 24, BC = 28, OA = 15, find OC.
Solution: 41 . Let M be the midpoint of AB; then 4OM B is a right triangle with
OB = 15, M B = 12, so OM = 9. Now 4OM C is a right triangle with OM = 9, M C = 40,
so OC = 41.

2
10. Let 4ABC be equilateral, and let D, E, F be points on sides BC, CA, AB respec-
tively, with F A = 9, AE = EC = 6, CD = 4. Determine the measure (in degrees) of
∠DEF .
Solution: 60 . Let H, I be the respective midpoints of sides BC, AB, and also extend
CB and EF to intersect at J. By equal angles, 4EIF ∼ 4JBF . However, BF = 12 − 9 =
3 = 9 − 6 = IF , so in fact 4EIF ∼ = 4JBF , and then JB = 6. Now let HI intersect EF
at K, and notice that 4EIK ∼ 4JHK ⇒ IK/HK = EI/JH = 6/12 = 1/2 ⇒ HK = 4,
since IK + HK = HI = 6. Now consider the 60◦ rotation about E carrying triangle CHE
to triangle HIE; we see that it also takes D to K, and thus ∠DEF = ∠DEK = 60◦ .

3
Harvard-MIT Math Tournament
March 17, 2002
Individual General Test: Part 1

1. What is the maximum number of lattice points (i.e. points with integer coordinates)
in the plane that can be contained strictly inside a circle of radius 1?
Solution: 4 . The circle centered at (1/2, 1/2) shows that 4 is achievable. On the other
hand, no two points within the circle can be at a mutual distance of 2 or greater. If there
are more than four lattice points, classify all such points by the parity of their coordinates:
(even, even), (even, odd), (odd, even), or (odd, odd). Then some two points lie in the same
class. Since they are distinct, this means either their first or second coordinates must differ
by at least 2, so their distance is at least 2, a contradiction.

2. Eight knights are randomly placed on a chessboard (not necessarily on distinct


squares). A knight on a given square attacks all the squares that can be reached by moving
either (1) two squares up or down followed by one squares left or right, or (2) two squares
left or right followed by one square up or down. Find the probability that every square,
occupied or not, is attacked by some knight.
Solution: 0 . Since every knight attacks at most eight squares, the event can only occur
if every knight attacks exactly eight squares. However, each corner square must be attacked,
and some experimentation readily finds that it is impossible to place a knight so as to attack
a corner and seven other squares as well.

3. How many triples (A, B, C) of positive integers (positive integers are the numbers
1, 2, 3, 4, . . .) are there such that A + B + C = 10, where order does not matter (for instance
the triples (2, 3, 5) and (3, 2, 5) are considered to be the same triple) and where two of the
integers in a triple could be the same (for instance (3, 3, 4) is a valid triple).
Solution: The triples can merely be enumerated: (1, 1, 8), (1, 2, 7), (1, 3, 6), (1, 4, 5),
(2, 2, 6), (2, 3, 5), (2, 4, 4), and (3, 3, 4). There are 8 elements.

4. We call a set of professors and committees on which they serve a university if


(1) given two distinct professors there is one and only one committee on which they both
serve,
(2) given any committee, C, and any professor, P , not on that committee, there is exactly
one committee on which P serves and no professors on committee C serve, and
(3) there are at least two professors on each committee; there are at least two committees.
What is the smallest number of committees a university can have?
Solution: Let C be any committee. Then there exists a professor P not on C (or else
there would be no other committees). By axiom 2, P serves on a committee D having no
common members with C. Each of these committees has at least two members, and for
each Q ∈ C, R ∈ D, there exists (by axiom 1) a committee containing Q and R, which
(again by axiom 1) has no other common members with C or D. Thus we have at least

1
2 + 2 · 2 = 6 committees. This minimum is attainable - just take four professors and let any
two professors form a committee.

5. A square
√ and a regular hexagon are drawn with the same side length. If the area of
the square is 3, what is the area of the hexagon? √
Solution: The hexagon is √composed of six equilateral triangles each of side length 4 3

(with base b = 4 3 and height 23 b), so the total area is 92 .

6. A man, standing on a lawn, is wearing a circular sombrero of radius 3 feet. Unfortu-


nately, the hat blocks the sunlight so effectively that the grass directly under it dies instantly.
If the man walks in a circle of radius 5 feet, what area of dead grass will result?
Solution: 60π ft2 Let O be the center of the man’s circular trajectory. The sombrero
kills all the grass that is within 3 feet of any point that is 5 feet away from O — i.e. all
the grass at points P with 2 ≤ OP ≤ 8. The area of this annulus is then π(82 − 22 ) = 60π
square feet.

7. A circle is inscribed in a square dartboard. If a dart is thrown at the dartboard and


hits the dartboard in a random location, with all locations having the same probability of
being hit, what is the probability that it lands within the circle?
Solution: The answer is the area of the circle over the area of the square, which is π4 .

8. Count the number of triangles with positive area whose vertices are points whose
(x, y)-coordinates lie in theset {(0, 0), (0, 1), (0, 2), (1, 0), (1, 1), (1, 2), (2, 0), (2, 1), (2, 2)}.
Solution: There are 93 = 84 triples of points. 8 of them form degenerate triangles (the
ones that lie on a line), so there are 84 − 8 = 76 nondegenerate triangles.

9. Real numbers a, b, c satisfy the equations a + b + c = 26, 1/a + 1/b + 1/c = 28. Find
the value of
a b c a c b
+ + + + + .
b c a c b a
Solution: 725 Multiplying the two given equations gives

a a a b b b c c c
+ + + + + + + + = 26 · 28 = 728,
a b c a b c a b c
and subtracting 3 from both sides gives the answer, 725.

10. A certain cafeteria serves ham and cheese sandwiches, ham and tomato sandwiches,
and tomato and cheese sandwiches. It is common for one meal to include multiple types of
sandwiches. On a certain day, it was found that 80 customers had meals which contained
both ham and cheese; 90 had meals containing both ham and tomatoes; 100 had meals
containing both tomatoes and cheese. 20 customers’ meals included all three ingredients.
How many customers were there?

2
Solution: 230 . Everyone who ate just one sandwich is included in exactly one of the
first three counts, while everyone who ate more than one sandwich is included in all four
counts. Thus, to count each customer exactly once, we must add the first three figures and
subtract the fourth twice: 80 + 90 + 100 − 2 · 20 = 230.

3
Harvard-MIT Math Tournament
March 17, 2002
Individual General Test: Part 2

1. The squares of a chessboard are numbered from left to right and top to bottom (so
that the first row reads 1, 2, . . . , 8, the second reads 9, 10, . . . , 16, and so forth). The number
1 is on a black square. How many black squares contain odd numbers?
Solution: 16 . The black squares in the nth row contain odd numbers when n is odd
and even numbers when n is even; thus there are four rows where the black squares contain
odd numbers, and each such row contributes four black squares.

2. You are in a completely dark room with a drawer containing 10 red, 20 blue, 30 green,
and 40 khaki socks. What is the smallest number of socks you must randomly pull out in
order to be sure of having at least one of each color?
Solution: 91 . The maximum number of socks that can be pulled out without repre-
senting every color is 20 blue + 30 green + 40 khaki = 90, so 91 is the minimum needed to
ensure that this doesn’t happen.

q p
3. Solve for x in 3 = x + x + x + · · ·.
Solution: Squaring both sides and subtracting x from both sides, we get 9 − x = 3, or
x=6.

4. Dan is holding one end of a 26 inch long piece of light string that has a heavy bead
on it with each hand (so that the string lies along two straight lines). If he starts with his
hands together at the start and leaves his hands at the same height, how far does he need
to pull his hands apart so that the bead moves upward by 8 inches?
Solution: After he pulls the bead is 5 inches below his hands, and it is 13 inches from
each hand. Using the Pythagorean theorem, his hands must be 2 · 12 = 24 inches apart.

5. A square
√ and a regular hexagon are drawn with the same side length. If the area of
the square is 3, what is the area of the hexagon? √
Solution: The hexagon is √composed of six equilateral triangles each of side length 4 3

(with base b = 4 3 and height 23 b), so the total area is 92 .

6. Nine nonnegative numbers have average 10. What is the greatest possible value for
their median?
Solution: 18 If the median is m, then the five highest numbers are all at least m, so
the sum of all the numbers is at least 5m. Thus 90 ≥ 5m ⇒ m ≤ 18. Conversely, we can
achieve m = 18 by taking four 0’s and five 18’s.

7. p and q are primes such that the numbers p + q and p + 7q are both squares. Find
the value of p.

1
Solution: 2 . Writing x2 = p + q, y 2 = p + 7q, we have 6q = y 2 − x2 = (y − x)(y + x).
Since 6q is even, one of the factors y − x, y + x is even, and then the other is as well; thus
6q is divisible by 4 ⇒ q is even ⇒ q = 2 and 6q = 12. We may assume x, y are both taken
to be positive; then we must have y − x = 2, y + x = 6 ⇒ x = 2, so p + 2 = 22 = 4 ⇒ p = 2
also.

8. Two fair coins are simultaneously flipped. This is done repeatedly until at least one
of the coins comes up heads, at which point the process stops. What is the probability that
the other coin also came up heads on this last flip?
Solution: 1/3 . Let the desired probability be p. There is a 1/4 chance that both coins
will come up heads on the first toss. Otherwise, both can come up heads simultaneously
only if both are tails on the first toss, and then the process restarts as if from the beginning;
thus this situation occurs with probability p/4. Thus p = 1/4 + p/4; solving, p = 1/3.
Alternate Solution: The desired event is equivalent to both coins coming up tails for n
successive turns (for some n ≥ 0), then both coins coming up heads. For any fixed value of
n, the probability of this occurring is 1/4n+1 . Since all these events are disjoint, the total
probability is 1/4 + 1/42 + 1/43 + · · · = 1/3.

9. A and B are two points on a circle with center O, and C lies outside the circle, on
ray AB. Given that AB = 24, BC = 28, OA = 15, find OC.
Solution: 41 . Let M be the midpoint of AB; then 4OM B is a right triangle with
OB = 15, M B = 12, so OM = 9. Now 4OM C is a right triangle with OM = 9, M C = 40,
so OC = 41.

10. How many four-digit numbers are there in which at least one digit occurs more than
once?
Solution: 4464 . There are 9000 four-digit numbers altogether. If we consider how
many four-digit numbers have all their digits distinct, there are 9 choices for the first digit
(since we exclude leading zeroes), and then 9 remaining choices for the second digit, then
8 for the third, and 7 for the fourth, for a total of 9 · 9 · 8 · 7 = 4536. Thus the remaining
9000 − 4536 = 4464 numbers have a repeated digit.

2
Harvard-MIT Math Tournament
March 17, 2002
Individual Subject Test: Advanced Topics

1. Eight knights are randomly placed on a chessboard (not necessarily on distinct


squares). A knight on a given square attacks all the squares that can be reached by moving
either (1) two squares up or down followed by one squares left or right, or (2) two squares
left or right followed by one square up or down. Find the probability that every square,
occupied or not, is attacked by some knight.
Solution: 0 . Since every knight attacks at most eight squares, the event can only occur
if every knight attacks exactly eight squares. However, each corner square must be attacked,
and some experimentation readily finds that it is impossible to place a knight so as to attack
a corner and seven other squares as well.

2. A certain cafeteria serves ham and cheese sandwiches, ham and tomato sandwiches,
and tomato and cheese sandwiches. It is common for one meal to include multiple types of
sandwiches. On a certain day, it was found that 80 customers had meals which contained
both ham and cheese; 90 had meals containing both ham and tomatoes; 100 had meals
containing both tomatoes and cheese. 20 customers’ meals included all three ingredients.
How many customers were there?
Solution: 230 . Everyone who ate just one sandwich is included in exactly one of the
first three counts, while everyone who ate more than one sandwich is included in all four
counts. Thus, to count each customer exactly once, we must add the first three figures and
subtract the fourth twice: 80 + 90 + 100 − 2 · 20 = 230.

3. How many four-digit numbers are there in which at least one digit occurs more than
once?
Solution: 4464 . There are 9000 four-digit numbers altogether. If we consider how
many four-digit numbers have all their digits distinct, there are 9 choices for the first digit
(since we exclude leading zeroes), and then 9 remaining choices for the second digit, then
8 for the third, and 7 for the fourth, for a total of 9 · 9 · 8 · 7 = 4536. Thus the remaining
9000 − 4536 = 4464 numbers have a repeated digit.

4. Two fair coins are simultaneously flipped. This is done repeatedly until at least one
of the coins comes up heads, at which point the process stops. What is the probability that
the other coin also came up heads on this last flip?
Solution: 1/3 . Let the desired probability be p. There is a 1/4 chance that both coins
will come up heads on the first toss. Otherwise, both can come up heads simultaneously
only if both are tails on the first toss, and then the process restarts as if from the beginning;
thus this situation occurs with probability p/4. Thus p = 1/4 + p/4; solving, p = 1/3.
Alternate Solution: The desired event is equivalent to both coins coming up tails for n
successive turns (for some n ≥ 0), then both coins coming up heads. For any fixed value of

1
n, the probability of this occurring is 1/4n+1 . Since all these events are disjoint, the total
probability is 1/4 + 1/42 + 1/43 + · · · = 1/3.

5. Determine the number of subsets S of {1, 2, 3, . . . , 10} with the following property:
there exist integers a < b < c with a ∈ S, b ∈
/ S, c ∈ S.
Solution: 968 There are 210 = 1024 subsets of {1, 2, . . . , 10} altogether. Any subset
without the specified property must be either the empty set or a block of consecutive integers.
To specify a block of consecutive
 integers, we either have just one element (10 choices) or a
10
pair of distinct endpoints ( 2 = 45 choices). So the number of sets with our property is
1024 − (1 + 10 + 45) = 968.

6. In how many ways can the numbers 1, 2, . . . , 2002 be placed at the vertices of a regular
2002-gon so that no two adjacent numbers differ by more than 2? (Rotations and reflections
are considered distinct.)
Solution: 4004 . There are 2002 possible positions for the 1. The two numbers adjacent
to the 1 must be 2 and 3; there are two possible ways of placing these. The positions of
these numbers uniquely determine the rest: for example, if 3 lies clockwise from 1, then the
number lying counterclockwise from 2 must be 4; the number lying clockwise from 3 must
be 5; the number lying counterclockwise from 4 must now be 6; and so forth. Eventually,
2002 is placed adjacent to 2000 and 2001, so we do get a valid configuration. Thus there are
2002 · 2 possible arrangements.

7. A manufacturer of airplane parts makes a certain engine that has a probability p of


failing on any given flight. Their are two planes that can be made with this sort of engine,
one that has 3 engines and one that has 5. A plane crashes if more than half its engines fail.
For what values of p do the two plane models have the same probability ofcrashing?
Solution: They have the same probability of failing if 52 p3 (1 − p)2 + 51 p4 (1 − p) + p5 =
3 2

1
p (1 − p) + p3 , which is true iff p2 (6p3 − 15p2 + 12p − 3) = 0. This is clearly true for p = 0.
We know it is true for p = 1, since both probabilities would be 1 in this case, so we know
p − 1 is a factor of 6p3 − 15p2 + 12p − 3. Thus, factoring gives that the engines have the
same probability of failing if p2 (p − 1)(6p2 − 9p + 3) = 0. By the quadratic formula (or by
factoring), the quadratic has roots p = 21 , 1, so the answer is 0, 12 , 1 .

8. Given a 9 × 9 chess board, we consider all the rectangles whose edges lie along grid
lines (the board consists of 81 unit squares, and the grid lines lie on the borders of the unit
squares). For each such rectangle, we put a mark in every one of the unit squares inside
it. When this process is completed, how many unit squares will contain an even number of
marks?
Solution: 56 . Consider the rectangles which contain the square in the ith row and jth
column. There are i possible positions for the upper edge of such a rectangle, 10 − i for the
lower edge, j for the left edge, and 10 − j for the right edge; thus we have i(10 − i)j(10 − j)
rectangles altogether, which is odd iff i, j are both odd, i.e. iff i, j ∈ {1, 3, 5, 7, 9}. There are
thus 25 unit squares which lie in an odd number of rectangles, so the answer is 81 − 25 = 56.

2
9. Given that a, b, c are positive real numbers and loga b + logb c + logc a = 0, find the
value of (loga b)3 + (logb c)3 + (logc a)3 .
Solution: 3 . Let x = loga b and y = logb c; then logc a = −(x + y). Thus we want to
compute the value of x3 + y 3 − (x + y)3 = −3x2 y − 3xy 2 = −3xy(x + y). On the other hand,
−xy(x + y) = (loga b)(logb c)(logc a) = 1, so the answer is 3.

10. One fair die is rolled; let a denote the number that comes up. We then roll a dice;
let the sum of the resulting a numbers be b. Finally, we roll b dice, and let c be the sum of
the resulting b numbers. Find the expected (average) value of c.
Solution: 343/8 The expected result of an individual die roll is (1+2+3+4+5+6)/6 =
7/2. For any particular value of b, if b dice are rolled independently, then the expected sum
is (7/2)b. Likewise, when we roll a dice, the expected value of their sum b is (7/2)a, so the
expected value of c is (7/2)2 a. Similar reasoning again shows us that the expected value of
a is 7/2 and so the expected value of c overall is (7/2)3 = 343/8.

3
Harvard-MIT Math Tournament
March 17, 2002
Guts Round

1. An (l, a)-design of a set is a collection of subsets of that set such that each subset
contains exactly l elements and that no two of the subsets share more than a elements. How
many (2,1)-designs are there  of a set containing 8 elements?
8
Solution: There are 2 = 28 2-element subsets. Any two distinct such subsets have at
most 1 common element; hence, for each subset, we can decide independently whether or
not it belongs to the design, and we thus obtain 228 designs.

2. A lattice point in the plane is a point of the form (n, m), where n and m are integers.
Consider a set S of lattice points. We construct the transform of S, denoted by S 0 , by the
following rule: the pair (n, m) is in S 0 if and only if any of (n, m − 1), (n, m + 1), (n − 1, m),
(n + 1, m), and (n, m) is in S. How many elements are in the set obtained by successively
transforming {(0, 0)} 14 times?
Solution: Transforming it k ≥ 1 times yields the “diamond” of points (n, m) such that
|n| + |m| ≤ k. The diamond contains (k + 1)2 + k 2 lattice points (this can be seen by rotating
the plane 45 degrees and noticing the lattice points in the transforms form two squares, one
of which is contained in the other), so the answer is 421 .

3. How many elements are in the set obtained by transforming {(0, 0), (2, 0)} 14 times?
Solution: Transforming it k ≥ 1 times yields the diamond {(n, m) : |n−1|+|m| ≤ k +1}
with the points (1, k), (1, k + 1), (1, −k), (1, −k − 1) removed (this can be seen inductively).
So we get (k + 1)2 + k 2 − 4 lattice points, making the answer 477 .

4. How many ways are there of using diagonals to divide a regular 6-sided polygon into
triangles such that at least one side of each triangle is a side of the original polygon and that
each vertex of each triangle is a vertex of the original polygon?
Solution: The number of ways of triangulating a convex (n+2)-sided polygon is 2n
 1
n n+1
,
which is 14 in this case. However, there are two triangulations of a hexagon which produce
one triangle sharing no sides with the original polygon, so the answer is 14 − 2 = 12 .

5. Two 4 × 4 squares are randomly placed on an 8 × 8 chessboard so that their sides lie
along the grid lines of the board. What is the probability that the two squares overlap?
Solution: 529/625 . Each square has 5 horizontal · 5 vertical = 25 possible positions,
so there are 625 possible placements of the squares. If they do not overlap, then either
one square lies in the top four rows and the other square lies in the bottom four rows, or
one square lies in the left four columns and the other lies in the right four columns. The
first possibility can happen in 2 · 5 · 5 = 50 ways (two choices of which square goes on
top, and five horizontal positions for each square); likewise, so can the second. However,

1
this double-counts the 4 cases in which the two squares are in opposite corners, so we have
50 + 50 − 4 = 96 possible non-overlapping arrangements ⇒ 252 − 96 = 529 overlapping
arrangements.

6. Find all values of x that satisfy x = 1 − x + x2 − x3 + x4 − x5 + · · · (be careful; this


is tricky). √
Solution: Multiplying both sides by 1 + x gives (1 + x)x = 1, or x = −1±2 5 . However,

−1+ 5
the series only converges for |x| < 1, so only the answer x = 2
makes sense.

7. A rubber band is 4 inches long. An ant begins at the left end. Every minute, the
ant walks one inch along rightwards along the rubber band, but then the band is stretched
(uniformly) by one inch. For what value of n will the ant reach the right end during the nth
minute?
Solution: 7 The ant traverses 1/4 of the band’s length in the first minute, 1/5 of the
length in the second minute (the stretching does not affect its position as a fraction of the
band’s length), 1/6 of the length in the third minute, and so on. Since

1/4 + 1/5 + · · · + 1/9 < 0.25 + 0.20 + 0.167 + 0.143 + 0.125 + 0.112 = 0.997 < 1,

the ant does not cover the entire band in six minutes. However,

1/4 + · · · + 1/10 > 0.25 + 0.20 + 0.16 + 0.14 + 0.12 + 0.11 + 0.10 = 1.08 > 1,

so seven minutes suffice.

8. Draw a square of side length 1. Connect its sides’ midpoints to form a second square.
Connect the midpoints of the sides of the second square to form a third square. Connect the
midpoints of the sides of the third square to form a fourth square. And so forth. What is
the sum of the areas of all the squares in this infinite series?
Solution: The area of the first square is 1, the area of the second is 21 , the area of the
third is 41 , etc., so the answer is 1 + 12 + 14 + 18 + · · · = 2 .

9. Find all values of x with 0 ≤ x < 2π that satisfy sin x + cos x = 2.
Solution: Squaring both sides gives sin2 x + cos2 x + 2 sin x cos x = 1 + sin 2x = 2, so
x = π4 , 5π
4
.

10. The mathematician John is having trouble remembering his girlfriend Alicia’s 7-digit
phone number. He remembers that the first four digits consist of one 1, one 2, and two 3s.
He also remembers that the fifth digit is either a 4 or 5. While he has no memory of the
sixth digit, he remembers that the seventh digit is 9 minus the sixth digit. If this is all the
information he has, how many phone numbers does he have to try if he is to make sure he
dials the correct number?

2
Solution: There are 4! 2!
= 12 possibilities for the first four digits. There are two pos-
sibilities for the fifth digit. There are 10 possibilities for the sixth digit, and this uniquely
determines the seventh digit. So he has to dial 12 · 2 · 10 = 240 numbers.
11. How many real solutions are there to the equation
||||x| − 2| − 2| − 2| = ||||x| − 3| − 3| − 3| ?
Solution: 6 . The graphs of the two sides of the equation can be graphed on the same
plot to reveal six intersection points.
12. This question forms a three question multiple choice test. After each question, there
are 4 choices, each preceded by a letter. Please write down your answer as the ordered triple
(letter of the answer of Question #1, letter of the answer of Question #2, letter of the answer
of Question #3). If you find that all such ordered triples are logically impossible, then write
“no answer” as your answer. If you find more than one possible set of answers, then provide
all ordered triples as your answer.
When we refer to “the correct answer to Question X” it is the actual answer, not the
letter, to which we refer. When we refer to “the letter of the correct answer to question X”
it is the letter contained in parentheses that precedes the answer to which we refer.
You are given the following condition: No two correct answers to questions on the test
may have the same letter.
Question 1. If a fourth question were added to this test, and if the letter of its correct answer
were (C), then:
(A) This test would have no logically possible set of answers.
(B) This test would have one logically possible set of answers.
(C) This test would have more than one logically possible set of answers.
(D) This test would have more than one logically possible set of answers.
Question 2. If the answer to Question 2 were “Letter (D)” and if Question 1 were not on
this multiple-choice test (still keeping Questions 2 and 3 on the test), then the letter of the
answer to Question 3 would be:
(A) Letter (B)
(B) Letter (C)
(C) Letter (D)
(D) Letter (A)
Question 3. Let P1 = 1. Let P2 = 3. For all i > 2, define Pi = Pi−1 Pi−2 − Pi−2 . Which is a
factor of P2002 ?
(A) 3
(B) 4
(C) 7
(D) 9
Solution: (A, C, D) . Question 2: In order for the answer to be consistent with the
condition, “If the answer to Question 2 were Letter (D),” the answer to this question actually
must be “Letter (D).” The letter of this answer is (C).

3
Question 1: If a fourth question had an answer with letter (C), then at least two answers
would have letter (C) (the answers to Questions 2 and 4). This is impossible. So, (A) must
be the letter of the answer to Question 1.
Question 3: If we inspect the sequence Pi modulo 3, 4, 7, and 9 (the sequences quickly
become periodic), we find that 3, 7, and 9 are each factors of P2002 . We know that letters
(A) and (C) cannot be repeated, so the letter of this answer must be (D).

13. A domino is a 1-by-2 or 2-by-1 rectangle. A domino tiling of a region of the plane
is a way of covering it (and only it) completely by nonoverlapping dominoes. For instance,
there is one domino tiling of a 2-by-1 rectangle and there are 2 tilings of a 2-by-2 rectangle
(one consisting of two horizontal dominoes and one consisting of two vertical dominoes).
How many domino tilings are there of a 2-by-10 rectangle?
Solution: The number of tilings of a 2-by-n, rectangle is the nth Fibonacci number Fn ,
where F0 = F1 = 1 and Fn = Fn−1 + Fn−1 for n ≥ 2. (This is not hard to show by induction.)
The answer is 89 .

14. An omino is a 1-by-1 square or a 1-by-2 horizontal rectangle. An omino tiling of a


region of the plane is a way of covering it (and only it) by ominoes. How many omino tilings
are there of a 2-by-10 horizontal rectangle?
Solution: There are exactly as many omino tilings of a 1-by-n rectangle as there are
domino tilings of a 2-by-n rectangle. Since the rows don’t interact at all, the number of
omino tilings of an m-by-n rectangle is the number of omino tilings of a 1-by-n rectangle
raised to the mth power, Fnm . The answer is 892 = 7921 .

15. How many sequences of 0s and 1s are there of length 10 such that there are no three
0s or 1s consecutively anywhere in the sequence?
Solution: We can have blocks of either 1 or 2 0s and 1s, and these blocks must be
alternating between 0s and 1s. The number of ways of arranging blocks to form a sequence
of length n is the same as the number of omino tilings of a 1-by-n rectangle, and we may
start each sequence with a 0 or a 1, making 2Fn or, in this case, 178 sequences.

16. Divide an m-by-n rectangle into mn nonoverlapping 1-by-1 squares. A polyomino of


this rectangle is a subset of these unit squares such that for any two unit squares S, T in the
polyomino, either
(1) S and T share an edge or
(2) there exists a positive integer n such that the polyomino contains unit squares
S1 , S2 , S3 , . . . , Sn such that S and S1 share an edge, Sn and T share an edge, and for all
positive integers k < n, Sk and Sk+1 share an edge.
We say a polyomino of a given rectangle spans the rectangle if for each of the four edges
of the rectangle the polyomino contains a square whose edge lies on it.
What is the minimum number of unit squares a polyomino can have if it spans a 128-by-
343 rectangle?

4
Solution: To span an a × b rectangle, we need at least a + b − 1 squares. Indeed, consider
a square of the polyomino bordering the left edge of the rectangle and one bordering the
right edge. There exists a path connecting these squares; suppose it runs through c different
rows. Then the path requires at least b − 1 horizontal and c − 1 vertical steps, so it uses at
least b + c − 1 different squares. However, since the polyomino also hits the top and bottom
edges of the rectangle, it must run into the remaining a − c rows as well, so altogether we
need at least a + b − 1 squares. On the other hand, this many squares suffice — just consider
all the squares bordering the lower or right edges of the rectangle. So, in our case, the answer
is 128 + 343 − 1 = 470 .

17. Find the number of pentominoes (5-square polyominoes) that span a 3-by-3 rect-
angle, where polyominoes that are flips or rotations of each other are considered the same
polyomino.
Solution: By enumeration, the answer is 6 .

18. Call the pentominoes found in the last problem square pentominoes. Just like
dominos and ominos can be used to tile regions of the plane, so can square pentominoes. In
particular, a square pentomino tiling of a region of the plane is a way of covering it (and only
it) completely by nonoverlapping square pentominoes. How many square pentomino tilings
are there of a 12-by-12 rectangle?
Solution: Since 5 does not divide 144, there are 0 .

19. For how many integers a (1 ≤ a ≤ 200) is the number aa a square?


Solution: 107 If a is even, we have aa = (aa/2 )2 . If a is odd, aa = (a(a−1)/2 )2 · a, which
is a square precisely when a is. Thus we have 100 even values of a and 7 odd square values
(12 , 32 , . . . , 132 ) for a total of 107.

20. The Antarctican language has an alphabet of just 16 letters. Interestingly, every
word in the language has exactly 3 letters, and it is known that no word’s first letter equals
any word’s last letter (for instance, if the alphabet were {a, b} then aab and aaa could not
both be words in the language because a is the first letter of a word and the last letter
of a word; in fact, just aaa alone couldn’t be in the language). Given this, determine the
maximum possible number of words in the language.
Solution: 1024 Every letter can be the first letter of a word, or the last letter of a
word, or possibly neither, but not both. If there are a different first letters and b different
last letters, then we can form a · 16 · b different words (and the desired conditions will be
met). Given the constraints 0 ≤ a, b; a + b ≤ 16, this product is maximized when a = b = 8,
giving the answer.

21. The Dyslexian alphabet consists of consonants and vowels. It so happens that a
finite sequence of letters is a word in Dyslexian precisely if it alternates between consonants
and vowels (it may begin with either). There are 4800 five-letter words in Dyslexian. How
many letters are in the alphabet?

5
Solution: 12 Suppose there are c consonants, v vowels. Then there are c · v · c · v · c +
v · c · v · c · v = (cv)2 (c + v) five-letter words. Thus, c + v = 4800/(cv)2 = 3 · (40/cv)2 , so cv is
a divisor of 40. If cv ≤ 10, we have c + v ≥ 48, impossible for c, v integers; if cv = 40, then
c + v = 3 which is again impossible. So cv = 20, giving c + v = 12, the answer. As a check,
this does have integer solutions: (c, v) = (2, 10) or (10, 2).

22. A path of length n is a sequence of points (x1 , y1 ), (x2 , y2 ), . . . , (xn , yn ) with integer
coordinates such that for all i between 1 and n − 1 inclusive, either
(1) xi+1 = xi + 1 and yi+1 = yi (in which case we say the ith step is rightward ) or
(2) xi+1 = xi and yi+1 = yi + 1 (in which case we say that the ith step is upward ).
This path is said to start at (x1 , y1 ) and end at (xn , yn ). Let P (a, b), for a and b nonneg-
ative integers, be the number of paths that start at (0, 0) and end at (a, b).
10
P
Find P (i, 10 − i).
i=0
Solution: This is just the number of paths of length 10. The ith step can be either
upward or rightward, so there are 210 = 1024 such paths.

23. Find P (7, 3).


Solution: The number of paths that start at (0, 0) and end at (n, m) is n+m

n
, since we
must choose n of our n + m steps to be rightward steps. In this case, the answer is 120 .

24. A restricted path of length n is a path of length n such that for all i between 1 and
n − 2 inclusive, if the ith step is upward, the i + 1st step must be rightward.
Find the number of restricted paths that start at (0, 0) and end at (7, 3).
Solution: This is equal to the number of lattice paths from (0, 0) to (7, 3) that use only
rightward and diagonal (upward+rightward) steps plus the number of lattice paths from
(0, 0) to (7, 2) that use only rightward and diagonal steps, which is equal to the number of
paths (as
 defined above) from (0, 0) to (4, 3) plus the number of paths from (0, 0) to (5, 2),
or 4+33
+ 5+2
2
= 56 .

25. A math professor stands up in front of a room containing 100 very smart math
students and says, “Each of you has to write down an integer between 0 and 100, inclusive,
to guess ‘two-thirds of the average of all the responses.’ Each student who guesses the highest
integer that is not higher than two-thirds of the average of all responses will receive a prize.”
If among all the students it is common knowledge that everyone will write down the best
response, and there is no communication between students, what single integer should each
of the 100 students write down?
Solution: Since the average cannot be greater than 100, no student will write down
a number greater than 23 · 100. But then the average cannot be greater than 23 · 100, and,
realizing this, each student will write down a number no greater than ( 23 )2 · 100. Continuing
in this manner, we eventually see that no student will write down an integer greater than
0 , so this is the answer.

6
26. Another professor enters the same room and says, “Each of you has to write down an
integer between 0 and 200. I will then compute X, the number that is 3 greater than half the
average of all the numbers that you will have written down. Each student who writes down
the number closest to X (either above or below X) will receive a prize.” One student, who
misunderstood the question, announces to the class that he will write the number 107. If
among the other 99 students it is common knowledge that all 99 of them will write down the
best response, and there is no further communication between students, what single integer
should each of the 99 students write down?
Solution: Use the same logic to get 7 . Note 6 and 8 do not work.

27. Consider the two hands of an analog clock, each of which moves with constant
angular velocity. Certain positions of these hands are possible (e.g. the hour hand halfway
between the 5 and 6 and the minute hand exactly at the 6), while others are impossible (e.g.
the hour hand exactly at the 5 and the minute hand exactly at the 6). How many different
positions are there that would remain possible if the hour and minute hands were switched?
Solution: 143 We can look at the twelve-hour cycle beginning at midnight and ending
just before noon, since during this time, the clock goes through each possible position exactly
once. The minute hand has twelve times the angular velocity of the hour hand, so if the
hour hand has made t revolutions from its initial position (0 ≤ t < 1), the minute hand
has made 12t revolutions. If the hour hand were to have made 12t revolutions, the minute
hand would have made 144t. So we get a valid configuration by reversing the hands precisely
when 144t revolutions land the hour hand in the same place as t revolutions — i.e. when
143t = 144t − t is an integer, which clearly occurs for exactly 143 values of t corresponding
to distinct positions on the clock (144 − 1 = 143).

28. Count how many 8-digit 4 numbers there are that contain exactly four nines as digits.
8
Solution: There are 4 · 9 sequences of 8 numbers with exactly four nines. A sequence
of digits of length8 is not an 8-digit number, however, if and only if the first digit is
zero. There are 74 93 8-digit sequences that are not 8-digit numbers. The answer is thus
8 7 3
 4 
4
· 9 − 4
9 = 433755 .

29. A sequence s0 , s1 , s2 , s3 , . . . is defined by s0 = s1 = 1 and, for every positive integer


n, s2n = sn , s4n+1 = s2n+1 , s4n−1 = s2n−1 + s22n−1 /sn−1 . What is the value of s1000 ?
Solution: 720 Some experimentation with small values may suggest that sn = k!, where
k is the number of ones in the binary representation of n, and this formula is in fact provable
by a straightforward induction. Since 100010 = 11111010002 , with six ones, s1000 = 6! = 720.

30. A conical flask contains some water. When the flask is oriented so that its base is
horizontal and lies at the bottom (so that the vertex is at the top), the water is 1 inch deep.
When the flask is turned upside-down, so that the vertex is at the bottom, the water is 2
inches deep. What is the height of the cone?

7

Solution: 12 + 693 . Let h be the height, and let V be such that V h3 equals the volume
of the flask. When the base is at the bottom, the portion of the flask not occupied by water
forms a cone similar to the entire flask, with a height of h − 1; thus its volume is V (h − 1)3 .
When the base is at the top, the water occupies a cone with a height of 2, so its volume is
V · 23 . Since the water’s volume does not change,

V h3 − V (h − 1)3 = 8V

⇒ 3h2 − 3h + 1 = h3 − (h − 1)3 = 8
⇒ 3h2 − 3h − 7 = 0.

1 93
Solving via the quadratic formula and taking the positive root gives h = 2
+ 6
.

31. Express, as concisely as possible, the value of the product

(03 − 350)(13 − 349)(23 − 348)(33 − 347) · · · (3493 − 1)(3503 − 0).

Solution: 0 . One of the factors is 73 − 343 = 0, so the whole product is zero.

32. Two circles have radii 13 and 30, and their centers are 41 units apart. The line
through the centers of the two circles intersects the smaller circle at two points; let A be the
one outside the larger circle. Suppose B is a point on the smaller circle and C a point on
the larger circle such that B is the midpoint of AC. Compute the distance AC.

Solution: 12 13 Call the large circle’s center O1 . Scale the small circle by a factor of
2 about A; we obtain a new circle whose center O2 is at a distance of 41 − 13 = 28 from O1 ,
and whose radius is 26. Also, the dilation sends B to C, which thus lies on circles O1 and O2 .
So points O1 , O2 , C form a 26-28-30 triangle. Let H be the foot of the √ altitude from √C to
O1 O2 ; we have CH = 24 and HO2 = 10. Thus, HA = 36, and AC = 242 + 362 = 12 13.

33. The expression bxc denotes the greatest integer less than or equal to x. Find the
value of  
2002!
.
2001! + 2000! + 1999! + · · · + 1!
Solution: 2000 We break up 2002! = 2002(2001)! as

2000(2001!) + 2 · 2001(2000!) = 2000(2001!) + 2000(2000!) + 2002 · 2000(1999!)

> 2000(2001! + 2000! + 1999! + · · · + 1!).


On the other hand,

2001(2001! + 2000! + · · · + 1!) > 2001(2001! + 2000!) = 2001(2001!) + 2001! = 2002!.

Thus we have 2000 < 2002!/(2001! + · · · + 1!) < 2001, so the answer is 2000.

8

34. Points P and Q are 3 units apart. A circle centered at P with a radius of 3 units
intersects a circle centered at Q with a radius of 3 units at points A and B. Find the area
of quadrilateral APBQ.
Solution: The area is twice the area of triangle AP Q, whichqis isosceles with side lengths
√ √ √
3, 3, 3. By Pythagoras, the altitude to the base has length 32 − ( 3/2)2 = 33/2, so
√ √
99 3 11
the triangle has area 4
. Double this to get 2
.

35. Suppose a, b, c, d are real numbers such that

|a − b| + |c − d| = 99; |a − c| + |b − d| = 1.

Determine all possible values of |a − d| + |b − c|.


Solution: 99 If w ≥ x ≥ y ≥ z are four arbitrary real numbers, then |w − z| + |x − y| =
|w − y| + |x − z| = w + x − y − z ≥ w − x + y − z = |w − x| + |y − z|. Thus, in our case,
two of the three numbers |a − b| + |c − d|, |a − c| + |b − d|, |a − d| + |b − c| are equal, and the
third one is less than or equal to these two. Since we have a 99 and a 1, the third number
must be 99.
2 3
36. Find the set consisting of all real values of x such that the three numbers 2x , 2x , 2x
form a non-constant arithmetic progression (in that order).
Solution: The empty set, Ø . Trivially, x = 0, 1 yield constant arithmetic progressions;
we show that there are no other possibilities. If these numbers do form a progression, then,
by the AM-GM (arithmetic mean-geometric mean) inequality,
2 3

2 · 2x = 2x + 2x ≥ 2 2x · 2x3
2 3 )/2
⇒ 2x ≥ 2(x+x ⇒ x2 ≥ (x + x3 )/2
⇒ x(x − 1)2 = x3 − 2x2 + x ≤ 0.
3
Assuming x 6= 0, 1, we can divide by (x−1)2 > 0 and obtain x < 0. However, then 2x , 2x are
2
less than 1, while 2x is more than 1, so the given sequence cannot possibly be an arithmetic
progression.

37. Call a positive integer “mild” if its base-3 representation never contains the digit 2.
How many values of n (1 ≤ n ≤ 1000) have the property that n and n2 are both mild?
Solution: 7 Such a number, which must consist entirely of 0’s and 1’s in base 3, can
never have more than one 1. Indeed, if n = 3a + 3b + higher powers where b > a, then
n2 = 32a + 2 · 3a+b + higher powers which will not be mild. On the other hand, if n does just
have one 1 in base 3, then clearly n and n2 are mild. So the values of n ≤ 1000 that work
are 30 , 31 , . . . , 36 ; there are 7 of them.

9
38. Massachusetts Avenue is ten blocks long. One boy and one girl live on each block.
They want to form friendships such that each boy is friends with exactly one girl and vice-
versa. Nobody wants a friend living more than one block away (but they may be on the
same block). How many pairings are possible?
Solution: 89 Let an be the number of pairings if there are n blocks; we have a1 =
1, a2 = 2, and we claim the Fibonacci recurrence is satisfied. Indeed, if there are n blocks,
either the boy on block 1 is friends with the girl on block 1, leaving an−1 possible pairings
for the people on the remaining n − 1 blocks, or he is friends with the girl on block 2, in
which case the girl on block 1 must be friends with the boy on block 2, and then there are
an−2 possibilities for the friendships among the remaining n − 2 blocks. So an = an−1 + an−2 ,
and we compute: a3 = 3, a4 = 5, . . . , a10 = 89.

39. In the x-y plane, draw a circle of radius 2 centered at (0, 0). Color the circle red
above the line y = 1, color the circle blue below the line y = −1, and color the rest of the
circle white. Now consider an arbitrary straight line at distance 1 from the circle. We color
each point P of the line with the color of the closest point to P on the circle. If we pick such
an arbitrary line, randomly oriented, what is the probability that it contains red, white, and
blue points?
Solution: Let O = (0, 0), P = (1, 0), and H the foot of the perpendicular from O to
the line. If ∠P OH (as measured counterclockwise) lies between π/3 and 2π/3, the line will
fail to contain blue points; if it lies between 4π/3 and 5π/3, the line will fail to contain red
points. Otherwise, it has points of every color. Thus, the answer is 1 − 2π 3
/2π = 23 .

p
40. Find the volume of the three-dimensional solid given by the inequality x2 + y 2 +
|z| ≤ 1.
Solution: 2π/3 . The solid consists of two cones, one whose base is the circle x2 +y 2 = 1
in the xy-plane and whose vertex is (0, 0, 1), and the other with the same base but vertex
(0, 0, −1). Each cone has a base area of π and a height of 1, for a volume of π/3, so the
answer is 2π/3.

41. For any integer n, define bnc as the greatest integer less than or equal to n. For any
positive integer n, let jnk jnk jnk
f (n) = bnc + + + ··· + .
2 3 n
For how many values of n, 1 ≤ n ≤ 100, is f (n) odd?
Solution: 55 Notice that, for fixed a, bn/ac counts the number of integers b ∈
{1, 2, . . . , n} which are divisible by a; hence, f (n) counts the number of pairs (a, b), a, b ∈
{1, 2, . . . , n} with b divisible by a. For any fixed b, the number of such pairs is d(b) (the
number of divisors of b), so the total number of pairs f (n) equals d(1)+d(2)+· · ·+d(n). But
d(b) is odd precisely when b is a square, so f (n) is odd precisely when there are an odd num-
ber of squares in {1, 2, . . . , n}. This happens for 1 ≤ n < 4; 9 ≤ n < 16; . . . ; 81 ≤ n < 100.
Adding these up gives 55 values of n.

10
42. Find all the integers n > 1 with the following property: the numbers 1, 2, . . . , n can
be arranged in a line so that, of any two adjacent numbers, one is divisible by the other.
Solution: 2, 3, 4, 6 The values n = 2, 3, 4, 6 work, as shown by respective examples
1, 2; 2, 1, 3; 2, 4, 1, 3; 3, 6, 2, 4, 1, 5. We shall show that there are no other possibilities. If
n = 2k + 1 is odd, then none of the numbers k + 1, k + 2, . . . , 2k + 1 can divide any other,
so no two of these numbers are adjacent. This is only possible if they occupy the 1st, 3rd,
. . . , (2k + 1)th positions in the line, which means every number ≤ k is adjacent to two of
these and hence divides two of them. But k only divides one of these numbers when k ≥ 2.
Thus no odd n ≥ 5 works. If n = 2k is even, the numbers k + 1, k + 2, . . . , 2k again must
be mutually nonadjacent, but now this means we can have up to two numbers ≤ k each of
which is adjacent to only one number > k, and if there are two such numbers, they must
be adjacent. If k ≥ 4, then each of k − 1, k divides only one of the numbers k + 1, . . . , 2k,
so k − 1, k must be adjacent, but this is impossible. Thus no even k ≥ 8 works, and we are
done.

43. Given that a, b, c are positive integers satisfying


a + b + c = gcd(a, b) + gcd(b, c) + gcd(c, a) + 120,
determine the maximum possible value of a.
Solution: 240 . Notice that (a, b, c) = (240, 120, 120) achieves a value of 240. To see
that this is maximal, first suppose that a > b. Notice that a + b + c = gcd(a, b) + gcd(b, c) +
gcd(c, a) + 120 ≤ gcd(a, b) + b + c + 120, or a ≤ gcd(a, b) + 120. However, gcd(a, b) is a
proper divisor of a, so a ≥ 2 · gcd(a, b). Thus, a − 120 ≤ gcd(a, b) ≤ a/2, yielding a ≤ 240.
Now, if instead a ≤ b, then either b > c and the same logic shows that b ≤ 240 ⇒ a ≤ 240,
or b ≤ c, c > a (since a, b, c cannot all be equal) and then c ≤ 240 ⇒ a ≤ b ≤ c ≤ 240.
44. The unknown real numbers x, y, z satisfy the equations
x+y 1 − z + z2 x−y 9 + 3z + z 2
= 2 ; = 2 .
1+z x − xy + y 2 3−z x + xy + y 2
Find x. √
Solution: 3 14 Cross-multiplying in both equations, we get, respectively, x3 + y 3 =

1 + z 3 , x3 − y 3 = 27 − z 3 . Now adding gives 2x3 = 28, or x = 3 14.
45. Find the number of sequences a1 , a2 , . . . , a10 of positive integers with the property
that an+2 = an+1 + an for n = 1, 2, . . . , 8, and a10 = 2002.
Solution: 3 Let a1 = a, a2 = b; we successively compute a3 = a + b; a4 = a +
2b; . . . ; a10 = 21a + 34b. The equation 2002 = 21a + 34b has three positive integer
solutions, namely (84, 7), (50, 28), (16, 49), and each of these gives a unique sequence.

46. Points A, B, C in the plane satisfy AB = 2002, AC = 9999. The circles with
diameters AB and AC intersect at A and D. If AD = 37, what is the shortest distance from
point A to line BC?

11
Solution: ∠ADB = ∠ADC = π/2 since D lies on the circles with AB and AC as
diameters, so D is the foot of the perpendicular from A to line BC, and the answer is the
given 37 .

47. The real function f has the property that, whenever a, b, n are positive integers such
that a + b = 2n , the equation f (a) + f (b) = n2 holds. What is f (2002)?
Solution: We know f (a) = n2 − f (2n − a) for any a, n with 2n > a; repeated application
gives
f (2002) = 112 − f (46) = 112 − (62 − f (18)) = 112 − (62 − (52 − f (14)))
= 112 − (62 − (52 − (42 − f (2)))).
But f (2) = 22 − f (2), giving f (2) = 2, so the above simplifies to 112 − (62 − (52 − (42 −
2))) = 96 .

48. A permutation of a finite set is a one-to-one function from the set to itself; for
instance, one permutation of {1, 2, 3, 4} is the function π defined such that π(1) = 1, π(2) = 3,
π(3) = 4, and π(4) = 2. How many permutations π of the set {1, 2, . . . , 10} have the property
that π(i) 6= i for each i = 1, 2, . . . , 10, but π(π(i)) = i for each i?
Solution: For each such π, the elements of {1, 2, . . . , 10} can be arranged into pairs {i, j}
such that π(i) = j; π(j) = i. Choosing a permutation π is thus tantamount to choosing a
partition of {1, 2, . . . , 10} into five disjoint pairs. There are 9 ways to pair off the number
1, then 7 ways to pair off the smallest number not yet paired, and so forth, so we have
9 · 7 · 5 · 3 · 1 = 945 partitions into pairs.

49. Two integers are relatively prime if they don’t share any common factors, i.e. if their
greatest common divisor is 1. Define ϕ(n) as the number of positive integers that are less
than n and relatively prime to n. Define ϕd (n) as the number of positive integers that are
less than dn and relatively prime to n.
What is the least n such that ϕx (n) = 64000, where x = ϕy (n), where y = ϕ(n)?
Solution: For fixed n, the pattern of integers relatively prime to n repeats every n
integers, so ϕd (n) = dϕ(n). Therefore the expression in the problem equals ϕ(n)3 . The cube
root of 64000 is 40. ϕ(p) = p − 1 for any prime p. Since 40 is one less than a prime, the least
n such that ϕ(n) = 40 is 41 .

50. Give the set of all positive integers n such that ϕ(n) = 20022 − 1.
Solution: The empty set, Ø . If m is relatively prime to n and m < n, then n − m
must likewise be relatively prime to n, and these are distinct for n > 2 since n/2, n are
not relatively prime. Therefore, for all n > 2, ϕ(n) must be even. 20022 − 1 is odd, and
ϕ(2) = 1 6= 20022 − 1, so no numbers n fulfill the equation.

51. Define ϕk (n) as the number of positive integers that are less than or equal to n/k
and relatively prime to n. Find φ2001 (20022 − 1). (Hint: φ(2003) = 2002.)

12
Solution: ϕ2001 (20022 − 1) = ϕ2001 (2001 · 2003) = the number of m that are relatively
prime to both 2001 and 2003, where m ≤ 2003. Since φ(n) = n − 1 implies that n is
prime, we must only check for those m relatively prime to 2001, except for 2002, which is
relatively prime to 20022 − 1. So ϕ2001 (20022 − 1) = ϕ(2001) + 1 = ϕ(3 · 23 · 29) + 1 =
(3 − 1)(23 − 1)(29 − 1) + 1 = 1233 .

52. Let ABCD be a quadrilateral, and let E, F, G, H be the respective midpoints of


AB, BC, CD, DA. If EG = 12 and F H = 15, what is the maximum possible area of
ABCD?
Solution: The area of EF GH is EG · F H sin θ/2, where θ is the angle between EG and
F H. This is at most 90. However, we claim the area of ABCD is twice that of EF GH. To
see this, notice that EF = AC/2 = GH, F G = BD/2 = HE, so EF GH is a parallelogram.
The half of this parallelogram lying inside triangle DAB has area (BD/2)(h/2), where h is
the height from A to BD, and triangle DAB itself has area BD · h/2 = 2 · (BD/2)(h/2). A
similar computation holds in triangle BCD, proving the claim. Thus, the area of ABCD is
at most 180 . And this maximum is attainable — just take a rectangle with AB = CD =
15, BC = DA = 12.

53. ABC is a triangle with points E, F on sides AC, AB, respectively. Suppose that
BE, CF intersect at X. It is given that AF/F B = (AE/EC)2 and that X is the midpoint
of BE. Find the ratio CX/XF .
Solution: Let x = AE/EC. By Menelaus’s theorem applied to triangle ABE and line
CXF ,
AF BX EC x2
1= · · = .
F B XE CA x+1

Thus, x2 = x + 1, and x must be positive, so x = (1 + 5)/2. Now apply Menelaus to
triangle ACF and line BXE, obtaining
AE CX F B CX x
1= · · = · 2 ,
EC XF BA XF x + 1

so CX/XF = (x2 + 1)/x = (2x2 − x)/x = 2x − 1 = 5.

54. How many pairs of integers (a, b), with 1 ≤ a ≤ b ≤ 60, have the property that b is
divisible by a and b + 1 is divisible by a + 1?
Solution: The divisibility condition is equivalent to b − a being divisible by both a
and a + 1, or, equivalently (since these are relatively prime), by a(a + 1). Any b satisfying
the condition is automatically ≥ a, so it suffices to count the number of values b − a ∈
{1 − a, 2 − a, . . . , 60 − a} that are divisible by a(a + 1) and sum over all a. The number
of such values will be precisely 60/[a(a + 1)] whenever this quantity is an integer, which
fortunately happens for every a ≤ 5; we count:
a = 1 gives 30 values of b;

13
a = 2 gives 10 values of b;
a = 3 gives 5 values of b;
a = 4 gives 3 values of b;
a = 5 gives 2 values of b;
a = 6 gives 2 values (b = 6 or 48);
any a ≥ 7 gives only one value, namely b = a, since b > a implies b ≥ a + a(a + 1) > 60.
Adding these up, we get a total of 106 pairs.

55. A sequence of positive integers is given by a1 = 1 and an = gcd(an−1 , n) + 1 for


n > 1. Calculate a2002 .
Solution: 3 . It is readily seen by induction that an ≤ n for all n. On the other hand,
a1999 is one greater than a divisor of 1999. Since 1999 is prime, we have a1999 = 2 or 2000;
the latter is not possible since 2000 > 1999, so we have a1999 = 2. Now we straightforwardly
compute a2000 = 3, a2001 = 4, and a2002 = 3.

56. x, y are positive real numbers such that x + y 2 = xy. What is the smallest possible
value of x?
Solution: 4 Notice that x = y 2 /(y −1) = 2+(y −1)+1/(y −1) ≥ 2+2 = 4. Conversely,
x = 4 is achievable, by taking y = 2.

57. How many ways, without taking order into consideration, can 2002 be expressed
as the sum of 3 positive integers (for instance, 1000 + 1000 + 2 and 1000 + 2 + 1000 are
considered to be the same way)?
Solution: Call the three numbers that sum to 2002 A, B, and C. In order to prevent
redundancy, we will consider only cases where A ≤ B ≤ C. Then A can range from 1 to
667, inclusive. For odd A, there are 1000 − 3(A−1)
2
possible values for B. For each choice
of A and B, there can only be one possible C, since the three numbers must add up to a
fixed value. We can add up this arithmetic progression to find that there are 167167 possible
combinations of A, B, C, for odd A. For each even A, there are 1002 − 3A 2
possible values
for B. Therefore, there are 166833 possible combinations for even A. In total, this makes
334000 possibilities.

58. A sequence is defined by a0 = 1 and an = 2an−1 for n ≥ 1. What is the last digit (in
base 10) of a15 ?
Solution: 6 . Certainly a13 ≥ 2, so a14 is divisible by 22 = 4. Writing a14 = 4k, we have
a15 = 24k = 16k . But every power of 16 ends in 6, so this is the answer.

59. Determine the value of


1 · 2 − 2 · 3 + 3 · 4 − 4 · 5 + · · · + 2001 · 2002.
Solution: 2004002 . Rewrite the expression as
2 + 3 · (4 − 2) + 5 · (6 − 4) + · · · + 2001 · (2002 − 2000)

14
= 2 + 6 + 10 + · · · + 4002.
This is an arithmetic progression with (4002 − 2)/4 + 1 = 1001 terms and average 2002, so
its sum is 1001 · 2002 = 2004002.

60. A 5 × 5 square grid has the number −3 written in the upper-left square and the
number 3 written in the lower-right square. In how many ways can the remaining squares
be filled in with integers so that any two adjacent numbers differ by 1, where two squares
are adjacent if they share a common edge (but not if they share only a corner)?
Solution: 250 If the square in row i, column j contains the number k, let its “index”
be i + j − k. The constraint on adjacent squares now says that if a square has index r,
the squares to its right and below it each have index r or r + 2. The upper-left square has
index 5, and the lower-right square has index 7, so every square must have index 5 or 7. The
boundary separating the two types of squares is a path consisting of upward and rightward
steps; it can be extended along the grid’s border so as to obtain a path between the lower-left
and upper-right corners. Conversely, any such path uniquely determines each square’s index
and hence the entire array of numbers — except that the two paths lying entirely along the
border of the grid fail to separate the upper-left from the lower-right square and thus do not
create valid arrays (since these two squares should have different indices). Each path consists
10

of 5 upward and 5 rightward steps, so there are 5 = 252 paths, but two are impossible, so
the answer is 250.

61. Bob Barker went back to school for a PhD in math, and decided to raise the
intellectual level of The Price is Right by having contestants guess how many objects exist
of a certain type, without going over. The number of points you will get is the percentage
of the correct answer, divided by 10, with no points for going over (i.e. a maximum of 10
points).
Let’s see the first object for our contestants...a table of shape (5, 4, 3, 2, 1) is an arrange-
ment of the integers 1 through 15 with five numbers in the top row, four in the next, three
in the next, two in the next, and one in the last, such that each row and each column is
increasing (from left to right, and top to bottom, respectively). For instance:
1 2 3 4 5
6 7 8 9
10 11 12
13 14
15
is one table. How many tables are there?
Solution: 15!/(34 · 53 · 72 · 9) = 292864 . These are Standard Young Tableaux.

62. Our next object up for bid is an arithmetic progression of primes. For example,
the primes 3, 5, and 7 form an arithmetic progression of length 3. What is the largest
possible length of an arithmetic progression formed of positive primes less than 1,000,000?
Be prepared to justify your answer.

15
Solution: 12 . We can get 12 with 110437 and difference 13860.

63. Our third and final item comes to us from Germany, I mean Geometry. It is known
that a regular n-gon can be constructed with straightedge and compass if n is a prime
that is 1 plus a power of 2. It is also possible to construct a 2n-gon whenever an n-gon is
constructible, or a p1 p2 · · · pm -gon where the pi ’s are distinct primes of the above form. What
is really interesting is that these conditions, together with the fact that we can construct a
square, is that they give us all constructible regular n-gons. What is the largest n less than
4,300,000,000 such that a regular n-gon is constructible?
Solution: The known primes of this form (Fermat primes) are 3, 5, 17, 257, and 65537,
and the result is due to Gauss (German). If there are other such primes (unknown), then
they are much bigger than 1010 . So for each product of these primes, we can divide 4.3 · 109
by that number and take log2 to find the largest power of 2 to multiply by, then compare the
resulting numbers. There are 32 cases to check, or just observe that 232 = 4, 294, 967, 296 is
so close that there’s likely a shortcut. Note that 232 + 1 is divisible by 641, and hence not
prime. 3 · 5 · 17 · 257 · 65537 = 232 − 1 is smaller; replacing any of the factors by the closest
power of 2 only decreases the product, and there’s not enough room to squeeze in an extra
factor of 2 without replacing all of them, and that gives us 232 , so indeed that it is the
answer.

Help control the pet population. Have your pets spayed or neutered. Bye-bye.

16
Team Event Solutions
HMMT 2002

Palindromes. A palindrome is a positive integer n not divisible by 10 such that if you write the
decimal digits of n in reverse order, the number you get is n itself. For instance, the numbers 4
and 25752 are palindromes.

1. [15] Determine the number of palindromes that are less than 1000.

Solution. Every one-digit number (there are nine) is a palindrome. The two-digit palindromes
have the form a a for a nonzero digit a, so there are nine of them. A three-digit palindrome is a b a
with a a nonzero digit and b any digit, so there are 9 × 10 = 90 of these. Thus the number of
palindromes less than 1000 is 9 + 9 + 90 = 108.

2. [30] Determine the number of four-digit integers n such that n and 2n are both palindromes.

Solution. Let n = a b b a. If a, b ≤ 4 then there are no carries in the multiplication n × 2, and


2n = (2a) (2b) (2b) (2a) is a palindrome. We shall show conversely that if n and 2n are palindromes,
then necessarily a, b ≤ 4. Hence the answer to the problem is 4 × 5 = 20 (because a cannot be
zero).

If a ≥ 5 then 2n is a five-digit number whose most significant digit is 1, but because 2n is even,
its least significant digit is even, contradicting the assumption that 2n is a palindrome. Therefore
a ≤ 4. Consequently 2n is a four-digit number, and its tens and hundreds digits must be equal.
Because a ≤ 4, there is no carry out of the ones place in the multiplication n × 2, and therefore the
tens digit of 2n is the ones digit of 2b. In particular, the tens digit of 2n is even. But if b ≥ 5, the
carry out of the tens place makes the hundreds digit of 2n odd, which is impossible. Hence b ≤ 4
as well.

3. [40] Suppose that a positive integer n has the property that n, 2n, 3n, . . ., 9n are all palindromes.
Prove that the decimal digits of n are all zeros or ones.

Solution. First consider the ones digit a of n; we claim that a = 1. Certainly a cannot be even,
for then 5n would be divisible by 10. If a is 5, 7, or 9, then 2n has an even ones digit, while its
most significant digit is 1. If a is 3, then 4n has an even ones digit but most significant digit 1.
Thus a = 1 is the only possibility. Moreover 9n has the same number of digits as n, for otherwise
9n would have most significant digit 1 but least significant digit 9, which is forbidden.

Now suppose n has at least one digit that is neither a zero nor a one. Let b be the leftmost (i.e.,
most significant) such digit, so that the left end of the decimal representation of n looks like

a1 . . . ar b . . .

for some r ≥ 1 and digits ai ∈ {0, 1}. When n is multiplied by 9, there will be a carry out of the
column containing b. In particular, the rth digit from the left in 9n will not be 9ar . But the right
end of the decimal representation of n is

. . . ar . . . a1 ;

because each ai is 0 or 1, there are no carries out of the first r − 1 columns, so the rth digit from the
right in 9n will be 9ar . Thus 9n is not a palindrome, a contradiction. This completes the proof.

1
Floor functions. The notation bxc stands for the largest integer less than or equal to x.

4. [15] Let n be an integer. Prove that


   
n n+1
+ = n.
2 2

Solution. Suppose n = 2m is even. Then bn/2c = bmc = m and b(n + 1)/2c = bm + 1/2c = m,
whose sum is m + m = 2m = n. Otherwise n = 2m + 1 is odd. In this case bn/2c = bm + 1/2c = m
and b(n + 1)/2c = bm + 1c = m + 1, whose sum is m + (m + 1) = 2m + 1 = n, as desired.

5. [20] Prove for integers n that     2


n n+1 n
= .
2 2 4

Solution. Suppose n = 2m
 is  then bn/2c
 even; = bmc = m and b(n + 1)/2c = bm + 1/2c = m,
whose product is m2 = m2 = (2m)2 /4 . Otherwise n = 2m + 1 is odd, so that bn/2c =


bm + 1/2c = m and b(n + 1)/2c = bm + 1c = m + 1, whose product is m2 + m. On the other side,


we find that  2  2   
n 4m + 4m + 1 2 1
= = m +m+ = m2 + m,
4 4 4
as desired.

In problems 6–7 you may use without proof the known summations
L
X L
X
n = n(n + 1)/2 and n3 = n2 (n + 1)2 /4 for positive integers L.
n=1 n=1

PL
6. [20] For positive integers L, let SL = n=1 bn/2c. Determine all L for which SL is a square
number.

Solution. We distinguish two cases depending on the parity of L. Suppose first that L = 2k − 1
is odd, where k ≥ 1. Then
X jnk X k(k − 1)
SL = =2 m=2· = k(k − 1).
2 2
1≤n≤2k−1 0≤m<k

If k = 1, this is the square number 0. If k > 1 then (k − 1)2 < k(k − 1) < k 2 , so k(k − 1) is not
square. Now suppose L = 2k is even, where k ≥ 1. Then SL = SL−1 + k = k 2 is always square.
Hence SL is square exactly when L = 1 or L is even.

PL
n3/9 for positive integers L. Determine all L for which TL is a square
 
7. [45] Let TL = n=1
number.

Solution. Since TL is square if and only if 9TL is square, we may consider 9TL instead of TL .

It is well known that n3 is congruent to 0, 1, or 8 modulo 9 according as n is congruent to 0, 1,


or 2 modulo 3. (Proof: (3m + k)3 = 27m3 + 3(9m2 )k + 3(3m)k 2 + k 3 ≡ k 3 (mod 9).) Therefore

2
n3 − 9 n3/9 is 0, 1, or 8 according as n is congruent to 0, 1, or 2 modulo 3. We find therefore that
 

X  n3 
9TL = 9
9
1≤n≤L
X
= n3 − #{1 ≤ n ≤ L : n ≡ 1 (mod 3)} − 8#{1 ≤ n ≤ L : n ≡ 2 (mod 3)}
1≤n≤L
 2    
1 L+2 L+1
= L(L + 1) − −8 .
2 3 3
2 2
Clearly 9TL < L(L + 1)/2 for L ≥ 1. We shall prove that 9TL > L(L + 1)/2 − 1 for L ≥ 4,
whence 9TL is not square for L ≥ 4. Because
2 2
L(L + 1)/2 − 1 = L(L + 1)/2 − L(L + 1) + 1,

we need only show that    


L+2 L+1
+8 ≤ L2 + L − 2.
3 3
But the left-hand side of this is bounded above by 3L+10/3, and the inequality 3L+10/3 ≤ L2 +L−2
means exactly L2 − 2L − 16/3 ≥ 0 or (L − 1)2 ≥ 19/3, which is true for L ≥ 4, as desired.

Hence TL is not square for L ≥ 4. By direct computation we find T1 = T2 = 0 and T3 = 3, so TL is


square only for L ∈ {1, 2}.

Luck of the dice. Problems 8–12 concern a two-player game played on a board consisting of fourteen
spaces in a row. The leftmost space is labeled START, and the rightmost space is labeled END.
Each of the twelve other squares, which we number 1 through 12 from left to right, may be blank
or may be labeled with an arrow pointing to the right. The term blank square will refer to one of
these twelve squares that is not labeled with an arrow. The set of blank squares on the board will
be called a board configuration; the board below uses the configuration {1, 2, 3, 4, 7, 8, 10, 11, 12}.

START ⇒ ⇒ ⇒ END
1 2 3 4 5 6 7 8 9 10 11 12

For i ∈ {1, 2}, player i has a die that produces each integer from 1 to si with probability 1/si . Here
s1 and s2 are positive integers fixed before the game begins. The game rules are as follows:

1. The players take turns alternately, and player 1 takes the first turn.

2. On each of his turns, player i rolls his die and moves his piece to the right by the number
of squares that he rolled. If his move ends on a square marked with an arrow, he moves his
piece forward another si squares. If that move ends on an arrow, he moves another si squares,
repeating until his piece comes to rest on a square without an arrow.

3. If a player’s move would take him past the END square, instead he lands on the END square.

4. Whichever player reaches the END square first wins.

As an example, suppose that s1 = 3 and the first player is on square 4 in the sample board shown
above. If the first player rolls a 2, he moves to square 6, then to square 9, finally coming to rest on

3
square 12. If the second player does not reach the END square on her next turn, the first player
will necessarily win on his next turn, as he must roll at least a 1.

8. [35] In this problem only, assume that s1 = 4 and that exactly one board square, say square
number n, is marked with an arrow. Determine all choices of n that maximize the average distance
in squares the first player will travel in his first two turns.

Solution. Because expectation is linear, the average distance the first player travels in his first two
turns is the average sum of two rolls of his die (which does not depend on the board configuration)
plus four times the probability that he lands on the arrow on one of his first two turns. Thus we just
need to maximize the probability that player 1 lands on the arrow in his first two turns. If n ≥ 5,
player 1 cannot land on the arrow in his first turn, so he encounters the arrow with probability at
most 1/4. If instead n ≤ 4, player 1 has a 1/4 chance of landing on the arrow on his first turn.
If he misses, then he has a 1/4 chance of hitting the arrow on his second turn provided that he
is not beyond square n already. The chance that player 1’s first roll left him on square n − 1 or
farther left is (n − 1)/4. Hence his probability of benefiting from the arrow in his first two turns is
1/4 + (1/4)(n − 1)/4, which is maximized for n = 4, where it is greater than the value of 1/4 that
we get from n ≥ 5. Hence the answer is n = 4.

9. [30] In this problem suppose that s1 = s2 . Prove that for each board configuration, the first
player wins with probability strictly greater than 21 .

Solution. Let σ1 and σ2 denote the sequence of the next twelve die rolls that players 1 and 2
respectively will make. The outcome of the game is completely determined by the σi . Now player
1 wins in all cases in which σ1 = σ2 , for then each of player 2’s moves bring her piece to a square
already occupied by player 1’s piece. It is sufficient, therefore, to show that player 1 wins at least
half the cases in which σ1 6= σ2 . But all these cases can be partitioned into disjoint pairs

{(σ1 , σ2 ), (σ2 , σ1 )},

and player 1 wins in at least one case in each pair. For if player 2 wins in the case (σ1 , σ2 ), say
on her nth turn, the first n elements of σ1 do not take player 1 beyond space 12, while the first n
elements of σ2 must take player 2 beyond space 12. Clearly, then, player 1 wins (σ2 , σ1 ) on his nth
turn.

10. [30] Exhibit a configuration of the board and a choice of s1 and s2 so that s1 > s2 , yet the
second player wins with probability strictly greater than 21 .

Solution. Let s1 = 3 and s2 = 2 and place an arrow on all the even-numbered squares. In this
configuration, player 1 can move at most six squares in a turn: up to three from his roll and an
additional three if his roll landed him on an arrow. Hence player 1 cannot win on his first or
second turn. Player 2, however, wins immediately if she ever lands on an arrow. Thus player 2 has
probability 1/2 of winning on her first turn, and failing that, she has probability 1/2 of winning on
her second turn. Hence player 2 wins with probability at least 1/2 + (1/2)(1/2) = 3/4.

11. [55] In this problem assume s1 = 3 and s2 = 2. Determine, with proof, the nonnegative integer
k with the following property:

1. For every board configuration with strictly fewer than k blank squares, the first player wins
with probability strictly greater than 12 ; but

4
2. there exists a board configuration with exactly k blank squares for which the second player
wins with probability strictly greater than 12 .

Solution. The answer is k = 3. Consider the configuration whose blank squares are 2, 6, and 10.
Because these numbers represent all congruence classes modulo 3, player 1 cannot win on his first
turn: he will come to rest on one of the blank squares. But player 2 will win on her first turn if she
rolls a 1, for 2, 6, and 10 are all even. Thus player 2 wins on her first turn with probability 1/2.
Failing this, player 1 may fail to win on his second turn, for instance, if he rolled a 2 previously
and now rolls a 1, ending up on square 6. Then player 2 will again have probability 1/2 of winning
on her next turn. Thus player 2 wins the game with probability exceeding 1/2.

We must now prove that all configurations with fewer than three blanks favor player 1. If the
numbers of the blank squares represent at most one residue class modulo 3, then clearly player
1 wins on his first turn with probability at least 2/3. This disposes of the cases of no blanks,
just one blank, and two blanks that are congruent modulo 3. In the remaining case, there are
two blank squares whose indices are incongruent modulo 3. Then player 1 wins on his first turn
with probability only 1/3. If he does not win immediately, player 2 wins on her first turn with
probability at most 1/2, for there is a blank in at least one congruence class modulo 2. If player
2 does not win on her first turn, then player 1 wins on his second turn with probability at least
2/3, for there is only one blank square in front of him now. Thus player 1 wins the game with
probability at least 1/3 + (2/3)(1/2)(2/3) = 5/9 > 1/2, as desired.

12. [65] Now suppose that before the game begins, the players choose the initial game state as
follows:

1. The first player chooses s1 subject to the constraint that 2 ≤ s1 ≤ 5; then

2. the second player chooses s2 subject to the constraint that 2 ≤ s2 ≤ 5 and then specifies the
board configuration.

Prove that the second player can always make her decisions so that she will win the game with
probability strictly greater than 21 .

Solution. If s1 ∈ {3, 5}, take s2 = 2 and put arrows on the even-numbered squares. Player 1
cannot win on his first turn because he can move at most 2s1 spaces in a turn. Player 2 wins on
her first turn with probability 1/2. Failing that, player 1 might fail to win on his second turn,
and player 2 will again have probability 1/2 of winning on her second turn, so her probability of
winning the game is certainly greater than 1/2.

If s1 = 4, take s2 = 3 and leave blank only squares 1, 4, 7, and 10. These occupy all congruence
classes modulo 4, so player 1 cannot win on his first turn. But the blank squares lie in the same
congruence class modulo 3, so player 2 then wins on her first turn with probability 2/3.

Finally, if s1 = 2, take s2 = 5 and leave all squares blank. Then player 2 moves 3 squares in a turn
on average, hence covers 15 squares on average in her first five turns. Moreover, the distribution
of player 2’s distance traveled in five turns is symmetric about 15. Thus player 2 has probability
greater than 1/2 of reaching END by the end of her fifth turn. Player 1, on the other hand, cannot
win in five turns because he can move at most 10 squares in those turns.

5
Harvard-MIT Mathematics Tournament
March 15, 2003

Individual Round: Algebra Subject Test — Solutions



1. Find the smallest value of x such that a ≥ 14 a − x for all nonnegative a.
Solution: 49
We want to find the smallest value of x
√ such that x ≥ √
14sqrta − a for all a. This is
just the maximum possible value of 14 a − a = 49 − ( a − 7)2 , which is clearly 49,
achieved when a = 49.
tan2 (20◦ )−sin2 (20◦ )
2. Compute tan2 (20◦ ) sin2 (20◦ )
.
Solution: 1
If we multiply top and bottom by cos2 (20◦ ), the numerator becomes sin2 (20◦ ) · (1 −
cos2 20◦ ) = sin4 (20◦ ), while the denominator becomes sin4 (20◦ ) also. So they are equal,
and the ratio is 1.

3. Find the smallest n such that n! ends in 290 zeroes.


Solution: 1170
Each 0 represents a factor of 10 = 2 · 5. Thus, we wish to find the smallest factorial
that contains at least 290 2’s and 290 5’s in its prime factorization. Let this number
be n!, so the factorization of n! contains 2 to the power p and 5 to the power q, where
¹ º ¹ º ¹ º ¹ º ¹ º ¹ º
n n n n n n
p= + 2 + 3 + · · · and q = + 2 + 3 + ···
2 2 2 5 5 5
(this takes into account one factor for each single multiple of 2 or 5 that is ≤ n,
an additional factor for each multiple of 22 or 52 , and so on). Naturally, p ≥ q
because 2 is smaller than 5. Thus, we want to bring q as low to 290 as possible.
If q = b n5 c + b 5n2 c + b 5n3 c + · · ·, we form a rough geometric sequence (by taking away
n/5
the floor function) whose sum is represented by 290 ≈ 1−1/5 . Hence we estimate
n = 1160, and this gives us q = 288. Adding 10 to the value of n gives the necessary
two additional factors of 5, and so the answer is 1170.
q √ √
4. Simplify: 2 1.5 + 2 − (1.5 + 2).
Solution: 1/2
q √ √ q √ √
The given expression equals 6
√ √ 2 + 4 2 − (1.5
√ + 2) = 6 + 2 8
√ √ − (1.5 + 2). But
√ on
inspection, we see that ( 2+ 4) = 6+2 8, so the answer is ( 2+ 4)−(1.5+ 2) =
2 − 3/2 = 1/2.

5. Several positive integers are given, not necessarily all different. Their sum is 2003.
Suppose that n1 of the given numbers are equal to 1, n2 of them are equal to 2, . . .,
n2003 of them are equal to 2003. Find the largest possible value of

n2 + 2n3 + 3n4 + · · · + 2002n2003 .

1
Solution: 2002
The sum of all the numbers is n1 + 2n2 + · · · + 2003n2003 , while the number of numbers
is n1 + n2 + · · · + n2003 . Hence, the desired quantity equals
(n1 + 2n2 + · · · + 2003n2003 ) − (n1 + n2 + · · · + n2003 )
= (sum of the numbers) − (number of numbers)
= 2003 − (number of numbers),
which is maximized when the number of numbers is minimized. Hence, we should have
just one number, equal to 2003, and then the specified sum is 2003 − 1 = 2002.
Comment: On the day of the contest, a protest was lodged (successfully) on the
grounds that the use of the words “several” and “their” in the problem statement
implies there must be at least 2 numbers. Then the answer is 2001, and this maximum
is achieved by any two numbers whose sum is 2003.me way.)
6. Let a1 = 1, and let an = bn3 /an−1 c for n > 1. Determine the value of a999 .
Solution: 999
We claim that for any odd n, an = n. The proof is by induction. To get the base
cases n = 1, 3, we compute a1 = 1, a2 = b23 /1c = 8, a3 = b33 /8c = 3. And if
the claim holds for odd n ≥ 3, then an+1 = b(n + 1)3 /nc = n2 + 3n + 3, so an+2 =
2 +3n+2
b(n+2)3 /(n2 +3n+3)c = b(n3 +6n2 +12n+8)/(n2 +3n+2)c = bn+2+ nn2 +3n+3 c = n+2.
So the claim holds, and in particular, a999 = 999.
7. Let a, b, c be the three roots of p(x) = x3 + x2 − 333x − 1001. Find a3 + b3 + c3 .
Solution: 2003
We know that x3 + x2 − 333x − 1001 = (x − a)(x − b)(x − c) = x3 − (a + b + c)x2 +
(ab + bc + ca)x − abc. Also, (a + b + c)3 − 3(a + b + c)(ab + bc + ca) + 3abc = a3 + b3 + c3 .
Thus, a3 + b3 + c3 = (−1)3 − 3(−1)(−333) + 3 · 1001 = 2003.
1 1 1
8. Find the value of 32 +1
+ 42 +2
+ 52 +3
+ · · ·.
Solution: 13/36
Each term takes the form
1 1
= .
n2 + (n − 2) (n + 2) · (n − 1)
Using the method of partial fractions, we can write (for some constants A, B)
1 A B
= +
(n + 2) · (n − 1) (n + 2) (n − 1)
⇒ 1 = A · (n − 1) + B · (n + 2)
Setting n = 1 we get B = 13 , and similarly with n = −2 we get A = − 13 . Hence the
sum becomes
·µ ¶ µ ¶ µ ¶ µ ¶ ¸
1 1 1 1 1 1 1 1 1
· − + − + − + − + ··· .
3 2 5 3 6 4 7 5 8
1
Thus, it telescopes, and the only terms that do not cancel produce a sum of 3
· ( 21 +
1
3
+ 14 ) = 13
36
.

2
³ ´
1 2n
9. For how many integers n, for 1 ≤ n ≤ 1000, is the number 2 n
even?
Solution: 990
³ ´
In fact, the expression 2nn
is always even, and it is not a multiple of four if and only
if n is a power of 2, and there are 10 powers of 2 between 1 and 1000.
Let f (N ) denote the number of factors of 2 in N . Thus,
¹ º ¹ º X∞ ¹ º ¹ º
n n n n
f (n!) = + + + ··· = k
.
2 4 8 k=1 2

Also, it is clear that f (ab) = f (a) + f (b) and f ( ab ) = f (a) − f (b) for integers a, b. Now
for any positive integer n, let m be the integer such that 2m ≤ n < 2m+1 . Then
ÃÃ !! Ã ! ∞ ¹ º Ã ∞ ¹ º!
2n (2n)! X 2n X n
f =f = k
−2 k
n n!n! k=1 2 k=1 2
∞ ¹ º Ã∞ ¹ º!
X n X n
= −2
k=1 2k−1 k=1 2k
à ∞ ¹ º!
X n
= bnc −
k=1 2k
Ãm ¹ º!
X n
= n−
k=1 2k
Ãm !
X n
≥ n−
k=1 2k
µ m ¶
2 −1 n
= n−n = ≥ 1.
2m 2m
³ ´
2n
Both equalities hold when n = 2m , and otherwise, f ( n
) > 1.

10. Suppose P (x) is a polynomial such that P (1) = 1 and


P (2x) 56
=8−
P (x + 1) x+7
for all real x for which both sides are defined. Find P (−1).
Solution: −5/21
Cross-multiplying gives (x + 7)P (2x) = 8xP (x + 1). If P has degree n and leading
coefficient c, then the leading coefficients of the two sides are 2n c and 8c, so n = 3.
Now x = 0 is a root of the right-hand side, so it’s a root of the left-hand side, so
that P (x) = xQ(x) for some polynomial Q ⇒ 2x(x + 7)Q(2x) = 8x(x + 1)Q(x + 1) or
(x+7)Q(2x) = 4(x+1)Q(x+1). Similarly, we see that x = −1 is a root of the left-hand
side, giving Q(x) = (x + 2)R(x) for some polynomial R ⇒ 2(x + 1)(x + 7)R(2x) =
4(x + 1)(x + 3)R(x + 1), or (x + 7)R(2x) = 2(x + 3)R(x + 1). Now x = −3 is a root
of the left-hand side, so R(x) = (x + 6)S(x) for some polynomial S.
At this point, P (x) = x(x+2)(x+6)S(x), but P has degree 3, so S must be a constant.
Since P (1) = 1, we get S = 1/21, and then P (−1) = (−1)(1)(5)/21 = −5/21.

3
Harvard-MIT Mathematics Tournament
March 15, 2003

Individual Round: Calculus Subject Test — Solutions


1. A point is chosen randomly with uniform distribution in the interior of a circle of radius
1. What is its expected distance from the center of the circle?
Solution: 2/3
The probability of the point falling between a distance r and r + dr from the center
is the ratio of the area of the corresponding annulus to the area of the whole circle:
π[(r+dr)2 −r2 ]
π
→ 2πrπ dr = 2r dr for small values of dr. Then the expected distance is
R1 2
0 r cot 2r dr = 3 .

2. A particle moves along the x-axis in such a way that its velocity at position x is given
by the formula v(x) = 2 + sin x. What is its acceleration at x = π6 ?

Solution: 5 3/4

Acceleration is given by a = dv
dt
= dv dx
·
dx dt
= dv
dx
· v = cos x · (2 + sin x) = 5 3/4.
3. What is the area of the region bounded by the curves y = x2003 and y = x1/2003 and
lying above the x-axis?
Solution: 1001/1002
The two curves intersect at (0, 0) and (1, 1), so the desired area is
Z 1³ ´ · ¸1
1/2003 2003 x2004/2003 x2004 1001
x −x dx = − = .
0 2004/2003 2004 0 1002
4. The sequence of real numbers x1 , x2 , x3 , . . . satisfies limn→∞ (x2n + x2n+1 ) = 315 and
limn→∞ (x2n + x2n−1 ) = 2003. Evaluate limn→∞ (x2n /x2n+1 ).
Solution: −1
We have limn→∞ (x2n+1 −x2n−1 ) = limn→∞ [(x2n +x2n+1 )−(x2n +x2n−1 )] = 315−2003 =
−1688; it follows that x2n+1 → −∞ as n → ∞. Then
x2n x2n + x2n+1
lim = lim − 1 = −1,
n→∞ x n→∞ x2n+1
2n+1

since x2n + x2n+1 → 315 while x2n+1 → −∞.


5. Find the minimum distance from the point (0, 5/2) to the graph of y = x4 /8.

Solution: 17/2
We want to minimize x2 +(x4 /8−5/2)2 = x8 /64−5x4 /8+x2 +25/4, which is equivalent
to minimizing z 4 /4 − 10z 2 + 16z, where we have set z = x2 . The derivative of this
expression is z 3 − 20z + 16, which √
is seen on inspection
√ to have 4 as a root, leading to
the factorization (z − 4)(z + 2 − 2 2)(z + 2 −√2 2). Since z = x2 ranges over [0, ∞),
the possible minima are√at z = 0, z = −2 + 2 2, and z = 4. However, the derivative
is positive on (0, −2 + 2 2), so this leaves only 0 and 4 to be tried. We find that the
minimum is in fact achieved
q at z = 4, so the closest point on the graph is given by

x = ±2, with distance 2 + (24 /8 − 5/2)2 = 17/2.
2

1
6. For n an integer, evaluate
µ ¶
1 1 1
lim √ + √ + · · · + q .
n→∞ n2 − 02 n2 − 12 n2 − (n − 1)2

Solution: π/2
Note that √ 1
n2 −i2
= 1
n
·√ 1
, so that the sum we wish to evaluate is just a Riemann
1−( ni )2
sum. Then,
µ ¶ Z 1
1 n−1
X 1 1 −1 1 π
lim q = √ dx = [sin x]0 = .
n→∞ n i=0 1 − ( i )2 0 1 − x2 2
n

7. For what value of a > 1 is Z a2


1 x−1
log dx
a x 32
minimum?
Solution: 3
R 2
Let f (a) = aa x1 log x−1
32
df
dx. Then we want da = 0; by the Fundamental Theorem of
Calculus and the chain rule, this implies that
à ! à !
1 a2 − 1 1 a−1 d Z a2 1 x−1 Z a
1 x−1
2a 2 log − log = log dx − log dx = 0,
a 32 a 32 da c x 32 c x 32
2 2
where c is any constant with 1 < c < a. Then 2 log a 32−1 = log a−1 32
, so that ( a 32−1 )2 =
a−1
32
. After canceling factors of (a−1)/32 (since a > 1), this simplifies to (a2 −1)(a+1) =
32 ⇒ a3 + a2 − a − 33 = 0, which in turn factors as (a − 3)(a2 + 4a + 11) = 0. The
quadratic factor has no real solutions, so this leaves only a = 3. However, we have that
a > 1, and we can check that f (1) = 0, lima→∞ f (a) > 0, and f (3) < 0, so the global
minimum does occur at a = 3.

8. A right circular cone with a height of 12 inches and a base radius of 3 inches is filled
with water and held with its vertex pointing downward. Water flows out through a
hole at the vertex at a rate in cubic inches per second numerically equal to the height
of the water in the cone. (For example, when the height of the water in the cone is 4
inches, water flows out at a rate of 4 cubic inches per second.) Determine how many
seconds it will take for all of the water to flow out of the cone.
Solution: 9π/2
When the water in the cone is h inches high, it forms a cone similar to the original, so
that its base has radius h/4 and its volume is hence πh3 /48. The given condition then
states that à !
d πh3 πh2 dh dh 32
= −h ⇒ · = −h ⇒ 2h · =− .
dt 48 16 dt dt π
Integrating with respect to t, we get that h2 = −32t/π + C; setting t = 0, h = 12, we
get C = 144. The cone empties when h = 0, so 0 = −32t/π + 144 ⇒ t = 9π/2.

2
9. Two differentiable real functions f (x) and g(x) satisfy

f 0 (x)
= ef (x)−g(x)
g 0 (x)

for all x, and f (0) = g(2003) = 1. Find the largest constant c such that f (2003) > c
for all such functions f, g.
Solution: 1 − ln 2
d
Rearranging the given equation gives f 0 (x)e−f (x) = g 0 (x)e−g(x) for all x, so dx (e−f (x) −
−g(x) 0 −f (x) 0 −g(x) −f (x) −g(x)
e ) = −f (x)e + g (x)e = 0. Thus, e −e is a constant, and it must
−f (0) −1 −f (2003) −g(2003)
be less than e = e . Thus, e <e +e = 2e−1 = eln 2−1 ⇒ f (2003) >
−1

1 − ln 2. On the other hand, we can find positive-valued functions e−f (x) , e−g(x) that
take on the required values at 0 and 2003 and have constant difference arbitrarily close
to e−1 . For example, for arbitrarily large t, we can set e−f (x) = e−(t(2003−x)+1) + e−1 −
e−(2003t+1) and e−g(x) = e−(t(2003−x)+1) , and we can check that the resulting functions
f, g satisfy the required conditions. Thus, we can make f (2003) arbitrarily close to
1 − ln 2, so this is the answer.

10. Evaluate Z ∞
1 − x2
dx.
−∞ 1 + x4

Solution: 0
R∞
Let S = 0 1/(x4 + 1) dx; note that the integral converges absolutely. Substituting
x = 1/u, so that dx = −1/u2 du, we have
Z ∞ Z 0 Z 0
1 1 du −u2
S= dx = = du
0 1 + x4 ∞ 1 + u−4 −u2 ∞ u4 + 1

Z ∞ Z ∞
u2 x2
= du = dx
0 1 + u4 0 1 + x4
(the
R∞
manipulations are justified by absolute convergence), from which we see that
2 4
0 (1 − x )/(1 + x ) dx = 0. Since the integrand is an even function, it follows that the
integral from −∞ to ∞ is zero as well.

3
Harvard-MIT Mathematics Tournament
March 15, 2003

Individual Round: Combinatorics Subject Test — Solutions

1. You have 2003 switches, numbered from 1 to 2003, arranged in a circle. Initially, each
switch is either ON or OFF, and all configurations of switches are equally likely. You
perform the following operation: for each switch S, if the two switches next to S were
initially in the same position, then you set S to ON; otherwise, you set S to OFF.
What is the probability that all switches will now be ON?
Solution: 1/22002
There are 22003 equally likely starting configurations. All switches end up ON if and
only if switches 1, 3, 5, 7, . . . , 2003, 2, 4, . . . , 2002 — i.e. all 2003 of them — were initially
in the same position. This initial position can be ON or OFF, so this situation occurs
with probability 2/22003 = 1/22002 .

2. You are given a 10 × 2 grid of unit squares. Two different squares are adjacent if they
share a side. How many ways can one mark exactly nine of the squares so that no two
marked squares are adjacent?
Solution: 36
Since each row has only two squares, it is impossible for two marked squares to be
in the same row. Therefore, exactly nine of the ten rows contain marked squares.
Consider two cases:
Case 1: The first or last row is empty. These two cases are symmetrical, so assume
without loss of generality that the first row is empty. There are two possibilities for the
second row: either the first square is marked, or the second square is marked. Since
the third row must contain a marked square, and it cannot be in the same column
as the marked square in the second row, the third row is determined by the second.
Similarly, all the remaining rows are determined. This leaves two possibilities if the
first row is empty. Thus, there are four possibilities if the first or last row is empty.
Case 2: The empty row is not the first or last. Then, there are two blocks of (one of
more) consecutive rows of marked squares. As above, the configuration of the rows in
each of the two blocks is determined by the position of the marked square in the first
of its rows. That makes 2 × 2 = 4 possible configurations. There are eight possibilities
for the empty row, making a total of 32 possibilities in this case.
Together, there are 36 possible configurations of marked squares.

3. Daniel and Scott are playing a game where a player wins as soon as he has two points
more than his opponent. Both players start at par, and points are earned one at a
time. If Daniel has a 60% chance of winning each point, what is the probability that
he will win the game?
Solution: 9/13
Consider the situation after two points. Daniel has a 9/25 chance of winning, Scott,
4/25, and there is a 12/25 chance that the players will be tied. In the latter case, we
revert to the original situation. In particular, after every two points, either the game

1
returns to the original situation, or one player wins. If it is given that the game lasts
2k rounds, then the players must be at par after 2(k − 1) rounds, and then Daniel wins
with probability (9/25)/(9/25 + 4/25) = 9/13. Since this holds for any k, we conclude
that Daniel wins the game with probability 9/13.

4. In a certain country, there are 100 senators, each of whom has 4 aides. These senators
and aides serve on various committees. A committee may consist either of 5 senators,
of 4 senators and 4 aides, or of 2 senators and 12 aides. Every senator serves on 5
committees, and every aide serves on 3 committees. How many committees are there
altogether?
Solution: 160
If each senator gets a point for every committee on which she serves, and every aide
gets 1/4 point for every committee on which he serves, then the 100 senators get 500
points altogether, and the 400 aides get 300 points altogether, for a total of 800 points.
On the other hand, each committee contributes 5 points, so there must be 800/5 = 160
committees.

5. We wish to color the integers 1, 2, 3, . . . , 10 in red, green, and blue, so that no two
numbers a and b, with a − b odd, have the same color. (We do not require that all
three colors be used.) In how many ways can this be done?
Solution: 186
The condition is equivalent to never having an odd number and an even number in
the same color. We can choose one of the three colors for the odd numbers and
distribute the other two colors freely among the 5 even numbers; this can be done in
3 · 25 = 96 ways. We can also choose one color for the even numbers and distribute
the other two colors among the 5 odd numbers, again in 96 ways. This gives a total
of 192 possibilities. However, we have double-counted the 3 · 2 = 6 cases where all
odd numbers are the same color and all even numbers are the same color, so there are
actually 192 − 6 = 186 possible colorings.

6. In a classroom, 34 students are seated in 5 rows of 7 chairs. The place at the center of
the room is unoccupied. A teacher decides to reassign the seats such that each student
will occupy a chair adjacent to his/her present one (i.e. move one desk forward, back,
left or right). In how many ways can this reassignment be made?
Solution: 0
Color the chairs red and black in checkerboard fashion, with the center chair black.
Then all 18 red chairs are initially occupied. Also notice that adjacent chairs have dif-
ferent colors. It follows that we need 18 black chairs to accommodate the reassignment,
but there are only 17 of them. Thus, the answer is 0.

7. You have infinitely many boxes, and you randomly put 3 balls into them. The boxes
are labeled 1, 2, . . .. Each ball has probability 1/2n of being put into box n. The balls
are placed independently of each other. What is the probability that some box will
contain at least 2 balls?
Solution: 5/7

2
Notice that the answer is the sum of the probabilities that boxes 1, 2, . . ., respectively,
contain at least 2 balls, since those events are mutually exclusive. For box n, the
probability of having at least 2 balls is

3[(1/2n )2 (1 − 1/2n )] + (1/2n )3 = 3/22n − 2/23n = 3/4n − 2/8n .

Summing to infinity using the geometric series formula, we get the answer (3/4)/(1 −
1/4) − (2/8)/(1 − 1/8), which is equal to 5/7.

8. For any subset S ⊆ {1, 2, . . . , 15}, a number n is called an “anchor” for S if n and
n + |S| are both members of S, where |S| denotes the number of members of S. Find
the average number of anchors over all possible subsets S ⊆ {1, 2, . . . , 15}.
Solution: 13/8
We first find the sum of the numbers of anchors of all subsets S; this is equivalent to
finding, for each n, the number of sets for which n is an anchor, and then summing
over all n. Suppose that n is an anchor for S, and S has k elements. Then n, n + k ∈
S ⇒ k ≥ 2, and also n + k ≤ 15, or k ≤ 15 − n. The remaining k − 2 elements of
S (other than n and n +³k) may ´ be freely chosen from the remaining 13 members of
13
{1, 2, . . . , 15}, so we get k−2 possible sets S. Summing over all allowed values of k,
³ ´ ³ ´ ³ ´ ³ ´
we then have 13 0
+ 13 1
+ 132
13
+ · · · + 13−n sets with n as an anchor. If we sum
over all n = 1, 2, . . . , 13 (since there are no possible values of k when n > 13), we get
a total of à ! à ! à ! à !
13 13 13 13
13 + 12 + 11 + ··· + .
0 1 2 12
If we call this quantity A, then, by symmetry, 2A equals
³ ´ ³ ´ ³ ´ ³ ´
13
13 0
+ 12 13 + 11 13 + ··· + 13
³1´ ³2´ ³12´ ³ ´
13 13 13 13
+ 1
+ 2 2
+ · · · + 12 12
+ 13 13
h³ ´ ³ ´ ³ ´ ³ ´ ³ ´i
13 13 13 13 13
= 13 0
+ 1
+ 2
+ ··· + 12
+ 13
= 13 · 213 .

So A = 13 · 212 is the total number of anchors over all possible sets S. Finally, to find
the average number of anchors, we divide by the number of sets, which is 215 ; thus,
the answer is 13 · 212 /215 = 13/8.

9. At a certain college, there are 10 clubs and some number of students. For any two
different students, there is some club such that exactly one of the two belongs to that
club. For any three different students, there is some club such that either exactly one
or all three belong to that club. What is the largest possible number of students?
Solution: 513
Let C be the set of clubs; each student then corresponds to a subset of C (the clubs
to which that student belongs). The two-student condition implies that these subsets
must be all distinct. Now (assuming there is more than one student) some student
belongs to a nonempty set S of clubs. For every subset T ⊆ C, let f (T ) be the subset
of C consisting of those clubs that are in exactly one of S and T (so that f (T ) =
(S ∪ T ) − (S ∩ T )). It is straightforward to check that f (f (T )) = T and f (T ) 6= T , so
that the collection of all 210 subsets of C is partitioned into pairs {T, f (T )}. Moreover,

3
as long as S is distinct from T and f (T ), every club is in either none or exactly two of
the sets S, T , and f (T ), so we cannot have a student corresponding to T and another
corresponding to f (T ). This puts an upper bound of 513 possible students (one for S,
one for ∅ = f (S), and one for each of the 511 other pairs). On the other hand, if we
take some club c, we can have one student belonging to no clubs and 512 other students
all belonging to c and to the 512 possible subsets of the other 9 clubs, respectively. It
is readily checked that this arrangement meets the conditions — for the three-student
condition, either all three students are in c, or one is the student who belongs to no
clubs and we reduce to the two-student condition — so 513 is achievable.

10. A calculator has a display, which shows a nonnegative integer N , and a button, which
replaces N by a random integer chosen uniformly from the set {0, 1, . . . , N − 1}, pro-
vided that N > 0. Initially, the display holds the number N = 2003. If the button is
pressed repeatedly until N = 0, what is the probability that the numbers 1, 10, 100,
and 1000 will each show up on the display at some point?
Solution: 1/2224222
First, we claim that if the display starts at some N , the probability that any given
number M < N will appear at some point is 1/(M +1). We can show this by induction
on N . If N = M + 1 (the base case), M can only be reached if it appears after the first
step, and this occurs with probability 1/N = 1/(M + 1). If N > M + 1 and the claim
holds for N − 1, then there are two possibilities starting from N . If the first step leads
to N − 1 (this occurs with probability 1/N ), the probability of seeing M subsequently
is 1/(M + 1) by the induction hypothesis. If the first step leads to something less than
N −1 (probability (N −1)/N ), then it leads to any of the integers {0, 1, . . . , N −2} with
equal probability. But this is exactly what the first step would have been if we had
started from N −1; hence, the probability of seeing M is again 1/(M +1) by induction.
Thus, the overall probability of seeing M is N1 · M1+1 + NN−1 · M1+1 = 1/(M + 1), proving
the induction step and the claim.
Now let P (N, M ) (M < N ) be the probability of eventually seeing the number M if
we start at N ; note that this is the same as the conditional probability of seeing M
given that we see N . Hence, the desired probability is
1 1 1 1 1
P (2003, 1000) · P (1000, 100) · P (100, 10) · P (10, 1) = · · · = .
1001 101 11 2 2224222

4
Harvard-MIT Mathematics Tournament
March 15, 2003

Individual Round: Geometry Subject Test — Solutions

1. AD and BC are both perpendicular to AB, and CD is perpendicular to AC. If AB = 4


and BC = 3, find CD.
D

A B

Solution: 20/3
By Pythagoras in 4ABC, AC = 5. But 6 CAD = 90◦ − 6 BAC = 6 ACB, so right
triangles CAD, BCA are similar, and CD/AC = BA/CB = 4/3 ⇒ CD = 20/3.

2. As shown, U and C are points on the sides of triangle M N H such that M U = s,


U N = 6, N C = 20, CH = s, HM = 25. If triangle U N C and quadrilateral M U CH
have equal areas, what is s?
N

6
20
U
s
M C
s
25 H

Solution: 4
Using brackets to denote areas, we have [M CH] = [U N C] + [M U CH] = 2[U N C]. On
the other hand, triangles with equal altitudes have their areas in the same ratio as
their bases, so

[M N H] [M N H] [M N C] NH MN s + 20 s + 6
2= = · = · = · .
[U N C] [M N C] [U N C] NC UN 20 6

Clearing the denominator gives (s + 20)(s + 6) = 240, and solving the quadratic gives
s = 4 or −30. Since s > 0, we must have s = 4.

1
3. A room is built in the shape of the region between two semicircles with the same center
and parallel diameters. The farthest distance between two points with a clear line of
sight is 12m. What is the area (in m2 ) of the room?
Solution: 18π
The maximal distance is as shown in the figure. Call the radii R and r, R > r. Then
R2 − r2 = 62 by the Pythagorean theorem, so the area is (π/2) · (R2 − r2 ) = 18π.

4. Farmer John is inside of an ellipse with reflective sides, given by the equation x2 /a2 +
y 2 /b2 = 1, with a > b > 0. He is standing at the point (3, 0), and he shines a laser
pointer in the y-direciton. The light reflects off the ellipse and proceeds directly toward
Farmer Brown, traveling a distance of 10 before reaching him. Farmer John then spins
around in a circle; wherever he points the laser, the light reflects off the wall and hits
Farmer Brown. What is the ordered pair (a, b)?
Solution: (5, 4)
The points where the farmers are standing must be the foci of the ellipse, so they are
(3, 0) and (−3, 0). If the total distance traveled is 10, then a must be half of that, or
5, since the distance traveled by a ray reflecting off the wall from when it leaves one
focus to when it reaches the other
√ focus is√2a, the length of the major axis. If a focus
is at x = 3, then we have 3 = a2 − b2 = 25 − b2 , yielding b = 4.

5. Consider a 2003-gon inscribed in a circle and a triangulation of it with diagonals


intersecting only at vertices. What is the smallest possible number of obtuse triangles
in the triangulation?
Solution: 1999
By induction, it follows easily that any triangulation of an n-gon inscribed in a circle
has n − 2 triangles. A triangle is obtuse unless it contains the center of the circle in its
interior (in which case it is acute) or on one of its edges (in which case it is right). It
is then clear that there are at most 2 non-obtuse triangles, and 2 is achieved when the
center of the circle is on one of the diagonals of the triangulation. So the minimum
number of obtuse triangles is 2001 − 2 = 1999.

6. Take a clay sphere of radius 13, and drill a circular hole of radius 5 through its center.
Take the remaining “bead” and mold it into a new sphere. What is this sphere’s
radius?
Solution: 12
Let r be the radius of the sphere. We take cross sections of the bead perpendicular
to the line of the drill and compare them to cross sections of the sphere at the same

2
distance√from its center. At a height h, the cross section of the sphere is a circle with
radius r2 − h2 and thus area π(r2 − h2√ ). At the same height, the cross section of
the bead is an annulus with outer radius 132 − h2 and inner radius 5, for an area of
π(132 − h2 ) − π(52 ) = π(122 − h2 ) (since 132 − 52 = 122 ). Thus, if r = 12, the sphere
and the bead will have the same cross-sectional area π(122 − h2 ) for |h| ≤ 12 and 0
for |h| > 12. Since all the cross sections have the same area, the two clay figures then
have the same volume. And certainly there is only one value of r for which the two
volumes are equal, so r = 12 is the answer.

7. Let RST U V be a regular pentagon. Construct an equilateral triangle P RS with point


P inside the pentagon. Find the measure (in degrees) of angle P T V .
Solution: 6
We have 6 P RV = 6 SRV − 6 SRP = 108◦ −60◦ = 48◦ . Since P R = RS = RV , triangle
P RV is isosceles, so that 6 V P R = 6 RV P = (180◦ − 6 P RV )/2 = 66◦ . Likewise, we
have 6 T P S = 66◦ , so that

6 T P V = 360◦ − (6 V P R + 6 RP S + 6 SP T ) = 360◦ − (66◦ + 60◦ + 66◦ ) = 168◦ .

Finally, by symmetry, triangle P T V is isosceles (P T = T V ), so 6 P T V = 6 T V P =


(180◦ − 6 T P V )/2 = 6◦ . (See the figure.)

V T
P

R S

8. Let ABC be an equilateral triangle of side length 2. Let ω be its circumcircle, and let
ωA , ωB , ωC be circles congruent to ω centered at each of its vertices. Let R be the set
of all points in the plane contained in exactly two of these four circles. What is the
area of R?

Solution: 2 3
ωA , ωB , ωC intersect at the circumcenter; thus, every point within the circumcircle, and
no point outside of it, is in two or more circles. The area inside exactly two circles
is shaded in the figure. The two intersection points of ωA and ωB , together with A,
form the vertices of an equilateral triangle. As shown, this equilateral triangle cuts off
a “lip” of ω (bounded by a 60◦ arc of ω and the corresponding chord) and another,
congruent lip of ωB that is not part of the region of interest. By rotating the first lip
to the position of the second, we can reassemble the equilateral triangle. Doing this for
each of the 6 such triangles, we see that the desired area equals the √ area of a regular

hexagon inscribed√in ω. The √ side length
√ of this hexagon is (2/3) · ( 3/2) · 2 = 2 3/3,
so its area is 6 · ( 3/4) · (2 3/3)2 = 2 3, and this is the answer.

3
ωA

ωB ωC

B C
ω

9. In triangle ABC, 6 ABC = 50◦ and 6 ACB = 70◦ . Let D be the midpoint of side
BC. A circle is tangent to BC at B and is also tangent to segment AD; this circle
instersects AB again at P . Another circle is tangent to BC at C and is also tangent
to segment AD; this circle intersects AC again at Q. Find 6 AP Q (in degrees).
Solution: 70
Suppose the circles are tangent to AD at E, F , respectively; then, by equal tangents,
DE = DB = DC = DF ⇒ E = F (as shown). So, by the Power of a Point Theorem,
AP · AB = AE 2 = AF 2 = AQ · AC ⇒ AP/AQ = AC/AB ⇒ 4AP Q ∼ 4ACB,
giving 6 AP Q = 6 ACB = 70◦ .

A
P
Q
E F

B D C

10. Convex quadrilateral M AT H is given with HM/M T = 3/4, and 6 AT M = 6 M AT =


6 AHM = 60◦ . N is the midpoint of M A, and O is a point on T H such that lines

M T, AH, N O are concurrent. Find the ratio HO/OT .


Solution: 9/16
Triangle M AT is equilateral, so HM/AT = HM/M T = 3/4. Also, 6 AHM = 6 AT M ,
so the quadrilateral is cyclic. Now, let P be the intersection of M T, AH, N O. Extend
M H and N O to intersect at point Q. Then by Menelaus’s theorem, applied to triangle

4
AHM and line QN P , we have
HQ M N AP
· · = 1,
QM N A P H
while applying the same theorem to triangle T HM and line QP O gives
HQ M P T O
· · = 1.
QM P T OH

Combining gives HO/OT = (M P/P T )·(AN/N M )·(HP/P A) = (M P/P A)·(HP/P T )


(because AN/N M = 1). But since M AT H is cyclic, 4AP T ∼ 4M P H, so M P/P A =
HP/P T = HM/AT = 3/4, and the answer is (3/4)2 = 9/16. (See figure.)

O H

A N M

5
Harvard-MIT Mathematics Tournament
March 15, 2003

Individual Round: General Test, Part 1 — Solutions

1. 10 people are playing musical chairs with n chairs in a circle. They can be seated
in 7! ways (assuming only one person fits on each chair, of course), where different
arrangements of the same people on chairs, even rotations, are considered different.
Find n.
Solution: 4
The number of ways 10 people can be seated on n chairs is n! multiplied by the number
of ways one can choose n people out of 10. Hence we must solve 7! = n! · 10!/(n! · (10 −
n)!). This is equivalent to (10 − n)! = 10!/7! = 8 · 9 · 10 = 720 = 6!. We therefore have
n = 4.

2. OP EN is a square, and T is a point on side N O, such that triangle T OP has area 62


and triangle T EN has area 10. What is the length of a side of the square?
Solution: 12
62 = P O · OT /2 and 10 = EN · T N/2 = P O · T N/2, so adding gives 72 = P O · (OT +
T N )/2 = P O · ON/2 = P O2 /2 ⇒ P O = 12.

3. There are 16 members on the Height-Measurement Matching Team. Each member was
asked, “How many other people on the team — not counting yourself — are exactly
the same height as you?” The answers included six 1’s, six 2’s, and three 3’s. What
was the sixteenth answer? (Assume that everyone answered truthfully.)
Solution: 3
For anyone to have answered 3, there must have been exactly 4 people with the same
height, and then each of them would have given the answer 3. Thus, we need at least
four 3’s, so 3 is the remaining answer. (More generally, a similar argument shows that
the number of members answering n must be divisible by n + 1.)

4. How many 2-digit positive integers have an even number of positive divisors?
Solution: 84
An integer has an odd number of divisors precisely if it is a square. So we take the 90
2-digit numbers (10, 11, . . . , 99) and remove the 6 squares (42 , 52 , . . . , 92 ), for a total of
84.

5. A room is built in the shape of the region between two semicircles with the same center
and parallel diameters. The farthest distance between two points with a clear line of
sight is 12m. What is the area (in m2 ) of the room?
Solution: 18π
The maximal distance is as shown in the figure (next page). Call the radii R and r,
R > r. Then R2 −r2 = 62 by the Pythagorean theorem, so the area is (π/2)·(R2 −r2 ) =
18π.

1
6. In how many ways can 3 bottles of ketchup and 7 bottles of mustard be arranged in a
row so that no bottle of ketchup is immediately between two bottles of mustard? (The
bottles of ketchup are mutually indistinguishable, as are the bottles of mustard.)
Solution: 22
Consider the blocks of consecutive bottles of ketchup in such an arrangement. A block
of just one bottle must occur at the beginning or the end of the row, or else it would
be between two bottles of mustard. However, a block of two or three bottles can occur
anywhere. We cannot have three blocks of one bottle each, since there are only two
possible locations for such blocks. Thus, we either have a block of one bottle and a
block of two, or one block of all three bottles. In the first case, if the single bottle
occurs at the beginning of the row, then anywhere from 1 to 7 bottles of mustard may
intervene before the block of 2 ketchup bottles, giving 7 possible arrangements. We
likewise have 7 arrangements if the single bottle occurs at the end of the row. Finally,
if there is just one block of three bottles, anywhere from 0 to 7 mustard bottles may
precede it, giving 8 possible arrangements. So, altogether, we have 7 + 7 + 8 = 22
configurations.

7. Find the real value of x such that x3 + 3x2 + 3x + 7 = 0.



Solution: −1 − 3 6

Rewrite
√ the equation as (x + 1)3 + 6 = 0 to get (x + 1)3 = −6 ⇒ x + 1 = 3
−6 ⇒ x =
−1 − 3 6.

8. A broken calculator has the + and × keys switched. For how many ordered pairs (a, b)
of integers will it correctly calculate a + b using the labelled + key?
Solution: 2
b 1
We need a + b = ab, or a = b−1 = 1 − b−1 , so 1/(b − 1) is an integer. Thus b must be
0 or 2, and a is 0 or 2, respectively. So there are 2.

9. Consider a 2003-gon inscribed in a circle and a triangulation of it with diagonals


intersecting only at vertices. What is the smallest possible number of obtuse triangles
in the triangulation?
Solution: 1999
By induction, it follows easily that any triangulation of an n-gon inscribed in a circle
has n − 2 triangles. A triangle is obtuse unless it contains the center of the circle in its
interior (in which case it is acute) or on one of its edges (in which case it is right). It
is then clear that there are at most 2 non-obtuse triangles, and 2 is achieved when the
center of the circle is on one of the diagonals of the triangulation. So the minimum
number of obtuse triangles is 2001 − 2 = 1999.

2
10. Bessie the cow is trying to navigate her way through a field. She can travel only
from lattice point to adjacent lattice point, can turn only at lattice points, and can
travel only to the east or north. (A lattice point is a point whose coordinates are both
integers.) (0, 0) is the southwest corner of the field. (5, 5) is the northeast corner of
the field. Due to large rocks, Bessie is unable to walk on the points (1, 1), (2, 3), or
(3, 2). How many ways are there for Bessie to travel from (0, 0) to (5, 5) under these
constraints?
Solution: 32
In the figure, each point is labeled with the number of ways to reach that point. The
numbers are successively computed as follows: The point (0, 0) can trivially be reached
in 1 way. When Bessie reaches any subsequent point (x, y) (other than a rock), she
can arrive either via a northward or an eastward step, so the number of ways she can
reach that point equals the number of ways of reaching (x − 1, y) plus the number of
ways of reaching (x, y − 1). By iterating this calculation, we eventually find that (5, 5)
can be reached in 32 ways.

(5,5)

1 4 7 10 16 32

1 3 3 3 6 16

1 2 0 3 10

1 1 2 3 7

1 1 2 3 4

1 1 1 1 1 1

(0,0)

3
Harvard-MIT Mathematics Tournament
March 15, 2003

Individual Round: General Test, Part 2 — Solutions

1. A compact disc has the shape of a circle of diameter 5 inches with a 1-inch-diameter
circular hole in the center. Assuming the capacity of the CD is proportional to its
area, how many inches would need to be added to the outer diameter to double the
capacity?
Solution: 2
Doubling the capacity is equivalent to doubling the area, which is initially π[(5/2)2 −
(1/2)2 ] = 6π. Thus we want to achieve an area of 12π, so if the new diameter is d, we
want π[(d/2)2 − (1/2)2 ] = 12π ⇒ d = 7. Thus we need to add 2 inches to the diameter.
2. You have a list of real numbers, whose sum is 40. If you replace every number x on
the list by 1 − x, the sum of the new numbers will be 20. If instead you had replaced
every number x by 1 + x, what would the sum then be?
Solution: 100
Let n be the number of numbers on the list. If each initial number is replaced by
its negative, the sum will then be −40, and adding 1 to every number on this list
increases the sum by n, so n − 40 = 20 ⇒ n = 60. Then, if we had simply added
1 to each of the initial numbers (without negating first), the sum would increase to
40 + n = 40 + 60 = 100.
3. How many positive rational numbers less than π have denominator at most 7 when
written in lowest terms? (Integers have denominator 1.)
Solution: 54
We can simply list them. The table shows that there are 3+3+6+6+12+6+18 = 54.

Denominator Values
1 2 3
1 1
, 1
, 1
1 3 5
2 2
, 2
, 2
1 2 4 5
3 3
, 3
, ,
3 3
, 73 , 83
1 3
4 4
, 4
, 45 , 74 , 94 , 11 4
1 2 3 4 6 7 8 9 11 12 13 14
5 5
, , , , 5, 5, 5, 5,
5 5 5
, , ,
5 5 5 5
1 5
6 6
, 6
, 67 , 11 6
, 136 6
, 17
1 2
7 7
, 7
, . . . , 76 , 87 , 79 , . . . , 13
7
, 15 16
, , . . . , 20
7 7 7

4. In triangle ABC with area 51, points D and E trisect AB and points F and G trisect
BC. Find the largest possible area of quadrilateral DEF G.
Solution: 17
Assume E is between D and B, and F is between G and B (the alternative is to switch
two points, say D and E, which clearly gives a non-convex quadrilateral with smaller

1
area). If two triangles have their bases on the same line and the same opposite vertex,
then it follows from the 21 bh formula that their areas are in the same ratio as their
bases. In particular (brackets denote areas),
[DBG] [DBG] [ABG] DB BG 2 2 4
= · = · = · = ,
[ABC] [ABG] [ABC] AB BC 3 3 9
and similarly [EBF ]/[ABC] = 1/9. Subtracting gives [DEF G]/[ABC] = 1/3, so the
answer is [ABC]/3 = 17.

5. You are given a 10 × 2 grid of unit squares. Two different squares are adjacent if they
share a side. How many ways can one mark exactly nine of the squares so that no two
marked squares are adjacent?
Solution: 36
Since each row has only two squares, it is impossible for two marked squares to be
in the same row. Therefore, exactly nine of the ten rows contain marked squares.
Consider two cases:
Case 1: The first or last row is empty. These two cases are symmetrical, so assume
without loss of generality that the first row is empty. There are two possibilities for the
second row: either the first square is marked, or the second square is marked. Since
the third row must contain a marked square, and it cannot be in the same column
as the marked square in the second row, the third row is determined by the second.
Similarly, all the remaining rows are determined. This leaves two possibilities if the
first row is empty. Thus, there are four possibilities if the first or last row is empty.
Case 2: The empty row is not the first or last. Then, there are two blocks of (one of
more) consecutive rows of marked squares. As above, the configuration of the rows in
each of the two blocks is determined by the position of the marked square in the first
of its rows. That makes 2 × 2 = 4 possible configurations. There are eight possibilities
for the empty row, making a total of 32 possibilities in this case.
Together, there are 36 possible configurations of marked squares.

6. The numbers 112, 121, 123, 153, 243, 313, and 322 are among the rows, columns, and
diagonals of a 3 × 3 square grid of digits (rows and diagonals read left-to-right, and
columns read top-to-bottom). What 3-digit number completes the list?
Solution: 524

1 1 2
5 2 4
3 1 3

The center digit is the middle digit of 4 numbers (hence at least 3 members of the
above list), so it must be 2. The top-left digit begins at least 2 members of the above
list, so it must be 1 or 3. If it is 3, then after placing 313 we see that we need three
more numbers starting with 3, impossible; hence, it is 1. So 243 and 313 must (in some
order) be the last row and the last column, and now it is easy to complete the grid as
shown; the answer is 524.

2
7. Daniel and Scott are playing a game where a player wins as soon as he has two points
more than his opponent. Both players start at par, and points are earned one at a
time. If Daniel has a 60% chance of winning each point, what is the probability that
he will win the game?
Solution: 9/13
Consider the situation after two points. Daniel has a 9/25 chance of winning, Scott,
4/25, and there is a 12/25 chance that the players will be tied. In the latter case, we
revert to the original situation. In particular, after every two points, either the game
returns to the original situation, or one player wins. If it is given that the game lasts
2k rounds, then the players must be at par after 2(k − 1) rounds, and then Daniel wins
with probability (9/25)/(9/25 + 4/25) = 9/13. Since this holds for any k, we conclude
that Daniel wins the game with probability 9/13.

8. If x ≥ 0, y ≥ 0 are integers, randomly chosen with the constraint x + y ≤ 10, what is


the probability that x + y is even?
Solution: 6/11
For each p ≤ 10, if x + y = p, x can range from 0 to p, yielding p + 1 ordered pairs
(x, y). Thus there are a total of 1 + 2 + 3 + · · · + 11 allowable ordered pairs (x, y), but
1 + 3 + 5 + · · · + 11 of these pairs have an even sum. So the desired probability is

1 + 3 + 5 + · · · + 11 62 6
= = .
1 + 2 + 3 + · · · + 11 11 · 6 11

9. In a classroom, 34 students are seated in 5 rows of 7 chairs. The place at the center of
the room is unoccupied. A teacher decides to reassign the seats such that each student
will occupy a chair adjacent to his/her present one (i.e. move one desk forward, back,
left or right). In how many ways can this reassignment be made?
Solution: 0
Color the chairs red and black in checkerboard fashion, with the center chair black.
Then all 18 red chairs are initially occupied. Also notice that adjacent chairs have dif-
ferent colors. It follows that we need 18 black chairs to accommodate the reassignment,
but there are only 17 of them. Thus, the answer is 0.

10. Several positive integers are given, not necessarily all different. Their sum is 2003.
Suppose that n1 of the given numbers are equal to 1, n2 of them are equal to 2, . . .,
n2003 of them are equal to 2003. Find the largest possible value of

n2 + 2n3 + 3n4 + · · · + 2002n2003 .

Solution: 2002
The sum of all the numbers is n1 + 2n2 + · · · + 2003n2003 , while the number of numbers
is n1 + n2 + · · · + n2003 . Hence, the desired quantity equals

(n1 + 2n2 + · · · + 2003n2003 ) − (n1 + n2 + · · · + n2003 )

= (sum of the numbers) − (number of numbers)

3
= 2003 − (number of numbers),
which is maximized when the number of numbers is minimized. Hence, we should have
just one number, equal to 2003, and then the specified sum is 2003 − 1 = 2002.
Comment: On the day of the contest, a protest was lodged (successfully) on the
grounds that the use of the words “several” and “their” in the problem statement
implies there must be at least 2 numbers. Then the answer is 2001, and this maximum
is achieved by any two numbers whose sum is 2003.

4
Harvard-MIT Mathematics Tournament
March 15, 2003

Guts Round — Solutions


q √ q
√ 4006 √
2003
1. Simplify 2 11 − 3 5 · 89 + 12 55.
Solution: −1
√ √ √
Note that (2 11 + 3 5)2 = 89 + 12 55. So, we have
q √ √ q √ q √ √ 2003q √ √
2003 4006 2003
2 11 − 3 5 · 89 + 12 55 = 2 11 − 3 5 · 2 11 + 3 5
q √ √ √
2003
= (2 11)2 − (3 5)2 = 2003 −1 = −1.

2. The graph of x4 = x2 y 2 is a union of n different lines. What is the value of n?


Solution: 3
The equation x4 − x2 y 2 = 0 factors as x2 (x + y)(x − y) = 0, so its graph is the union
of the three lines x = 0, x + y = 0, and x − y = 0.

3. If a and b are positive integers that can each be written as a sum of two squares, then
ab is also a sum of two squares. Find the smallest positive integer c such that c = ab,
where a = x3 + y 3 and b = x3 + y 3 each have solutions in integers (x, y), but c = x3 + y 3
does not.
Solution: 4
We can’t have c = 1 = 13 + 03 or c = 2 = 13 + 13 , and if c = 3, then a or b = ±3 which
is not a sum of two cubes (otherwise, flipping signs of x and y if necessary, we would
get either a sum of two nonnegative cubes to equal 3, which clearly does not happen,
or a difference of two nonnegative cubes to equal 3, but the smallest difference between
two successive cubes ≥ 1 is 23 − 13 = 7). However, c = 4 does meet the conditions,
with a = b = 2 = 13 + 13 (an argument similar to the above shows that there are no
x, y with 4 = x3 + y 3 ), so 4 is the answer.
1 2 3
4. Let z = 1 − 2i. Find z
+ z2
+ z3
+ · · ·.
Solution: (2i − 1)/4
³ ´
Let x = z1 + z22 + z33 + · · ·, so z · x = 1 + z2 + z32 + z43 + · · · . Then z · x − x =
1 + z1 + z12 + z13 + · · · = 1−1/z
1 z
= z−1 z
. Solving for x in terms of z, we obtain x = (z−1)2.

Plugging in the original value of z produces x = (2i − 1)/4.

5. Compute the surface area of a cube inscribed in a sphere of surface area π.


Solution: 2
r satisfies 4πr2 = π ⇒ r = 1/2,√so the cube has body diagonal 1,
The sphere’s radius √
hence side length 1/ 3. So, its surface area is 6(1/ 3)2 = 2.

6. Define the Fibonacci numbers by F0 = 0, F1 = 1, Fn = Fn−1 + Fn−2 for n ≥ 2. For


how many n, 0 ≤ n ≤ 100, is Fn a multiple of 13?

1
Solution: 15
The sequence of remainders modulo 13 begins 0, 1, 1, 2, 3, 5, 8, 0, and then we have
Fn+7 ≡ 8Fn modulo 13 by a straightforward induction. In particular, Fn is a multiple
of 13 if and only if 7 | n, so there are 15 such n.
√ q √
7. a and b are integers such that a + b = 15 + 216. Compute a/b.
Solution: 1/2
√ √
Squaring both sides gives a2 +b+2a b = 15+ 216; separating rational from irrational
parts, we get a2 + b = 15, 4a2 b = 216, so a2 and b equal 6 and 9. a is√ an integer, so
a2 = 9, b = 6 ⇒ a/b = 3/6 = 1/2. (We cannot have a = −3, since a + b is positive.)

8. How many solutions in nonnegative integers (a, b, c) are there to the equation

2a + 2b = c! ?

Solution: 5
We can check that 2a + 2b is never divisible by 7, so we must have c < 7. The binary
representation of 2a + 2b has at most two 1’s. Writing 0!, 1!, 2!, . . . , 6! in binary, we can
check that the only possibilities are c = 2, 3, 4, giving solutions (0, 0, 2), (1, 2, 3), (2, 1, 3),
(3, 4, 4), (4, 3, 4).

9. For x a real number, let f (x) = 0 if x < 1 and f (x) = 2x − 2 if x ≥ 1. How many
solutions are there to the equation

f (f (f (f (x)))) = x?

Solution: 2
Certainly 0, 2 are fixed points of f and therefore solutions. On the other hand, there
can be no solutions for x < 0, since f is nonnegative-valued; for 0 < x < 2, we have
0 ≤ f (x) < x < 2 (and f (0) = 0), so iteration only produces values below x, and for
x > 2, f (x) > x, and iteration produces higher values. So there are no other solutions.

10. Suppose that A, B, C, D are four points in the plane, and let Q, R, S, T, U, V be the
respective midpoints of AB, AC, AD, BC, BD, CD. If QR = 2001, SU = 2002, T V =
2003, find the distance between the midpoints of QU and RV .
Solution: 2001
This problem has far more information than necessary: QR and U V are both parallel
to BC, and QU and RV are both parallel to AD. Hence, QU V R is a parallelogram,
and the desired distance is simply the same as the side length QR, namely 2001. (See
figure, next page.)

11. Find the smallest positive integer n such that 12 + 22 + 32 + 42 + · · · + n2 is divisible


by 100.
Solution: 24
The sum of the first n squares equals n(n + 1)(2n + 1)/6, so we require n(n + 1)(2n + 1)
to be divisible by 600 = 24 · 25. The three factors are pairwise relatively prime, so

2
A
R

C Q

one of them must be divisible by 25. The smallest n for which this happens is n = 12
(2n + 1 = 25), but then we do not have enough factors of 2. The next smallest is
n = 24 (n + 1 = 25), and this works, so 24 is the answer.

12. As shown in the figure, a circle of radius 1 has two equal circles whose diameters cover
a chosen diameter of the larger circle. In each of these smaller circles we similarly draw
three equal circles, then four in each of those, and so on. Compute the area of the
region enclosed by a positive even number of circles.

Solution: π/e
At the nth step, we have n! circles of radius 1/n! each, for a total area of n! · π/(n!)2 =
π/n!. The desired area is obtained by adding the areas of the circles at step 2, then
subtracting those at step 3, then adding those at step 4, then subtracting those at step
5, and so forth. Thus, the answer is
µ ¶
π π π 1 1 1 1
− + − ··· = π − + − + · · · = πe−1 .
2! 3! 4! 0! 1! 2! 3!

13. If xy = 5 and x2 + y 2 = 21, compute x4 + y 4 .


Solution: 391
We have 441 = (x2 + y 2 )2 = x4 + y 4 + 2(xy)2 = x4 + y 4 + 50, yielding x4 + y 4 = 391.

3
14. A positive integer will be called “sparkly” if its smallest (positive) divisor, other than
1, equals the total number of divisors (including 1). How many of the numbers
2, 3, . . . , 2003 are sparkly?
Solution: 3
Suppose n is sparkly; then its smallest divisor other than 1 is some prime p. Hence,
n has p divisors. However, if the full prime factorization of n is pe11 pe22 · · · perr , the
number of divisors is (e1 + 1)(e2 + 1) · · · (er + 1). For this to equal p, only one factor
can be greater than 1, so n has only one prime divisor — namely p — and we get
e1 = p − 1 ⇒ n = pp−1 . Conversely, any number of the form pp−1 is sparkly. There are
just three such numbers in the desired range (21 , 32 , 54 ), so the answer is 3.

15. The product of the digits of a 5-digit number is 180. How many such numbers exist?
Solution: 360
Let the digits be a, b, c, d, e. Then abcde = 180 = 22 · 32 · 5. We observe that there
are 6 ways to factor 180 into digits a, b, c, d, e (ignoring differences in ordering): 180 =
1 · 1 · 4 · 5 · 9 = 1 · 1 · 5 · 6 · 6 = 1 · 2 · 2 · 5 · 9 = 1 · 2 · 3 · 5 · 6 = 1 · 3 · 3 · 4 · 5 = 2 · 2 · 3 · 3 · 5.
There are (respectively) 60, 30, 60, 120, 60, and 30 permutations of these breakdowns,
for a total of 360 numbers.

16. What fraction of the area of a regular hexagon of side length 1 is within distance 12 of
at least one of the vertices?

Solution: π 3/9
√ √
The hexagon has area 6( 3/4)(1)2 = 3 3/2. The region we want consists of six 120◦
arcs of circles of radius 1/2, whichcan be reassembled
√ into two circles of radius 1/2. So
its area is π/2, and the ratio of areas is π 3/9.

17. There are 10 cities in a state, and some pairs of cities are connected by roads. There
are 40 roads altogether. A city is called a “hub” if it is directly connected to every
other city. What is the largest possible number of hubs?
Solution: 6
³ ´
h
If there are h hubs, then 2
roads connect the hubs to each other, and each hub is
³ ´
h
connected to the other 10 − h cities; we thus get 2
+ h(10 − h) distinct roads. So,
³ ´
h 2
40 ≥ +h(10−h) = −h /2+19h/2, or 80 ≥ h(19−h). The largest h ≤ 10 satisfying
2
this condition is h = 6, and conversely,
³ ´ if we connect each of 6 cities to every other
6
city and place the remaining 40 − [ 2 + 6(10 − 6)] = 1 road wherever we wish, we can
achieve 6 hubs. So 6 is the answer.

18. Find the sum of the reciprocals of all the (positive) divisors of 144.
Solution: 403/144
As d ranges over the divisors of 144, so does 144/d, so the sum of 1/d is 1/144 times the
sum of the divisors of 144. Using the formula for the sum of the divisors of a number
(or just counting them out by hand), we get that this sum is 403, so the answer is
403/144.

4
19. Let r, s, t be the solutions to the equation x3 + ax2 + bx + c = 0. What is the value of
(rs)2 + (st)2 + (rt)2 in terms of a, b, and c?
Solution: b2 − 2ac
We have (x − r)(x − s)(x − t) = x3 + ax2 + bx + c, so

a = −(r + s + t), b = rs + st + rt, c = −rst.

So we have

(rs)2 + (st)2 + (rt)2 = (rs + st + rt)2 − 2rst(r + s + t) = b2 − 2ac.

20. What is the smallest number of regular hexagons of side length 1 needed to completely
cover a disc of radius 1?
Solution: 3
First, we show that two hexagons do not suffice. Specifically, we claim that a hexagon
covers less than half of the disc’s boundary. First, a hexagon of side length 1 may be
inscribed in a circle, and this covers just 6 points. Translating the hexagon vertically
upward (regardless of its orientation) will cause it to no longer touch any point on
the lower half of the circle, so that it now covers less than half of the boundary. By
rotational symmetry, the same argument applies to translation in any other direction,
proving the claim. Then, two hexagons cannot possibly cover the disc.
The disc can be covered by three hexagons as follows. Let P be the center of the circle.
Put three non-overlapping hexagons together at point P . This will cover the circle,
since each hexagon will cover a 120◦ sector of the circle.

21. r and s are integers such that

3r ≥ 2s − 3 and 4s ≥ r + 12.

What is the smallest possible value of r/s?


Solution: 1/2
We simply plot the two inequalities in the sr-plane and find the lattice point satisfying
both inequalities such that the slope from it to the origin is as low as possible. We find
that this point is (2, 4) (or (3, 6)), as circled in the figure, so the answer is 2/4 = 1/2.

5
r 4s = r + 12

3r = 2s - 3

22. There are 100 houses in a row on a street. A painter comes and paints every house red.
Then, another painter comes and paints every third house (starting with house number
3) blue. Another painter comes and paints every fifth house red (even if it is already
red), then another painter paints every seventh house blue, and so forth, alternating
between red and blue, until 50 painters have been by. After this is finished, how many
houses will be red?
Solution: 52
House n ends up red if and only if the largest odd divisor of n is of the form 4k + 1. We
have 25 values of n = 4k + 1; 13 values of n = 2(4k + 1) (given by k = 0, 1, 2, . . . , 12);
7 values of n = 4(4k + 1) (k = 0, 1, . . . , 6); 3 values of n = 8(4k + 1) (k = 0, 1, 2); 2 of
the form n = 16(4k + 1) (for k = 0, 1); 1 of the form n = 32(4k + 1); and 1 of the form
n = 64(4k + 1). Thus we have a total of 25 + 13 + 7 + 3 + 2 + 1 + 1 = 52 red houses.
23. How many lattice points are enclosed by the triangle with vertices (0, 99), (5, 100), and
(2003, 500)? Don’t count boundary points.
Solution: 0
Using the determinant formula, we get that the area of the triangle is
¯ ¯
¯ 5 1 ¯.
¯ ¯
¯ ¯ 2 = 1.
¯ 2003 401 ¯

There are 4 lattice points on the boundary of the triangle (the three vertices and
(1004, 300)), so it follows from Pick’s Theorem that there are 0 in the interior.
24. Compute the radius of the inscribed circle of a triangle with sides 15, 16, and 17.

Solution: 21
√ √
Hero’s formula gives that the area is 24 · 9 · 8 · 7 = 24 21. Then, using the result
that the area
√ of a triangle equals the inradius times half the perimeter, we see that the
radius is 21.

6
25. Let ABC be an isosceles triangle with apex A. Let I be the incenter. If AI = 3 and
the distance from I to BC is 2, then what is the length of BC?

Solution: 4 5
Let X and Y be the points where the incircle touches AB and BC, respectively.
Then AXI and AY B are similar right triangles. Since I is the incenter, we have

IX = IY = 2. Using the Pythagorean theorem on triangle AXI, we √ find AX = 5.
By similarity,
√ AY /AX = BY /IX. Plugging in the
√ numbers given, 5/ 5 = BY /2, so
BY = 2 5. Y is the midpoint of BC, so BC = 4 5.

26. Find all integers m such that m2 + 6m + 28 is a perfect square.


Solution: 6, −12
We must have m2 +6m+28 = n2 , where n is an integer. Rewrite this as (m+3)2 +19 =
n2 ⇒ n2 − (m + 3)2 = 19 ⇒ (n − m − 3)(n + m + 3) = 19. Let a = n − m − 3 and
b = n + m + 3, so we want ab = 19. This leaves only 4 cases:

• a = 1, b = 19. Solve the system n − m − 3 = 1 and n + m + 3 = 19 to get n = 10


and m = 6, giving one possible solution.
• a = 19, b = 1. Solve the system, as above, to get n = 10 and m = −12.
• a = −1, b = −19. We get n = −10 and m = −12.
• a = −19, b = −1. We get n = −10 and m = 6.

Thus the only m are 6 and −12.

27. The rational numbers x and y, when written in lowest terms, have denominators 60
and 70, respectively. What is the smallest possible denominator of x + y?
Solution: 84
Write x + y = a/60 + b/70 = (7a + 6b)/420. Since a is relatively prime to 60 and b is
relatively prime to 70, it follows that none of the primes 2, 3, 7 can divide 7a + 6b, so we
won’t be able to cancel any of these factors in the denominator. Thus, after reducing
to lowest terms, the denominator will still be at least 22 · 3 · 7 = 84 (the product of
the powers of 2, 3, and 7 dividing 420). On the other hand, 84 is achievable, by taking
(e.g.) 1/60 + 3/70 = 25/420 = 5/84. So 84 is the answer.

28. A point in three-space has distances 2, 6, 7, 8, 9 from five of the vertices of a regular
octahedron. What is its distance from the sixth vertex?

Solution: 21
By a simple variant of the British Flag Theorem, if ABCD is a square and P any point
in space, AP 2 + CP 2 = BP 2 + DP 2 . Four of the five given vertices must form a square
ABCD, and by experimentation we find their distances to the given point P must be
AP = 2, BP = 6, CP = 9, DP = 7. Then A, C, and the other two vertices E, F√also
form a square AECF , so 85 = AP 2 + CP 2 = EP 2 + F P 2 = 82 + F P 2 ⇒ F P = 21.

29. A palindrome is a positive integer that reads the same backwards as forwards, such as
82328. What is the smallest 5-digit palindrome that is a multiple of 99?
Solution: 54945

7
Write the number as XY ZY X. This is the same as 10000X+1000Y +100Z+10Y +X =
99(101X +10Y +Z)+20Y +2X +Z. We thus want 20Y +2X +Z to be a multiple of 99,
with X as small as possible. This expression cannot be larger than 20·9+2·9+9 = 207,
and it is greater than 0 (since X 6= 0), so for this to be a multiple of 99, it must equal
99 or 198. Consider these two cases.
To get 198, we must have Y = 9, which then leaves 2X +Z = 18. The smallest possible
X is 5, and then Z becomes 8 and we have the number 59895.
To get 99, we must have Y = 4. Then, 2X + Z = 19, and, as above, we find the
minimal X is 5 and then Z = 9. This gives us the number 54945. This is smaller than
the other number, so it is the smallest number satisfying the conditions of the problem.
30. The sequence a1 , a2 , a3 , . . . of real numbers satisfies the recurrence
a2n − an−1 + 2an
an+1 = .
an−1 + 1
Given that a1 = 1 and a9 = 7, find a5 .
Solution: 3
Let bn = an +1. Then the recurrence becomes bn+1 −1 = (b2n −bn−1 )/bn−1 = b2n /bn−1 −1,
so bn+1 = b2n /bn−1 . It follows that the sequence (bn ) is a geometric progression, from
which b25 = b1 b9 = 2 · 8 = 16 ⇒ b5 = ±4. However, since all bn are real, they either
alternate in sign or all have the same sign (depending on the sign of the progression’s
common ratio); either way, b5 has the same sign as b1 , so b5 = 4 ⇒ a5 = 3.
31. A cylinder of base radius 1 is cut into two equal parts along a plane passing through
the center of the cylinder and tangent to the two base circles. Suppose that each piece’s
surface area is m times its volume. Find the greatest lower bound for all possible values
of m as the height of the cylinder varies.
Solution: 3
Let h be the height of the cylinder. Then the volume of each piece is half the volume
of the cylinder, so it is 12 πh.
q
The base of the piece has area π, and the ellipse formed
2
by the cut has area π · 1 · 1 + h4 because its area is the product of the semiaxes times
π. The rest of the area of the piece is half the lateral area of the cylinder, so it is πh.
Thus, the value of m is
q √
π + π 1 + h2 /4 + πh 2 + 2h + 4 + h2
=
πh/2 h
s
2 4
= +2+ + 1,
h h2
a decreasing function of h whose limit as h → ∞ is 3. Therefore the greatest lower
bound of m is 3.
32. If x, y, and z are real numbers such that 2x2 + y 2 + z 2 = 2x − 4y + 2xz − 5, find the
maximum possible value of x − y + z.
Solution: 4
The equation rearranges as (x − 1)2 + (y + 2)2 + (x − z)2 = 0, so we must have x = 1,
y = −2, z = 1, giving us 4.

8
33. We are given triangle ABC, with AB = 9, AC = 10, and BC = 12, and a point D on
BC. B and C are reflected in AD to B 0 and C 0 , respectively. Suppose that lines BC 0
and B 0 C never meet (i.e., are parallel and distinct). Find BD.
Solution: 6
The lengths of AB and AC are irrelevant. Because the figure is symmetric about AD,
lines BC 0 and B 0 C meet if and only if they meet at a point on line AD. So, if they
never meet, they must be parallel to AD. Because AD and BC 0 are parallel, triangles
ABD and ADC 0 have the same area. Then ABD and ADC also have the same area.
Hence, BD and CD must have the same length, so BD = 21 BC = 6.

34. OKRA is a trapezoid with OK parallel to RA. If OK = 12 and RA is a positive


integer, how many integer values can be taken on by the length of the segment in the
trapezoid, parallel to OK, through the intersection of the diagonals?
Solution: 10
Let RA = x. If the diagonals intersect at X, and the segment is P Q with P on KR,
then 4P KX ∼ 4RKA and 4OKX ∼ 4RAX (by equal angles), giving RA/P X =
AK/XK = 1 + AX/XK = 1 + AR/OK = (x + 12)/12, so P X = 12x/(12 + x).
288
Similarly XQ = 12x/(12 + x) also, so P Q = 24x/(12 + x) = 24 − 12+x . This has to be
5 2
an integer. 288 = 2 3 , so it has (5 + 1)(3 + 1) = 18 divisors. 12 + x must be one of
these. We also exclude the 8 divisors that don’t exceed 12, so our final answer is 10.

35. A certain lottery has tickets labeled with the numbers 1, 2, 3, . . . , 1000. The lottery is
run as follows: First, a ticket is drawn at random. If the number on the ticket is odd,
the drawing ends; if it is even, another ticket is randomly drawn (without replacement).
If this new ticket has an odd number, the drawing ends; if it is even, another ticket
is randomly drawn (again without replacement), and so forth, until an odd number
is drawn. Then, every person whose ticket number was drawn (at any point in the
process) wins a prize.
You have ticket number 1000. What is the probability that you get a prize?
Solution: 1/501
Notice that the outcome is the same as if the lottery instead draws all the tickets, in
random order, and awards a prize to the holder of the odd ticket drawn earliest and each
even ticket drawn before it. Thus, the probability of your winning is the probability
that, in a random ordering of the tickets, your ticket precedes all the odd tickets. This,
in turn, equals the probability that your ticket is the first in the setconsisting of your
own and all odd-numbered tickets, irrespective of the other even-numbered tickets.
Since all 501! orderings of these tickets are equally likely, the desired probability is
1/501.

36. A teacher must divide 221 apples evenly among 403 students. What is the minimal
number of pieces into which she must cut the apples? (A whole uncut apple counts as
one piece.)
Solution: 611
Consider a bipartite graph, with 221 vertices representing the apples and 403 vertices
representing the students; each student is connected to each apple that she gets a

9
piece of. The number of pieces then equals the number of edges in the graph. Each
student gets a total of 221/403 = 17/31 apple, but each component of the graph
represents a complete distribution of an integer number of apples to an integer number
of students and therefore uses at least 17 apple vertices and 31 student vertices. Then
we have at most 221/17 = 403/31 = 13 components in the graph, so there are at
least 221 + 403 − 13 = 611 edges. On the other hand, if we simply distribute in the
straightforward manner — proceeding through the students, cutting up a new apple
whenever necessary but never returning to a previous apple or student — we can create
a graph without cycles, and each component does involve 17 apples and 31 students.
Thus, we get 13 trees, and 611 edges is attainable.
37. A quagga is an extinct chess piece whose move is like a knight’s, but much longer: it
can move 6 squares in any direction (up, down, left, or right) and then 5 squares in
a perpendicular direction. Find the number of ways to place 51 quaggas on an 8 × 8
chessboard in such a way that no quagga attacks another. (Since quaggas are naturally
belligerent creatures, a quagga is considered to attack quaggas on any squares it can
move to, as well as any other quaggas on the same square.)
Solution: 68
Represent the 64 squares of the board as vertices of a graph, and connect two vertices
by an edge if a quagga can move from one to the other. The resulting graph consists of
4 paths of length 5 and 4 paths of length 3 (given by the four rotations of the two paths
shown, next page), and 32 isolated vertices. Each path of length 5 can accommodate
at most 3 nonattacking quaggas in a unique way (the first, middle, and last vertices),
and each path of length 3 can accommodate at most 2 nonattacking quaggas in a
unique way; thus, the maximum total number of nonattacking quaggas we can have is
4 · 3 + 4 · 2 + 32 = 52. For 51 quaggas to fit, then, just one component of the graph
must contain one less quagga than its maximum.
³ ´
If this component is a path of length 5, there are 52 − 4 = 6 ways to place the two
quaggas on nonadjacent vertices, and then all the other locations are forced; the 4 such
paths then give us 4 · 6 = 24 possibilities this way. If it is a path of length 3, there are
3 ways to place one quagga, and the rest of the board is forced, so we have 4 · 3 = 12
possibilities here. Finally, if it is one of the 32 isolated vertices, we simply leave this
square empty, and the rest of the board is forced, so we have 32 possibilities here. So
the total is 24 + 12 + 32 = 68 different arrangements.
38. Given are real numbers x, y. For any pair of real numbers a0 , a1 , define a sequence by
an+2 = xan+1 + yan for n ≥ 0. Suppose that there exists a fixed nonnegative integer
m such that, for every choice of a0 and a1 , the numbers am , am+1 , am+3 , in this order,
form an arithmetic progression. Find all possible values of y.

Solution: 0, 1, (1 ± 5)/2
Note that x = 1 (or x = 0), y = 0 gives a constant sequence, so it will always have
the desired property. Thus, y = 0 is one possibility. For the rest of the proof, assume
y 6= 0.
We will prove that am and am+1 may take on any pair of values, for an appropriate
choice of a0 and a1 . Use induction on m. The case m = 0 is trivial. Suppose that
am and am+1 can take on any value. Let p and q be any real numbers. By setting

10
am = q−xp
y
(remembering that y 6= 0) and am+1 = p, we get am+1 = p and am+2 = q.
Therefore, am+1 and am+2 can have any values if am and am+1 can. That completes
the induction.
Now we determine the nonzero y such that am , am+1 , am+3 form an arithmetic sequence;
that is, such that am+3 − am+1 = am+1 − am . But because am+3 = (x2 + y)am+1 + xyam
by the recursion formula, we can eliminate am+3 from the equation, obtaining the
equivalent condition (x2 + y − 2)am+1 + (xy + 1)am = 0. Because the pair am , am+1
can take on any values, this condition means exactly that x2 + y − 2 = xy + 1 = 0.
Then x = −1/y, and 1/y 2 + y − 2 = 0, or y 3 − 2y 2 + 1 = 0. One root of√this cubic is
y = 1, and the remaining quadratic factor y 2 − y − 1 has the roots (1 ± 5)/2. Since
each such y gives an x for which√the condition holds, we conclude that the answer to
the problem is y = 0, 1, or (1 ± 5)/2.

39. In the figure, if AE = 3, CE = 1, BD = CD = 2, and AB = 5, find AG.


A

E
G
F
C

D
B


Solution: 3 66/7
By Stewart’s Theorem, AD2 · BC + CD · BD · BC = AB 2 · CD + AC 2 · BD, so
AD2 = (52 · 2 + 42 · 2 − 2 · 2 · 4)/4 = (50 + 32 − 16)/4 = 33/2. By Menelaus’s
Theorem applied to line BGE and triangle ACD, DG/GA · AE/EC√ · CB/BD = 1,
so DG/GA = 1/6 ⇒ AD/AG = 7/6. Thus AG = 6 · AD/7 = 3 66/7.

40. All the sequences consisting of five letters from the set {T, U, R, N, I, P } (with repeti-
tions allowed) are arranged in alphabetical order in a dictionary. Two sequences are
called “anagrams” of each other if one can be obtained by rearranging the letters of the

11
other. How many pairs of anagrams are there that have exactly 100 other sequences
between them in the dictionary?
Solution: 0
Convert each letter to a digit in base 6: I 7→ 0, N 7→ 1, P 7→ 2, R 7→ 3, T 7→ 4, U 7→ 5.
Then the dictionary simply consists of all base-6 integers from 000006 to 555556 in
numerical order. If one number can be obtained from another by a rearrangement of
digits, then the numbers are congruent modulo 5 (this holds because a number abcde6
= 64 · a + 63 · b + 62 · c + 6 · d + e is congruent modulo 5 to a + b + c + d + e), but if
there are 100 other numbers between them, then their difference is 101, which is not
divisible by 5. So there are no such pairs.

41. A hotel consists of a 2 × 8 square grid of rooms, each occupied by one guest. All the
guests are uncomfortable, so each guest would like to move to one of the adjoining
rooms (horizontally or vertically). Of course, they should do this simultaneously, in
such a way that each room will again have one guest. In how many different ways can
they collectively move?
Solution: 1156
Imagine that the rooms are colored black and white, checkerboard-style. Each guest in
a black room moves to an adjacent white room (and vice versa). If, for each such guest,
we place a domino over the original room and the new room, we obtain a covering of
the 2 × n grid by n dominoes, since each black square is used once and each white
square is used once. Applying a similar procedure to each guest who begins in a white
room and moves to a black room, we obtain a second domino tiling. Conversely, it is
readily verified that any pair of such tilings uniquely determines a movement pattern.
Also, it is easy to prove by induction that the number of domino tilings of a 2 × n
grid is the (n + 1)th Fibonacci number (this holds for the base cases n = 1, 2, and
for a 2 × n rectangle, the two rightmost squares either belong to one vertical domino,
leaving a 2 × (n − 1) rectangle to be covered arbitrarily, or to two horizontal dominoes
which also occupy the adjoining squares, leaving a 2 × (n − 2) rectangle to be covered
freely; hence, the numbers of tilings satisfy the Fibonacci recurrence). So the number
of domino tilings of a 2 × 8 grid is 34, and the number of pairs of such tilings is
342 = 1156, the answer.

42. A tightrope walker stands in the center of a rope of length 32 meters. Every minute she
walks forward one meter with probability 3/4 and backward one meter with probability
1/4. What is the probability that she reaches the end in front of her before the end
behind her?
Solution: 316 /(316 + 1)
After one minute, she is three times as likely to be one meter forward as one meter
back. After two minutes she is either in the same place, two meters forward, or two
meters back. The chance of being two meters forward is clearly (3/4)2 and thus 32 = 9
times greater than the chance of being two back ((1/4)2 ). We can group the minutes
into two-minute periods and ignore the periods of no net movement, so we can consider
her to be moving 2 meters forward or backward each period, where forward movement
is 32 times as likely as backward movement. Repeating the argument inductively, we
eventually find that she is 316 times more likely to move 16 meters forward than 16

12
meters backward, and thus the probability is 316 /(316 + 1) that she will meet the front
end of the rope first.
43. Write down an integer N between 0 and 10, inclusive. You will receive N points —
unless some other team writes down the same N , in which case you receive nothing.
Comments: Somewhat surprisingly, this game does not appear to be well-documented
in the literature. A better-known relative is Undercut, described by Douglas Hofstadter
in his book Metamagical Themas: Questing for the Essence of Mind and Pattern. In
this game, two players each choose an integer in a specified interval and receive that
many points — unless the two numbers differ by 1, in which case the lower player
receives a number of points equal to the sum of the two numbers. See, for example,
http://mathforum.org/mam/96/undercut/ .
44. A partition of a number n is a sequence of positive integers, arranged in descending
order, whose sum is n. For example, n = 4 has 5 partitions: 1 + 1 + 1 + 1 = 2 +
1 + 1 = 2 + 2 = 3 + 1 = 4. Given two different partitions of the same number,
n = a1 + a2 + · · · + ak = b1 + b2 + · · · + bl , where k ≤ l, the first partition is said to
dominate the second if all of the following inequalities hold:
a1 ≥ b1 ;
a1 + a2 ≥ b1 + b2 ;
a1 + a2 + a3 ≥ b1 + b2 + b3 ;
..
.
a1 + a2 + · · · + ak ≥ b1 + b2 + · · · + bk .
Find as many partitions of the number n = 20 as possible such that none of the
partitions dominates any other. Your score will be the number of partitions you find.
If you make a mistake and one of your partitions does dominate another, your score is
the largest m such that the first m partitions you list constitute a valid answer.
Comments: Computer searches have found that the maximum possible number of
partitions is 20, achieved e.g. by the following:

9+1+1+1+1+1+1+1+1+1+1+1 6+3+3+2+2+2+2
8+2+2+1+1+1+1+1+1+1+1 5+5+4+1+1+1+1+1+1
7+4+1+1+1+1+1+1+1+1+1 5+5+3+2+2+1+1+1
7+3+3+1+1+1+1+1+1+1 5+5+2+2+2+2+2
7+3+2+2+2+1+1+1+1 5+4+4+3+1+1+1+1
7+2+2+2+2+2+2+1 5+4+4+2+2+2+1
6+5+2+1+1+1+1+1+1+1 5+4+3+3+3+1+1
6+4+3+2+1+1+1+1+1 5+3+3+3+3+3
6+4+2+2+2+2+1+1 4+4+4+4+2+1+1
6+3+3+3+2+1+1+1 4+4+4+3+3+2

In general, as shown in

http://www-math.mit.edu/~ efedula/chains.ps
“Chain Lengths in the Dominance Lattice,” by Edward Early, 2002

13
the maximum√ number of mutually nondominating partitions of n is roughly on the
π 2n/3
order of e (in fact, the ratio of the total number of partitions to n to the maximum
number of nondominating partitions has polynomial order of magnitude), although a
proof by explicit construction is not known.

45. Find a set S of positive integers such that no two distinct subsets of S have the same
sum. Your score will be b20(2n /r − 2)c, where n is the number of elements in the set S,
and r is the largest element of S (assuming, of course, that this number is nonnegative).
Comments: The obvious such sets are those of the form {1, 2, 4, . . . , 2n−1 }, but these
achieve a score of 0. A bit of experimentation finds that the set {3, 5, 6, 7} also works;
this answer achieves 5 points. The set {6, 9, 11, 12, 13} achieves 9 points. Doing signif-
icantly better than this by hand computation becomes difficult. The reference

http://www.combinatorics.org/Volume 5/PDF/v5i1r3.pdf
“A Construction for Sets of Integers with Distinct Subset Sums,”
by Tom Bohman, 1997

(from whence the above examples) provides a construction that achieves a score of 50.
Erdős conjectured in effect that a maximum possible score exists, but this conjecture
remains open. The strongest known result in this direction
√ is that, if S has n elements,
n
the largest element is at least a constant times 2 / n (proven by Erdős and Moser in
1955, with an improvement on the constant by Elkies in 1986).

Hej då!

14
Harvard-MIT Mathematics Tournament
March 15, 2003

Team Round — Solutions

Completions and Configurations


Given a set A and a nonnegative integer k, the k-completion of A is the collection of all
k-element subsets of A, and a k-configuration of A is any subset of the k-completion of A
(including the empty set and the entire k-completion). For instance, the 2-completion of
A = {1, 2, 3} is {{1, 2}, {1, 3}, {2, 3}}, and the 2-configurations of A are
{} {{1, 2}}
{{1, 3}} {{2, 3}}
{{1, 2}, {1, 3}} {{1, 2}, {2, 3}}
{{1, 3}, {2, 3}} {{1, 2}, {1, 3}, {2, 3}}
The order of an element a of A with respect to a given k-configuration of A is the number
of subsets in the k-configuration that contain a. A k-configuration of a set A is consistent if
the order of every element of A is the same, and the order of a consistent k-configuration is
this common value.

1. (a) How many k-configurations are there of a set that has n elements?
³ ´
Solution: An n-element set has nk subsets of size k, and we can construct a k-
configuration by independently choosing, for each subset, whether or not to include it,
so there are 2(k ) k-configurations.
n

(b) How many k-configurations that have m elements are there of a set that has n
elements?
³ ´ ³ n ´
Solution: Again, an n-element set has n subsets of size k, so there are (k ) k-
k m
configurations with m elements.
2. Suppose A is a set with n elements, and k is a divisor of n. Find the number of
consistent k-configurations of A of order 1.
Solution: Given such a k-configuration, we can write out all the elements of one of
the k-element subsets, then all the elements of another subset, and so forth, eventually
obtaining an ordering of all n elements of A. Conversely, given any ordering of the
elements of A, we can construct a consistent k-configuration of order 1 from it by
grouping together the first k elements, then the next k elements, and so forth. In fact,
each consistent k-configuration of order 1 corresponds to (n/k)!(k!)n/k different such
orderings, since the elements of A within each of the n/k k-element subsets can be
ordered in k! ways, and the various subsets can also be ordered with respect to each
other in (n/k)! different ways. Thus, since there are n! orderings of the elements of A,
n!
we get (n/k)!(k!) n/k different consistent k-configurations of order 1.

3. (a) Let An = {a1 , a2 , a3 , . . . , an , b}, for n ≥ 3, and let Cn be the 2-configuration


consisting of {ai , ai+1 } for all 1 ≤ i ≤ n − 1, {a1 , an }, and {ai , b} for 1 ≤ i ≤ n.
Let Se (n) be the number of subsets of Cn that are consistent of order e. Find
Se (101) for e = 1, 2, and 3.

1
Solution: For convenience, we assume the ai are indexed modulo 101, so that ai+1 = a1
when ai = a101 .
In any consistent subset of C101 of order 1, b must be paired with exactly one ai , say
a1 . Then, a2 cannot be paired with a1 , so it must be paired with a3 , and likewise we find
we use the pairs {a4 , a5 }, {a6 , a7 }, . . . , {a100 , a101 } — and this does give us a consistent
subset of order 1. Similarly, pairing b with any other ai would give us a unique extension
to a consistent configuration of order 1. Thus, we have one such 2-configuration for each
i, giving S1 (101) = 101 altogether.
In a consistent subset of order 2, b must be paired with two other elements. Suppose
one of them is ai . Then ai is also paired with either ai−1 or ai+1 , say ai+1 . But then
ai−1 needs to be paired up with two other elements, and ai is not available, so it must
be paired with ai−2 and b. Now b has its two pairs determined, so nothing else can be
paired with b. Thus, for j 6= i − 1, i, we have that aj must be paired with aj−1 and
aj+1 . So our subset must be of the form

{{b, ai }, {ai , ai+1 }, {ai+1 , ai+2 }, . . . , {a101 , a1 }, . . . , {ai−2 , ai−1 }, {ai−1 , b}}

for some i. On the other hand, for any i = 1, . . . , 101, this gives a subset meeting our
requirements. So, we have 101 possibilities, and S2 (101) = 101.
Finally, in a consistent subset of order 3, each ai must be paired with ai−1 , ai+1 ,
and b. But then b occurs in 101 pairs, not just 3, so we have a contradiction. Thus, no
such subset exists, so S3 (101) = 0.
(b) Let A = {V, W, X, Y, Z, v, w, x, y, z}. Find the number of subsets of the 2-
configuration

{ {V, W }, {W, X}, {X, Y }, {Y, Z}, {Z, V }, {v, x}, {v, y}, {w, y}, {w, z}, {x, z},

{V, v}, {W, w}, {X, x}, {Y, y}, {Z, z} }


that are consistent of order 1.
Solution: No more than two of the pairs {v, x}, {v, y}, {w, y}, {w, z}, {x, z} may be
included in a 2-configuration of order 1, since otherwise at least one of v, w, x, y, z would
occur more than once. If exactly one is included, say {v, x}, then w, y, z must be paired
with W, Y, Z, respectively, and then V and X cannot be paired. So either none or
exactly two of the five pairs above must be used. If none, then v, w, x, y, z must be
paired with V, W, X, Y, Z, respectively, and we have 1 2-configuration arising in this
manner. If exactly two are used, we can check that there are 5 ways to do this without
duplicating an element:

{v, x}, {w, y} {v, x}, {w, z} {v, y}, {w, z} {v, y}, {x, z} {w, y}, {x, z}

In each case, it is straightforward to check that there is a unique way of pairing up the
remaining elements of A. So we get 5 2-configurations in this way, and the total is 6.
(c) Let A = {a1 , b1 , a2 , b2 , . . . , a10 , b10 }, and consider the 2-configuration C consisting
of {ai , bi } for all 1 ≤ i ≤ 10, {ai , ai+1 } for all 1 ≤ i ≤ 9, and {bi , bi+1 } for all
1 ≤ i ≤ 9. Find the number of subsets of C that are consistent of order 1.
Solution: Let An = {a1 , b1 , a2 , b2 , . . . , an , bn } for n ≥ 1, and consider the 2-configuration
Cn consisting of {ai , bi } for all 1 ≤ i ≤ n, {ai , ai+1 } for all 1 ≤ i ≤ n − 1, and {bi , bi+1 }

2
for all 1 ≤ i ≤ n − 1. Let Nn be the number of subsets of Cn that are consistent of order
1 (call these “matchings” of Cn ). Consider any matching of Cn+2 . Either an+2 is paired
with bn+2 , in which case the remaining elements of our matching form a matching of
Cn+1 ; or an+2 is paired with an+1 , in which case bn+2 must be paired with bn+1 , and
the remaining elements form a matching of Cn . It follows that Nn+2 = Nn+1 + Nn . By
direct calculation, N1 = 1 and N2 = 2, and now computing successive values of Nn
using the recurrence yields N10 = 89.

Define a k-configuration of A to be m-separable if we can label each element of A with an


integer from 1 to m (inclusive) so that there is no element E of the k-configuration all of
whose elements are assigned the same integer. If C is any subset of A, then C is m-separable
if we can assign an integer from 1 to m to each element of C so that there is no element E
of the k-configuration such that E ⊆ C and all elements of E are assigned the same integer.

4. (a) Suppose A has n elements, where n ≥ 2, and C is a 2-configuration of A that is


not m-separable for any m < n. What is (in terms of n) the smallest number of
elements that C can have?
Solution:³ We
´ claim that every pair of elements of A must belong to C, so that the
answer is n2 . Indeed, if a, b ∈ A and {a, b} is not in the 2-configuration, then we can
assign the other elements of A the numbers 1, 2, . . . , n − 2 and assign a and b both
the number n − 1, so that C is (n − 1)-separable. On the other hand, if every pair of
elements of A is in the configuration, then A cannot be m-separable for m < n, since
this would require assigning the same number to at least two elements, and then we
would have a pair whose elements have the same number.
(b) Show that every 3-configuration of an n-element set A is m-separable for every
integer m ≥ n/2.
Solution: We can successively label the elements of A with the numbers 1, 1, 2, 2,
3, 3, . . . , dn/2e. Then surely no 3-element subset can have all its elements labeled with
the same number, since no label is assigned to more than two elements. Thus, when
m ≥ n/2 ⇒ m ≥ dn/2e, this labeling shows that any 3-configuration is m-separable.
2
(c) Fix k ≥ 2, and suppose
³ 2 ´ A has k elements. Show that any k-configuration of A
with fewer than kk−1
−1
elements is k-separable.
Solution: The argument is similar to that used in problem 2. Suppose the config-
uration is not k-separable. Consider all possible orderings of the k 2 elements of A.
For each ordering, assign the first k elements the number 1, the next k elements the
number 2, and so forth. By assumption, for each such assignment, there exists some
element of the k-configuration whose elements are all assigned the same number. Now
consider any given element E of the k-configuration. For each i, we count the orderings
in which all k elements of E receive the number i: there are k! possible orderings for
the elements of E, and there are (k 2 − k)! possible orderings for the remaining elements
of A. Altogether, this gives k · k! · (k 2 − k)! orderings in which the elements of E all
receive the same label. So if, in each of the (k 2 )! orderings of the elements of A, there
is some E all of whose members receive the same label, then there must be at least
à !
(k 2 )! (k 2 − 1)! k2 − 1
= =
k · k! · (k 2 − k)! (k − 1)!(k 2 − k)! k−1

3
elements E of the k-configuration. Hence, if there are fewer elements, the k-configuration
is k-separable, as desired.
5. Let Bk (n) be the largest possible number of elements in a 2-separable k-configuration of
a set with 2n elements (2 ≤ k ≤ n). Find a closed-form expression (i.e. an expression
not involving any sums or products with a variable number of terms) for Bk (n).
³ ´ ³ ´ ³ ´
a 2n−a n
Solution: First, a lemma: For any a with 0 ≤ a ≤ 2n, k
+ k
≥ 2 k
. (By
³ ´
a
convention, we set k
= 0 when a < k.) Proof: We may assume a ≥ n, since otherwise
we can replace a with 2n − a. Now we prove ³the
´ result
³ ´by induction on a. For the base
case, if a = n, then the lemma states that 2 k ≥ 2 nk , which is trivial. If the lemma
n

holds for some a > 0, then by the familiar identity,


"Ã ! Ã !# "Ã ! Ã !#
a+1 2n − a − 1 a 2n − a
+ − +
k k k k
"Ã ! Ã !# "Ã ! Ã !#
a+1 a 2n − a 2n − a − 1
= − − +
k k k k
à ! à !
a 2n − a − 1
= − >0
k−1 k−1
³ ´ ³ ´ ³ ´ ³ ´ ³ ´
a+1 2n−a−1 a 2n−a n
(since a > 2n − a − 1), so k
+ k
> k
+ k
≥2 k
, giving the induction
step. The lemma follows.
Now suppose that the elements of A are labeled such that a elements of the set
A receive the number 1 and 2n − a elements receive the number 2. Then the k-
configuration can include all k-element subsets of A except those contained among the
a elements
³ ´ ³ ´ numbered
³ ´ 1 or the 2n − a elements numbered 2. Thus, we have at most
2n a 2n−a
k
− k − k elements in the k-configuration, and by the lemma, this is at most
à ! à !
2n n
−2 .
k k
³ ´ ³ ´
On the other hand, we can achieve 2n k
− 2 nk via the recipe above — take all the
k-element subsets of A, except those contained entirely within the first n elements
or entirely within the last n elements. Then, labeling the first n elements with the
number 1 and the last n elements
³ ´ ³with
´ the number 2 shows that the configuration is
2n n
2-separable. So, Bk (n) = k − 2 k .
6. Prove
q that any 2-configuration containing e elements is m-separable for some m ≤
1
2
+ 2e + 14 .
Solution: Suppose m is the minimum integer for which the given configuration C on
set A is m-separable, and fix a corresponding labeling of the elements of A. Let Ai be
the set of all elements with the label i. Then, for any i, j with 1 ≤ i < j ≤ m, there
must exist ai ∈ Ai , aj ∈ Aj with {ai , aj } ∈ C, since otherwise the elements of Aj could
have been reassigned the label i, decreasing the number of distinct
³ ´ labels necessary and
thus contradicting the minimality of m. We thus get at least m2 different elements of
³ ´
m
C. Therefore, e ≥ 2
= m(m − 1)/2 = [(m − 21 )2 − 41 ]/2, and solving for m gives the
desired result.

4
A cell of a 2-configuration of a set A is a nonempty subset C of A such that
i. for any two distinct elements a, b of C, there exists a sequence c0 , c1 , . . . , cn of elements
of A with c0 = a, cn = b, and such that {c0 , c1 }, {c1 , c2 }, . . . , {cn−1 , cn } are all elements
of the 2-configuration, and
ii. if a is an element of C and b is an element of A but not of C, there does NOT
exist a sequence c0 , c1 , . . . , cn of elements of A with c0 = a, cn = b, and such that
{c0 , c1 }, {c1 , c2 }, . . ., {cn−1 , cn } are all elements of the 2-configuration.
Also, we define a 2-configuration of A to be barren if there is no subset {a0 , a1 , . . . , an } of A,
with n ≥ 2, such that {a0 , a1 }, {a1 , a2 }, . . . , {an−1 , an } and {an , a0 } are all elements of the
2-configuration.

7. Show that, given any 2-configuration of a set A, every element of A belongs to exactly
one cell.
Solution: First, given a, let Ca be the set of all b ∈ A for which there exists a sequence
a = c0 , c1 , . . . , cn = b as in the definition of a cell. Certainly a ∈ Ca (take n = 0); we
claim that Ca is a cell. If b, b0 ∈ Ca , then there exist sequences a = c0 , c1 , . . . , cn = b
and a = c00 , c01 , . . . , c0m = b0 , so the sequence b = cn , cn−1 , . . . , c1 , c0 , c01 , . . . , c0m = b0 shows
that the first condition is met. For the second, suppose that there does exist a sequence
b = c0 , c1 , . . . , cn = b0 with b ∈ Ca , b0 ∈ / Ca . Then, concatenating with our sequence
from a to b, we get a sequence from a to b0 , contradicting the assumption b0 ∈ / Ca .
Thus, the second condition holds, and Ca is a cell. So a lies in at least one cell.
But now, note that if C is a cell containing a, then all b for which such a sequence
from a to b exists must lie in C (or the second condition is violated), and if no such
sequence exists, then b cannot lie in C (or the first condition is violated). Thus, the
elements of C are uniquely determined, so there is exactly one cell containing a, and
the proof is complete.

8. (a) Given a set A with n ≥ 1 elements, find the number of consistent 2-configurations
of A of order 1 with exactly 1 cell.
Solution: There must be some pair {a, b} in the 2-configuration, since each element
a ∈ A must belong to one pair. Since neither a nor b can now belong to any other pair,
this must be the entire cell. Thus, there is 1 such 2-configuration when n = 2, and
there are none when n 6= 2.
(b) Given a set A with 10 elements, find the number of consistent 2-configurations of
A of order 2 with exactly 1 cell.
Solution: Consider such a configuration; let {a1 , a2 } be an element of it. Then a2 be-
longs to exactly one other pair; call it {a2 , a3 }. Likewise, a3 belongs to exactly one other
pair {a3 , a4 }, and so forth; since we have finitely many elements, we must eventually
reach some pair {am , ak } that revisits a previously used element (m > k). But this is
only possible if k = 1, since each other ak with k < m is already used in two pairs. Now,
{a1 , . . . , am } constitutes a complete cell, because none of these elements can be used
in any more pairs, so m = 10. Thus, every consistent 2-configuration of order 2 with
exactly 1 cell gives rise to a permutation a1 , . . . , a10 of the elements of A, and conversely,
each such permutation gives us a 2-configuration {{a1 , a2 }, . . . , {a9 , a10 }, {a10 , a1 }}. In
fact, each configuration corresponds to exactly 20 permutations, depending which of the

5
10 elements of the 2-configuration we choose as {a1 , a2 } and which of these two elements
of A we in turn choose to designate as a1 (as opposed to a2 ). Therefore, the number of
such 2-configurations is 10!/20 = 9!/2 = 181440. (More generally, the same argument
shows that, when A has n ≥ 3 elements, there are (n − 1)!/2 such 2-configurations.)
(c) Given a set A with 10 elements, find the number of consistent 2-configurations of
order 2 with exactly 2 cells.
Solution: Notice that if we look only at the pairs contained within any fixed cell, each
element of that cell still lies in 2 such pairs, since all the pairs it belongs to are contained
within that cell. Thus we have an induced consistent 2-configuration of order 2 of each
cell.
Now, each cell must have at least 3 elements for the configuration to be 2-consistent.
So we can have either two 5-element cells, a 4-element cell and a 6-element cell, or a
3-element cell and a 7-element ³cell. ´ If there are two 5-element cells, we can choose
10
the members of the first cell in 5 ways, and then (by the reasoning in the previous
problem) we have 4!/2 ways to build a consistent 2-configuration of order 2 of each
cell. However, choosing 5 elements for the first cell is equivalent to choosing the other
5 elements for the first cell, since the two ³ ´cells are indistinguishable; thus, we have
overcounted by a factor of 2. So we have 10 5
· (4!/2)2 /2 = 252 · 144/2 = 18144 ways to
form our configuration if we require it to have two cells of 5 elements each. ³ ´
If we have one 4-element cell and one 6-element cell, then there are 10 4
ways
to determine which 4 elements go in the smaller cell, and then 3!/2 ways and 5!/2
ways,
³ ´ respectively, to construct the 2-configurations of the two cells, for a total of
10
4
· (3!/2) · (5!/2) = 210 · 3 · 60 = 37800 configurations (no overcounting here), and
³ ´
by similar reasoning, we have 10
3
· (2!/2) · (6!/2) = 120 · 1 · 360 = 43200 configurations
with one 3-element cell and one 7-element cell. Thus, altogether, we have a total of
18144 + 37800 + 43200 = 99144 consistent 2-configurations of order 2 with exactly 2
cells.

9. (a) Show that if every cell of a 2-configuration of a finite set A is m-separable, then
the whole 2-configuration is m-separable.
Solution: Let C be a 2-configuration of A with cells A1 , . . . , An , so that there is
no element of C with one element in Ai and another in Aj for i 6= j. Suppose that
each cell is m-separable, so that for each i, 1 ≤ i ≤ n, there is a labeling function
fi : Ai → {1, . . . , m} such that no two elements in the same pair of C are assigned the
same number. Then, by combining, we get a function f on all of A whose restriction
to Ai is fi for each i. By the definition of fi , within each Ai there is no element of C
both of whose elements are mapped to the same integer; and as above, we know that
there are no elements of C not contained inside any Ai . Thus, C is m-separable, by the
existence of f .
(b) Show that any barren 2-configuration of a finite set A is 2-separable.
Solution: It is sufficient to show each cell is 2-separable, by part (a). A barren 2-
configuration by definition cannot have any cycles (i.e. subsets {a0 , . . . , an }, n ≥ 2,
where each {ai , ai+1 } and {an , a0 } all belong to the 2-configuration). For any two
distinct elements a, b of A in the same cell of a 2-configuration C, define the distance
between them to be the smallest n such that there exists a sequence a = a0 , a1 , . . . , an =

6
b with {a0 , a1 }, {a1 , a2 }, . . . , {an−1 , an } all belonging to the 2-configuration. Notice that
the terms of this sequence are all distinct: if ai = aj for i < j, then we have the shorter
sequence a0 , a1 , . . . , ai , aj+1 , . . . , an , contradicting minimality.
Now let C be a barren 2-configuration of A. Pick any element a of A; label it and
all elements at even distance from it with the integer 1, and label all elements at odd
distance from it with the integer 2. We claim no two different elements with the same
label appear in the same element of C. Otherwise, let b and c be such elements, and
let a = a0 , a1 , . . . , an = b and a = a00 , a01 , . . . , a0m = c be the corresponding minimal
sequences. Consider the largest p such that ap ∈ {a00 , . . . , a0m }; write ap = a0q . We claim
the set {an , an−1 , . . . , ap , a0q+1 , . . . , a0m } is then a cycle. It is straightforward to check
that all its elements are distinct; the only issue is whether it has at least 3 elements. If
not, we would have ap = an or a0m . Assume that ap = an ⇒ p = n ⇒ q = m − 1 (by
minimality of our sequence (a0i )), but this means that m = n + 1, so the distances of b
and c from a have opposite parities, contradicting the assumption that they have the
same label. The case a0q = a0m is similar. Thus, our set really does have at least three
elements, and it is a cycle. But since A is barren, it contains no cycles, and we have a
contradiction.
Thus, after all, no two elements with the same label appear in the same pair of C,
so the cell containing a is 2-separable, and we are done.

10. Show that every consistent 2-configuration of order 4 on a finite set A has a subset
that is a consistent 2-configuration of order 2.
Solution: First, assume the 2-configuration has just one cell. We claim there exists a
sequence a0 , a1 , . . . , an of elements of A (not necessarily all distinct) such that the list

{a0 , a1 }, {a1 , a2 }, . . . , {an−1 , an }, {an , a0 }

contains each element of the 2-configuration exactly once. To see this, consider the
longest sequence such that {a0 , a1 }, . . . , {an−1 , an }, {an , a0 } are all distinct elements of
the 2-configuration. (We may take n = 0 if necessary. Note that the finiteness condition
ensures such a maximal sequence exists.) Each element of A occurs an even number
of times among these pairs (since each occurrence in the sequence contributes to two
pairs). If every element occurs 4 times or 0 times, then the elements occurring in the
sequence form a cell, since they cannot occur in any other pairs in the 2-configuration.
Hence, they are all of A, and our sequence uses all the pairs in the 2-configuration,
so the claim follows. Otherwise, there is some element ai occurring exactly twice.
Choose b1 so that {ai , b1 } is one of the two pairs in the 2-configuration not used by our
sequence. Then choose b2 so that {b1 , b2 } be another pair not used thus far. Continue
in this manner, choosing new elements bk with {bk , bk+1 } a pair not already used, until
we reach a point where finding another unused pair is impossible. Now, our pairs so
far are
{a0 , a1 }, . . . , {an−1 , an }, {an , a0 },
{ai , b1 }, {b1 , b2 }, . . . , {bk−1 , bk }.
Every element is used in an even number of these pairs, except possibly ai , which is
used in three pairs, and bk , which is used in an odd number of pairs (so one or three) —
unless ai = bk , in which case this element occurs four times. But since it is impossible
to continue the sequence, bk must indeed have been used four times, so bk = ai .

7
But now we can construct the following sequence of distinct elements of the 2-configuration:

{a0 , a1 }, . . . , {ai−1 , ai }, {ai , b1 }, {b1 , b2 }, . . . , {bk−1 , ai },

{ai , ai+1 }, . . . , {an−1 , an }, {an , a0 }.


This contradicts the maximality of our original sequence. This contradiction means
that our original sequence must have used all the pairs in the 2-configuration, after all.
So we can express the 2-configuration via such a sequence of pairs, where each pair’s
second element equals the first element of the next pair. If A has n elements, then
(since each element appears in four pairs) we have 2n pairs. So we can choose the 1st,
3rd, 5th, . . . , (2n − 1)th pairs, and then each element of A belongs to just two of these
pairs, because each occurrence of the element as an ai contributes to two consecutive
pairs from our original sequence (or the first and last such pairs). Thus, we have our
consistent 2-configuration of order 2, as desired.
Finally, if A consists of more than one cell, then the pairs within any given cell form
a consistent 2-configuration of order 4 on that cell. So we simply apply the above
procedure to obtain a consistent 2-configuration of order 2 on each cell, and then
combining these gives a consistent 2-configuration of order 2 on A, as desired.

Comments: A note for those who might have found these problems rather foreign
— the objects described here are actually of considerable importance; they constitute the
elements of graph theory, one of the major research areas of modern mathematics. What
we have called a “2-configuration” is generally called a graph, and what we have called a
“k-configuration” (k > 2) is generally called a hypergraph. The graph in problem 3b is the
Petersen graph, a ubiquitous counterexample in graph theory. A consistent 2-configuration
of order n is an n-regular graph; a cell is a component; a barren 2-configuration is a forest
(and a forest with one component is a tree); and an m-separable configuration is m-colorable
(and the minimum m for which a graph is m-colorable is its chromatic number).

8
Harvard-MIT Mathematics Tournament
February 28, 2004

Individual Round: Algebra Subject Test — Solutions

1. How many ordered pairs of integers (a,b) satisfy all of the following inequalities?
a2 + b2 < 16
a2 + b2 < 8a
a2 + b2 < 8b

Solution: 6
This is easiest to see by simply graphing the inequalities. They correspond to the
(strict) interiors of circles of radius 4 and centers at (0, 0), (4, 0), (0, 4), respectively. So
we can see that there are 6 lattice points in their intersection (circled in the figure).

2. Find the largest number n such that (2004!)! is divisible by ((n!)!)!.


Solution: 6
For positive integers a, b, we have
a! | b! ⇔ a! ≤ b! ⇔ a ≤ b.
Thus,
((n!)!)! | (2004!)! ⇔ (n!)! ≤ 2004! ⇔ n! ≤ 2004 ⇔ n ≤ 6.

3. Compute: $ %
20053 20033
− .
2003 · 2004 2004 · 2005

Solution: 8
Let x = 2004. Then the expression inside the floor brackets is
(x + 1)3 (x − 1)3 (x + 1)4 − (x − 1)4 8x3 + 8x 16x
− = = 3 =8+ 3 .
(x − 1)x x(x + 1) (x − 1)x(x + 1) x −x x −x
Since x is certainly large enough that 0 < 16x/(x3 − x) < 1, the answer is 8.

1
4. Evaluate the sum
1 1 1 1
√ + √ + √ + ··· + √ .
2b 1c + 1 2b 2c + 1 2b 3c + 1 2b 100c + 1

Solution: 190/21
The first three terms all equal 1/3, thenj√theknext five all equal
j√ 1/5; kmore generally,
for each a = 1, 2, . . . , 9, the terms 1/(2 a + 1) to 1/(2 a2 + 2a + 1) all equal
2

1/(2a + 1), and there are 2a + 1 such terms. Thus ourjtermskcan be arranged into

9 groups, each with sum 1, and only the last term 1/(2 100 + 1) remains, so the
answer is 9 + 1/21 = 190/21.

5. There exists a positive real number x such that cos(tan−1 (x)) = x. Find the value of
x2 .

Solution: (−1 + 5)/2
with legs 1, x; then the angle θ opposite x is tan√−1 x, and we can
Draw a right triangle √
compute cos(θ)√= 1/ x2 + 1. Thus, we only need to solve x = 1/ x2 + 1. This is
equivalent to x x2 + 1 = 1. Square both sides to get x4 + √ x2 = 1 ⇒ x4 + x2 − 1 = 0.
Use the quadratic formula to get the solution x2 = (−1 + 5)/2 (unique since x2 must
be positive).

6. Find all real solutions to x4 + (2 − x)4 = 34.



Solution: 1 ± 2
Let y = 2 − x, so x + y = 2 and x4 + y 4 = 34. We know

(x + y)4 = x4 + 4x3 y + 6x2 y 2 + 4xy 3 + y 4 = x4 + y 4 + 2xy(2x2 + 2y 2 + 3xy).

Moreover, x2 +y 2 = (x+y)2 −2xy, so the preceding equation becomes 24 = 34+2xy(2·


22 −xy), or (xy)2 −8xy−9 = 0. Hence xy = 9 or −1. Solving xy = 9, x + y =√2 produces

complex√solutions,
√ and solving xy =√−1, x + y = 2 produces (x, y) = (1 + 2, 1 − 2)
or (1 − 2, 1 + 2). Thus, x = 1 ± 2.

7. If x, y, k are positive reals such that


à ! à !
x2 y 2 x y
3 = k2 + +k + ,
y 2 x2 y x

find the maximum possible value of k.



Solution: (−1 + 7)/2
We have 3 = k 2 (x2 /y√
2
+ y 2 /x2 ) + k(x/y + y/x) ≥ 2k 2 + 2k, hence 7 ≥ 4k 2 + 4k + 1 =
(2k +1)2 , hence k ≤ ( 7−1)/2. Obviously k can assume this value, if we let x = y = 1.

8. Let x be a real number such that x3 + 4x = 8. Determine the value of x7 + 64x2 .


Solution: 128

2
For any integer n ≥ 0, the given implies xn+3 = −4xn+1 + 8xn , so we can rewrite any
such power of x in terms of lower powers. Carrying out this process iteratively gives
x7 = −4x5 + 8x4
= 8x4 + 16x3 − 32x2
= 16x3 − 64x2 + 64x
= −64x2 + 128.
Thus, our answer is 128.
9. A sequence of positive integers is defined by a0 = 1 and an+1 = a2n + 1 for each n ≥ 0.
Find gcd(a999 , a2004 ).
Solution: 677
If d is the relevant greatest common divisor, then a1000 = a2999 + 1 ≡ 1 = a0 (mod d),
which implies (by induction) that the sequence is periodic modulo d, with period 1000.
In particular, a4 ≡ a2004 ≡ 0. So d must divide a4 . Conversely, we can see that
a5 = a24 + 1 ≡ 1 = a0 modulo a4 , so (again by induction) the sequence is periodic
modulo a4 with period 5, and hence a999 , a2004 are indeed both divisible by a4 . So the
answer is a4 , which we can compute directly; it is 677.
10. There exists a polynomial P of degree 5 with the following property: if z is a complex
number such that z 5 +2004z = 1, then P (z 2 ) = 0. Calculate the quotient P (1)/P (−1).
Solution: −2010012/2010013
Let z1 , . . . , z5 be the roots of Q(z) = z 5 +2004z −1. We can check these are distinct (by
using the fact that there’s one in a small neighborhood of each root of z 5 + 2004z, or by
noting that Q(z) is relatively prime to its derivative). And certainly none of the roots
of Q is the negative of another, since z 5 + 2004z = 1 implies (−z)5 + 2004(−z) = −1,
so their squares are distinct as well. Then, z12 , . . . , z52 are the roots of P , so if we write
C for the leading coefficient of P , we have
P (1) C(1 − z12 ) · · · (1 − z52 )
=
P (−1) C(−1 − z12 ) · · · (−1 − z52 )
[(1 − z1 ) · · · (1 − z5 )] · [(1 + z1 ) · · · (1 + z5 )]
=
[(i − z1 ) · · · (i − z5 )] · [(i + z1 ) · · · (i + z5 )]
[(1 − z1 ) · · · (1 − z5 )] · [(−1 − z1 ) · · · (−1 − z5 )]
=
[(i − z1 ) · · · (i − z5 )] · [(−i − z1 ) · · · (−i − z5 )]
(15 + 2004 · 1 − 1)(−15 + 2004 · (−1) − 1)
=
(i5 + 2004 · i − 1)(−i5 + 2004 · (−i) − 1)
(2004)(−2006)
=
(−1 + 2005i)(−1 − 2005i)
20052 − 1
= −
20052 + 1
= −4020024/4020026 = −2010012/2010013.

Alternative Solution: In fact, we can construct the polynomial P explicitly (up to


multiplication by a constant). We write P (z 2 ) as a polynomial in z; it must use only

3
even powers of z and be divisible by z 5 +2004z −1, so we are inspired to try a difference
of squares,

P (z 2 ) = (z 5 + 2004z − 1)(z 5 + 2004z + 1) = (z 5 + 2004z)2 − 12 = z 2 (z 4 + 2004)2 − 1,

giving
P (z) = z(z 2 + 2004)2 − 1.
Now plugging in z = 1 and z = −1 rapidly gives (20052 − 1)/(−20052 − 1) as before.

4
Harvard-MIT Mathematics Tournament
February 28, 2004

Individual Round: Calculus Subject Test — Solutions


f (x+h)−f (h)
1. Let f (x) = sin(sin x). Evaluate limh→0 x
at x = π.
Solution: 0
The expression f (x+h)−fx
(h)
is continuous at h = 0, so the limit is just f (x)−f (0)
x
. Letting
sin(sin π)−sin(sin 0)
x = π yields π
= 0.

2. Suppose the function f (x) − f (2x) has derivative 5 at x = 1 and derivative 7 at x = 2.


Find the derivative of f (x) − f (4x) at x = 1.
Solution: 19
Let g(x) = f (x) − f (2x). Then we want the derivative of

f (x) − f (4x) = (f (x) − f (2x)) + (f (2x) − f (4x)) = g(x) + g(2x)

at x = 1. This is g 0 (x) + 2g 0 (2x) at x = 1, or 5 + 2 · 7 = 19.


√ √
3. Find limx→∞ ( 3 x3 + x2 − 3 x3 − x2 ).
Solution: 2/3
Observe that
h √
3
i x/3 + 1/27
lim (x + 1/3) − x3 + x2 = lim √ √ ,
x→∞ x→∞ ( 3 x3 + x2 )2 + ( 3 x3 + x2 )(x + 1/3) + (x + 1/3)2

by factoring the numerator as a difference of cubes. The numerator is linear in x, while


the denominator is at√ least 3x2 , so the limit as x → ∞ is 0. By similar arguments,
3
limx→∞ [(x − 1/3) − x3 − x2 ] = 0. So, the desired limit equals
√3
√3
2/3 + lim [(x − 1/3) − x3 − x2 ] − lim [(x + 1/3) − x3 + x2 ] = 2/3.
x→∞ x→∞

4. Let f (x) = cos(cos(cos(cos(cos(cos(cos(cos x))))))), and suppose that the number a


satisfies the equation a = cos a. Express f 0 (a) as a polynomial in a.
Solution: a8 − 4a6 + 6a4 − 4a2 + 1
This is an exercise using the chain rule. Define f0 (x) = x and fn (x) = cos fn−1 (x) for
n ≥ 0. We will show by induction that fn (a) = a and fn0 (a) = (− sin a)n for all n. The
case n = 0 is clear. Then fn (a) = cos fn−1 (a) = cos a = a, and
0
fn0 (a) = fn−1 (a) · (− sin fn−1 (a)) = (− sin a)n−1 · (− sin a) = (− sin a)n

by induction. Now, f (x) = f8 (x), so f 0 (a) = (− sin a)8 = sin8 a. But sin2 a = 1 −
cos2 a = 1 − a2 , so f 0 (a) = (1 − a2 )4 = a8 − 4a6 + 6a4 − 4a2 + 1.

1
5. A mouse is sitting in a toy car on a negligibly small turntable. The car cannot turn on
its own, but the mouse can control when the car is launched and when the car stops
(the car has brakes). When the mouse chooses to launch, the car will immediately
leave the turntable on a straight trajectory at 1 meter per second.
Suddenly someone turns on the turntable; it spins at 30 rpm. Consider the set S of
points the mouse can reach in his car within 1 second after the turntable is set in
motion. (For example, the arrows in the figure below represent two possible paths the
mouse can take.) What is the area of S, in square meters?
.5 m

30 rpm

1m

Solution: π/6
The mouse can wait while the table rotates through some angle θ and then spend the
remainder of the time moving along that ray at 1 m/s. He can reach any point between
the starting point and the furthest reachable point along the ray, (1 − θ/π) meters out.
So the area is given by the polar integral
Z π
(1 − θ/π)2 1 1 Zπ 2
dθ = · 2 φ dφ = π/6
0 2 2 π 0
(where we have used the change of variables φ = π − θ).

6. For x > 0, let f (x) = xx . Find all values of x for which f (x) = f 0 (x).
Solution: 1
Let g(x) = log f (x) = x log x. Then f 0 (x)/f (x) = g 0 (x) = 1 + log x. Therefore
f (x) = f 0 (x) when 1 + log x = 1, that is, when x = 1.

7. Find the area of the region in the xy-plane satisfying x6 − x2 + y 2 ≤ 0.


Solution: π/2

Rewrite the condition as |y| ≤ x2 − x6 . The right side is zero when x is −1, 0, or 1,
and it bounds an area symmetric about the x- and y-axes. Therefore, we can calculate
the area by the integral
Z 1 √ Z 1 √ Z 1√
2 x2 − x6 dx = 4 x 1− x4 dx = 2 1 − u2 du = π/2.
−1 0 0

8. If x and y are real numbers with (x + y)4 = x − y, what is the maximum possible value
of y?

Solution: 3 3 2/16

2
By drawing the graph of the curve (as shown), which is just a 135◦ clockwise rotation
and scaling of y = x4 , we see that the maximum is achieved at the unique point where
dy/dx = 0. Implicit differentiation gives 4(dy/dx
√ +√1)(x + y)3 = 1 − dy/dx, so setting

3 4
dy/dx = 0 gives 4(x + y) = 1. So x + y = √ 1/ 4 = √3 2/2, and x −
3
√ y = (x + y) = 3
2/8.
Subtracting and dividing by 2 gives y = ( 3 2/2 − 3 2/8)/2 = 3 3 2/16.

0.5 1 1.5

-0.5

-1

-1.5

9. Find the positive constant c0 such that the series



X n!
n
n=0 (cn)

converges for c > c0 and diverges for 0 < c < c0 .


Solution: 1/e
The ratio test tells us that the series converges if
µ ¶n
(n + 1)!/(c(n + 1))n+1 1 n
lim n
= · lim
n→∞ n!/(cn) c n→∞ n + 1
is less than one and diverges if it is greater than one. But
µ ¶n µ ¶−n
n 1 1
lim = n→∞
lim 1+ = .
n→∞ n+1 n e
Then the limit above is just 1/ce, so the series converges for c > 1/e and diverges for
0 < c < 1/e.
10. Let P (x) = x3 − 23 x2 + x + 14 . Let P [1] (x) = P (x), and for n ≥ 1, let P [n+1] (x) =
R
P [n] (P (x)). Evaluate 01 P [2004] (x) dx.
Solution: 1/2
By Note that P (1 − x) = 1 − P (x). It follows easily by induction that P [k] (1 − x) =
1 − P [k] (x) for all positive integers k. Hence
Z 1 Z 1
P [2004] (x) dx = 1 − P [2004] (1 − x) dx
0 0
Z 1
= 1− P [2004] (1 − x) dx
0
Z 1
= 1− P [2004] (u) du (u = 1 − x).
0

R1
Hence 0 P [2004] (x) dx = 1/2.

3
Harvard-MIT Mathematics Tournament
February 28, 2004

Individual Round: Combinatorics Subject Test — Solutions

1. There are 1000 rooms in a row along a long corridor. Initially the first room contains
1000 people and the remaining rooms are empty. Each minute, the following happens:
for each room containing more than one person, someone in that room decides it is
too crowded and moves to the next room. All these movements are simultaneous (so
nobody moves more than once within a minute). After one hour, how many different
rooms will have people in them?
Solution: 31
We can prove by induction on n that the following pattern holds for 0 ≤ n ≤ 499:
after 2n minutes, the first room contains 1000 − 2n people and the next n rooms each
contain 2 people, and after 2n + 1 minutes, the first room contains 1000 − (2n + 1)
people, the next n rooms each contain 2 people, and the next room after that contains
1 person. So, after 60 minutes, we have one room with 940 people and 30 rooms with
2 people each.

2. How many ways can you mark 8 squares of an 8 × 8 chessboard so that no two marked
squares are in the same row or column, and none of the four corner squares is marked?
(Rotations and reflections are considered different.)
Solution: 21600
In the top row, you can mark any of the 6 squares that is not a corner. In the
bottom row, you can then mark any of the 5 squares that is not a corner and not
in the same column as the square just marked. Then, in the second row, you have
6 choices for a square not in the same column as either of the two squares already
marked; then there are 5 choices remaining for the third row, and so on down to 1
for the seventh row, in which you make the last mark. Thus, altogether, there are
6 · 5 · (6 · 5 · · · 1) = 30 · 6! = 30 · 720 = 21600 possible sets of squares.

3. A class of 10 students took a math test. Each problem was solved by exactly 7 of the
students. If the first nine students each solved 4 problems, how many problems did
the tenth student solve?
Solution: 6
Suppose the last student solved n problems, and the total number of problems on
the test was p. Then the total number of correct solutions written was 7p (seven per
problem), and also equal to 36 + n (the sum of the students’ scores), so p = (36 + n)/7.
The smallest n ≥ 0 for which this is an integer is n = 6. But we also must have n ≤ p,
so 7n ≤ 36 + n, and solving gives n ≤ 6. Thus n = 6 is the answer.

4. Andrea flips a fair coin repeatedly, continuing until she either flips two heads in a row
(the sequence HH) or flips tails followed by heads (the sequence T H). What is the
probability that she will stop after flipping HH?
Solution: 1/4

1
The only way that Andrea can ever flip HH is if she never flips T , in which case she
must flip two heads immediately at the beginning. This happens with probability 14 .

5. A best-of-9 series is to be played between two teams; that is, the first team to win 5
games is the winner. The Mathletes have a chance of 2/3 of winning any given game.
What is the probability that exactly 7 games will need to be played to determine a
winner?
Solution: 20/81
If the Mathletes are to win, they must win exactly 5 out of the 7 games. One of
the 5 games they win must be the 7th game, because otherwise they would win the
tournament before 7 games are completed. Thus, in the first 6 games, the Mathletes
must win 4 games and lose 2. The probability of this happening and the Mathletes
winning the last game is
"µ ¶ µ ¶ µ ¶ # µ ¶
4 2
6 2 1 2
· · · .
2 3 3 3

Likewise, the probability of the other team winning on the 7th game is
"µ ¶ µ ¶ µ ¶ # µ ¶
4 2
6 1 2 1
· · · .
2 3 3 3

Summing these values, we obtain 160/729 + 20/729 = 20/81.

6. A committee of 5 is to be chosen from a group of 9 people. How many ways can it be


chosen, if Bill and Karl must serve together or not at all, and Alice and Jane refuse to
serve with each other?
Solution: 41
¡¢
If Bill and Karl are on the committee, there are 73 = 35 ways for the other group
members to be ¡chosen.
¢ However, if Alice and Jane are on the committee with Bill and
Karl, there are 51 = 5 ways for the last member to be chosen, yielding
¡7¢ 5 unacceptable
committees. If Bill and Karl are not on the committee, there are 5 = 21 ways for
the 5 members to¡5¢be chosen, but again if Alice and Jane were to be on the committee,
there would be 3 = 10 ways to choose the other three members, yielding 10 more
unacceptable committees. So, we obtain (35 − 5) + (21 − 10) = 41 ways the committee
can be chosen.

7. We have a polyhedron such that an ant can walk from one vertex to another, traveling
only along edges, and traversing every edge exactly once. What is the smallest possible
total number of vertices, edges, and faces of this polyhedron?
Solution: 20
This is obtainable by construction. Consider two tetrahedrons glued along a face; this
gives us 5 vertices, 9 edges, and 6 faces, for a total of 20, and one readily checks that
the required Eulerian path exists. Now, to see that we cannot do better, first notice
that the number v of vertices is at least 5, since otherwise we must have a tetrahedron,
which does not have an Eulerian path. Each vertex is incident to at least 3 edges, and

2
in fact, since there is an Eulerian path, all except possibly two vertices are incident to
an even number of edges. So the number of edges is at least (3 + 3 + 4 + 4 + 4)/2 (since
each edge meets two vertices) = 9. Finally, if f = 4 then each face must be a triangle,
because there are only 3 other faces for it to share edges with, and we are again in the
case of a tetrahedron, which is impossible; therefore f ≥ 5. So f +v+e ≥ 5+5+9 = 19.
But since f +v −e = 2−2g (where g is the number of holes in the polyhedron), f +v +e
must be even. This strengthens our bound to 20 as needed.

8. Urn A contains 4 white balls and 2 red balls. Urn B contains 3 red balls and 3 black
balls. An urn is randomly selected, and then a ball inside of that urn is removed. We
then repeat the process of selecting an urn and drawing out a ball, without returning
the first ball. What is the probability that the first ball drawn was red, given that the
second ball drawn was black?
Solution: 7/15
This is a case of conditional probability; the answer is the probability that the first
ball is red and the second ball is black, divided by the probability that the second ball
is black.
First, we compute the numerator. If the first ball is drawn from Urn A, we have a
probability of 2/6 of getting a red ball, then a probability of 1/2 of drawing the second
ball from Urn B, and a further probability of 3/6 of drawing a black ball. If the first
ball is drawn from Urn B, we have probability 3/6 of getting a red ball, then 1/2 of
drawing the second ball from Urn B, and 3/5 of getting a black ball. So our numerator
is µ ¶
1 2 1 3 3 1 3 7
· · + · · = .
2 6 2 6 6 2 5 60
We similarly compute the denominator: if the first ball is drawn from Urn A, we have
a probability of 1/2 of drawing the second ball from Urn B, and 3/6 of drawing a
black ball. If the first ball is drawn from Urn B, then we have probability 3/6 that
it is red, in which case the second ball will be black with probability (1/2) · (3/5),
and probability 3/6 that the first ball is black, in which case the second is black with
probability (1/2) · (2/5). So overall, our denominator is
µ · ¸¶
1 1 3 3 1 3 1 2 1
· + · + · = .
2 2 6 6 2 5 2 5 4

Thus, the desired conditional probability is (7/60) / (1/4) = 7/15.

9. A classroom consists of a 5 × 5 array of desks, to be filled by anywhere from 0 to 25


students, inclusive. No student will sit at a desk unless either all other desks in its row
or all others in its column are filled (or both). Considering only the set of desks that are
occupied (and not which student sits at each desk), how many possible arrangements
are there?
Solution: 962
The set of empty desks must be of the form (non-full rows) × (non-full columns): each
empty desk is in a non-full column and a non-full row, and the given condition implies
that each desk in such a position is empty. So if there are fewer than 25 students, then

3
both of these sets are nonempty; we have 25 − 1 = 31 possible sets of non-full rows, and
31 sets of non-full columns, for 961 possible arrangements. Alternatively, there may be
25 students, and then only 1 arrangement is possible. Thus there are 962 possibilities
altogether.

10. In a game similar to three card monte, the dealer places three cards on the table: the
queen of spades and two red cards. The cards are placed in a row, and the queen
starts in the center; the card configuration is thus RQR. The dealer proceeds to move.
With each move, the dealer randomly switches the center card with one of the two
edge cards (so the configuration after the first move is either RRQ or QRR). What is
the probability that, after 2004 moves, the center card is the queen?
Solution: 1/3 + 1/(3 · 22003 )
If the probability that the queen is the center card after move n is pn , then the prob-
ability that the queen is an edge card is 1 − pn , and the probability that the queen
is the center card after move n + 1 is pn+1 = (1 − pn )/2. This recursion allows us
to calculate the first few values of pn . We might then notice in 1, 0, 12 , 14 , 38 , 16 5 11
, 32 , · · · ,
that the value of each fraction is close to 1/3, and getting closer for larger n. In fact
subtracting 1/3 from each fraction yields 23 , − 13 , 16 , − 12
1 1 1
, 24 , − 48 , · · · . This suggests the
1 2 1 n
formula pn = 3 + 3 (− 2 ) , and one can then prove that this formula is in fact correct
by induction. Thus, p(2004) = 13 + 23 (− 12 )2004 = 13 + 3·220031
.
The recurrence can also be solved without guessing — by generating functions, for
example, or by using the fundamental theorem of linear recurrences, which ensures
that the solution is of the form pn = a + b(−1/2)n for some constants a, b.

4
Harvard-MIT Mathematics Tournament
February 28, 2004

Individual Round: Geometry Subject Test — Solutions

1. In trapezoid ABCD, AD is parallel to BC. 6 A = 6 D = 45◦ , while 6 B = 6 C = 135◦ .


If AB = 6 and the area of ABCD is 30, find BC.

Solution: 2 2
Draw altitudes from B and C to AD and label the points of intersection X and√Y ,
respectively. Then ABX and CDY are 45◦ −45◦ −90◦ triangles with BX = CY = 3 2.
So, the area of ABX and the area
√ of CDY are each
√ 9, meaning
√ that the area of rectangle
BCY X is 12. Since BX = 3 2, BC = 12/(3 2) = 2 2.
B 2✓2 C

6
3✓2
9 9

A 3✓2 X Y D
Area BCYX = 30 − 18 = 12

2. A parallelogram has 3 of its vertices at (1, 2), (3,8), and (4, 1). Compute the sum of
the possible x-coordinates for the 4th vertex.
Solution: 8
There are 3 possible locations for the 4th vertex. Let (a, b) be its coordinates. If it is
opposite to vertex (1, 2), then since the midpoints of the diagonals of a parallelogram
coincide, we get ( a+1
2
, b+2
2
) = ( 3+4
2
, 8+1
2
). Thus (a, b) = (6, 7). By similar reasoning for
the other possible choices of opposite vertex, the other possible positions for the fourth
vertex are (0, 9) and (2, −5), and all of these choices do give parallelograms. So the
answer is 6 + 0 + 2 = 8.
3. A swimming pool is in the shape of a circle with diameter 60 ft. The depth varies
linearly along the east-west direction from 3 ft at the shallow end in the east to 15 ft
at the diving end in the west (this is so that divers look impressive against the sunset)
but does not vary at all along the north-south direction. What is the volume of the
pool, in ft3 ?
Solution: 8100π
Take another copy of the pool, turn it upside-down, and put the two together to form
a cylinder. It has height 18 ft and radius 30 ft, so the volume is π(30 ft)2 · 18 ft =
16200π ft3 ; since our pool is only half of that, the answer is 8100π ft3 .
4. P is inside rectangle ABCD. P A = 2, P B = 3, and P C = 10. Find P D.

Solution: 95
Draw perpendiculars from P to E on AB, F on BC, G on CD, and H on DA, and
let AH = BF = w, HD = F C = x, AE = DG = y, and EB = GC = z. Then
P A2 = w2 + y 2 , P B 2 = w2 + z 2 , P C 2 = x2 + z 2 , and P D2 = x2 +√y 2 . Adding and
subtracting, we see that P D2 = P A2 − P B 2 + P C 2 = 95, so P D = 95.

1
A y E z B

w w

H F
P

x x

D y G z C

5. Find the area of the region of the xy-plane defined by the inequality |x|+|y|+|x+y| ≤ 1.
Solution: 3/4
To graph this region we divide the xy-plane into six sectors depending on which of
x, y, x + y are ≥ 0, or ≤ 0. The inequality simplifies in each case:

Sector Inequality Simplified inequality


x ≥ 0, y ≥ 0, x + y ≥0 x + y + (x + y) ≤ 1 x + y ≤ 1/2
x ≥ 0, y ≤ 0, x + y ≥0 x − y + (x + y) ≤ 1 x ≤ 1/2
x ≥ 0, y ≤ 0, x + y ≤0 x − y − (x + y) ≤ 1 y ≥ −1/2
x ≤ 0, y ≥ 0, x + y ≥0 −x + y + (x + y) ≤ 1 y ≤ 1/2
x ≤ 0, y ≥ 0, x + y ≤0 −x + y − (x + y) ≤ 1 x ≥ −1/2
x ≤ 0, y ≤ 0, x + y ≤0 −x − y − (x + y) ≤ 1 x + y ≥ −1/2

We then draw the region; we get a hexagon as shown. The hexagon intersects each
region in an isosceles right triangle of area 1/8, so the total area is 6 · 1/8 = 3/4.

6. In trapezoid ABCD shown, AD is parallel to BC, and AB = 6, BC = 7, CD =


8, AD = 17. If sides AB and CD are extended to meet at E, find the resulting angle
at E (in degrees).
Solution: 90
Choose point F on AD so that BCDF is a parallelogram. Then BF = CD = 8, and
AF = AD − DF = AD − BC = 10, so 4ABF is a 6-8-10 right triangle. The required
angle is equal to 6 ABF = 90◦ .

2
B 7 C

6 8 8

A D
10 F 7

7. Yet another trapezoid ABCD has AD parallel to BC. AC and BD intersect at P .


If [ADP ]/[BCP ] = 1/2, find [ADP ]/[ABCD]. (Here the notation [P1 · · · Pn ] denotes
the area of the polygon P1 · · · Pn .)

Solution: 3 − 2 2
A homothety (scaling) about P takes triangle ADP into BCP , since AD, BC √ are
parallel and A, P, C; B, P, D are collinear. The ratio of homothety √ is thus 2. It
follows that, if we rescale to put [ADP ] = 1, then
√ [ABP ] = [CDP ] = 2, just by√the
ratios of lengths of bases. So [ABCD]
√ = 3 + 2 2, so [ADP ]/[ABCD] = 1/(3 + 2 2).
Simplifying this, we get 3 − 2 2.
8. A triangle has side lengths 18, 24, and 30. Find the area of the triangle whose vertices
are the incenter, circumcenter, and centroid of the original triangle.
Solution: 3
There are many solutions to this problem, which is straightforward. The given triangle
is a right 3-4-5 triangle, so the circumcenter is the midpoint of the hypotenuse. Co-
ordinatizing for convenience, put the vertex at (0, 0) and the other vertices at (0, 18)
and (24, 0). Then the circumcenter is (12, 9). The centroid is at one-third the sum of
the three vertices, which is (8, 6). Finally, since the area equals the inradius times half
the perimeter, we can see that the inradius is (18 · 24/2) / ([18 + 24 + 30]/2) = 6. So
the incenter of the triangle is (6, 6). So the small triangle has a base of length 2 and a
height of 3, hence its area is 3.
9. Given is a regular tetrahedron of volume 1. We obtain a second regular tetrahedron by
reflecting the given one through its center. What is the volume of their intersection?
Solution: 1/2
Imagine placing the tetrahedron ABCD flat on a table with vertex A at the top.
By vectors or otherwise, we see that the center is 3/4 of the way from A to the
bottom face, so the reflection of this face lies in a horizontal plane halfway between A
and BCD. In particular, it cuts off the smaller tetrahedron obtained by scaling the
original tetrahedron by a factor of 1/2 about A. Similarly, the reflections of the other
three faces cut off tetrahedra obtained by scaling ABCD by 1/2 about B, C, and D.
On the other hand, the octahedral piece remaining remaining after we remove these
four smaller tetrahedra is in the intersection of ABCD with its reflection, since the
reflection sends this piece to itself. So the answer we seek is just the volume of this
piece, which is
(volume of ABCD) − 4 · (volume of ABCD scaled by a factor of 1/2)
= 1 − 4(1/2)3 = 1/2.

3
10. Right triangle XY Z has right angle at Y and XY = 228, Y Z = 2004. Angle Y is
trisected, and the angle trisectors intersect XZ at P and Q so that X, P, Q, Z lie on
XZ in that order. Find the value of (P Y + Y Z)(QY + XY ).
Solution: 1370736
The triangle’s area is (228 · 2004)/2 = 228456. All the angles at Y are 30 degrees,
so by the sine area formula, the areas of the three small triangles in the diagram are
QY · Y Z/4, P Y · QY /4, and XY · P Y /4, which sum to the area of the triangle. So
expanding (P Y + Y Z)(QY + XY ), we see that it equals

4 · 228456 + XY · Y Z = 6 · 228456 = 1370736.

X
P
Q

Y Z

4
Harvard-MIT Mathematics Tournament
February 28, 2004

Individual Round: General Test, Part 1 — Solutions

1. There are 1000 rooms in a row along a long corridor. Initially the first room contains
1000 people and the remaining rooms are empty. Each minute, the following happens:
for each room containing more than one person, someone in that room decides it is
too crowded and moves to the next room. All these movements are simultaneous (so
nobody moves more than once within a minute). After one hour, how many different
rooms will have people in them?
Solution: 31
We can prove by induction on n that the following pattern holds for 0 ≤ n ≤ 499:
after 2n minutes, the first room contains 1000 − 2n people and the next n rooms each
contain 2 people, and after 2n + 1 minutes, the first room contains 1000 − (2n + 1)
people, the next n rooms each contain 2 people, and the next room after that contains
1 person. So, after 60 minutes, we have one room with 940 people and 30 rooms with
2 people each.

2. What is the largest whole number that is equal to the product of its digits?
Solution: 9
Suppose the number n has k + 1 digits, the first of which is d. Then the number is at
least d · 10k . On the other hand, each of the digits after the first is at most 9, so the
product of the digits is at most d · 9k . Thus, if n equals the product of its digits, then

d · 10k ≤ n ≤ d · 9k

which forces k = 0, i.e., the number has only one digit. So n = 9 is clearly the largest
possible value.

3. Suppose f is a function that assigns to each real number x a value f (x), and suppose
the equation

f (x1 + x2 + x3 + x4 + x5 ) = f (x1 ) + f (x2 ) + f (x3 ) + f (x4 ) + f (x5 ) − 8

holds for all real numbers x1 , x2 , x3 , x4 , x5 . What is f (0)?


Solution: 2
Plug in x1 = x2 = x3 = x4 = x5 = 0. Then the equation reads f (0) = 5f (0) − 8, so
4f (0) = 8, so f (0) = 2.

4. How many ways can you mark 8 squares of an 8 × 8 chessboard so that no two marked
squares are in the same row or column, and none of the four corner squares is marked?
(Rotations and reflections are considered different.)
Solution: 21600
In the top row, you can mark any of the 6 squares that is not a corner. In the
bottom row, you can then mark any of the 5 squares that is not a corner and not
in the same column as the square just marked. Then, in the second row, you have

1
6 choices for a square not in the same column as either of the two squares already
marked; then there are 5 choices remaining for the third row, and so on down to 1
for the seventh row, in which you make the last mark. Thus, altogether, there are
6 · 5 · (6 · 5 · · · 1) = 30 · 6! = 30 · 720 = 21600 possible sets of squares.

5. A rectangle has perimeter 10 and diagonal 15. What is its area?
Solution: 5
√ √
If the sides are x and y, we have 2x + 2y = 10, so x + y = 5, and x2 + y 2 = 15, so
x2 + y 2 = 15. Squaring the first equation gives x2 + 2xy + y 2 = 25, and subtracting
the second equation gives 2xy = 10, so the area is xy = 5.

6. Find the ordered quadruple of digits (A, B, C, D), with A > B > C > D, such that

ABCD
− DCBA
= BDAC.

Solution: (7, 6, 4, 1)
Since D < A, when A is subtracted from D in the ones’ column, there will be a borrow
from C in the tens’ column. Thus, D + 10 − A = C. Next, consider the subtraction in
the tens’ column, (C − 1) − B. Since C < B, there will be a borrow from the hundreds’
column, so (C − 1 + 10) − B = A. In the hundreds’ column, B − 1 ≥ C, so we do
not need to borrow from A in the thousands’ column. Thus, (B − 1) − C = D and
A − D = B. We thus have a system of four equations in four variables A, B, C, D, and
solving by standard methods (e.g. substitution) produces (A, B, C, D) = (7, 6, 4, 1).

7. Let ACE be a triangle with a point B on segment AC and a point D on segment CE


such that BD is parallel to AE. A point Y is chosen on segment AE, and segment
CY is drawn. Let X be the intersection of CY and BD. If CX = 5, XY = 3, what is
the ratio of the area of trapezoid ABDE to the area of triangle BCD?
Solution: 39/25

B
K
X D
A

L
Y
E

Draw the altitude from C to AE, intersecting line BD at K and line AE at L. Then
CK is the altitude of triangle BCD, so triangles CKX and CLY are similar. Since
CY /CX = 8/5, CL/CK = 8/5. Also triangles CKB and CLA are similar, so that
CA/CB = 8/5, and triangles BCD and ACE are similar, so that AE/BD = 8/5.
The area of ACE is (1/2)(AE)(CL), and the area of BCD is (1/2)(BD)(CK), so the

2
ratio of the area of ACE to the area of BCD is 64/25. Therefore, the ratio of the area
of ABDE to the area of BCD is 39/25.

8. You have a 10 × 10 grid of squares. You write a number in each square as follows: you
write 1, 2, 3, . . . , 10 from left to right across the top row, then 11, 12, . . . , 20 across the
second row, and so on, ending with a 100 in the bottom right square. You then write
a second number in each square, writing 1, 2, . . . , 10 in the first column (from top to
bottom), then 11, 12, . . . , 20 in the second column, and so forth.
When this process is finished, how many squares will have the property that their two
numbers sum to 101?
Solution: 10
The number in the ith row, jth column will receive the numbers 10(i − 1) + j and
10(j − 1) + i, so the question is how many pairs (i, j) (1 ≤ i, j ≤ 10) will have

101 = [10(i − 1) + j] + [10(j − 1) + i] ⇔ 121 = 11i + 11j = 11(i + j).

Now it is clear that this is achieved by the ten pairs (1, 10), (2, 9), (3, 8), . . . , (10, 1) and
no others.

9. Urn A contains 4 white balls and 2 red balls. Urn B contains 3 red balls and 3 black
balls. An urn is randomly selected, and then a ball inside of that urn is removed. We
then repeat the process of selecting an urn and drawing out a ball, without returning
the first ball. What is the probability that the first ball drawn was red, given that the
second ball drawn was black?
Solution: 7/15
This is a case of conditional probability; the answer is the probability that the first
ball is red and the second ball is black, divided by the probability that the second ball
is black.
First, we compute the numerator. If the first ball is drawn from Urn A, we have a
probability of 2/6 of getting a red ball, then a probability of 1/2 of drawing the second
ball from Urn B, and a further probability of 3/6 of drawing a black ball. If the first
ball is drawn from Urn B, we have probability 3/6 of getting a red ball, then 1/2 of
drawing the second ball from Urn B, and 3/5 of getting a black ball. So our numerator
is µ ¶
1 2 1 3 3 1 3 7
· · + · · = .
2 6 2 6 6 2 5 60
We similarly compute the denominator: if the first ball is drawn from Urn A, we have
a probability of 1/2 of drawing the second ball from Urn B, and 3/6 of drawing a
black ball. If the first ball is drawn from Urn B, then we have probability 3/6 that
it is red, in which case the second ball will be black with probability (1/2) · (3/5),
and probability 3/6 that the first ball is black, in which case the second is black with
probability (1/2) · (2/5). So overall, our denominator is
µ · ¸¶
1 1 3 3 1 3 1 2 1
· + · + · = .
2 2 6 6 2 5 2 5 4

Thus, the desired conditional probability is (7/60) / (1/4) = 7/15.

3
10. A floor is tiled with equilateral triangles of side length 1, as shown. If you drop a
needle of length 2 somewhere on the floor, what is the largest number of triangles it
could end up intersecting? (Only count the triangles whose interiors are met by the
needle — touching along edges or at corners doesn’t qualify.)

Solution: 8
Let L be the union of all the lines of the tiling. Imagine walking from one end of the
needle to the other. We enter a new triangle precisely when we cross one of the lines
of the tiling. Therefore, the problem is equivalent to maximizing the number of times
the needle crosses L. Now, the lines of the tiling each run in one of three directions.
It is clear that the needle cannot
√ cross more than three lines in any given direction,
since the lines are a distance
√ 3/2 apart and the needle would therefore have to be of
length greater than 3 3/2 > 2. Moreover, it cannot cross three lines in each of two
different directions. To see this, notice that its endpoints would have to lie in either
the two light-shaded regions or the two dark-shaded regions shown, but the closest two
points of such opposite regions are at a distance of 2 (twice the length of a side of a
triangle), so the needle cannot penetrate both regions.

Therefore, the needle can cross at most three lines in one direction and two lines in
each of the other two directions, making for a maximum of 3 + 2 + 2 = 7 crossings and
7 + 1 = 8 triangles intersected. √
The example shows that 8 is achievable, as long as the
needle has length greater than 3 < 2.

4
Harvard-MIT Mathematics Tournament
February 28, 2004

Individual Round: General Test, Part 2 — Solutions

1. Find the largest number n such that (2004!)! is divisible by ((n!)!)!.


Solution: 6
For positive integers a, b, we have

a! | b! ⇔ a! ≤ b! ⇔ a ≤ b.

Thus,

((n!)!)! | (2004!)! ⇔ (n!)! ≤ 2004! ⇔ n! ≤ 2004 ⇔ n ≤ 6.

2. Andrea flips a fair coin repeatedly, continuing until she either flips two heads in a row
(the sequence HH) or flips tails followed by heads (the sequence T H). What is the
probability that she will stop after flipping HH?
Solution: 1/4
The only way that Andrea can ever flip HH is if she never flips T , in which case she
must flip two heads immediately at the beginning. This happens with probability 14 .

3. How many ordered pairs of integers (a, b) satisfy all of the following inequalities?

a2 + b2 < 16
a2 + b2 < 8a
a2 + b2 < 8b

Solution: 6
This is easiest to see by simply graphing the inequalities. They correspond to the
(strict) interiors of circles of radius 4 and centers at (0, 0), (4, 0), (0, 4), respectively. So
we can see that there are 6 lattice points in their intersection (circled in the figure).

1
4. A horse stands at the corner of a chessboard, a white square. With each jump, the
horse can move either two squares horizontally and one vertically or two vertically and
one horizontally (like a knight moves). The horse earns two carrots every time it lands
on a black square, but it must pay a carrot in rent to rabbit who owns the chessboard
for every move it makes. When the horse reaches the square on which it began, it can
leave. What is the maximum number of carrots the horse can earn without touching
any square more than twice?

Solution: 0
The horse must alternate white and black squares, and it ends on the same square
where it started. Thus it lands on the same number of black squares (b) as white
squares (w). Thus, its net earnings will be 2b − (b + w) = b − w = 0 carrots, regardless
of its path.

5. Eight strangers are preparing to play bridge. How many ways can they be grouped
into two bridge games — that is, into unordered pairs of unordered pairs of people?
Solution: 315
Putting 8 people into 4 pairs and putting those 4 pairs into 2 pairs of pairs are inde-
pendent. If the people are numbered from 1 to 8, there are 7 ways to choose the person
to pair with person 1. Then there are 5 ways to choose the person to pair with the
person who has the lowest remaining number, 3 ways to choose the next, and 1 way to
choose the last (because there are only 2 people remaining). Thus, there are 7 · 5 · 3 · 1
ways to assign 8 people to pairs and similarly there are 3 · 1 ways to assign 4 pairs to
2 pairs of pairs, so there are 7 · 5 · 3 · 3 = 315 ways.

6. a and b are positive integers. When written in binary, a has 2004 1’s, and b has 2005 1’s
(not necessarily consecutive). What is the smallest number of 1’s a + b could possibly
have?
Solution: 1
Consider the following addition:

111 · · · 100 · · · 01
+ 11 · · · 11
= 1000 · · · · · · · · · 00

2
By making the blocks of 1’s and 0’s appropriately long, we can ensure that the addends
respectively contain 2004 and 2005 1’s. (To be precise, we get a = 24008 − 22005 + 1 and
b = 22005 − 1.) Then the sum has only one 1. And clearly it is not possible to get any
less than one 1.
7. Farmer John is grazing his cows at the origin. There is a river that runs east to west
50 feet north of the origin. The barn is 100 feet to the south and 80 feet to the east
of the origin. Farmer John leads his cows to the river to take a swim, then the cows
leave the river from the same place they entered and Farmer John leads them to the
barn. He does this using the shortest path possible, and the total distance he travels
is d feet. Find the value of d.

Solution: 40 29
Suppose we move the barn to its reflection across the river’s edge. Then paths from
the origin to the river and then to the old barn location correspond to paths from
the origin to the river and then to the new barn location, by reflecting the second
part of the path across the river, and corresponding paths have the same length. Now
the shortest path from the origin to the river and then to the new barn location is a
straight line. The√new barn location√ is 200 feet north and 80 feet east of the origin, so
2 2
the value of d is 200 + 80 = 40 29.

(20,50)

(0,0)

(80,−100)

8. A freight train leaves the town of Jenkinsville at 1:00 PM traveling due east at constant
speed. Jim, a hobo, sneaks onto the train and falls asleep. At the same time, Julie
leaves Jenkinsville on her bicycle, traveling along a straight road in a northeasterly
direction (but not due northeast) at 10 miles per hour. At 1:12 PM, Jim rolls over
in his sleep and falls from the train onto the side of the tracks. He wakes up and
immediately begins walking at 3.5 miles per hour directly towards the road on which
Julie is riding. Jim reaches the road at 2:12 PM, just as Julie is riding by. What is the
speed of the train in miles per hour?
Solution: 62.5
Julie’s distance is (10 mph) · (6/5 hrs) = 12 miles. Jim’s walking distance, after
falling off the train, is (3.5 mph) ·√(1 hr) = 3.5 miles
√ at a right angle to the road.
Therefore, Jim rode the train for 122 + 3.52 = 21 242 + 72 = 25/2 miles, and its
speed is (25/2 mi)/(1/5 hr) = 62.5 mph.

3
9. Given is a regular tetrahedron of volume 1. We obtain a second regular tetrahedron by
reflecting the given one through its center. What is the volume of their intersection?
Solution: 1/2
Imagine placing the tetrahedron ABCD flat on a table with vertex A at the top.
By vectors or otherwise, we see that the center is 3/4 of the way from A to the
bottom face, so the reflection of this face lies in a horizontal plane halfway between A
and BCD. In particular, it cuts off the smaller tetrahedron obtained by scaling the
original tetrahedron by a factor of 1/2 about A. Similarly, the reflections of the other
three faces cut off tetrahedra obtained by scaling ABCD by 1/2 about B, C, and D.
On the other hand, the octahedral piece remaining remaining after we remove these
four smaller tetrahedra is in the intersection of ABCD with its reflection, since the
reflection sends this piece to itself. So the answer we seek is just the volume of this
piece, which is

(volume of ABCD) − 4 · (volume of ABCD scaled by a factor of 1/2)

= 1 − 4(1/2)3 = 1/2.

10. A lattice point is a point whose coordinates are both integers. Suppose Johann walks
in a line from the point (0, 2004) to a random lattice point in the interior (not on
the boundary) of the square with vertices (0, 0), (0, 99), (99, 99), (99, 0). What is the
probability that his path, including the endpoints, contains an even number of lattice
points?
Solution: 3/4
If Johann picks the point (a, b), the path will contain gcd(a, 2004 − b) + 1 points. There
will be an odd number of points in the path if gcd(a, 2004 − b) is even, which is true if
and only if a and b are both even. Since there are 492 points with a, b both even and
982 total points, the probability that the path contains an even number of points is

982 − 492 492 (22 − 12 ) 3


2
= 2 2
= .
98 49 (2 ) 4

4
Harvard-MIT Mathematics Tournament
February 28, 2004

Guts Round — Solutions


1. Find the value of
µ ¶ µ ¶ µ ¶ µ ¶ µ ¶ µ ¶
6 1 6 2 6 3 6 4 6 5 6 6
2 + 2 + 2 + 2 + 2 + 2.
1 2 3 4 5 6

Solution: 728
This
¡6¢ 0 sum is the binomial expansion of (1 + 2)6 , except that it is missing the first term,
0
2 = 1. So we get 36 − 1 = 728.
2. If the three points
(1, a, b)
(a, 2, b)
(a, b, 3)
are collinear (in 3-space), what is the value of a + b?
Solution: 4
The first two points are distinct (otherwise we would have a = 1 and a = 2 simulta-
neously), and they both lie on the plane z = b, so the whole line is in this plane and
b = 3. Reasoning similarly with the last two points gives a = 1, so a + b = 4.
3. If the system of equations
|x + y| = 99
|x − y| = c
has exactly two real solutions (x, y), find the value of c.
Solution: 0
If c < 0, there are no solutions. If c > 0 then we have four possible systems of linear
equations given by x + y = ±99, x − y = ±c, giving four solutions (x, y). So we must
have c = 0, and then we do get two solutions (x = y, so they must both equal ±99/2).
4. A tree grows in a rather peculiar manner. Lateral cross-sections of the trunk, leaves,
branches, twigs, and so forth are circles. The trunk is 1 meter in diameter to a height of
1 meter, at which point it splits into two sections, each with diameter .5 meter. These
sections are each one meter long, at which point they each split into two sections, each
with diameter .25 meter. This continues indefinitely: every section of tree is 1 meter
long and splits into two smaller sections, each with half the diameter of the previous.
What is the total volume of the tree?
Solution: π/2
If we count the trunk as level 0, the two sections emerging from it as level 1, and so
forth, then the nth level consists of 2n sections each with diameter 1/2n , for a volume
of 2n (π/4 · 2−2n ) = (π/4) · 2−n . So the total volume is given by a simple infinite sum,
.25π · (1 + 1/2 + 1/4 + ...) = .25π · 2 = π/2.

1
5. Augustin has six 1 × 2 × π bricks. He stacks them, one on top of another, to form a
tower six bricks high. Each brick can be in any orientation so long as it rests flat on
top of the next brick below it (or on the floor). How many distinct heights of towers
can he make?
Solution: 28
If there are k bricks which are placed so that they contribute either 1 or 2 height,
then the height of these k bricks can be any integer from k to 2k. Furthermore,
towers with different values of k cannot have the same height. Thus, for each k there
are k + 1 possible tower heights, and since k is any integer from 0 to 6, there are
1 + 2 + 3 + 4 + 5 + 6 + 7 = 28 possible heights.
√ √
6. Find the smallest integer n such that n + 99 − n < 1.
Solution: 2402
This is equivalent to
√ √
n + 99 < n+1

n + 99 < n + 1 + 2 n

49 < n

So the smallest integer n with this property is 492 + 1 = 2402.

7. Find the shortest distance from the line 3x + 4y = 25 to the circle x2 + y 2 = 6x − 8y.
Solution: 7/5
The circle is (x − 3)2 + (y + 4)2 = 52 . The center (3, −4) is a distance of

|3 · 3 + 4 · −4 − 25| 32
√ =
32 + 42 5
from the line, so we subtract 5 for the radius of the circle and get 7/5.

8. I have chosen five of the numbers {1, 2, 3, 4, 5, 6, 7}. If I told you what their product
was, that would not be enough information for you to figure out whether their sum
was even or odd. What is their product?
Solution: 420
Giving you the product of the five numbers is equivalent to telling you the product
of the two numbers I didn’t choose. The only possible products that are achieved by
more than one pair of numbers are 12 ({3, 4} and {2, 6}) and 6 ({1, 6} and {2, 3}). But
in the second case, you at least know that the two unchosen numbers have odd sum
(and so the five chosen numbers have odd sum also). Therefore, the first case must
hold, and the product of the five chosen numbers is

1 · 2 · 5 · 6 · 7 = 1 · 3 · 4 · 5 · 7 = 420.

9. A positive integer n is picante if n! ends in the same number of zeroes whether written
in base 7 or in base 8. How many of the numbers 1, 2, . . . , 2004 are picante?
Solution: 4

2
The number of zeroes in base 7 is the total number of factors of 7 in 1 · 2 · · · n, which is
¥ ¦ ¥ ¦
bn/7c + n/72 + n/73 + · · · .

The number of zeroes in base 8 is bac, where


¥ ¦ ¥ ¦
a = (bn/2c + n/22 + n/23 + · · · )/3
¥ ¦ ¥ ¦
is one-third the number of factors of 2 in the product n!. Now n/2k /3 ≥ n/7k for all
k, since (n/2k )/3 ≥ n/7k . But n can only be picante if the two sums differ by at most
2/3, so in particular this requires (bn/22 c)/3 ≤ bn/72 c + 2/3 ⇔ bn/4c ≤ 3 bn/49c + 2.
This cannot happen for n ≥ 12; checking the remaining few cases by hand, we find
n = 1, 2, 3, 7 are picante, for a total of 4 values.

10. Let f (x) = x2 + x4 + x6 + x8 + · · · , for all real x such that the sum converges. For how
many real numbers x does f (x) = x?
Solution: 2
Clearly x = 0 works. Otherwise, we want x = x2 /(1 − x2 ), or x2 + x −
√ 1 = 0. Discard
the negative root (since the sum doesn’t converge there), but (−1 + 5)/2 works, for
a total of 2 values.

11. Find all numbers n with the following property: there is exactly one set of 8 different
positive integers whose sum is n.
Solution: 36, 37
The sum of 8 different positive integers is at least 1 + 2 + 3 + · · · + 8 = 36, so we must
have n ≥ 36. Now n = 36 satisfies the desired property, since in this case we must
have equality — the eight numbers must be 1, . . . , 8. And if n = 37 the eight numbers
must be 1, 2, . . . , 7, 9: if the highest number is 8 then the sum is 36 < n, while if the
highest number is more than 9 the sum is > 1 + 2 + · · · + 7 + 9 = 37 = n. So the
highest number must be 9, and then the remaining numbers must be 1, 2, . . . , 7. Thus
n = 37 also has the desired property.
However, no other values of n work: if n > 37 then {1, 2, 3, . . . , 7, n − 28} and
{1, 2, . . . , 6, 8, n − 29} are both sets of 8 distinct positive integers whose sum is n.
So n = 36, 37 are the only solutions.

12. A convex quadrilateral is drawn in the coordinate plane such that each of its vertices
(x, y) satisfies the equations x2 + y 2 = 73 and xy = 24. What is the area of this
quadrilateral?
Solution: 110
The vertices all satisfy (x + y)2 = x2 + y 2 + 2xy = 73 + 2 · 24 = 121, so x + y = ±11.
Similarly, (x−y)2 = x2 +y 2 −2xy = 73−2·24 = 25, so x−y = ±5. Thus, there are four
solutions: (x, y) = (8, 3), (3, 8), (−3, −8), (−8, −3). All four of these solutions satisfy
the
√ original √equations. The quadrilateral is therefore a rectangle with side lengths of
5 2 and 11 2, so its area is 110.

13. Find all positive integer solutions (m, n) to the following equation:

m2 = 1! + 2! + · · · + n!.

3
Solution: (1, 1), (3, 3)
A square must end in the digit 0, 1, 4, 5, 6, or 9. If n ≥ 4, then 1! + 2! + · · · + n! ends
in the digit 3, so cannot be a square. A simple check for the remaining cases reveals
that the only solutions are (1, 1) and (3, 3).
an+2
14. If a1 = 1, a2 = 0, and an+1 = an + 2
for all n ≥ 1, compute a2004 .
Solution: −21002
By writing out the first few terms, we find that an+4 = −4an . Indeed,

an+4 = 2(an+3 − an+2 ) = 2(an+2 − 2an+1 ) = 2(−2an ) = −4an .

Then, by induction, we get a4k = (−4)k for all positive integers k, and setting k = 501
gives the answer.

15. A regular decagon A0 A1 A2 · · · A9 is given in the plane. Compute ∠A0 A3 A7 in degrees.


Solution: 54◦
Put the decagon in a circle. Each side subtends an arc of 360◦ /10 = 36◦ . The in-
scribed angle ∠A0 A3 A7 contains 3 segments, namely A7 A8 , A8 A9 , A9 A0 , so the angle
is 108◦ /2 = 54◦ .

16. An n-string is a string of digits formed by writing the numbers 1, 2, . . . , n in some order
(in base ten). For example, one possible 10-string is

35728910461

What is the smallest n > 1 such that there exists a palindromic n-string?
Solution: 19
The following is such a string for n = 19:

9|18|7|16|5|14|3|12|1|10|11|2|13|4|15|6|17|8|19

where the vertical bars indicate breaks between the numbers. On the other hand, to
see that n = 19 is the minimum, notice that only one digit can occur an odd number
of times in a palindromic n-string (namely the center digit). If n ≤ 9, then (say)
the digits 1, 2 each appear once in any n-string, so we cannot have a palindrome. If
10 ≤ n ≤ 18, then 0, 9 each appear once, and we again cannot have a palindrome. So
19 is the smallest possible n.

17. Kate has four red socks and four blue socks. If she randomly divides these eight socks
into four pairs, what is the probability that none of the pairs will be mismatched?
That is, what is the probability that each pair will consist either of two red socks or
of two blue socks?
Solution: 3/35
¡¢
The number of ways Kate can divide the four red socks into two pairs is 42 /2 = 3. The
number of ways she can divide the four blue socks into two pairs is also 3. Therefore,
the number of ways she can form two pairs of red socks and two pairs of blue socks is

4
3 · 3 = 9. The total number of ways she can divide the eight socks into four pairs is
[8!/(2! · 2! · 2! · 2!)]/4! = 105, so the probability that the socks come out paired correctly
is 9/105 = 3/35.
To see why 105 is the correct denominator, we can look at each 2! term as representing
the double counting of pair (ab) and pair (ba), while the 4! term represents the number
of different orders in which we can select the same four pairs. Alternatively, we know
that there are three ways to select two pairs from four socks. To select three pairs from
six socks, there are five different choices for the first sock’s partner and then three ways
to pair up the remaining four socks, for a total of 5 · 3 = 15 pairings. To select four
pairs from eight socks, there are seven different choices for the first sock’s partner and
then fifteen ways to pair up the remaining six socks, for a total of 7 · 15 = 105 pairings.

18. On a spherical planet with diameter 10, 000 km, powerful explosives are placed at
the north and south poles. The explosives are designed to vaporize all matter within
5, 000 km of ground zero and leave anything beyond 5, 000 km untouched. After the
explosives are set off, what is the new surface area of the planet, in square kilometers?
Solution: 100, 000, 000π
The explosives have the same radius as the planet, so the surface area of the “cap”
removed is the same as the new surface area revealed in the resulting “dimple.” Thus
the area is preserved by the explosion and remains π · (10, 000)2 .

19. The Fibonacci numbers are defined by F1 = F2 = 1, and Fn = Fn−1 + Fn−2 for n ≥ 3.
If the number
F2003 F2004

F2002 F2003
is written as a fraction in lowest terms, what is the numerator?
Solution: 1
2
Before reducing, the numerator is F2003 − F2002 F2004 . We claim Fn2 − Fn−1 Fn+1 =
(−1)n+1 , which will immediately imply that the answer is 1 (no reducing required).
This claim is straightforward to prove by induction on n: it holds for n = 2, and if it
holds for some n, then
2
Fn+1 −Fn Fn+2 = Fn+1 (Fn−1 +Fn )−Fn (Fn +Fn+1 ) = Fn+1 Fn−1 −Fn2 = −(−1)n+1 = (−1)n+2 .

20. Two positive rational numbers x and y, when written in lowest terms, have the property
that the sum of their numerators is 9 and the sum of their denominators is 10. What
is the largest possible value of x + y?
Solution: 73/9
For fixed denominators a < b (with sum 10), we maximize the sum of the fractions by
giving the smaller denominator as large a numerator as possible: 8/a + 1/b. Then, if
a ≥ 2, this quantity is at most 8/2 + 1/1 = 5, which is clearly smaller than the sum
we get by setting a = 1, namely 8/1 + 1/9 = 73/9. So this is the answer.

21. Find all ordered pairs of integers (x, y) such that 3x 4y = 2x+y + 22(x+y)−1 .
Solution: (0, 1), (1, 1), (2, 2)

5
The right side is 2x+y (1 + 2x+y−1 ). If the second factor is odd, it needs to be a power
of 3, so the only options are x + y = 2 and x + y = 4. This leads to two solutions,
namely (1,1) and (2,2). The second factor can also be even, if x + y − 1 = 0. Then
x + y = 1 and 3x 4y = 2 + 2, giving (0, 1) as the only other solution.

22. I have written a strictly increasing sequence of six positive integers, such that each
number (besides the first) is a multiple of the one before it, and the sum of all six
numbers is 79. What is the largest number in my sequence?
Solution: 48
If the fourth number is ≥ 12, then the last three numbers must sum to at least 12 +
2 · 12 + 22 · 12 = 84 > 79. This is impossible, so the fourth number must be less than
12. Then the only way we can have the required divisibilities among the first four
numbers is if they are 1, 2, 4, 8. So the last two numbers now sum to 79 − 15 = 64. If
we call these numbers 8a, 8ab (a, b > 1) then we get a(1 + b) = a + ab = 8, which forces
a = 2, b = 3. So the last two numbers are 16, 48.

23. Find the largest integer n such that 3512 − 1 is divisible by 2n .


Solution: 11
Write
3512 − 1 = (3256 + 1)(3256 − 1) = (3256 + 1)(3128 + 1)(3128 − 1)
= · · · = (3256 + 1)(3128 + 1) · · · (3 + 1)(3 − 1).
k k
Now each factor 32 + 1, k ≥ 1, is divisible by just one factor of 2, since 32 + 1 =
k−1 k−1
(32 )2 +1 ≡ 12 +1 = 2 (mod 4). Thus we get 8 factors of 2 here, and the remaining
terms (3 + 1)(3 − 1) = 8 give us 3 more factors of 2, for a total of 11.

24. We say a point is contained in a square if it is in its interior or on its boundary. Three
unit squares are given in the plane such that there is a point contained in all three.
Furthermore, three points A, B, C, are given, each contained in at least one of the
squares. Find the maximum area of triangle ABC.

Solution: 3 3/2
Let X be a point contained in all three √ squares. The distance from X to any point
in any of the three squares is at most 2, the length of √ the diagonal of the squares.
Therefore,
√ triangle ABC is contained in a circle of radius 2, so its circumradius is at
most 2. The triangle with √ greatest area that satisfies this property is the equilateral
triangle in a circle of radius 2. (This can be proved, for example, by considering that
the maximum altitude to any given side is obtained by putting the opposite vertex
at the midpoint of its arc, and it follows that all the vertices are equidistant.) The
equilateral triangle is also attainable, since making X the circumcenter and positioning
the squares such that AX, BX, and CX are diagonals (of the three √ squares) and ABC
is equilateral, leads to such a triangle. This triangle has area 3 3/2, which may be
calculated, for example,
√ using the sine formula for area applied to ABX, ACX, and
BCX, to get 3/2( 2)2 sin 120◦ . (See diagram, next page.)

25. Suppose x3 − ax2 + bx − 48 is a polynomial with three positive roots p, q, and r such
that p < q < r. What is the minimum possible value of 1/p + 2/q + 3/r?

6
C

X
A B

Solution: 3/2
We know pqr = 48 since the product of the roots of a cubic is the constant term. Now,
r
1 2 3 6 3
+ + ≥33 =
p q r pqr 2

by AM-GM, with equality when 1/p = 2/q = 3/r. This occurs when p = 2, q = 4,
r = 6, so 3/2 is in fact the minimum possible value.

26. How many of the integers 1, 2, . . . , 2004 can be represented as (mn + 1)/(m + n) for
positive integers m and n?
Solution: 2004
For any positive integer a, we can let m = a2 + a − 1, n = a + 1 to see that every
positive integer has this property, so the answer is 2004.

27. A regular hexagon has one side along the diameter of a semicircle, and the two opposite
vertices on the semicircle. Find the area of the hexagon if the diameter of the semicircle
is 1.

Solution: 3 3/26
The midpoint of the side of the hexagon on the diameter is the center of the circle.
Draw the segment from this center to a vertex of the hexagon on the circle. This
segment, whose length
√ is 1/2, is the hypotenuse of a right triangle whose legs have
2
lengths a/2 and a 3, where a is a side of the hexagon. So 1/4 = a (1/4 + 3), so
a2 = 1/13. The hexagon√ consists
√ of 6 equilateral triangles of side length a, so the area
2
of the hexagon is 3a 3/2 = 3 3/26.

7
28. Find the value of
µ ¶ µ ¶ µ ¶ µ ¶
2003 2003 2003 2003
+ + + ··· + .
1 4 7 2002

Solution: (22003 − 2)/3



Let ω = −1/2+i 3/2 be a complex cube root of unity. Then, by the binomial theorem,
we have
µ ¶ µ ¶ µ ¶ µ ¶
2 2003 2003 2 2003 3 2003 4 2003 2005
ω (ω + 1) = ω + ω + ω + ··· + ω
0 1 2 2003
µ ¶ µ ¶ µ ¶ µ ¶
2003 2003 2003 2003 2003
2 = + + + ··· +
0 1 2 2003
µ ¶ µ ¶ µ ¶ µ ¶
−2 −1 2003 2003 −2 2003 −3 2003 −4 2003 −2005
ω (ω + 1) = ω + ω + ω + ··· + ω
0 1 2 2003
¡ ¢
If we add these together, then the terms 2003 n
for n ≡ 1 (mod 3) appear with coefficient
3, while the remaining terms appear with coefficient 1 + ω + ω 2 = 0. Thus the desired
sum is just (ω 2 (ω + 1)2003 + 22003 + ω −2 (ω −1 + 1)2003 )/3. Simplifying using ω + 1 = −ω 2
and ω −1 + 1 = −ω gives (−1 + 22003 + −1)/3 = (22003 − 2)/3.

29. A regular dodecahedron is projected orthogonally onto a plane, and its image is an
n-sided polygon. What is the smallest possible value of n?
Solution: 6
We can achieve 6 by projecting onto a plane perpendicular to an edge of the dodeca-
heron. Indeed, if we imagine viewing the dodecahedron in such a direction, then 4 of
the faces are projected to line segments (namely, the two faces adjacent to the edge
and the two opposite faces), and of the remaining 8 faces, 4 appear on the front of the
dodecahedron and the other 4 are on the back. Thus, the dodecahedron appears as
shown.

To see that we cannot do better, note that, by central symmetry, the number of edges
of the projection must be even. So we just need to show that the answer cannot be 4.
But if the projection had 4 sides, one of the vertices would give a projection forming
an acute angle, which is not possible. So 6 is the answer.

30. We have an n-gon, and each of its vertices is labeled with a number from the set
{1, . . . , 10}. We know that for any pair of distinct numbers from this set there is

8
at least one side of the polygon whose endpoints have these two numbers. Find the
smallest possible value of n.
Solution: 50
Each number be paired with each of the 9 other numbers, but each vertex can be used
in at most 2 different pairs, so each number must occur on at least d9/2e = 5 different
vertices. Thus, we need at least 10 · 5 = 50 vertices, so n ≥ 50.
To see that n = 50 is feasible, let the numbers 1, . . .¡, 10
¢ be the vertices of a complete
graph. Then each vertex has degree 9, and there are 10 2
= 45 edges. If we attach extra
copies of the edges 1-2, 3-4, 5-6, 7-8, and 9-10, then every vertex will have degree 10.
In particular, the graph has an Eulerian tour, so we can follow this tour, successively
numbering vertices of the 50-gon according to the vertices of the graph we visit. Then,
for each edge of the graph, there will be a corresponding edge of the polygon with the
same two vertex labels on its endpoints. It follows that every pair of distinct numbers
occurs at the endpoints of some edge of the polygon, and so n = 50 is the answer.
31. P is a point inside triangle ABC, and lines AP, BP, CP intersect the opposite sides
BC, CA, AB in points D, E, F , respectively. It is given that ∠AP B = 90◦ , and that
AC = BC and AB = BD. We also know that BF = 1, and that BC = 999. Find
AF .
Solution: 499/500

A P
D

C
Let AC = BC = s, AB = BD = t. Since BP is the altitude in isosceles triangle
ABD, it bisects angle B. So, the Angle Bisector Theorem in triangle ABC given
AE/EC = AB/BC = t/s. Meanwhile, CD/DB = (s − t)/t. Now Ceva’s theorem
gives us µ ¶ µ ¶
AF AE CD s−t
= · =
FB EC DB s
AB s−t 2s − t st
⇒ =1+ = ⇒ FB = .
FB s s 2s − t
Now we know s = 999, but we need to find t given that st/(2s − t) = F B = 1. So
st = 2s − t ⇒ t = 2s/(s + 1), and then
AF s−t (s2 − s)/(s + 1) s−1 499
AF = F B · =1· = = = .
FB s s s+1 500

9
32. Define the sequence b0 , b1 , . . . , b59 by
(
1 if i is a multiple of 3
bi =
0 otherwise.

Let {ai } be a sequence of elements of {0, 1} such that

bn ≡ an−1 + an + an+1 (mod 2)

for 0 ≤ n ≤ 59 (a0 = a60 and a−1 = a59 ). Find all possible values of 4a0 + 2a1 + a2 .
Solution: 0, 3, 5, 6
Try the four possible combinations of values for a0 and a1 . Since we can write an ≡
bn−1 −an−2 −an−1 , these two numbers completely determine the solution {ai } beginning
with them (if there is one). For a0 = a1 = 0, we can check that the sequence beginning
0, 0, 0, 0, 1, 1 and repeating every 6 indices is a possible solution for {ai }, so one possible
value for 4a0 + 2a1 + a2 is 0. The other three combinations for a0 and a1 similarly lead
to valid sequences (produced by repeating the sextuples 0, 1, 1, 1, 0, 1; 1, 0, 1, 1, 1, 0;
1, 1, 0, 1, 0, 1, respectively); we thus obtain the values 3, 5, and 6.

33. A plane P slices through a cube of volume 1 with a cross-section in the shape of a
regular hexagon. This cube also has an inscribed sphere, whose intersection with P
is a circle. What is the area of the region inside the regular hexagon but outside the
circle?

Solution: (3 3 − π)/4
One can show that the hexagon must have as its vertices the midpoints of six edges of
the cube, as illustrated; for example, this readily follows from the fact that opposite
sides of the hexagons and the medians
√ between them are parallel. We then conclude
that the side of the hexagon is 2/2 (since
√ it √
cuts off an
√ isosceles triangle of leg 1/2
2
from each face), so the area is (3/2)( 2/2) ( 3) = 3 3/4. Also, the plane passes
through the center of the sphere by symmetry, so it cuts out a cross section of radius
1/2, whose area (which √ is contained entirely inside the hexagon) is then π/4. The
sought area is thus (3 3 − π)/4.

34. Find the number of 20-tuples of integers x1 , . . . , x10 , y1 , . . . , y10 with the following prop-
erties:

• 1 ≤ xi ≤ 10 and 1 ≤ yi ≤ 10 for each i;

10
• xi ≤ xi+1 for i = 1, . . . , 9;
• if xi = xi+1 , then yi ≤ yi+1 .
¡109¢
Solution: 10

By setting zi = 10xi + yi , we see that the problem is equivalent to choosing a nonde-


creasing sequence of numbers z1 , z2 , . . . , z10 from the values 11, 12, . . . , 110. Making a
further substitution by setting wi = zi − 11 + i, we see that the problem is equivalent to
choosing a strictly increasing
¡ ¢ sequence of numbers w1 , . . . , w10 from among the values
1, 2, . . . , 109. There are 109
10
ways to do this.

35. There are eleven positive integers n such that there exists a convex polygon with n
sides whose angles, in degrees, are unequal integers that are in arithmetic progression.
Find the sum of these values of n.
Solution: 106
The sum of the angles of an n-gon is (n − 2)180, so the average angle measure is
(n − 2)180/n. The common difference in this arithmetic progression is at least 1, so
the difference between the largest and smallest angles is at least n − 1. So the largest
angle is at least (n − 1)/2 + (n − 2)180/n. Since the polygon is convex, this quantity is
no larger than 179: (n − 1)/2 − 360/n ≤ −1, so that 360/n − n/2 ≥ 1/2. Multiplying
by 2n gives 720 − n2 ≥ n. So n(n + 1) ≤ 720, which forces n ≤ 26. Of course, since the
common difference is an integer, and the angle measures are integers, (n−2)180/n must
be an integer or a half integer, so (n − 2)360/n = 360 − 720/n is an integer, and then
720/n must be an integer. This leaves only n = 3, 4, 5, 6, 8, 9, 10, 12, 15, 16, 18, 20, 24 as
possibilities. When n is even, (n−2)180/n is not an angle of the polygon, but the mean
of the two middle angles. So the common difference is at least 2 when (n − 2)180/n
is an integer. For n = 20, the middle angle is 162, so the largest angle is at least
162 + 38/2 = 181, since 38 is no larger than the difference between the smallest and
largest angles. For n = 24, the middle angle is 165, again leading to a contradiction.
So no solution exists for n = 20, 24. All of the others possess solutions:

n angles
3 59, 60, 61
4 87, 89, 91, 93
5 106, 107, 108, 109, 110
6 115, 117, 119, 121, 123, 125
8 128, 130, 132, 134, 136, 138, 140, 142
9 136, . . . , 144
10 135, 137, 139, . . . , 153
12 139, 141, 143, . . . , 161
15 149, 150, . . . , 163
16 150, 151, . . . , 165
18 143, 145, . . . , 177

(These solutions are quite easy to construct.) The desired value is then 3 + 4 + 5 + 6 +
8 + 9 + 10 + 12 + 15 + 16 + 18 = 106.

11
36. For a string of P ’s and Q’s, the value is defined to be the product of the positions of
the P ’s. For example, the string P P QP QQ has value 1 · 2 · 4 = 8.
Also, a string is called antipalindromic if writing it backwards, then turning all the
P ’s into Q’s and vice versa, produces the original string. For example, P P QP QQ is
antipalindromic.
There are 21002 antipalindromic strings of length 2004. Find the sum of the reciprocals
of their values.
Solution: 20051002 /2004!
Consider the product
µ ¶µ ¶µ ¶ µ ¶
1 1 1 1 1 1 1 1
+ + + ··· + .
1 2004 2 2003 3 2002 1002 1003
This product expands to 21002 terms, and each term gives the reciprocal of the value
of a corresponding antipalindromic string of P ’s and Q’s as follows: if we choose
the term 1/n for the nth factor, then our string has a P in position n and Q in
position 2005 − n; if we choose the term 1/(2005 − n), then we get a Q in position
n and P in position 2005 − n. Conversely, each antipalindromic string has its value
represented by exactly one of our 21002 terms. So the value of the product is the
number we are looking for. But when we simplify this product, the nth factor becomes
1/n + 1/(2005 − n) = 2005/n(2005 − n). Multiplying these together, we get 1002
factors of 2005 in the numerator and each integer from 1 to 2004 exactly once in the
denominator, for a total of 20051002 /2004!.
Q
37. Simplify 2004
k=1 sin(2πk/4009).

4009
Solution: 22004
k −k Q
Let ζ = e2πi/4009 so that sin(2πk/4009) = ζ −ζ and x4009 − 1 = 4008 − ζ k ). Hence
k=0 (x Q
Q 2i
1 + x + · · · + x4008 =Q 4008 k
k=1 (x − ζ ). Comparing constant coefficients gives
Q
4008 k
k=1 ζ = 1,
4008 4008
setting x = 1 gives k=1 (1 − ζ ) = 4009, and setting x = −1 gives k=1 (1 + ζ k ) = 1.
k

Now, note that sin(2π(4009 − k)/4009) = − sin(2πk/4009), so


Ã2004 !2 4008
Y Y
2004
sin(2πk/4009) = (−1) sin(2πk/4009)
k=1 k=1
4008
Y ζ k − ζ −k
=
k=1
2i
4008
Y ζ 2k − 1
1
=
(2i)4008 k=1 ζ k
4008
Y
1
= (ζ 2k − 1)
24008 k=1
4008
Y
1
= (ζ k − 1)(ζ k + 1)
24008 k=1
4009 · 1
= .
24008

12
However,

sin(x) is nonnegative on the interval [0, π], so our product is positive. Hence
4009
it is 22004 .

38. Let S = {p1 p2 · · · pn | p1 , p2 , . . . , pn are distinct primes and p1 , . . . , pn < 30}. Assume
1 is in S. Let a1 be an element of S. We define, for all positive integers n:

an+1 = an /(n + 1) if an is divisible by n + 1;

an+1 = (n + 2)an if an is not divisible by n + 1.


How many distinct possible values of a1 are there such that aj = a1 for infinitely many
j’s?
Solution: 512
If a1 is odd, then we can see by induction that aj = (j +1)a1 when j is even and aj = a1
when j is odd (using the fact that no even j can divide a1 ). So we have infinitely many
j’s for which aj = a1 .
If a1 > 2 is even, then a2 is odd, since a2 = a1 /2, and a1 may have only one factor of
2. Now, in general, let p = min({p1 , . . . , pn } \ {2}). Suppose 1 < j < p. By induction,
we have aj = (j + 1)a1 /2 when j is odd, and aj = a1 /2 when j is even. So ai 6= a1 for
all 1 < j < p. It follows that ap = a1 /2p. Then, again using induction, we get for all
nonnegative integers k that ap+k = ap if k is even, and ap+k = (p + k + 1)ap if k is odd.
Clearly, ap 6= a1 and p + k + 1 6= 2p when k is odd (the left side is odd, and the right
side even). It follows that aj = a1 for no j > 1. Finally, when a1 = 2, we can check
inductively that aj = j + 1 for j odd and aj = 1 for j even.
So our answer is just the number of odd elements in S. There are 9 odd prime numbers
smaller than 30, so the answer is 29 = 512.

39. You want to arrange the numbers 1, 2, 3, . . . , 25 in a sequence with the following prop-
erty: if n is divisible by m, then the nth number is divisible by the mth number. How
many such sequences are there?
Solution: 24
Let the rearranged numbers be a1 , . . . , a25 . The number of pairs (n, m) with n | m
must equal the number of pairs with an | am , but since each pair of the former type
is also of the latter type, the converse must be true as well. Thus, n | m if and only
if an | am . Now for each n = 1, 2, . . . , 6, the number of values divisible by n uniquely
determines n, so n = an . Similarly, 7, 8 must either be kept fixed by the rearrangement
or interchanged, because they are the only values that divide exactly 2 other numbers
in the sequence; since 7 is prime and 8 is not, we conclude they are kept fixed. Then we
can easily check by induction that n = an for all larger composite numbers n ≤ 25 (by
using m = am for all proper factors m of n) and n = 11 (because it is the only prime
that divides exactly 1 other number). So we have only the primes n = 13, 17, 19, 23
left to rearrange, and it is easily seen that these can be permuted arbitrarily, leaving
4! possible orderings altogether.

40. You would like to provide airline service to the 10 cities in the nation of Schizophrenia,
by instituting a certain number of two-way routes between cities. Unfortunately, the
government is about to divide Schizophrenia into two warring countries of five cities

13
each, and you don’t know which cities will be in each new country. All airplane service
between the two new countries will be discontinued. However, you want to make sure
that you set up your routes so that, for any two cities in the same new country, it will
be possible to get from one city to the other (without leaving the country).
What is the minimum number of routes you must set up to be assured of doing this,
no matter how the government divides up the country?
Solution: 30
Each city C must be directly connected to at least 6 other cities, since otherwise
the government could put C in one country and all its connecting cities in the other
country, and there would be no way out of C. This means that we have 6 routes for
each of 10 cities, counted twice (since each route has two endpoints) ⇒ 6 · 10/2 = 30
routes. On the other hand, this is enough: picture the cities arranged around a circle,
and each city connected to its 3 closest neighbors in either direction. Then if C and D
are in the same country but mutually inaccessible, this means that on each arc of the
circle between C and D, there must be (at least) three consecutive cities in the other
country. Then this second country would have 6 cities, which is impossible. So our
arrangement achieves the goal with 30 routes.

41. A tetrahedron has all its faces triangles with sides 13, 14, 15. What is its volume?

Solution: 42 55
Let ABC be a triangle with AB = 13, BC = 14, CA = 15. Let AD, BE be altitudes.
Then BD = 5, CD = 9. (If you don’t already know this, it can be deduced from the
Pythagorean Theorem: CD2 −BD2 = (CD2 +AD2 )−(BD2 +AD2 ) = AC 2 −AB 2 = 56,
while CD + BD = BC =√14, giving CD − BD = 56/14 = 4, and now solve the linear
system.) Also, AD = AB 2 − BD2 = 12. Similar reasoning gives AE = 33/5,
EC = 42/5.

E
I
G

C F D B

Now let F be the point on BC such that CF = BD = 5, and let G be on AC such


that CG = AE = 33/5. Imagine placing face ABC flat on the table, and letting X
be a point in space with CX = 13, BX = 14. By mentally rotating triangle BCX

14
about line BC, we can see that X lies on the plane perpendicular to BC through F .
In particular, this holds if X is the fourth vertex of our tetrahedron ABCX. Similarly,
X lies on the plane perpendicular to AC through G. Let the mutual intersection of
these two planes and plane ABC be H. Then XH is the altitude of the tetrahedron.
To find XH, extend F H to meet AC at I. Then 4CF I ∼ 4CDA, a 3-4-5 triangle,
so F I = CF · 4/3 = 20/3, and CI = CF · 5/3 = 25/3. Then IG = CI − CG = 26/15,
and HI√ = IG · 5/4 = √ 13/6. This leads√to HF = F I − HI = 9/2, and finally
XH = XF 2 − HF 2 = AD2 − HF 2 = 3 55/2.
Now XABC is a tetrahedron
√ whose base 4ABC has area√AD · BC/2 = 12 √ · 14/2 = 84,
and whose height XH is 3 55/2, so its volume is (84)(3 55/2)/3 = 42 55.

42. S is a set of complex numbers such that if u, v ∈ S, then uv ∈ S and u2 + v 2 ∈ S.


Suppose that the number N of elements of S with absolute value at most 1 is finite.
What is the largest possible value of N ?
Solution: 13
First, if S contained some u 6= 0 with absolute value < 1, then (by the first condition)
every power of u would be in S, and S would contain infinitely many different numbers
of absolute value < 1. This is a contradiction. Now suppose S contains some number
u of absolute value 1 and argument θ. If θ is not an integer multiple of π/6, then u
has some power v whose argument lies strictly between θ + π/3 and θ + π/2. Then
u2 + v 2 = u2 (1 + (v/u)2 ) has absolute value between 0 and 1, since (v/u)2 lies on
the unit circle with angle strictly between 2π/3 and π. But u2 + v 2 ∈ S, so this is a
contradiction.
This shows that the only possible elements of S with absolute value ≤ 1 are 0 and the
points on the unit circle whose arguments are multiples of π/6, giving N ≤ 1+12 = 13.
To show that N = 13 is attainable, we need to show that there exists a possible set S
containing all these points. Let T be the set of all numbers of the form a + bω, where
a, b are integers are ω is a complex cube root of 1. Since ω 2 = −1 − ω, T is closed
under multiplication and addition. Then, if we let S be the set of numbers u such
that u2 ∈ T , S has the required properties, and it contains the 13 complex numbers
specified, so we’re in business.

43. Write down an integer from 0 to 20 inclusive. This problem will be scored as follows: if
N is the second-largest number from among the responses submitted, then each team
that submits N gets N points, and everyone else gets zero. (If every team picks the
same number then nobody gets any points.)
Solution: The only Nash equilibria of this game (where each team plays its best
possible move given the other teams’ choices) are fairly degenerate: every team but
one plays 1, and the remaining team is more likely to choose 2 than any higher number.
Of course, we cannot assume perfectly rational play in reality — nor are the utility
functions the same, since the goal is to score higher than other teams, not to maximize
one’s own expected number of points. It will be interesting to see what the submissions
are.

44. Shown on your answer sheet is a 20 × 20 grid. Place as many queens as you can so
that each of them attacks at most one other queen. (A queen is a chess piece that can

15
move any number of squares horizontally, vertically, or diagonally.) It’s not very hard
to get 20 queens, so you get no points for that, but you get 5 points for each further
queen beyond 20. You can mark the grid by placing a dot in each square that contains
a queen.
Solution: An elementary argument shows there cannot be more than 26 queens: we
cannot have more than 2 in a row or column (or else the middle queen would attack
the other two), so if we had 27 queens, there would be at least 7 columns with more
than one queen and thus at most 13 queens that are alone in their respective columns.
Similarly, there would be at most 13 queens that are alone in their respective rows.
This leaves 27 − 13 − 13 = 1 queen who is not alone in her row or column, and she
therefore attacks two other queens, contradiction.
Of course, this is not a very strong argument since it makes no use of the diagonals.
The best possible number of queens is not known to us; the following construction
gives 23:

45. A binary string of length n is a sequence of n digits, each of which is 0 or 1. The


distance between two binary strings of the same length is the number of positions
in which they disagree; for example, the distance between the strings 01101011 and
00101110 is 3 since they differ in the second, sixth, and eighth positions.
Find as many binary strings of length 8 as you can, such that the distance between
any two of them is at least 3. You get one point per string.
Solution: The maximum possible number of such strings is 20. An example of a set

16
attaining this bound is

00000000 00110101
11001010 10011110
11100001 01101011
11010100 01100110
10111001 10010011
01111100 11001101
00111010 10101100
01010111 11110010
00001111 01011001
10100111 11111111

This example is taken from page 57 of F. J. MacWilliams and N. J. A. Sloane, The


Theory of Error Correcting Codes (New York: Elsevier Publishing, 1977). The proof
that 20 is the best possible is elementary but too long to reproduce here; see pages
537–541 of MacWilliams and Sloane for details.
In general, a set of M strings of length n such that any two have a distance of at least
d is called an (n, M, d)-code. These objects are of basic importance in coding theory,
which studies how to transmit information through a channel with a known error rate.
For example, since the code given above has minimum distance 3, I can transmit to
you a message consisting of strings in this code, and even if there is a possible error
rate of one digit in each string, you will still be able to determine the intended message
uniquely.

17
Harvard-MIT Mathematics Tournament
February 28, 2004

Team Round — Solutions


A Build-It-Yourself Table
An infinite table of nonnegative integers is constructed as follows: in the top row, some
number is 1 and all other numbers are 0’s; in each subsequent row, every number is the sum
of some two of the three closest numbers in the preceding row. An example of such a table
is shown below.
··· 0 0 0 0 1 0 0 0 0 ···
··· 0 0 0 0 1 1 0 0 0 ···
··· 0 0 0 1 1 2 1 0 0 ···
··· 0 0 1 1 3 3 2 0 0 ···
··· 0 1 2 4 4 6 3 2 0 ···
. .. .. .. .. .. .. .. .. .. ..
.. . . . . . . . . . .

The top row (with the one 1) is called row 0; the next row is row 1; the next row is row
2, and so forth.
Note that the following problems require you to prove the statements for every table that
can be constructed by the process described above, not just for the example shown.
1. Show that any number in row n (for n > 0) is at most 2n−1 .
Solution: We use induction on n. It is clear that any number in row 1 is at most
1 = 20 . Now, if every number in row n is at most 2n−1 , then every number in row n + 1
is the sum of two numbers in row n and so is at most 2n−1 + 2n−1 = 2n . This gives the
induction step, and the result follows.
2. What is the earliest row in which the number 2004 may appear?
Solution:
··· 0 0 1 0 0 ···
··· 0 0 1 1 0 ···
··· 0 1 2 2 0 ···
··· 0 3 4 4 0 ···
··· 0 7 8 8 0 ···
··· 0 15 16 16 0 ···
··· 0 31 31 32 0 ···
··· 0 62 63 63 0 ···
··· 0 125 125 126 0 ···
··· 0 250 251 251 0 ···
··· 0 501 501 502 0 ···
··· 0 1002 1002 1003 0 ···
··· 0 2004 2004 2005 0 ···

By the previous problem, it cannot appear before row 12. By starting off the table as
shown above, we see that row 12 is possible, so this is the answer.

1
3. Let µ ¶ µ ¶ µ ¶ µ ¶
n−1 n−1 n−1 n−1
S(n, r) = + + + ··· +
r−1 r r+1 n−1
for all n, r > 0, and in particular S(n, r) = 0 if r > n > 0. Prove that the number in
row n of the table, r columns to the left of the 1 in the top row, is at most S(n, r).
(Hint: First prove that S(n − 1, r − 1) + S(n − 1, r) = S(n, r).)
Solution: First, we prove the statement in the hint: adding the ith term of the sum
for S(n − 1, r − 1) to the ith term for S(n − 1, r), for each i, we get that S(n − 1, r −
1) + S(n − 1, r) equals
µµ ¶ µ ¶¶ µµ ¶ µ ¶¶
n−2 n−2 n−2 n−2
+ + + + ···
r−2 r−1 r−1 r
µµ ¶ µ ¶¶ µ ¶
n−2 n−2 n−2
··· + + +
n−3 n−2 n−2
µ ¶ µ ¶ µ ¶
n−1 n−1 n−1
= + + ··· + = S(n, r).
r−1 r n−1

Now we can prove the main statement by induction on n. The base case n = 1 is clear.
If the statement holds for n − 1, then first suppose r > 1. Then the number in row n,
r columns to the left, is the sum of two of the three numbers above it, which, by the
induction hypothesis, are at most S(n−1, r −1), S(n−1, r), S(n−1, r +1) respectively.
Since the first two of these are greater than
¡n−1 ¢ the last (because the summation formula
¡ ¢
gives S(n − 1, r − 1) = S(n − 1, r) + r−2 and S(n − 1, r) = S(n − 1, r + 1) + n−1 r−1
),
we have an upper bound of S(n − 1, r − 1) + S(n − 1, r) = S(n, r) by the above. So
the result follows by induction. Finally, in the case r = 1, the quantity in question is
just 2n−1 , and the result holds by Problem 1.
4. Show that the sum of all the numbers in row n is at most (n + 2)2n−1 .
Solution: The previous problem gives an upper bound on the number located r
columns to the left of the initial 1; adding over all r = 1, 2, . . . , n gives
n−1
X µ ¶
n−1
(s + 1)
s=0
s
¡ ¢
since the term n−1s
occurs for the s + 1 values r = 1, 2, . . . , s + 1. But this sum equals
n−2
(n + 1)2 . For example, add the sum to itself and reverse the terms of the second
sum to get
n−1
X µ ¶ X n−1 µ ¶
n−1 n−1
(s + 1) + ([n − 1 − s] + 1)
s=0
s s=0
n−1−s
Xn−1 µ ¶ Xn−1 µ ¶
n−1 n−1
= (n + 1) = (n + 1) = (n + 1)2n−1 ,
s=0
s s=0
s
and our original sum is half of this.
So the sum of the terms in row n to the left of the central column is at most (n+1)2n−2 .
Similarly, the sum of the terms to the right of the central column is at most (n+1)2n−2 .
Adding these together, plus the upper bound of 2n−1 for the central number (Problem
1), gives our upper bound of (n + 2)2n−1 for the sum of all the numbers in the row.

2
A pair of successive numbers in the same row is called a switch pair if one number in the
pair is even and the other is odd.

5. Prove that the number of switch pairs in row n is at most twice the number of odd
numbers in row n.
Solution: Each switch pair contains an odd number, and each odd number can belong
to at most two switch pairs (since it has only two neighbors).

6. Prove that the number of odd numbers in row n is at most twice the number of switch
pairs in row n − 1.
Solution: Each odd number in row n is the sum of two of the three numbers above it
in row n − 1; these three numbers cannot all have the same parity (or else any sum of
two of them would be even), so somewhere among them is a switch pair. Since each
switch pair in row n − 1 can contribute to at most two odd numbers in row n in this
manner (namely, the two numbers immediately below the members of the pair), the
result follows.

7. Prove that the number of switch pairs in row n is at most twice the number of switch
pairs in row n − 1.
Solution: If we go sufficiently far to the left or right in row n, we get to zeroes.
Therefore, row n consists of a finite number of “odd blocks” of the form

EOOO · · · OE

(where E represents an even number and O an odd number), which are separated by
even numbers, except that one even number may simultaneously be an endpoint of two
odd blocks. Each odd block contributes two switch pairs to row n, so it is enough to
show that each odd block has a switch pair somewhere above it in row n − 1. But the
odd block consists of at least one O between two E’s, making for at least three terms.
If there were no switch pairs above the block, then in particular, the first three terms
immediately above it would be all odd or all even, and then the second term in our
block would have to be even, contradicting the assumption that it was O. This proves
the result.

Written In The Stars

Suppose S is a finite set with a binary operation ? — that is, for any elements a, b of S,
there is defined an element a ? b of S. It is given that (a ? b) ? (a ? b) = b ? a for all a, b ∈ S.

8. Prove that a ? b = b ? a for all a, b ∈ S.


Solution: We have

a?b = (b ? a) ? (b ? a)
= ([a ? b] ? [a ? b]) ? ([a ? b] ? [a ? b])
= [a ? b] ? [a ? b]
= b ? a.

3
Let T be the set of elements of the form a ? a for a ∈ S.

9. If b is any element of T , prove that b ? b = b.


Solution: If b ∈ T , then b = a ? a for some a, so b ? b = (a ? a) ? (a ? a) = a ? a (by
the given property) = b.

Now suppose further that (a ? b) ? c = a ? (b ? c) for all a, b, c ∈ S. (Thus we can write an


expression like a ? b ? c ? d without ambiguity.)

10. Let a be an element of T . Let the image of a be the set of all elements of T that can be
represented as a ? b for some b ∈ T . Prove that if c is in the image of a, then a ? c = c.
Solution: Write c = a ? b, and then a ? c = a ? a ? b = a ? b (since a ? a = a) = c.

11. Prove that there exists an element a ∈ T such that the equation a ? b = a holds for all
b ∈ T.
Solution: Choose a whose image contains as few elements as possible — we know
we can do this, since T , being a subset of S, is finite. We claim that this a works.
Indeed, suppose c is in the image of a. Then, for any d in the image of c, a ? (c ? d) =
(a ? c) ? d = c ? d = d, so d is also in the image of a. So the image of c is contained in
the image of a. But a was chosen to have image as small as possible, so the two images
must coincide. In particular, a ? a = a is in the image of c. So

a = c ? a = a ? c = c.

This argument shows that a is the only element of the image of a, which gives what
we wanted.
Alternative Solution: This can also be solved without using Problem 10: The
product of any two elements of T is also in T , since commutativity and associativity
give (b ? b) ? (c ? c) = (b ? c) ? (b ? c) for b, c ∈ S. Then let a1 , a2 , . . . , an be all the
elements of T , and put a = a1 ? a2 ? · · · ? an ; this value does not depend on the ordering
of the elements. If b ∈ T , then a = c ? b, where c is the ?-product of all elements of T
different from b, and consequently a ? b = (c ? b) ? b = c ? (b ? b) = c ? b = a.

12. Prove that there exists an element a ∈ S such that the equation a ? b = a holds for all
b ∈ S.
Solution: The same a as in the previous problem will do the trick. Indeed, for any
b ∈ S, we have
a ? b = (b ? a) ? (b ? a) = (a ? a) ? (b ? b)
(we have used commutativity and associativity). But a ? a = a, and b ? b ∈ T , so this
expression equals a ? (b ? b) = a, as required.

Sigma City

13. Let n be a positive odd integer. Prove that

blog2 nc + blog2 (n/3)c + blog2 (n/5)c + blog2 (n/7)c + · · · + blog2 (n/n)c = (n − 1)/2.

4
Solution: Note that blog2 kc is the cardinality of the set {2, 4, 8, . . . , 2blog2 kc }, i.e.,
the number of powers of 2 that are even and are at most k. Then blog2 (n/k)c is the
number of even powers of 2 that are at most n/k, or equivalently (multiplying each such
number by k) the number of positive even numbers ≤ n whose greatest odd divisor is
k. Summing over all odd k, we get the number of even numbers ≤ n, which is just
(n − 1)/2.

Let σ(n) denote the sum of the (positive) divisors of n, including 1 and n itself.

14. Prove that


σ(1) + σ(2) + σ(3) + · · · + σ(n) ≤ n2
for every positive integer n.
Solution: The ith term on the left is the sum of all d dividing i. If we write this sum
out explicitly, then each term d = 1, 2, . . . , n appears bn/dc times — once for each
multiple of d that is ≤ n. Thus, the sum equals

bn/1c + 2 bn/2c + 3 bn/3c + · · · + n bn/nc ≤ n/1 + 2n/2 + 3n/3 + · · · + n/n


= n + n + ··· + n
= n2 .

15. Prove that


σ(1) σ(2) σ(3) σ(n)
+ + + ··· + ≤ 2n
1 2 3 n
for every positive integer n.
Solution: This is similar to the previous solution. If d is a divisor of i, then so is i/d,
and (i/d)/i = 1/d. Summing over all d, we see that σ(i)/i is the sum of the reciprocals
of the divisors of i, for each positive integer i. So, summing over all i from 1 to n, we
get the value 1/d appearing bn/dc times, once for each multiple of d that is at most n.
In particular, the sum is
1 jnk 1 jnk 1 jnk 1 jnk n n n
+ + + ··· + < 2 + 2 + ··· + 2.
1 1 2 2 3 3 n n 1 2 n
So now all we need is 1/12 + 1/22 + · · · + 1/n2 < 2. This can be obtained from the
classic formula 1/12 + 1/22 + · · · = π 2 /6, or from the more elementary estimate
¡ ¢
1/22 + 1/32 + · · · + 1/n2 < 1/(1 · 2) + 1/(2 · 3) + · · · + 1/ (n − 1) · n
¡ ¢
= (1/1 − 1/2) + (1/2 − 1/3) + · · · + 1/(n − 1) − 1/n
= 1 − 1/n
< 1.

16. Now suppose again that n is odd. Prove that

σ(1) blog2 nc + σ(3) blog2 (n/3)c + σ(5) blog2 (n/5)c + · · · + σ(n) blog2 (n/n)c < n2/8.

Solution: The term σ(i) blog2 (n/i)c is the sum of the divisors of i times the number
of even numbers ≤ n whose greatest odd divisor is i. Thus, summing over all odd i,

5
we get the sum of d over all pairs (d, j), where j < n is even and d is an odd divisor
of j. Each odd number d then appears bn/2dc times, since this is the number of even
numbers < n that have d as a divisor. So the sum equals

bn/2c + 3 bn/6c + 5 bn/10c + · · · + n bn/2nc


≤ (n − 1)/2 + 3(n − 1)/6 + · · · + m(n − 1)/2m,

where m is the greatest odd integer less than n/2. (We can ignore the terms d bn/2dc
for d > m because these floors are zero.) This expression equals

(n − 1)/2 + (n − 1)/2 + · · · + (n − 1)/2 = (n − 1)(m + 1)/4 ≤ (n − 1)(n + 1)/8,

which is less than n2 /8, as required.

6
Harvard-MIT Mathematics Tournament
February 19, 2005

Individual Round: Algebra Subject Test — Solutions

1. How many real numbers x are solutions to the following equation?

|x − 1| = |x − 2| + |x − 3|

Solution: 2
If x < 1, the equation becomes (1 − x) = (2 − x) + (3 − x) which simplifies to x = 4,
contradicting the assumption x < 1. If 1 ≤ x ≤ 2, we get (x − 1) = (2 − x) + (3 − x),
which gives x = 2. If 2 ≤ x ≤ 3, we get (x − 1) = (x − 2) + (3 − x), which again gives
x = 2. If x ≥ 3, we get (x − 1) = (x − 2) + (x − 3), or x = 4. So 2 and 4 are the only
solutions, and the answer is 2.

2. How many real numbers x are solutions to the following equation?

2003x + 2004x = 2005x

Solution: 1
Rewrite the equation as (2003/2005)x + (2004/2005)x = 1. The left side is strictly
decreasing in x, so there cannot be more than one solution. On the other hand, the
left side equals 2 > 1 when x = 0 and goes to 0 when x is very large, so it must equal
1 somewhere in between. Therefore there is one solution.

3. Let x, y, and z be distinct real numbers that sum to 0. Find the maximum possible
value of
xy + yz + zx
.
x2 + y 2 + z 2

Solution: −1/2
Note that 0 = (x + y + z)2 = x2 + y 2 + z 2 + 2xy + 2yz + 2zx. Rearranging, we get that
xy + yz + zx = − 12 (x2 + y 2 + z 2 ), so that in fact the quantity is always equal to −1/2.
¥ ¦ ¥ b+c ¦ ¥ c+a ¦
4. If a, b, c > 0, what is the smallest possible value of a+b c
+ a + b ? (Note that
bxc denotes the greatest integer less than or equal to x.)
Solution: 4
Since bxc > x − 1 for all x, we have that
¹ º ¹ º ¹ º
a+b b+c c+a a+b b+c c+a
+ + > + + −3
c a b c a b
µ ¶ µ ¶ ³
a b b c c a´
= + + + + + − 3.
b a c b a c
But by the AM-GM inequality, each of the first three terms in the last line is at least
2. Therefore, the lefthand side is greater than 2 + 2 + 2 − 3 = 3. Since it is an integer,
the smallest value it can be is 4. This is in fact attainable by letting (a, b, c) = (6, 8, 9).

1
5. Ten positive integers are arranged around a circle. Each number is one more than the
greatest common divisor of its two neighbors. What is the sum of the ten numbers?
Solution: 28
First note that all the integers must be at least 2, because the greatest common divisor
of any two positive integers is at least 1. Let n be the largest integer in the circle.
The greatest common divisor of its two neighbors is n − 1. Therefore, each of the two
neighbors is at least n − 1 but at most n, so since n − 1 - n for n − 1 ≥ 2, they must
both be equal to n − 1. Let m be one of the numbers on the other side of n − 1 from
n. Then gcd(n, m) = n − 2. Since n − 2 ≥ 0, n − 2 | n only for n = 3 or 4. If n = 3,
each number must be 2 or 3, and it is easy to check that there is no solution. If n = 4,
then it is again not hard to find that there is a unique solution up to rotation, namely
4322343223. The only possible sum is therefore 28.

6. Find the sum of the x-coordinates of the distinct points of intersection of the plane
curves given by x2 = x + y + 4 and y 2 = y − 15x + 36.
Solution: 0
Substituting y = x2 − x − 4 into the second equation yields

0 = (x2 − x − 4)2 − (x2 − x − 4) + 15x − 36


= x4 − 2x3 − 7x2 + 8x + 16 − x2 + x + 4 + 15x − 36
= x4 − 2x3 − 8x2 + 24x − 16
= (x − 2)(x3 − 8x + 8) = (x − 2)2 (x2 + 2x − 4).

This quartic has three distinct real roots at x = 2, −1 ± 5. Each of these yields a
distinct point of intersection, so the answer is their sum, 0.

10

-4 -2 2 4

-5

-10

7. Let x be a positive real number. Find the maximum possible value of



x2 + 2 − x4 + 4
.
x

2

Solution: 2 2−2
Rationalizing the numerator, we get
√ √
x2 + 2 − x4 + 4 x2 + 2 + x4 + 4 (x2 + 2)2 − (x4 + 4)
· √ = √
x x2 + 2 + x4 + 4 x(x2 + 2 + x4 + 4)
4x2
= √
x(x2 + 2 + x4 + 4)
4
= 1 √
(x 2+2+ x4 + 4)
x
4
= q .
x + x2 + x2 + x42

Since we wish to maximize


√ this quantity, we wish to minimize the denominator.
√ By
2 2 4
AM-GM, x + x ≥ 2 2 and x + x2 ≥ 4, so that the denominator is at least 2 2 + 2.
Therefore, √
x2 + 2 − x4 + 4 4 √
≤ √ = 2 2 − 2,
x 2 2+2

with equality when x = 2.

8. Compute

X n
.
n=0
n4 + n2 + 1

Solution: 1/2
Note that

n4 + n2 + 1 = (n4 + 2n2 + 1) − n2 = (n2 + 1)2 − n2 = (n2 + n + 1)(n2 − n + 1).

Decomposing into partial fractions, we find that


µ ¶
n 1 1 1
= − .
n4 + n2 + 1 2 n2 − n + 1 n2 + n + 1
1 1 1
Now, note that if f (n) = n2 −n+1
, then f (n + 1) = (n+1)2 −(n+1)+1
= n2 +n+1
. It follows
that

1³ ´

X n
= (f (0) − f (1)) + (f (1) − f (2)) + (f (2) − f (3)) + · · · .
n=0
n4 + n2 + 1 2

Since f (n) tends towards 0 as n gets large, this sum telescopes to f (0)/2 = 1/2.

9. The number 27,000,001 has exactly four prime factors. Find their sum.
Solution: 652

3
First, we factor

27x6 + 1 = (3x2 )3 + 1
= (3x2 + 1)(9x4 − 3x2 + 1)
= (3x2 + 1)((9x4 + 6x2 + 1) − 9x2 )
= (3x2 + 1)((3x2 + 1)2 − (3x)2 )
= (3x2 + 1)(3x2 + 3x + 1)(3x2 − 3x + 1).

Letting x = 10, we get that 27000001 = 301 · 331 · 271. A quick check shows that
301 = 7 · 43, so that 27000001 = 7 · 43 · 271 · 331. Each factor here is prime, and their
sum is 652.

10. Find the sum of the absolute values of the roots of x4 − 4x3 − 4x2 + 16x − 8 = 0.
√ √
Solution: 2 + 2 2 + 2 3

x4 − 4x3 − 4x2 + 16x − 8 = (x4 − 4x3 + 4x2 ) − (8x2 − 16x + 8)


= x2 (x − 2)2 − 8(x − 1)2
√ √
= (x2 − 2x)2 − (2 2x − 2 2)2
√ √ √ √
= (x2 − (2 + 2 2)x + 2 2)(x2 − (2 − 2 2)x − 2 2).
√ √
But noting that (1 + 2)2 = 3 + 2 2 and completing the square,
√ √ √ √
x2 − (2 + 2 2)x + 2 2 = x2 − (2 + 2 2)x + 3 + 2 2 − 3
√ √
= (x − (1 + 2))2 − ( 3)2
√ √ √ √
= (x − 1 − 2 + 3)(x − 1 − 2 − 3).

Likewise,
√ √ √ √ √ √
x2 − (2 − 2 2)x − 2 2 = (x − 1 + 2 + 3)(x − 1 + 2 − 3),
√ √
so the
√ roots
√ of the quartic are 1 ± 2 ± 3. Only one of these is negative, namely
1 − 2 − 3, so the sum of the absolute values of the roots is
√ √ √ √ √ √ √ √ √ √
(1 + 2 + 3) + (1 + 2 − 3) + (1 − 2 + 3) − (1 − 2 − 3) = 2 + 2 2 + 2 3.

4
Harvard-MIT Mathematics Tournament
February 19, 2005

Individual Round: Calculus Subject Test — Solutions

1. Let f (x) = x3 + ax + b, with a 6= b, and suppose the tangent lines to the graph of f at
x = a and x = b are parallel. Find f (1).
Solution: 1
Since f 0 (x) = 3x2 + a, we must have 3a2 + a = 3b2 + a. Then a2 = b2 , and since a 6= b,
a = −b. Thus f (1) = 1 + a + b = 1.
R ∞ cos u R
2. A plane curve is parameterized by x(t) = t u
du and y(t) = t∞ sinu u du for 1 ≤ t ≤
2. What is the length of the curve?
Solution: ln 2
dx dy
By the Second Fundamental Theorem of Calculus, dt
= − cost t and dt
= − sint t . There-
fore, the length of the curve is
v s
Z 2uà ! à !2 Z 2 Z 2
u dx 2 dy cos2 t sin2 t 1
t
+ dt = 2
+ 2 dt = dt = [ln t]21 = ln 2.
1 dt dt 1 t t 1 t

R1 R1
3. Let f : R → RR be a continuous function with 0 f (x)f 0 (x)dx = 0 and 0 f (x)2 f 0 (x)dx =
18. What is 01 f (x)4 f 0 (x)dx?
Solution: 486/5
Z 1 Z f (1)
0 1
0= f (x)f (x)dx = u du = (f (1)2 − f (0)2 ), and
0 f (0) 2
Z 1 Z f (1)
1
18 = f (x)2 f 0 (x)dx = u2 du = (f (1)3 − f (0)3 ).
0 f (0) 3
The first equation implies f (0) = ±f (1). The second equation shows that f (0) 6= f (1),
and in fact 54 = f (1)3 − f (0)3 = 2f (1)3 , so f (1) = 3 and f (0) = −3. Then
Z 1 Z f (1)
4 0 1 1 486
f (x) f (x)dx = u4 du = (f (1)5 − f (0)5 ) = (243 + 243) = .
0 f (0) 5 5 5

4. Let f : R → R be a smooth function such that f 0 (x)2 = f (x)f 00 (x) for all x. Suppose
f (0) = 1 and f (4) (0) = 9. Find all possible values of f 0 (0).

Solution: ± 3
Let f 0 (0) = a. Then the equation gives f 00 (0) = a2 . Differentiating the given equation
gives
2f 0 (x)f 00 (x) = f (x)f 000 (x) + f 0 (x)f 00 (x),
or f 0 (x)f 00 (x) = f (x)f 000 (x). Differentiating once more gives

f 0 (x)f 000 (x) + f 00 (x)2 = f (x)f (4) (x) + f 0 (x)f 000 (x)

1

or f 00 (x)2 = f (x)f (4) (x), giving

9 = f (4)
(0) = a 4
. Thus a = ± 3. These are indeed
±x 3
both attainable by f (x) = e .
00 0
Alternative Solution: Rewrite the given equation as ff 0 (x) (x)
= ff (x)
(x)
. Integrating both
0 0
sides gives ln f (x) = ln f (x) + C1 , and exponentiating gives f (x) = Cf (x). This has
solution f (x) = AeCx for constants A and C. Since f (0)√= 1, A = 1, and differentiating
we find that C 4 = f (4) (0) = 9, yielding f 0 (0) = C = ± 3.
5. Calculate ³ ´
x
lim+ xx − xx .
x→0

Solution: −1
We first calculate limx→0+ xx : it is just exp(limx→0+ x ln x). But
ln x 1/x
lim+ x ln x = lim+ = lim+ = lim −x = 0
x→0 x→0 1/x x→0 −1/x2 x→0+
x
by L’Hôpital’s
³ x Rule.
´ Therefore limx→0+ xx = 1. Then limx→0+ xx = 01 = 0, so
limx→0+ xx − xx = −1.
6. The graph of r = 2 + cos 2θ and its reflection over the line y = x bound five regions in
the plane. Find the area of the region containing the origin.
Solution: 9π/2 − 8

The original graph is closer to the origin than its reflection for θ ∈ ( π4 , 3π
4
) ∪ ( 5π
4
, 7π
4
),
and the region is symmetric about the origin. Therefore the area we wish to find is
the polar integral
Z 3π Z 3π
4 1 4
4 (2 + cos 2θ)2 dθ = 2 π (4 + 4 cos 2θ + cos2 2θ)dθ
π
4
2 4
Z 3π µ ¶
4 1
= 2 π 4 + 4 cos 2θ + (1 + cos 4θ) dθ
4
2
· ¸ 3π
1 4
= 9θ + 4 sin 2θ + sin 4θ
4 π
4
µ ¶ µ ¶
27π 9π 9π
= −4 − +4 = − 8.
4 4 2

2
7. Two ants, one starting at (−1, 1), the other at (1, 1), walk to the right along the
parabola y = x2 such that their midpoint moves along the line y = 1 with constant
speed 1. When the left ant first hits the line y = 12 , what is its speed?

Solution: 3 3 − 3
When the left ant first√ hits the line y = 12 , the right ant

hits the line y = 32 . The
left ant is then at (− 22 , 12 ), and the right ant is at ( 26 , 32 ). Let the left ant have
velocity with components vx√and vy , the right√ ant velocity with components wx and
dy vy wy
wy . Since dx = 2x, vx = − 2 and wx = 6. Since the midpoint of the ants moves
at speed 1 along the line y = 1, 12 (vx + wx ) = 1 and 12 (vy + w√y ) = 0. Therefore,
√ √ √ √
2vx = −vy = wy = 6wx = 6(2 − vx ). Solving for vx gives √26+√ 6
2
= 3 − 3. Then
the speed of the left ant is
q q √ √ √
vx2 + vy2 = vx2 + (− 2vx )2 = 3|vx | = 3 3 − 3.

8. If f is a continuous real function such that


R 2005
f (x − 1) + f (x + 1) ≥ x + f (x) for all x,
what is the minimum possible value of 1 f (x)dx?
Solution: 2010012
Let g(x) = f (x) − x. Then

g(x − 1) + x − 1 + g(x + 1) + x + 1 ≥ x + g(x) + x,

or g(x − 1) + g(x + 1) ≥ g(x). But now,

g(x + 3) ≥ g(x + 2) − g(x + 1) ≥ −g(x).

Therefore
Z a+6 Z a+3 Z a+6
g(x)dx = g(x)dx + g(x)dx
a a a+3
Z a+3
= (g(x) + g(x + 3))dx ≥ 0.
a

It follows that
Z 2005 333 Z 6n+7
X
g(x) = g(x)dx ≥ 0,
1 n=0 6n+1

so that
Z 2005 Z 2005 Z 2005 " #2005
x2 20052 − 1
f (x)dx = (g(x) + x)dx ≥ x dx = = = 2010012.
1 1 1 2 1
2

Equality holds for f (x) = x.

9. Compute

X 4
.
k=0 (4k)!

Solution: e + 1/e + 2 cos 1

3
This is the power series
4x4 4x8
4+ + + ···
4! 8!
evaluated at x = 1. But this power series can be written as the sum
à !
x x2 x3 x4 x5 x6 x7
1+ + + + + + + + ···
1! 2! 3! 4! 5! 6! 7!
à !
x x2 x3 x4 x5 x6 x7
+ 1− + − + − + − + ···
1! 2! 3! 4! 5! 6! 7!
à !
x2 x4 x6
+ 2 1− + − + ···
2! 4! 6!
= ex + e−x + 2 cos x.

It follows that the quantity is e + 1/e + 2 cos 1.

10. Let f : R → R be a smooth function such that f 0 (x) = f (1 − x) for all x and f (0) = 1.
Find f (1).
Solution: sec 1 + tan 1
Differentiating the given equation gives f 00 (x) = −f (x). This has solutions of the form
A cos(x) + B sin(x). Since f (0) = 1, A = 1. Then f 0 (x) = B cos(x) − sin(x) and

f (1 − x) = cos(1 − x) + B sin(1 − x)
= cos 1 cos x + sin 1 sin x + B sin 1 cos x − B cos 1 sin x
= (cos 1 + B sin 1) cos x + (sin 1 − B cos 1) sin x.

Therefore, B = cos 1+B sin 1 and −1 = sin 1−B cos 1, both of which yield as solutions
cos 1 1 + sin 1
B= = = sec 1 + tan 1.
1 − sin 1 cos 1

4
Harvard-MIT Mathematics Tournament
February 19, 2005

Individual Round: Combinatorics Subject Test — Solutions

1. A true-false test has ten questions. If you answer five questions “true” and five “false,”
your score is guaranteed to be at least four. How many answer keys are there for which
this is true?
Solution: 22
Suppose that either nine or ten of the questions have the same answer. Then no matter
which five questions we pick to have this answer, we will be right at least four times.
Conversely, suppose that there are at least two questions with each answer; we will
show that we can get a score less than four. By symmetry, assume there are at least
five questions whose answer is true. Then if we label five of these false, not only will
we get these five wrong, but we will also have answered all the false questions with
true, for a total of at least seven incorrect. There are 2 ways for all the questions to
have the same answer, and 2 · 10 = 20 ways for one question to have a different answer
from the others, for a total of 22 ways.

2. How many nonempty subsets of {1, 2, 3, . . . , 12} have the property that the sum of the
largest element and the smallest element is 13?
Solution: 1365
If a is the smallest element of such a set, then 13 − a is the largest element, and
for the remaining elements we may choose any (or none) of the 12 − 2a elements
a + 1, a + 2, . . . , (13 − a) − 1. Thus there are 212−2a such sets whose smallest element
is a. Also, 13 − a ≥ a clearly implies a < 7. Summing over all a = 1, 2, . . . , 6, we get a
total of

210 + 28 + 26 + · · · + 20 = 45 + 44 + · · · + 40 = (46 − 1)/(4 − 1) = 4095/3 = 1365

possible sets.

3. The Red Sox play the Yankees in a best-of-seven series that ends as soon as one team
wins four games. Suppose that the probability that the Red Sox win Game n is n−1 6
.
What is the probability that the Red Sox will win the series?
Solution: 1/2
Note that if we imagine that the series always continues to seven games even after one
team has won four, this will never change the winner of the series. Notice also that
the probability that the Red Sox will win Game n is precisely the probability that the
Yankees will win Game 8 − n. Therefore, the probability that the Yankees win at least
four games is the same as the probability that the Red Sox win at least four games,
namely 1/2.

4. In how many ways can 4 purple balls and 4 green balls be placed into a 4 × 4 grid such
that every row and column contains one purple ball and one green ball? Only one ball
may be placed in each box, and rotations and reflections of a single configuration are
considered different.

1
Solution: 216
There are 4! = 24 ways to place the four purple balls into the grid. Choose any purple
ball, and place two green balls, one in its row and the other in its column. There
are four boxes that do not yet lie in the same row or column as a green ball, and at
least one of these contains a purple ball (otherwise the two rows containing green balls
would contain the original purple ball as well as the two in the columns not containing
green balls). It is then easy to see that there is a unique way to place the remaining
green balls. Therefore, there are a total of 24 · 9 = 216 ways.

5. Doug and Ryan are competing in the 2005 Wiffle Ball Home Run Derby. In each round,
each player takes a series of swings. Each swing results in either a home run or an out,
and an out ends the series. When Doug swings, the probability that he will hit a home
run is 1/3. When Ryan swings, the probability that he will hit a home run is 1/2. In
one round, what is the probability that Doug will hit more home runs than Ryan hits?
Solution: 1/5
Denote this probability by p. Doug hits more home runs if he hits a home run on his
first try when Ryan does not, or if they both hit home runs on their first try and Doug
hits more home runs thereafter. The probability of the first case occurring is 13 · 12 = 16 ,
and the probability of the second case occurring is 13 · 12 · p = p6 . Therefore p = 61 + p6 ,
which we solve to find p = 15 .

6. Three fair six-sided dice, each numbered 1 through 6, are rolled. What is the probability
that the three numbers that come up can form the sides of a triangle?
Solution: 37/72
Denote this probability by p, and let the three numbers that come up be x, y, and z. We
will calculate 1−p instead: 1−p is the probability that x ≥ y+z, y ≥ z+x, or z ≥ x+y.
Since these three events are mutually exclusive, 1 − p is just 3 times the probability
that x ≥ y + z. This happens with probability (0 + 1 + 3 + 6 + 10 + 15)/216 = 35/216,
so the answer is 1 − 3 · (35/216) = 1 − 35/72 = 37/72.

7. What is the maximum number of bishops that can be placed on an 8 × 8 chessboard


such that at most three bishops lie on any diagonal?
Solution: 38
If the chessboard is colored black and white as usual, then any diagonal is a solid color,
so we may consider bishops on black and white squares separately. In one direction,
the lengths of the black diagonals are 2, 4, 6, 8, 6, 4, and 2. Each of these can have
at most three bishops, except the first and last which can have at most two, giving a
total of at most 2 + 3 + 3 + 3 + 3 + 3 + 2 = 19 bishops on black squares. Likewise there
can be at most 19 bishops on white squares for a total of at most 38 bishops. This is
indeed attainable as in the diagram below.

2
X X X X X X X X
X X X X X X X
X X
X X
X X
X X
X X X X X X X
X X X X X X X X

8. Every second, Andrea writes down a random digit uniformly chosen from the set
{1, 2, 3, 4}. She stops when the last two numbers she has written sum to a prime
number. What is the probability that the last number she writes down is 1?
Solution: 15/44
Let pn be the probability that the last number she writes down is 1 when the first
number she writes down is n. Suppose she starts by writing 2 or 4. Then she can
continue writing either 2 or 4, but the first time she writes 1 or 3, she stops. Therefore
p2 = p4 = 12 . Suppose she starts by writing 1. Then she stops if she writes 1,
2, or 4, but continues if she writes 3. Therefore p1 = 14 (1 + p3 ). If she starts by
writing 3, then she stops if she writes 2 or 4 and otherwise continues. Therefore
p3 = 14 (p1 + p3 ) = 16
1 1
(1 + 5p3 ). Solving gives p3 = 11 3
and p1 = 11 . The probability we
1 15
want to find is therefore 4 (p1 + p2 + p3 + p4 ) = 44 .
9. Eight coins are arranged in a circle heads up. A move consists of flipping over two
adjacent coins. How many different sequences of six moves leave the coins alternating
heads up and tails up?
Solution: 7680
Imagine we flip over two adjacent coins by pushing a button halfway between them.
Then the outcome depends only on the parities of the number of times that each button
is pushed. To flip any coin, we must push the two buttons adjacent to that coin a total
of an odd number of times. To flip every other coin, the parities must then progress
around the circle as even, even, odd, odd, even, even, odd, odd. There are 4 ways to
assign these parities. If we assume each button is pressed either once or not at all,
this accounts for only four presses, so some button is also pressed twice more. Suppose
this button was already pushed once. There are 4 of these, and the number of possible
sequences of presses is then 6!/3! = 120. Suppose it has not already been pressed.
There are 4 of these as well, and the number of possible sequences is 6!/2! = 360. The
total number of sequences is then 4(4 · 120 + 4 · 360) = 7680.
10. You start out with a big pile of 32004 cards, with the numbers 1, 2, 3, . . . , 32004 written
on them. You arrange the cards into groups of three any way you like; from each group,
you keep the card with the largest number and discard the other two. You now again
arrange these 32003 remaining cards into groups of three any way you like, and in each
group, keep the card with the smallest number and discard the other two. You now
have 32002 cards, and you again arrange these into groups of three and keep the largest
number in each group. You proceed in this manner, alternating between keeping the
largest number and keeping the smallest number in each group, until you have just one
card left.

3
How many different values are possible for the number on this final card?
Solution: 32004 − 2 · 31002 + 2
We claim that if you have cards numbered 1, 2, . . . , 32n and perform 2n successive
grouping operations, then c is a possible value for your last remaining card if and only
if
3n ≤ c ≤ 32n − 3n + 1.
This gives 32n − 2 · 3n + 2 possible values of c, for a final answer of 32004 − 2 · 31002 + 2.
Indeed, notice that the last remaining card c must have been the largest of some set of
three at the (2n − 1)th step; each of these was in turn the largest of some set of three
(and so c was the largest of some set of 9 cards) remaining at the (2n − 3)th step; each
of these was in turn the largest of some set of three (and so c was the largest of some
set of 27) remaining at the (2n − 5)th step; continuing in this manner, we get that c
was the largest of some 3n cards at the first step, so c ≥ 3n . A similar analysis of all
of the steps in which we save the smallest card gives that c is the smallest of some set
of 3n initial cards, so c ≤ 32n − 3n + 1.
To see that any c in this interval is indeed possible, we will carry out the groupings
inductively so that, after 2i steps, the following condition is satisfied: if the numbers
remaining are a1 < a2 < · · · < a32(n−i) , then c is one of these, and there are at least
3n−i − 1 numbers smaller than c and at least 3n−i − 1 numbers larger than c. This is
certainly true when i = 0, so it suffices to show that if it holds for some i < n, we
can perform the grouping so that the condition will still hold for i + 1. Well, we first
group the smallest numbers as {a1 , a2 , a3 }, {a4 , a5 , a6 }, . . . , {a3n−i −5 , a3n−i −4 , a3n−i −3 }.
We then group the remaining numbers in such a way that c and the largest 3n−i − 1
numbers are each the largest in its respective group; it is easy to see that we can do
this. After retaining the largest number in each group, we will then have at least
3n−i−1 − 1 numbers smaller than c remaining and at least 3n−i − 1 numbers larger
than c remaining. And for the next grouping, we similarly group the largest 3n−i − 3
numbers into 3n−i−1 −1 groups, and arrange the remaining numbers so that the smallest
3n−i−1 − 1 numbers and c are all the smallest in their groups. After this round of
discarding, then c will be retained, and we will still have at least 3n−i−1 − 1 numbers
larger than c and 3n−i−1 numbers smaller than c. This proves the induction step, and
now the solution is complete.

4
Harvard-MIT Mathematics Tournament
February 19, 2005

Individual Round: Geometry Subject Test — Solutions

1. The volume of a cube (in cubic inches) plus three times the total length of its edges
(in inches) is equal to twice its surface area (in square inches). How many inches long
is its long diagonal?

Solution: 6 3
If the side length of the cube is s inches, then the condition implies s3 + 3 · 12s = 2 · 6s2 ,
or√s(s2 −√12s + 36) = s(s − 6)2 = 0. Therefore s = 6, and the long diagonal has length
s 3 = 6 3.
2. Let ABCD be a regular tetrahedron with side length 2. The plane parallel to edges
AB and CD and lying halfway between them cuts ABCD into two pieces. Find the
surface area of one of these pieces.

Solution: 1 + 2 3
The plane intersects each face of the tetrahedron in a midline of the face; by symmetry
it follows that the intersection of the plane with the tetrahedron is a square of side
length 1. The surface area of each piece is half √ the total surface
√ area of the tetrahedron
2
plus the area of the square, that is, 21 · 4 · 2 4 3 + 1 = 1 + 2 3.
3. Let ABCD be a rectangle with area 1, and let E lie on side CD. What is the area of
the triangle formed by the centroids of triangles ABE, BCE, and ADE?
Solution: 1/9
Let the centroids of ABE, BCE, and ADE be denoted by X, Y , and Z, respectively.
Let d(P, QR) denote the distance from P to line QR. Since the centroid lies two-thirds
of the distance from each vertex to the midpoint of the opposite edge, d(X, AB) =
d(Y, CD) = d(Z, CD) = 13 BC, so Y Z is parallel to CD and d(X, Y Z) = BC −
2
3
BC = 13 BC. Likewise, d(Z, AD) = 13 DE and d(Y, BC) = 31 CE, so that since Y Z
is perpendicular to AD and BC, we have that Y Z = CD − 13 (DE + CE) = 23 CD.
Therefore, the area of XY Z is 12 ( 13 BC)( 23 CD) = 91 BC · CD = 19 .

A B

D E C

4. Let XY Z be a triangle with 6 X = 60◦ and 6 Y = 45◦ . A circle with center P passes
through points A and B on side XY , C and D on side Y Z, and E and F on side ZX.
Suppose AB = CD = EF . Find 6 XP Y in degrees.

1
Solution: 255/2
Since P AB, P CD, and P EF are all isosceles triangles with equal legs and equal
bases, they are congruent. It follows that the heights of each are the same, so that P is
equidistant from the sides of XY Z. Therefore, P is the incenter and therefore lies on
the angle bisectors of XY Z. Thus 6 Y XP = 12 6 Y XZ = 30◦ and 6 P Y X = 12 6 ZY X =
45 ◦ ◦ ◦
2
. Therefore 6 XP Y = 180◦ − 30◦ − 45
2
= 255
2
.

5. A cube with side length 2 is inscribed in a sphere. A second cube, with faces parallel
to the first, is inscribed between the sphere and one face of the first cube. What is the
length of a side of the smaller cube?
Solution: 2/3

First note√that the long diagonal of the cube has length 2 3, so the radius of the
sphere is 3. Let x be the side length of the smaller cube. Then the distance from the
center of the sphere to the far face of the smaller cube is 1 + x,
√ while the distance from
the center of the far face to a vertex lying on the sphere is x 2 2 . Therefore, the square
2
of the radius is 3 = (1 + x)2 + x2 , or 3x2 + 4x − 4 = (3x − 2)(x + 2) = 0, so x = 23 .

6. A triangular piece of paper of area 1 is folded along a line parallel to one of the sides
and pressed flat. What is the minimum possible area of the resulting figure?
Solution: 2/3
Let the triangle be denoted ABC, and suppose we fold parallel to BC. Let the distance
from A to BC be h, and suppose we fold along a line at a distance of ch from A. We
will assume that neither angle B nor C is obtuse, for the area of overlap will only be
smaller if either is obtuse. If c ≤ 12 , then A does not fold past the edge BC, so the
overlap is a triangle similar to the original with height ch; the area of the figure is
then 1 − c2 ≥ 34 . Suppose c > 12 , so that A does fold past BC. Then the overlap is a
trapezoid formed by taking a triangle of height ch similar to the original and removing
a triangle of height (2c − 1)h similar to the original. The area of the resulting figure is
thus 1 − c2 + (2c − 1)2 = 3c2 − 4c + 2. This is minimized when c = 32 , when the area
is 23 < 34 ; the minimum possible area is therefore 23 .

7. Let ABCD be a tetrahedron such that edges AB, AC, and AD are mutually perpen-
dicular. Let the areas of triangles ABC, ACD, and ADB be denoted by x, y, and z,
respectively. In terms of x, y, and z, find the area of triangle BCD.
√ 2
Solution: x + y2 + z2
Place A, B, C, and D at (0, 0, 0), (b, 0, 0), (0, c, 0), and (0, 0, d) in Cartesian coordinate
space, with b, c, and d positive. Then the plane through B, C, and D is given by the
equation xb + yc + dz = 1. The distance from the origin to this plane is then
1 bcd bcd
q =√ = √ 2 .
1
+ 1
+ 1 b2 c2 2 2 2
+c d +d b2 2 x + y2 + z2
b2 c2 d2

Then if the area of BCD is K, the volume of the tetrahedron is


bcd bcdK
= √ 2 ,
6 6 x + y2 + z2

2

implying K = x2 + y 2 + z 2 .
Alternative Solution: The area of BCD is also half the length of the cross prod-
−−→ −−→
uct of the vectors BC = (0, −c, d) and BD = (−b, √ 2 0, d). This cross product is
(−cd, −db,
√ −bc) = −2(y, z, x), which has length 2 x + y + z 2 . Thus the area of
2

BCD is x + y 2 + z 2 .
2

8. Let T be a triangle with side lengths 26, 51, and 73. Let S be the set of points inside
T which do not lie within a distance of 5 of any side of T . Find the area of S.
Solution: 135/28
Note that the sides of S are parallel to the sides of T , so S is a triangle similar to T .
The semiperimeter of T is s = 12 (26 + 51 + 73) = 75. By Heron’s formula, the area

of T is 75 · 49 · 24 · 2 = 420. If r is the inradius of T , then the area of T is rs, so
r = 420/75 = 28/5. It follows that the inradius of S is r − 5 = 3/5, and the ratio of
similitude between S and T is 3/28. Therefore, the area of S is 420 · (3/28)2 = 135/28.
9. Let AC be a diameter of a circle ω of radius 1, and let D be the point on AC such
that CD = 1/5. Let B be the point on ω such that DB is perpendicular to AC, and
let E be the midpoint of DB. The line tangent to ω at B intersects line CE at the
point X. Compute AX.
Solution: 3
We first show that AX is perpendicular to AC. Let the tangent to ω at A intersect
CB at Z and CE at X 0 . Since ZA is parallel to BD and BE = ED, ZX 0 = X 0 A.
Therefore, X 0 is the midpoint of the hypotenuse of the right triangle ABZ, so it is also
its circumcenter. Thus X 0 A = X 0 B, and since X 0 A is tangent to ω and B lies on ω,
we must have that X 0 B is tangent to ω, so X = X 0 .
Let O be the center of ω. Then OD = 54 , so BD = 3
5
and DE = 3
10
. Then AX =
AC 3 2
DE · DC = 10 · 1/5 = 3.

X’

E
A C
O D

3
10. Let AB be the diameter of a semicircle Γ. Two circles, ω1 and ω2 , externally tangent
to each other and internally tangent to Γ, are tangent to the line AB at P and Q,
respectively, and to semicircular arc AB at C and D, respectively, with AP < AQ.
Suppose F lies on Γ such that 6 F QB = 6 CQA and that 6 ABF = 80◦ . Find 6 P DQ
in degrees.
Solution: 35
Extend the semicircle centered at O to an entire circle ω, and let the reflection of F
over AB be F 0 . Then CQF 0 is a straight line. Also, the homothety centered at C
taking ω1 into ω takes P to a point X on ω and AB to the parallel line tangent to ω
at X. Therefore, X is the midpoint of semicircle AXB, and C, P , and X lie on a line.
Similarly, D, Q, and X lie on a line. So,

45◦ = 6 XCB = 6 P CB = 6 P CQ + 6 QCB = 6 P CQ + 10◦ ,

since 6 QCB = 6 F 0 CB = 6 F 0 AB = 6 F AB = 90◦ − 6 ABF = 10◦ . Thus 6 P CQ = 35◦ .


We will show that 6 P CQ = 6 P DQ to get that 6 P DQ = 35◦ .
Note that 6 XP Q subtends the sum of arcs AC and BX, which is equal to arc XC.
Therefore 6 XP Q = 6 CDX, so CDQP is cyclic and 6 P CQ = 6 P DQ. The conclusion
follows.

A B
P O Q

F’

4
Harvard-MIT Mathematics Tournament
February 19, 2005

Individual Round: General Test, Part 1 — Solutions

1. How many real numbers x are solutions to the following equation?

|x − 1| = |x − 2| + |x − 3|

Solution: 2
If x < 1, the equation becomes (1 − x) = (2 − x) + (3 − x) which simplifies to x = 4,
contradicting the assumption x < 1. If 1 ≤ x ≤ 2, we get (x − 1) = (2 − x) + (3 − x),
which gives x = 2. If 2 ≤ x ≤ 3, we get (x − 1) = (x − 2) + (3 − x), which again gives
x = 2. If x ≥ 3, we get (x − 1) = (x − 2) + (x − 3), or x = 4. So 2 and 4 are the only
solutions, and the answer is 2.

2. A true-false test has ten questions. If you answer five questions “true” and five “false,”
your score is guaranteed to be at least four. How many answer keys are there for which
this is true?
Solution: 22
Suppose that either nine or ten of the questions have the same answer. Then no matter
which five questions we pick to have this answer, we will be right at least four times.
Conversely, suppose that there are at least two questions with each answer; we will
show that we can get a score less than four. By symmetry, assume there are at least
five questions whose answer is true. Then if we label five of these false, not only will
we get these five wrong, but we will also have answered all the false questions with
true, for a total of at least seven incorrect. There are 2 ways for all the questions to
have the same answer, and 2 · 10 = 20 ways for one question to have a different answer
from the others, for a total of 22 ways.

3. Let ABCD be a regular tetrahedron with side length 2. The plane parallel to edges
AB and CD and lying halfway between them cuts ABCD into two pieces. Find the
surface area of one of these pieces.

Solution: 1 + 2 3
The plane intersects each face of the tetrahedron in a midline of the face; by symmetry
it follows that the intersection of the plane with the tetrahedron is a square of side
length 1. The surface area of each piece is half
√ the total surface
√ area of the tetrahedron
1 22 3
plus the area of the square, that is, 2 · 4 · 4 + 1 = 1 + 2 3.

4. Find all real solutions to x3 + (x + 1)3 + (x + 2)3 = (x + 3)3 .


Solution: 3
The equation simplifies to 3x3 + 9x2 + 15x + 9 = x3 + 9x2 + 27x + 27, or equivalently,
2x3 − 12x − 18 = 2(x − 3)(x2 + 3x + 3) = 0. The discriminant of x2 + 3x + 3 is −3 < 0,
so the only real solution is x = 3.

1
5. In how many ways can 4 purple balls and 4 green balls be placed into a 4 × 4 grid such
that every row and column contains one purple ball and one green ball? Only one ball
may be placed in each box, and rotations and reflections of a single configuration are
considered different.
Solution: 216
There are 4! = 24 ways to place the four purple balls into the grid. Choose any purple
ball, and place two green balls, one in its row and the other in its column. There
are four boxes that do not yet lie in the same row or column as a green ball, and at
least one of these contains a purple ball (otherwise the two rows containing green balls
would contain the original purple ball as well as the two in the columns not containing
green balls). It is then easy to see that there is a unique way to place the remaining
green balls. Therefore, there are a total of 24 · 9 = 216 ways.

6. In an election, there are two candidates, A and B, who each have 5 supporters. Each
supporter, independent of other supporters, has a 21 probability of voting for his or her
candidate and a 21 probability of being lazy and not voting. What is the probability of
a tie (which includes the case in which no one votes)?
Solution: 63/256
The probability that exactly k supporters of A vote and exactly k supporters of B vote
¡ ¢2
is k5 · 2110 . Summing over k from 0 to 5 gives
µ ¶
1 252 63
10
(1 + 25 + 100 + 100 + 25 + 1) = = .
2 1024 256
¥ ¦ ¥ b+c ¦ ¥ c+a ¦
7. If a, b, c > 0, what is the smallest possible value of a+b
c
+ a + b ? (Note that
bxc denotes the greatest integer less than or equal to x.)
Solution: 4
Since bxc > x − 1 for all x, we have that
¹ º ¹ º ¹ º
a+b b+c c+a a+b b+c c+a
+ + > + + −3
c a b c a b
µ ¶ µ ¶ ³
a b b c c a´
= + + + + + − 3.
b a c b a c

But by the AM-GM inequality, each of the first three terms in the last line is at least
2. Therefore, the lefthand side is greater than 2 + 2 + 2 − 3 = 3. Since it is an
integer, the smallest value it can be is therefore 4. This is in fact attainable by letting
(a, b, c) = (6, 8, 9).

8. Ten positive integers are arranged around a circle. Each number is one more than the
greatest common divisor of its two neighbors. What is the sum of the ten numbers?
Solution: 28
First note that all the integers must be at least 2, because the greatest common divisor
of any two positive integers is at least 1. Let n be the largest integer in the circle.
The greatest common divisor of its two neighbors is n − 1. Therefore, each of the two

2
neighbors is at least n − 1 but at most n, so since n − 1 - n for n − 1 ≥ 2, they must
both be equal to n − 1. Let m be one of the numbers on the other side of n − 1 from
n. Then gcd(n, m) = n − 2. Since n − 2 ≥ 0, n − 2 | n only for n = 3 or 4. If n = 3,
each number must be 2 or 3, and it is easy to check that there is no solution. If n = 4,
then it is again not hard to find that there is a unique solution up to rotation, namely
4322343223. The only possible sum is therefore 28.

9. A triangular piece of paper of area 1 is folded along a line parallel to one of the sides
and pressed flat. What is the minimum possible area of the resulting figure?
Solution: 2/3
Let the triangle be denoted ABC, and suppose we fold parallel to BC. Let the distance
from A to BC be h, and suppose we fold along a line at a distance of ch from A. We
will assume that neither angle B nor C is obtuse, for the area of overlap will only be
smaller if either is obtuse. If c ≤ 12 , then A does not fold past the edge BC, so the
overlap is a triangle similar to the original with height ch; the area of the figure is
then 1 − c2 ≥ 34 . Suppose c > 12 , so that A does fold past BC. Then the overlap is a
trapezoid formed by taking a triangle of height ch similar to the original and removing
a triangle of height (2c − 1)h similar to the original. The area of the resulting figure is
thus 1 − c2 + (2c − 1)2 = 3c2 − 4c + 2. This is minimized when c = 32 , when the area
is 23 < 34 ; the minimum possible area is therefore 23 .

10. What is the smallest integer x larger than 1 such that x2 ends in the same three digits
as x does?
Solution: 376
The condition is that 1000 | x2 − x = x(x − 1). Since 1000 = 23 · 53 , and 2 cannot
divide both x and x − 1, 23 = 8 must divide one of them. Similarly, 53 = 125 must
divide either x or x − 1. We try successive values of x that are congruent to 0 or 1
modulo 125 and see which ones have the property that x or x − 1 is divisible by 8. It
is easy to check that 125, 126, 250, 251, and 375 do not work, but the next value, 376,
does, so this is the answer.

3
Harvard-MIT Mathematics Tournament
February 19, 2005

Individual Round: General Test, Part 2 — Solutions

1. The volume of a cube (in cubic inches) plus three times the total length of its edges
(in inches) is equal to twice its surface area (in square inches). How many inches long
is its long diagonal?

Solution: 6 3
If the side length of the cube is s inches, then the condition implies s3 + 3 · 12s = 2 · 6s2 ,
or√s(s2 −√12s + 36) = s(s − 6)2 = 0. Therefore s = 6, and the long diagonal has length
s 3 = 6 3.

2. Find three real numbers a < b < c satisfying:

a + b + c = 21/4
1/a + 1/b + 1/c = 21/4
abc = 1.

Solution: 1/4, 1, 4
By inspection, one notices that if b is a number such that b+1/b = 17/4, then a = 1, c =
1/b will work. Again by inspection (or by solving the quadratic b2 − 17b/4 + 1 = 0),
one finds b = 1/4 or 4, so the numbers are 1/4, 1, and 4.
Alternative Solution: Note that ab + bc + ca = abc(1/a + 1/b + 1/c) = 21/4, so
a, b, and c are the roots of the polynomial x3 − 21x2 /4 + 21x/4 − 1, which factors as
1
4
(x − 1)(x − 4)(4x − 1), giving the same answer.

3. Working together, Jack and Jill can paint a house in 3 days; Jill and Joe can paint the
same house in 4 days; or Joe and Jack can paint the house in 6 days. If Jill, Joe, and
Jack all work together, how many days will it take them?
Solution: 8/3
Suppose that Jack paints x houses per day, Jill paints y houses per day, and Joe paints
z houses per day. Together, Jack and Jill paint 1/3 of a house in a day — that is,

x + y = 1/3.

Similarly,
y + z = 1/4,
and
z + x = 1/6.
Adding all three equations and dividing by 2 gives

x + y + z = 3/8.

So, working together, the three folks can paint 3/8 houses in a day, or 8/3 days per
house.

1
4. In how many ways can 8 people be arranged in a line if Alice and Bob must be next
to each other, and Carol must be somewhere behind Dan?
Solution: 5040
Let us place Alice and Bob as a single person; there are then 7! = 5040 different
arrangements. Alice can be in front of Bob or vice versa, multiplying the number of
possibilities by 2, but Carol is behind Dan in exactly half of those, so that the answer
is just 5040.

5. You and I play the following game on an 8 × 8 square grid of boxes: Initially, every
box is empty. On your turn, you choose an empty box and draw an X in it; if any
of the four adjacent boxes are empty, you mark them with an X as well. (Two boxes
are adjacent if they share an edge.) We alternate turns, with you moving first, and
whoever draws the last X wins. How many choices do you have for a first move that
will enable you to guarantee a win no matter how I play?
Solution: 0
I can follow a symmetry strategy: whenever you play in the box S, I play in the image
of S under the 180◦ rotation about the center of the board. This ensures that the board
will always be centrally symmetric at the beginning of your turn. Thus, if you play in
an empty box S, its symmetric image S 0 is also empty at the beginning of your turn,
and it remains so after your turn, since the even size of the board ensures that S can
be neither equal to nor adjacent to S 0 . In particular, I always have a move available.
Since the first person without an available move loses, you are guaranteed to lose. So
the answer is that you have 0 choices for a first move that will guarantee your win.

6. A cube with side length 2 is inscribed in a sphere. A second cube, with faces parallel
to the first, is inscribed between the sphere and one face of the first cube. What is the
length of a side of the smaller cube?
Solution: 2/3

First note√that the long diagonal of the cube has length 2 3, so the radius of the
sphere is 3. Let x be the side length of the smaller cube. Then the distance from the
center of the sphere to the far face of the smaller cube is 1 + x,
√ while the distance from
the center of the far face to a vertex lying on the sphere is x 2 2 . Therefore, the square
2
of the radius is 3 = (1 + x)2 + x2 , or 3x2 + 4x − 4 = (3x − 2)(x + 2) = 0, so x = 23 .

7. Three distinct lines are drawn in the plane. Suppose there exist exactly n circles in
the plane tangent to all three lines. Find all possible values of n.
Solution: 0, 2, 4
If the three lines form a triangle, then there are 4 circles, namely the incircle and the
three excircles. If the three lines concur or are all parallel, then there are 0 circles. If
two lines are parallel and the third is not, then there are 2 circles lying between the
two parallel lines, one on each side of the transverse line. These are the only possible
configurations, so the answers are 0, 2, and 4.

2
8. What is the maximum number of bishops that can be placed on an 8 × 8 chessboard
such that at most three bishops lie on any diagonal?
Solution: 38
If the chessboard is colored black and white as usual, then any diagonal is a solid color,
so we may consider bishops on black and white squares separately. In one direction,
the lengths of the black diagonals are 2, 4, 6, 8, 6, 4, and 2. Each of these can have
at most three bishops, except the first and last which can have at most two, giving a
total of at most 2 + 3 + 3 + 3 + 3 + 3 + 2 = 19 bishops on black squares. Likewise there
can be at most 19 bishops on white squares for a total of at most 38 bishops. This is
indeed attainable as in the diagram below.

X X X X X X X X
X X X X X X X
X X
X X
X X
X X
X X X X X X X
X X X X X X X X

9. In how many ways can the cells of a 4 × 4 table be filled in with the digits 1, 2, . . . , 9
so that each of the 4-digit numbers formed by the columns is divisible by each of the
4-digit numbers formed by the rows?
Solution: 9
If a and b are 4-digit numbers with the same first digit, and a divides b, then since
b < a + 1000 ≤ 2a, b must equal a. In particular, since the number formed by the first
row of the table divides the number in the first column (and both have the same first
digit), these numbers must be equal; call their common value n. Then, for k = 2, 3, or
4, we find that the number in the kth column and the number in the kth row have the
same first digit (namely the kth digit of n), so by the same reasoning, they are equal.
Also, the smallest number b formed by any column is divisible by the largest number
a formed by any row, but by the symmetry just proven, a is also the largest number
formed by any column, so a ≥ b. Since b is divisible by a, we must have equality. Then
all columns contain the same number — and hence all rows also contain the same

3
number — which is only possible if all 16 cells contain the same digit. Conversely, for
each d = 1, . . . , 9, filling in all 16 cells with the digit d clearly gives a table meeting
the required condition, so we have exactly 9 such tables, one for each digit.

10. Let bxc denote the greatest integer less than or equal to x. How many positive integers
less than 2005 can be expressed in the form bx bxcc for some positive real x?
Solution: 990
Let {x} = x − bxc be the fractional part of x. Note that

bx bxcc = b(bxc + {x}) bxcc = bxc2 + b{x} bxcc .

Because {x} may take on any value in the half-open interval [0, 1), the quantity
b{x} bxcc can take on any integer value between 0 and bxc − 1, inclusive.
If bxc = n, then bx bxcc can be any of the numbers n2 , n2 + 1, . . . , n2 + n − 1. In other
words, there are precisely n possible values that bx bxcc can take, and moreover, all of
them are less than (n + 1)2 . Because 442 + 43 = 1979 < 2005 and 452 = 2025 > 2005,
n can range between 1 and 44, inclusive. Therefore, the answer is
44
X 44 · 45
n= = 990.
n=1
2

4
Harvard-MIT Mathematics Tournament
February 19, 2005

Guts Round — Solutions

1. Find the largest positive integer n such that 1 + 2 + 3 + · · · + n2 is divisible by 1 + 2 +


3 + · · · + n.
Solution: 1
The statement is
n(n + 1) n2 (n2 + 1)
| ⇔ n + 1 | n(n2 + 1) = n3 + n.
2 2
But n + 1 also divides (n + 1)(n2 − n + 2) = n3 + n + 2, so n + 1 must divide 2. Hence,
n cannot be greater than 1. And n = 1 clearly works, so that is the answer.

2. Let x, y, and z be positive real numbers such that (x · y) + z = (x + z) · (y + z). What


is the maximum possible value of xyz?
Solution: 1/27
The condition is equivalent to z 2 + (x + y − 1)z = 0. Since z is positive, z = 1 − x − y,
so x + y + z = 1. By the AM-GM inequality,
µ ¶3
x+y+z 1
xyz ≤ = ,
3 27

with equality when x = y = z = 13 .

3. Find the sum


21 22 24 28
+ + + + ···.
41 − 1 42 − 1 44 − 1 48 − 1

Solution: 1
Notice that
k k
22 22 + 1 1 1 1 1 1
2 k = 2k − 2k = 2k − 2k = 2k−1 − 2k .
4 −1 4 −1 4 −1 2 −1 4 −1 4 −1 4 −1
Therefore, the sum telescopes as
µ ¶ µ ¶ µ ¶
1 1 1 1 1 1
− + − + − + ···
42−1 − 1 420 − 1 420 − 1 421 − 1 421 − 1 422 − 1
−1
and evaluates to 1/(42 − 1) = 1.

4. What is the probability that in a randomly chosen arrangement of the numbers and
letters in “HMMT2005,” one can read either “HMMT” or “2005” from left to right?
(For example, in “5HM0M20T,” one can read “HMMT.”)
Solution: 23/144

1
¡¢
To read “HMMT,” there are 84¡ ¢ways to place the letters, and 4!2 ways to place the
numbers. Similarly, there are 84 4!2 arrangements where one¡8¢can read ”2005.” The
number of arrangements in which one can read both is just 4 . The total number of
arrangements is 8!4 , thus the answer is
¡8¢ 4! ¡8¢ 4! ¡8¢ µ ¶
4 2
+ 4 2 − 4 8 4 23
8!
= · 23 = .
4
4 8! 144

5. For how many integers n between 1 and 2005, inclusive, is 2 · 6 · 10 · · · (4n − 2) divisible
by n!?
Solution: 2005
Note that

2 · 6 · 10 · · · (4n − 2) = 2n · 1 · 3 · 5 · · · (2n − 1)
1 · 2 · 3 · · · 2n
= 2n ·
2 · 4 · 6 · · · 2n
1 · 2 · 3 · · · 2n
= ,
1 · 2 · 3···n
¡ ¢
that is, it is just (2n)!/n!. Therefore, since (2n)!/(n!)2 = 2n n
is always an integer, the
answer is 2005.

6. Let m ◦ n = (m + n)/(mn + 4). Compute ((· · · ((2005 ◦ 2004) ◦ 2003) ◦ · · · ◦ 1) ◦ 0).


Solution: 1/12
1
Note that m ◦ 2 = (m + 2)/(2m + 4) = 2
, so the quantity we wish to find is just
( 21 ◦ 1) ◦ 0 = 13 ◦ 0 = 1/12.

7. Five people of different heights are standing in line from shortest to tallest. As it
happens, the tops of their heads are all collinear; also, for any two successive people,
the horizontal distance between them equals the height of the shorter person. If the
shortest person is 3 feet tall and the tallest person is 7 feet tall, how tall is the middle
person, in feet?

Solution: 21
If A, B, and C are the tops of the heads of three successive people and D, E, and F are
their respective feet, let P be the foot of the perpendicular from A to BE and let Q be
the foot of the perpendicular from B to CF . Then, by equal angles, 4ABP ∼ 4BCQ,
so
CF CF CQ BP BE BE
= = +1= +1= = .
BE BQ BQ AP AP AD
Therefore the heights of successive people are in geometric progression. Hence, √ the
heights
√ of all five people are in geometric progression, so the middle height is 3·7=
21 feet.

2
C

B Q

A P

D E F

8. Let ABCD be a convex quadrilateral inscribed in a circle with shortest side AB. The
ratio [BCD]/[ABD] is an integer (where [XY Z] denotes the area of triangle XY Z.)
If the lengths of AB, BC, CD, and DA are distinct integers no greater than 10, find
the largest possible value of AB.
Solution: 5
Note that
1
[BCD] BC · CD · sin C BC · CD
= 21 =
[ABD] 2
DA · AB · sin A DA · AB
since ∠A and ∠C are supplementary. If AB ≥ 6, it is easy to check that no assignment
of lengths to the four sides yields an integer ratio, but if AB = 5, we can let BC = 10,
CD = 9, and DA = 6 for a ratio of 3. The maximum value for AB is therefore 5.

9. Farmer Bill’s 1000 animals — ducks, cows, and rabbits — are standing in a circle.
In order to feel safe, every duck must either be standing next to at least one cow or
between two rabbits. If there are 600 ducks, what is the least number of cows there
can be for this to be possible?
Solution: 201
Suppose Bill has r rabbits and c cows. At most r −1 ducks can be between two rabbits:
each rabbit can serve up to two such ducks, so at most 2r/2 = r ducks will each be
served by two rabbits, but we cannot have equality, since this would require alternating
between rabbits and ducks all the way around the circle, contradicting the fact that
more than half the animals are ducks. Also, at most 2c ducks can each be adjacent to
a cow. So we need 600 ≤ r − 1 + 2c = (400 − c) − 1 + 2c, giving c ≥ 201. Conversely,
an arrangement with 201 cows is possible:

RDRDR
| {z · · · DR} |DCD DCD DCD
{z · · · DCD} .
199 R, 198 D 201 C, 402 D

So 201 is the answer.

10. You are given a set of cards labeled from 1 to 100. You wish to make piles of three cards
such that in any pile, the number on one of the cards is the product of the numbers
on the other two cards. However, no card can be in more than one pile. What is the
maximum number of piles you can form at once?
Solution: 8
Certainly, the two factors in any pile cannot both be at least 10, since then the product
would be at least 10 × 11 > 100. Also, the number 1 can not appear in any pile, since
then the other two cards in the pile would have to be the same. So each pile must use

3
one of the numbers 2, 3, . . . , 9 as one of the factors, meaning we have at most 8 piles.
Conversely, it is easy to construct a set of 8 such piles, for example:

{9, 11, 99} {8, 12, 96} {7, 13, 91} {6, 14, 84}

{5, 15, 75} {4, 16, 64} {3, 17, 51} {2, 18, 36}

11. The Dingoberry Farm is a 10 mile by 10 mile square, broken up into 1 mile by 1 mile
patches. Each patch is farmed either by Farmer Keith or by Farmer Ann. Whenever
Ann farms a patch, she also farms all the patches due west of it and all the patches
due south of it. Ann puts up a scarecrow on each of her patches that is adjacent to
exactly two of Keith’s patches (and nowhere else). If Ann farms a total of 30 patches,
what is the largest number of scarecrows she could put up?
Solution: 7
Whenever Ann farms a patch P , she also farms all the patches due west of P and
due south of P . So, the only way she can put a scarecrow on P is if Keith farms the
patch immediately north of P and the patch immediately east of P , in which case Ann
cannot farm any of the patches due north of P or due east of P . That is, Ann can
only put a scarecrow on P if it is the easternmost patch she farms in its east-west row,
and the northernmost in its north-south column. In particular, all of her scarecrow
patches are in different rows and columns. Suppose that she puts up n scarecrows.
The farthest south of these must be in the 10th row or above, so she farms at least 1
patch in that column; the second-farthest south must be in the 9th row above, so she
farms at least 2 patches in that column; the third-farthest south must be in the 8th
row or above, so she farms at least 3 patches in that column, and so forth, for a total
of at least
1 + 2 + · · · + n = n(n + 1)/2
patches. If Ann farms a total of 30 < 8 · 9/2 patches, then we have n < 8. On the
other hand, n = 7 scarecrows are possible, as shown:

S
S
S
S
S
S
S

12. Two vertices of a cube are given in space. The locus of points that could be a third
vertex of the cube is the union of n circles. Find n.
Solution: 10
Let the distance between the two given vertices be 1. If the two given vertices are
adjacent,
√ then the other vertices lie on four circles, two of radius 1 and two of radius
2. If the two vertices are separated by a diagonal of a face of the cube, then the
locus of possible vertices adjacent to both of them is a circle of radius 12 , the locus of

4

possible vertices adjacent to exactly one of them is two circles of radius√ 22 , and the
locus of possible vertices adjacent to neither of them is a circle of radius 23 . If the two
given vertices are separated√by a long diagonal, then each of the other vertices lie on
one of two circles of radius 32 , for a total of 10 circles.
√ √
13. Triangle ABC has AB = 1, BC = 7, and CA = 3. Let `1 be the line through A
perpendicular to AB, `2 the line through B perpendicular to AC, and P the point of
intersection of `1 and `2 . Find P C.
Solution: 3

By the Law of Cosines, ∠BAC = cos−1 3+1−7√
2 3
= cos−1 (− 23 ) = 150◦ . If we let Q be the
intersection of `2 and AC, we notice that ∠QBA = 90◦ − ∠QAB = 90◦ − 30◦ = 60 √ .

It follows that triangle ABP is a 30-60-90 triangle and thus P B = 2 and P A = 3.


Finally, we have ∠P AC = 360◦ − (90◦ + 150◦ ) = 120◦ , and

P C = (P A2 + AC 2 − 2P A · AC cos 120◦ )1/2 = (3 + 3 + 3)1/2 = 3.

A B

14. Three noncollinear points and a line ` are given in the plane. Suppose no two of the
points lie on a line parallel to ` (or ` itself). There are exactly n lines perpendicular
to ` with the following property: the three circles with centers at the given points and
tangent to the line all concur at some point. Find all possible values of n.
Solution: 1
The condition for the line is that each of the three points lies at an equal distance from
the line as from some fixed point; in other words, the line is the directrix of a parabola
containing the three points. Three noncollinear points in the coordinate plane deter-
mine a quadratic polynomial in x unless two of the points have the same x-coordinate.
Therefore, given the direction of the directrix, three noncollinear points determine a
parabola, unless two of the points lie on a line perpendicular to the directrix. This
case is ruled out by the given condition, so the answer is 1.

15. Let S be the set of lattice points inside the circle x2 + y 2 = 11. Let M be the greatest
area of any triangle with vertices in S. How many triangles with vertices in S have
area M ?
Solution: 16
The boundary of the convex hull of S consists of points with (x, y) or (y, x) = (0, ±3),
(±1, ±3), and (±2, ±2). For any triangle T with vertices in S, we can increase its
area by moving a vertex not on the boundary to some point on the boundary. Thus,

5
if T has area M , its vertices are all on the boundary of S. The next step is to see
(either by inspection or by noting that√T has area no larger than that of an equilateral
triangle inscribed in a circle of radius 10, which has area less than 13) that M = 12.
There are 16 triangles with area 12, all congruent to one of the following three: vertices
(2, 2), (1, −3), and (−3, 1); vertices (3, −1), (−3, −1), and (1, 3); or vertices (3, −1),
(−3, −1), and (0, 3).
16. A regular octahedron has a side length of 1. What is the distance between two opposite
faces?

Solution: 6/3
Imagine orienting the octahedron so that the two opposite faces are horizontal. Project
onto a horizontal plane; these two faces are congruent equilateral triangles which (when
projected) have the same center and opposite orientations. Hence, the vertices of the
octahedron project to the vertices of a regular hexagon ABCDEF .

F
A

E O M B

C
D

Let O be the center of the hexagon√ and M the midpoint


√ of
√ AC. Now ABM is a
30-60-90 triangle, so AB = AM/( 3/2) = (1/2)/( 3/2) = 3/3. If we let d denote
the desired vertical distance between the opposite faces (which project
√ to ACE
√ and
2 2 2 2
BDF ), then by the Pythagorean Theorem, AB + d = 1 , so d = 1 − AB = 6/3.
17. Compute s r
q
3 4 √
5
2 2 2 2 2 · · ·.

Solution: 2e−1
Taking the base 2 logarithm of the expression gives
µ µ ¶¶
1 1 1 1 1 1
1+ 1+ 1 + (1 + · · ·) = 1 + + + + · · · = e − 1.
2 3 4 2! 3! 4!
Therefore the expression is just 2e−1 .
18. If a, b, and c are random real numbers from 0 to 1, independently and uniformly chosen,
what is the average (expected) value of the smallest of a, b, and c?
Solution: 1/4
Let d be a fourth random variable, also chosen uniformly from [0, 1]. For fixed a, b,
and c, the probability that d < min{a, b, c} is evidently equal to min{a, b, c}. Hence,

6
if we average over all choices of a, b, c, the average value of min{a, b, c} is equal to
the probability that, when a, b, c, and d are independently randomly chosen, d <
min{a, b, c}, i.e., that d is the smallest of the four variables. On the other hand, by
symmetry, the probability that d is the smallest of the four is simply equal to 1/4, so
that is our answer.

19. Regular tetrahedron ABCD is projected onto a plane sending A, B, C, and D to


A0 , B 0 , C 0 , and D0 respectively. Suppose A0 B 0 C 0 D0 is a convex quadrilateral with
A0 B 0 = A0 D0 and C 0 B 0 = C 0 D0 , and suppose that the area of A0 B 0 C 0 D0 = 4. Given
these conditions, the set of possible lengths of AB consists of all real numbers in the
interval [a, b). Compute b.

Solution: 2 4 6
The value of b occurs when the quadrilateral A0 B 0 C 0 D0 degenerates to an isosceles
triangle. This occurs when the altitude from A to BCD is parallel to the plane. Let
s = AB. Then the altitude from A intersects
q the√center E of face BCD. Since
s2
EB = 3 , it follows that A C = AE = s − 3 = s 3 6 . Then since BD is parallel to
√s 0 0 2
2
√ √
the plane, B 0 D0 = s. Then the area of A0 B 0 C 0 D0 is 4 = 21 · s 3 6 , implying s2 = 4 6, or

s = 2 4 6.

20. If n is a positive integer, let s(n) denote the sum of the digits of n. We say that n
is zesty if there exist positive integers x and y greater than 1 such that xy = n and
s(x)s(y) = s(n). How many zesty two-digit numbers are there?
Solution: 34
Let n be a zesty two-digit number, and let x and y be as in the problem statement.
Clearly if both x and y are one-digit numbers, then s(x)s(y) = n 6= s(n). Thus either
x is a two-digit number or y is. Assume without loss of generality that it is x. If
x = 10a + b, 1 ≤ a ≤ 9 and 0 ≤ b ≤ 9, then n = 10ay + by. If both ay and by are
less than 10, then s(n) = ay + by, but if either is at least 10, then s(n) < ay + by. It
follows that the two digits of n share a common factor greater than 1, namely y. It is
now easy to count the zesty two-digit numbers by first digit starting with 2; there are
a total of 5 + 4 + 5 + 2 + 7 + 2 + 5 + 4 = 34.

21. In triangle ABC with altitude AD, ∠BAC = 45◦ , DB = 3, and CD = 2. Find the
area of triangle ABC.
Solution: 15
Suppose first that D lies between B and C. Let ABC be inscribed in circle ω, and
extend AD to intersect ω again at E. Note that A subtends a quarter of the circle, so
in particular, the chord through C perpendicular to BC and parallel to AD has length
BC = 5. Therefore, AD = 5 + DE. By power of a point, 6 = BD · DC = AD · DE =
AD2 − 5AD, implying AD = 6, so the area of ABC is 12 BC · AD = 15.

If D does not lie between B and C, then BC = 1, so A lies on a circle of radius 2/2
through B and C. But then it is easy to check that the perpendicular to BC through
D cannot intersect the circle, a contradiction.

7
B

E
D A

22. Find
{ln(1 + e)} + {ln(1 + e2 )} + {ln(1 + e4 )} + {ln(1 + e8 )} + · · · ,
where {x} = x − bxc denotes the fractional part of x.
Solution: 1 − ln(e − 1)
k k k
Since ln(1 + e2 ) is just larger than 2k , its fractional part is ln(1 + e2 ) − ln e2 =
k
ln(1 + e−2 ). But now notice that
n
Y k n+1 −1
(1 + x2 ) = 1 + x + x2 + · · · + x2 .
k=0

(This is easily proven by induction or by noting that every nonnegative integer less
than 2n+1 has a unique (n+1)-bit binary expansion.) If |x| < 1, this product converges
1
to 1−x as n goes to infinity. Therefore,

X ∞
Y
−2k k 1 e
ln(1 + e ) = ln (1 + (e−1 )2 ) = ln −1
= ln = 1 − ln(e − 1).
k=0 k=0
1−e e−1

23. The sides of a regular hexagon are trisected, resulting in 18 points, including vertices.
These points, starting with a vertex, are numbered clockwise as A1 , A2 , . . . , A18 . The
line segment Ak Ak+4 is drawn for k = 1, 4, 7, 10, 13, 16, where indices are taken modulo
18. These segments define a region containing the center of the hexagon. Find the
ratio of the area of this region to the area of the large hexagon.
Solution: 9/13
Let us assume all sides are of side length 3. Consider the triangle A1 A4 A5 . Let
P be the point of intersection of A1 A5 with A4 A8 . This is a vertex of the inner
hexagon. Then ∠A4 A1 A5 = ∠A5 A4 P , by symmetry. It follows that √ A1 A4 A5 ∼ A4 P A5 .

Also, ∠A1 A4 A5 = 120 , so by the Law √ of Cosines A1 A5 = 13. It follows that
P A5 = (A4 A5 ) · (A4 A5 )/(A1 A5 ) = 1/ √13. Let Q be the intersection of A1 A5√and
A16 A2 . By similar reasoning, A1 Q = 3/ 13, so P Q = A1 A5 − A1 Q − P A√5 = 9/ 13.
By symmetry, the inner region is a regular hexagon with √ side length 9/ 13. Hence
the ratio of the area of the smaller to larger hexagon is (3/ 13)2 = 9/13.

8
24. In the base 10 arithmetic problem HM M T + GU T S = ROU N D, each distinct letter
represents a different digit, and leading zeroes are not allowed. What is the maximum
possible value of ROU N D?
Solution: 16352
Clearly R = 1, and from the hundreds column, M = 0 or 9. Since H + G = 9 + O or
10 + O, it is easy to see that O can be at most 7, in which case H and G must be 8
and 9, so M = 0. But because of the tens column, we must have S + T ≥ 10, and in
fact since D cannot be 0 or 1, S + T ≥ 12, which is impossible given the remaining
choices. Therefore, O is at most 6.
Suppose O = 6 and M = 9. Then we must have H and G be 7 and 8. With the
remaining digits 0, 2, 3, 4, and 5, we must have in the ones column that T and S are
2 and 3, which leaves no possibility for N . If instead M = 0, then H and G are 7 and
9. Since again S + T ≥ 12 and N = T + 1, the only possibility is S = 8, T = 4, and
N = 5, giving ROU N D = 16352 = 7004 + 9348 = 9004 + 7348.

25. An ant starts at one vertex of a tetrahedron. Each minute it walks along a random
edge to an adjacent vertex. What is the probability that after one hour the ant winds
up at the same vertex it started at?
Solution: (359 + 1)/(4 · 359 )
Let pn be the probability that the ant is at the original vertex after n minutes; then
p0 = 1. The chance that the ant is at each of the other three vertices after n minutes is
1
3
(1−pn ). Since the ant can only walk to the original vertex from one of the three others,
and at each there is a 13 probability of doing so, we have that pn+1 = 13 (1 − pn ). Let
qn = pn − 14 . Substituting this¡into¢ the recurrence, we find that qn+1 = 14 + 31 (−qn − 43 ) =
n
− 13 qn . Since q0 = 34 , qn = 43 · − 13 . In particular, this implies that

1 1 3 1 359 + 1
p60 = + q60 = + · 60 = .
4 4 4 3 4 · 359

26. In triangle ABC, AC = 3AB. Let AD bisect angle A with D lying on BC, and let E
be the foot of the perpendicular from C to AD. Find [ABD]/[CDE]. (Here, [XY Z]
denotes the area of triangle XY Z).
Solution: 1/3
By the Angle Bisector Theorem, DC/DB = AC/AB = 3. We will show that AD =
DE. Let CE intersect AB at F . Then since AE bisects angle A, AF = AC = 3AB,
and EF = EC. Let G be the midpoint of BF . Then BG = GF , so GE k BC. But
then since B is the midpoint of AG, D must be the midpoint of AE, as desired. Then
[ABD]/[CDE] = (AD · BD)/(ED · CD) = 1/3.

9
C

A B G F

27. In a chess-playing club, some of the players take lessons from other players. It is
possible (but not necessary) for two players both to take lessons from each other. It
so happens that for any three distinct members of the club, A, B, and C, exactly one
of the following three statements is true: A takes lessons from B; B takes lessons from
C; C takes lessons from A. What is the largest number of players there can be?
Solution: 4
If P , Q, R, S, and T are any five distinct players, then consider all pairs A, B ∈
{P, Q, R, S, T } such that A takes lessons from B. Each pair contributes to exactly
three triples (A, B, C) (one for each of the choices of C distinct from A and B); three
triples (C, A, B); and three triples (B, C, A). On the other hand, there are 5×4×3 = 60
ordered triples of distinct players among these five, and each includes exactly one of
our lesson-taking pairs. That means that there are 60/9 such pairs. But this number
isn’t an integer, so there cannot be five distinct people in the club.
On the other hand, there can be four people, P , Q, R, and S: let P and Q both take
lessons from each other, and let R and S both take lessons from each other; it is easy
to check that this meets the conditions. Thus the maximum number of players is 4.

28. There are three pairs of real numbers (x1 , y1 ), (x2 ,³y2 ), and´ ³
(x3 , y3 ) ´that
³ satisfy´ both
x3 − 3xy 2 = 2005 and y 3 − 3x2 y = 2004. Compute 1 − xy11 1 − xy22 1 − xy33 .

Solution: 1/1002
By the given, 2004(x3 − 3xy 2 ) − 2005(y 3 − 3x2 y) = 0. Dividing both sides by y 3 and
setting t = xy yields 2004(t3 − 3t) − 2005(1 − 3t2 ) = 0. A quick check shows that this
cubic has three real roots. Since the three roots are precisely xy11 , xy22 , and xy33 , we must
³ ´³ ´³ ´
have 2004(t3 − 3t) − 2005(1 − 3t2 ) = 2004 t − xy11 t − xy22 t − xy33 . Therefore,
µ ¶µ ¶µ ¶
x1 x2 x3 2004(13 − 3(1)) − 2005(1 − 3(1)2 ) 1
1− 1− 1− = = .
y1 y2 y3 2004 1002

29. Let n > 0 be an integer. Each face of a regular tetrahedron is painted in one of n colors
(the faces are not necessarily painted different colors.) Suppose there are n3 possible
colorings, where rotations, but not reflections, of the same coloring are considered the
same. Find all possible values of n.
Solution: 1, 11

10
We count the possible number of colorings. If four colors are used,¡ there ¢ are two
different colorings that are mirror images of each other, for a total of 2 n4 colorings. If
three colors are¡used,
¢ we choose one color to use twice (which determines the coloring),
n
for a total of 3 3 colorings. If two colors are used, we can either choose one of those
colors
¡n¢ and color three faces with it, or we can color two faces ¡n¢each color, for a total of
3 2 colorings. Finally, we can also use only one color, for 1 colorings. This gives a
total of µ ¶ µ ¶ µ ¶ µ ¶
n n n n 1
2 +3 +3 + = n2 (n2 + 11)
4 3 2 1 12
colorings. Setting this equal to n3 , we get the equation n2 (n2 + 11) = 12n3 , or equiva-
lently n2 (n − 1)(n − 11) = 0, giving the answers 1 and 11.

30. A cuboctahedron is a polyhedron whose faces are squares and equilateral triangles such
that two squares and two triangles alternate around each vertex, as shown.

What is the volume of a cuboctahedron of side length 1?



Solution: 5 2/3
We can construct a cube such that the vertices of the cuboctahedron are the midpoints
of the edges of the cube.

Let s be the side length of this cube. Now, the cuboctahedron is obtained from the
cube by cutting a tetrahedron from each corner. Each such tetrahedron has a base in
the form of an isosceles right triangle of area (s/2)2 /2 and height s/2 for a volume of
(s/2)3 /6. The total volume of the cuboctahedron is therefore

s3 − 8 · (s/2)3 /6 = 5s3 /6.

Now, the side of the cuboctahedron


√ is the
√ hypotenuse of an isosceles right triangle
of√leg s/2; thus 1 = (s/2) 2, giving s = 2, so the volume of the cuboctahedron is
5 2/3.

11
31. The L shape made by adjoining three congruent squares can be subdivided into four
smaller L shapes.

Each of these can in turn be subdivided, and so forth. If we perform 2005 successive
subdivisions, how many of the 42005 L’s left at the end will be in the same orientation
as the original one?
Solution: 42004 + 22004
After n successive subdivisions, let an be the number of small L’s in the same orientation
as the original one; let bn be the number of small L’s that have this orientation rotated
counterclockwise 90◦ ; let cn be the number of small L’s that are rotated 180◦ ; and
let dn be the number of small L’s that are rotated 270◦ . When an L is subdivided,
it produces two smaller L’s of the same orientation, one of each of the neighboring
orientations, and none of the opposite orientation. Therefore,

(an+1 , bn+1 , cn+1 , dn+1 ) = (dn + 2an + bn , an + 2bn + cn , bn + 2cn + dn , cn + 2dn + an ).

It is now straightforward to show by induction that

(an , bn , cn , dn ) = (4n−1 + 2n−1 , 4n−1 , 4n−1 − 2n−1 , 4n−1 )

for each n ≥ 1. In particular, our desired answer is a2005 = 42004 + 22004 .

32. Let a1 = 3, and for n ≥ 1, let an+1 = (n + 1)an − n. Find the smallest m ≥ 2005 such
that am+1 − 1 | a2m − 1.
Solution: 2010
We will show that an = 2 · n! + 1 by induction. Indeed, the claim is obvious for n = 1,
and (n + 1)(2 · n! + 1) − n = 2 · (n + 1)! + 1. Then we wish to find m ≥ 2005 such that
2(m + 1)! | 4(m!)2 + 4m!, or dividing by 2 · m!, we want m + 1 | 2(m! + 1). Suppose
m + 1 is composite. Then it has a proper divisor d > 2, and since d | m!, we must have
d | 2, which is impossible. Therefore, m + 1 must be prime, and if this is the case, then
m + 1 | m! + 1 by Wilson’s Theorem. Therefore, since the smallest prime greater than
2005 is 2011, the smallest possible value of m is 2010.

33. Triangle ABC has incircle ω which touches AB at C1 , BC at A1 , and CA at B1 . Let


A2 be the reflection of A1 over the midpoint of BC, and define B2 and C2 similarly.
Let A3 be the intersection of AA2 with ω that is closer to A, and define B3 and C3
similarly. If AB = 9, BC = 10, and CA = 13, find [A3 B3 C3 ]/[ABC]. (Here [XY Z]
denotes the area of triangle XY Z.)
Solution: 14/65
Notice that A2 is the point of tangency of the excircle opposite A to BC. Therefore, by
considering the homothety centered at A taking the excircle to the incircle, we notice
that A3 is the intersection of ω and the tangent line parallel to BC. It follows that

12
A1 B1 C1 is congruent to A3 B3 C3 by reflecting through the center of ω. We therefore
need only find [A1 B1 C1 ]/[ABC]. Since

[A1 BC1 ] A1 B · BC1 ((9 + 10 − 13)/2)2 1


= = = ,
[ABC] AB · BC 9 · 10 10

and likewise [A1 B1 C]/[ABC] = 49/130 and [AB1 C1 ]/[ABC] = 4/13, we get that

[A3 B3 C3 ] 1 49 4 14
=1− − − = .
[ABC] 10 130 13 65

34. A regular octahedron ABCDEF is given such that AD, BE, and CF are perpen-
dicular. Let G, H, and I lie on edges AB, BC, and CA respectively such that
AG
GB
= BHHC
= CI
IA
= ρ. For some choice of ρ > 1, GH, HI, and IG are three edges
of a regular icosahedron, eight of whose faces are inscribed in the faces of ABCDEF .
Find ρ.

Solution: (1 + 5)/2
Let J lie on edge CE such that EJ
JC
= ρ. Then we must have that HIJ is another face of
the icosahedron,
√ so in particular, HI = HJ. But since BC and CE are perpendicular,
HJ = HC 2. By the Law of Cosines, HI = HC 2 + CI 2 − 2HC · CI cos
2
√ 60 =

HC 2 (1 + ρ2 − ρ). Therefore, 2 = 1 + ρ2 − ρ, or ρ2 − ρ − 1 = 0, giving ρ = 1+2 5 .

I
G

E
B
J
H C

35. Let p = 224036583 −1, the largest prime currently known. For how many positive integers
c do the quadratics ±x2 ± px ± c all have rational roots?
Solution: 0
This is equivalent to both discriminants p2 ± 4c being squares. In other words, p2 must
be the average of two squares a2 and b2 . Note that a and b must have the same parity,
2 2
and that ( a+b
2
)2 + ( a−b
2
)2 = a +b
2
= p2 . Therefore, p must be the hypotenuse in a
Pythagorean triple. Such triples are parametrized by k(m2 − n2 , 2mn, m2 + n2 ). But
p ≡ 3 (mod 4) and is therefore not the sum of two squares. This implies that p is not
the hypotenuse of any Pythagorean triple, so the answer is 0.

13
36. One hundred people are in line to see a movie. Each person wants to sit in the front
row, which contains one hundred seats, and each has a favorite seat, chosen randomly
and independently. They enter the row one at a time from the far right. As they walk,
if they reach their favorite seat, they sit, but to avoid stepping over people, if they
encounter a person already seated, they sit to that person’s right. If the seat furthest
to the right is already taken, they sit in a different row. What is the most likely number
of people that will get to sit in the first row?
Solution: 10
Let S(i) be the favorite seat of the ith person, counting from the right. Let P (n) be
the probability that at least n people get to sit. At least n people sit if and only if
S(1) ≥ n, S(2) ≥ n − 1, . . ., S(n) ≥ 1. This has probability:
100 − (n − 1) 100 − (n − 2) 100 100!
P (n) = · ··· = .
100 100 100 (100 − n)! · 100n
The probability, Q(n), that exactly n people sit is
100! 100! 100! · n
P (n) − P (n + 1) = − = .
(100 − n)! · 100n (99 − n)! · 100n+1 (100 − n)! · 100n+1
Now,
Q(n) 100! · n (101 − n)! · 100n n(101 − n) 101n − n2
= · = = ,
Q(n − 1) (100 − n)! · 100n+1 100! · (n − 1) 100(n − 1) 100n − 100
which is greater than 1 exactly when n2 − n − 100 < 0, that is, for n ≤ 10. Therefore,
the maximum value of Q(n) occurs for n = 10.

37. Let a1 , a2 , . . . , a2005 be real numbers such that


a1 · 1 + a2 · 2 + a3 · 3 + ··· + a2005 · 2005 = 0
a1 · 12 + a2 · 22 + a3 · 32 + ··· + a2005 · 20052 = 0
a1 · 13 + a2 · 23 + a3 · 33 + ··· + a2005 · 20053 = 0
.. .. .. .. ..
. . . . .
a1 · 12004 + a2 · 22004 + a3 · 32004 + · · · + a2005 · 20052004 = 0
and

a1 · 12005 + a2 · 22005 + a3 · 32005 + · · · + a2005 · 20052005 = 1.

What is the value of a1 ?


Solution: 1/2004!
The polynomial p(x) = x(x − 2)(x − 3) · · · (x − 2005)/2004! has zero constant term,
has the numbers 2, 3, . . . , 2005 as roots, and satisfies p(1) = 1. Multiplying the nth
equation by the coefficient of xn in the polynomial p(x) and summing over all n gives

a1 p(1) + a2 p(2) + a3 p(3) + · · · + a2005 p(2005) = 1/2004!

(since the leading coefficient is 1/2004!). The left side just reduces to a1 , so 1/2004! is
the answer.

14
38. In how many ways can the set of ordered pairs of integers be colored red and blue such
that for all a and b, the points (a, b), (−1 − b, a + 1), and (1 − b, a − 1) are all the same
color?
Solution: 16
Let ϕ1 and ϕ2 be 90◦ counterclockwise rotations about (−1, 0) and (1, 0), respectively.
Then ϕ1 (a, b) = (−1 − b, a + 1), and ϕ2 (a, b) = (1 − b, a − 1). Therefore, the possible
colorings are precisely those preserved under these rotations. Since ϕ1 (1, 0) = (−1, 2),
the colorings must also be preserved under 90◦ rotations about (−1, 2). Similarly, one
can show that they must be preserved under rotations about any point (x, y), where x
is odd and y is even. Decompose the lattice points as follows:
L1 = {(x, y) | x + y ≡ 0 (mod 2)}
L2 = {(x, y) | x ≡ y − 1 ≡ 0 (mod 2)}
L3 = {(x, y) | x + y − 1 ≡ y − x + 1 ≡ 0 (mod 4)}
L4 = {(x, y) | x + y + 1 ≡ y − x − 1 ≡ 0 (mod 4)}
Within any of these sublattices, any point can be brought to any other through appro-
priate rotations, but no point can be brought to any point in a different sublattice. It
follows that every sublattice must be colored in one color, but that different sublattices
can be colored differently. Since each of these sublattices can be colored in one of two
colors, there are 24 = 16 possible colorings.

1 2 1 2 1 2 1
4 1 3 1 4 1 3
1 2 1 2 1 2 1
3 1 4 1 3 1 4
1 2 1 2 1 2 1
4 1 3 1 4 1 3
1 2 1 2 1 2 1

39. How many regions of the plane are bounded by the graph of
x6 − x5 + 3x4 y 2 + 10x3 y 2 + 3x2 y 4 − 5xy 4 + y 6 = 0?

Solution: 5
The left-hand side decomposes as
(x6 + 3x4 y 2 + 3x2 y 4 + y 6 ) − (x5 − 10x3 y 2 + 5xy 4 ) = (x2 + y 2 )3 − (x5 − 10x3 y 2 + 5xy 4 ).
Now, note that
(x + iy)5 = x5 + 5ix4 y − 10x3 y 2 − 10ix2 y 3 + 5xy 4 + iy 5 ,
so that our function is just (x2 + y 2 )3 − <((x + iy)5 ). Switching to polar coordinates,
this is r6 − <(r5 (cos θ + i sin θ)5 ) = r6 − r5 cos 5θ by de Moivre’s rule. The graph of our
function is then the graph of r6 − r5 cos 5θ = 0, or, more suitably, of r = cos 5θ. This
is a five-petal rose, so the answer is 5.

15
40. In a town of n people, a governing council is elected as follows: each person casts one
vote for some person in the town, and anyone that receives at least five votes is elected
to council. Let c(n) denote the average number of people elected to council if everyone
votes randomly. Find limn→∞ c(n)/n.
Solution: 1 − 65/24e
Let ck (n) denote the expected number of people that will receive exactly k votes. We
will show that limn→∞ ck (n)/n = 1/(e · k!). The probability that any given person
receives exactly k votes, which is the same as the average proportion of people that
receive exactly k votes, is
µ ¶ µ ¶k µ ¶n−k µ ¶n
n 1 n−1 n−1 n(n − 1) · · · (n − k + 1)
· · = · .
k n n n k! · (n − 1)k
¡ ¢n
Taking the limit as n → ∞ and noting that limn→∞ 1 − n1 = 1e gives that the limit
is 1/(e · k!), as desired. Therefore, the limit of the average proportion of the town that
receives at least five votes is
µ ¶
1 1 1 1 1 1 65
1− + + + + =1− .
e 0! 1! 2! 3! 4! 24e

41. There are 42 stepping stones in a pond, arranged along a circle. You are standing
on one of the stones. You would like to jump among the stones so that you move
counterclockwise by either 1 stone or 7 stones at each jump. Moreover, you would like
to do this in such a way that you visit each stone (except for the starting spot) exactly
once before returning to your initial stone for the first time. In how many ways can
you do this?
Solution: 63
Number the stones 0, 1, . . . , 41, treating the numbers as values modulo 42, and let rn
be the length of your jump from stone n. If you jump from stone n to n + 7, then you
cannot jump from stone n + 6 to n + 7 and so must jump from n + 6 to n + 13. That
is, if rn = 7, then rn+6 = 7 also. It follows that the 7 values rn , rn+6 , rn+12 , . . . , rn+36
are all equal: if one of them is 7, then by the preceding argument applied repeatedly,
all of them must be 7, and otherwise all of them are 1. Now, for n = 0, 1, 2, . . . , 42,
let sn be the stone you are on after n jumps. Then sn+1 = sn + rsn , and we have
sn+1 = sn + rsn ≡ sn + 1 (mod 6). By induction, sn+i ≡ sn + i (mod 6); in particular

16
sn+6 ≡ sn , so rsn +6 = rsn . That is, the sequence of jump lengths is periodic with period
6 and so is uniquely determined by the first 6 jumps. So this gives us at most 26 = 64
possible sequences of jumps rs0 , rs1 , . . . , rs41 .
Now, the condition that you visit each stone exactly once before returning to the
original stone just means that s0 , s1 , . . . , s41 are distinct and s42 = s0 . If all jumps are
length 7, then s6 = s0 , so this cannot happen. On the other hand, if the jumps are not
all of length 7, then we claim s0 , . . . , s41 are indeed all distinct. Indeed, suppose si = sj
for some 0 ≤ i < j < 42. Since sj ≡ si + (j − i) (mod 6), we have j ≡ i (mod 6), so
j − i = 6k for some k. Moreover, since the sequence of jump lengths has period 6, we
have
si+6 − si = si+12 − si+6 = · · · = si+6k − si+6(k−1) .
Calling this common value l, we have kl ≡ 0 mod 42. But l is divisible by 6, and
j − i < 42 ⇒ k < 7 means that k is not divisible by 7, so l must be. So l, the sum of
six successive jump lengths, is divisible by 42. Hence the jumps must all be of length
7, as claimed.
This shows that, for the 64 − 1 = 63 sequences of jumps that have period 6 and are not
all of length 7, you do indeed reach every stone once before returning to the starting
point.
42. In how many ways can 6 purple balls and 6 green balls be placed into a 4 × 4 grid of
boxes such that every row and column contains two balls of one color and one ball of
the other color? Only one ball may be placed in each box, and rotations and reflections
of a single configuration are considered different.
Solution: 5184
In each row or column, exactly one box is left empty. There are 4! = 24 ways to choose
the empty spots. Once that has been done, there are 6 ways to choose which two rows
have 2 purple balls each. Now, assume without loss of generality that boxes (1, 1),
(2, 2), (3, 3), and (4, 4) are the empty ones, and that rows 1 and 2 have two purple
balls each. Let A, B, C, and D denote the 2 × 2 squares in the top left, top right,
bottom left, and bottom right corners, respectively (so A is formed by the first two
rows and first two columns, etc.). Let a, b, c, and d denote the number of purple balls
in A, B, C, and D, respectively. Then 0 ≤ a, d ≤ 2, a + b = 4, and b + d ≤ 4, so a ≥ d.
Now suppose we are given the numbers a and d, satisfying 0 ≤ d ≤ a ≤ 2. Fortunately,
the numbers of ways to color the balls in A, B, C, and D are independent of each
other. For example, given a = 1 and d = 0, there are 2 ways to color A and 1 way to
color D and, no matter how the coloring of A is done, there are always 2 ways to color
B and 3 ways to color C. The numbers of ways to choose the colors of all the balls is
as follows:
a\d 0 1 2
0 1 · (1 · 2) · 1 = 2 0 0
1 2 · (2 · 3) · 1 = 12 2 · (1 · 1) · 2 = 4 0
2 1 · (2 · 2) · 1 = 4 1 · (3 · 2) · 2 = 12 1 · (2 · 1) · 1 = 2
In each square above, the four factors are the number of ways of arranging the balls in A,
B, C, and D, respectively. Summing this over all pairs (a, d) satisfying 0 ≤ d ≤ a ≤ 2
gives a total of 36. The answer is therefore 24 · 6 · 36 = 5184.

17
43. Write down an integer N between 0 and 20 inclusive. If more than N teams write
down N , your score is N ; otherwise it is 0.
Remark: Well, maybe you can get 1 point...

44. Write down a set S of positive integers, all greater than 1, such that for each x ∈ S, x
is a proper divisor of (P/x) + 1, where P is the product of all the elements of S. Your
score is 2n, where n = |S|.
Remark: This question was posed by Stefan Znám in 1972. Solutions exist for |S| ≥ 5,
but an explicit solution has been found only for |S| ≤ 13. The simplest solution is S =
{2, 3, 11, 23, 31}. A solution for |S| = 7 is given by S = {2, 3, 11, 17, 101, 149, 3109}.
For more information, see http://mathworld.wolfram.com/ZnamsProblem.html.

45. A binary word is a finite sequence of 0’s and 1’s. A square subword is a subsequence
consisting of two identical chunks next to each other. For example, the word 100101011
contains the square subwords 00, 0101 (twice), 1010, and 11.
Find a long binary word containing a small number of square subwords. Specifically,
write down a binary word of any length n ≤ 50. Your score will be max{0, n − s},
where s is the number of occurrences of square subwords. (That is, each different
square subword will be counted according to the number of times it appears.)
Remark: See http://www.combinatorics.org/Volume 10/PDF/v10i1r12.pdf for
analysis of this problem. The maximum possible score is on the order of 25. In general,
the minimum number of square subwords of a word of length n tends to roughly 0.551n
as n → ∞.

18
Harvard-MIT Mathematics Tournament
February 19, 2005

Team Round A — Solutions

Disconnected Domino Rally [175]

On an infinite checkerboard, the union of any two distinct unit squares is called a (dis-
connected) domino. A domino is said to be of type (a, b), with a ≤ b integers not both zero,
if the centers of the two squares are separated by a distance of a in one orthogonal direction
and b in the other. (For instance, an ordinary connected domino is of type (0, 1), and a
domino of type (1, 2) contains two squares separated by a knight’s move.)

Each of the three pairs of squares above forms a domino of type (1, 2).
Two dominoes are said to be congruent if they are of the same type. A rectangle is said to
be (a, b)-tileable if it can be partitioned into dominoes of type (a, b).

1. [15] Prove that for any two types of dominoes, there exists a rectangle that can be
tiled by dominoes of either type.
Solution: Note that a type (a, b) domino tiles a max{1, 2a}×2b rectangle (see diagram
for a > 0). Then both type (a, b) and type (a0 , b0 ) dominoes tile a (max{1, 2a} ·
max{1, 2a0 }) × (2b · 2b0 ) rectangle.

b b

2. [25] Suppose 0 < a ≤ b and 4 - mn. Prove that the number of ways in which an m × n
rectangle can be partitioned into dominoes of type (a, b) is even.
Solution: If the rectangle is tileable, it can be partitioned into an odd number of
dominoes. Consider the reflection of the partitioned rectangle over one axis. This
gives another partition of the rectangle. In fact, it cannot be the same partition, for
suppose it were. Then we can pair each domino with its reflected image, but since
there are an odd number of dominoes, one must reflect into itself. Since a > 0, this is
not possible. Therefore, we can pair off partitions and their reflections, and it follows
that the total number of partitions is even.

1
3. [10] Show that no rectangle of the form 1 × k or 2 × n, where 4 - n, is (1, 2)-tileable.
Solution: The claim is obvious for 1 × k rectangles. For the others, color the first two
columns black, the next two white, the next two black, etc. Each (1, 2) domino will
contain one square of each color, so in order to be tileable, the rectangle must contain
the same number of black and white squares. This is the case only when 4 | n.
4. [35] Show that all other rectangles of even area are (1, 2)-tileable.
Solution: First, we demonstrate that there exist (1, 2)-tilings of 2 × 4, 3 × 4, 3 × 6,
and 5 × 6 rectangles.

Now, notice that by combining these rectangles, we can form any rectangle of even
area other than those described in the previous problem: using the first rectangle, we
can form any 2 × n rectangle with 4 | n. By combining the first two, we can form any
m × 4 rectangle with m ≥ 2, and by combining the last three, we can form any m × 6
rectangle with m ≥ 3. From these, we can form any m × n rectangle with m ≥ 3 and
n even and greater than 2, completing the proof.
5. [25] Show that for b even, there exists some M such that for every n > M , a 2b × n
rectangle is (1, b)-tileable.
Solution: Recall from above that we can tile a 2 × 2b rectangle. Four columns of a
(b + 1) × 2b rectangle can be tiled as shown below, and repeating this 2b times tiles
the entire rectangle. Since any integer at least b can be written as a positive linear
combination of 2 and b + 1, we can tile any 2b × n rectangle for n ≥ b.

6. [40] Show that for b even, there exists some M such that for every m, n > M with mn
even, an m × n rectangle is (1, b)-tileable.
Solution: By the diagram below, it is possible to tile a (2b + 2) × (4b + 1) rectangle.
Since we can already tile a (2b + 2) × 2b rectangle by above, and 2b is relatively prime
to 4b + 1, this will allow us to tile any (2b + 2) × n rectangle for n sufficiently large.
Combining this with the previous problem, this will allow us to tile any m×n rectangle
for m and n sufficiently large and m even, completing the proof.
To tile the (2b + 2) × (4b + 1) rectangle, we first tile the following piece:

2
2

2b-1 2b

b+1

This is then combined with two 2 × 2b rectangles, a 2b × b rectangle, and a 2b × (2b + 1)


rectangle as follows:

2 x 2b

2b x 2b+1

2b x b

2 x 2b

7. [25] Prove that neither of the previous two problems holds if b is odd.
Solution: Color the grid black and white in checkerboard fashion. Then if b is odd,
the two squares that make up a (1, b) domino always have the same color. Therefore,
for an m × n rectangle to be (1, b)-tileable, it must have an even number of squares
of each color. Then for any M , we can choose m and n larger than M such that n is
odd and 4 - m. A 2b × n rectangle and an m × n rectangle then contain bn and mn/2
squares of each color, respectively. Since both bn and mn/2 are odd, neither of these
rectangles is (1, b)-tileable.

An Interlude — Discovering One’s Roots [100]

A kth root of unity is any complex number ω such that ω k = 1. You may use the following
facts: if ω 6= 1, then
1 + ω + ω 2 + · · · + ω k−1 = 0,
and if 1, ω, . . . , ω k−1 are distinct, then

(xk − 1) = (x − 1)(x − ω)(x − ω 2 ) · · · (x − ω k−1 ).

8. [25] Suppose x is a fifth root of unity. Find, in radical form, all possible values of

1 x x2 x3
2x + + + + .
1 + x 1 + x2 1 + x3 1 + x4

3
Solution: Note that
x x2 x6 x4 x4 + x6 1
2
+ 3
= 2
+ 2 5
= 2
= x4 = , and
1+x 1+x 1+x x +x 1+x x
1 x3 x5 x4 x4 + x5 1
+ 4
= + 5
= = x4 = .
1+x 1+x 1+x x+x 1+x x
Therefore, the sum is just 2x + x . If x = 1, this is 4. Otherwise, let y = x + x1 . Then
2

x satisfies
µ ¶ µ ¶
2 3 4 2 1 1
0=1+x+x +x +x = x + 2 +2 + x+ − 1 = y 2 + y − 1,
x x
√ √
so solving this quadratic yields y = −1±2 5 , or 2y = −1 ± 5. Since each value of y can
correspond to only 2 possible values of x, and there are 4 possible values of x besides

1, both of these values for y are possible, which yields the answers, 4 and −1 ± 5.
9. [25] Let A1 A2 . . . Ak be a regular k-gon inscribed in a circle of radius 1, and let P
be a point lying on or inside the circumcircle. Find the maximum possible value of
(P A1 )(P A2 ) · · · (P Ak ).
Solution: Place the vertices at the kth roots of unity, 1, ω, . . . , ω k−1 , and place P at
some complex number p. Then
k−1
Y
2
((P A1 )(P A2 ) · · · (P Ak )) = |p − ω i |2
i=0
= |p − 1|2 ,
k

since xk − 1 = (x − 1)(x − ω) · · · (x − ω k−1 ). This is maximized when pk is as far as


possible from 1, which occurs when pk = −1. Therefore, the maximum possible value
of (P A1 )(P A2 ) · · · (P Ak ) is 2.
10. [25] Let P be a regular k-gon inscribed in a circle of radius 1. Find the sum of the
squares of the lengths of all the sides and diagonals of P .
Solution: Place the vertices of P at the kth roots of unity, 1, ω, ω 2 , . . . , ω k−1 . We
will first calculate the sum of the squares of the lengths of the sides and diagonals that
contain the vertex 1. This is
k−1
X k−1
X
i 2
|1 − ω | = (1 − ω i )(1 − ω̄ i )
i=0 i=0
k−1
X
= (2 − ω i − ω̄ i )
i=0
k−1
X
= 2k − 2 ωi
i=0
= 2k,
using the fact that 1 + ω + · · · + ω k−1 = 0. Now, by symmetry, this is the sum of the
squares of the lengths of the sides and diagonals emanating from any vertex. Since
there are k vertices and each segment has two endpoints, the total sum is 2k · k/2 = k 2 .

4
11. [25] Let P (x) = an xn + an−1 xn−1 + · · · + a0 be a polynomial with real coefficients,
an 6= 0. Suppose every root of P is a root of unity, but P (1) 6= 0. Show that the
coefficients of P are symmetric; that is, show that an = a0 , an−1 = a1 , . . .
Solution: Since the coefficients of P are real, the complex conjugates of the roots of
P are also roots of P . Now, if x is a root of unity, then x−1 = x̄. But the roots of
xn P (x−1 ) = a0 xn + a1 xn−1 + · · · + an
are then just the complex conjugates of the roots of P , so they are the roots of P .
Therefore, P (x) and xn P (x−1 ) differ by a constant multiple c. Since an = ca0 and
a0 = can , c is either 1 or −1. But if it were −1, then
1
P (1) = an + an−1 + · · · + a0 = ((an + a0 ) + (an−1 + a1 ) + · · · + (a0 + an )) = 0,
2
a contradiction. Therefore c = 1, giving the result.

Early Re-tile-ment [125]


Let S = {s0 , . . . , sn } be a finite set of integers, and define S + k = {s0 + k, . . . , sn + k}. We
say that two sets S and T are equivalent, written S ∼ T , if T = S + k for some k. Given a
(possibly infinite) set of integers A, we say that S tiles A if A can be partitioned into subsets
equivalent to S. Such a partition is called a tiling of A by S.

12. [20] Suppose the elements of A are either bounded below or bounded above. Show
that if S tiles A, then it does so uniquely, i.e., there is a unique tiling of A by S.
Solution: Assume A is bounded below; the other case is analogous. In choosing
the tiling of A, note that there is a unique choice for the set S0 that contains the
minimum element of A. But then there is a unique choice for the set S1 that contains
the minimum element of A\S0 . Continuing in this manner, there is a unique choice
for the set containing the minimum element not yet covered, so we see that the tiling
is uniquely determined.
13. [35] Let B be a set of integers either bounded below or bounded above. Then show
that if S tiles all other integers Z\B, then S tiles all integers Z.
Solution: Assume B is bounded above; the other case is analogous. Let a be the dif-
ference between the largest and smallest element of S. Denote the sets in the partition
of Z\B by Sk , k ∈ Z, such that the minimum element of Sk , which we will denote ck ,
is strictly increasing as k increases. Since B is bounded above, there exists some k0
such that ck0 is larger than all the elements of B. Let

[
Tl = Sk .
k=l

Suppose l ≥ k0 . Note that any element in Sk , k < l, is at most cl − 1 + a. Therefore,


Tl contains all integers that are at least cl + a. Since the minimum element of Tl is
cl , Tl is completely determined by which of the integers cl + 1, cl + 2, . . . , cl + a − 1 it
contains. This implies that there are at most 2a−1 possible nonequivalent sets Tl when
l ≥ k0 (here we extend the notion of equivalence to infinite sets in the natural way.)
By the Pigeonhole Principle, there must then be some l2 > l1 ≥ k0 such that Tl1 ∼ Tl2 .
But then it is easy to see that the set Sl1 ∪ Sl1 +1 ∪ · · · ∪ Sl2 −1 tiles Z, so S tiles Z.

5
14. [35] Suppose S tiles the natural numbers N. Show that S tiles the set {1, 2, . . . , k} for
some positive integer k.
Solution: Using the notation from above, we can find l1 < l2 such that Tl1 ∼ Tl2 . By
the same argument as in problem 12, as long as Tl1 6= N, there is a unique choice for
Sl1 −1 that contains the largest integer not in Tl1 . Since the same can be said for Tl2 ,
we must have that Tl1 −1 ∼ Tl2 −1 . Continuing in this manner, we find that there must
exist some l for which N ∼ Tl ; then S tiles N\Tl = {1, 2, · · · , cl − 1}.

15. [35] Suppose S tiles N. Show that S is symmetric; that is, if −S = {−sn , . . . , −s0 },
show that S ∼ −S.
Solution: Assume without loss of generality that the minimum element of S is 0. By
the previous problem, S P tiles the set {1, 2, . . . , k} for some positive integer k. Then let
P (x) be the polynomial ni=0 xsi . To say that the set {1, 2, . . . , k}, or equivalently the
set {0, 1, . . . , k − 1}, is tiled by S is to say that there is some polynomial Q(x) with
coefficients 0 or 1 such that P (x)Q(x) = 1 + x + · · · + xk−1 = (xk − 1)/(x − 1). It follows
that all the roots of P (x) are roots of unity, but P (1) 6= 0. By question 11 above, this
implies that P (x) is symmetric. Therefore, s0 + sn = s1 + sn−1 = · · · = sn + s0 , so S
is symmetric.

6
Harvard-MIT Mathematics Tournament
February 19, 2005

Team Round B — Solutions

Disconnected Domino Rally [150]

On an infinite checkerboard, the union of any two distinct unit squares is called a (dis-
connected) domino. A domino is said to be of type (a, b), with a ≤ b integers not both zero,
if the centers of the two squares are separated by a distance of a in one orthogonal direction
and b in the other. (For instance, an ordinary connected domino is of type (0, 1), and a
domino of type (1, 2) contains two squares separated by a knight’s move.)

Each of the three pairs of squares above forms a domino of type (1, 2).

Two dominoes are said to be congruent if they are of the same type. A rectangle is said to
be (a, b)-tileable if it can be partitioned into dominoes of type (a, b).

1. [15] Let 0 < m ≤ n be integers. How many different (i.e., noncongruent) dominoes
can be formed by choosing two squares of an m × n array?
Solution: We must have 0 ≤ a < m, 0 ≤ b < n, a ≤ b, and a and b not both 0. The
number of pairs (a, b) with b < a < m is m(m − 1)/2, so the answer is

m(m − 1) m2 − m + 2
mn − − 1 = mn − .
2 2

2. [10] What are the dimensions of the rectangle of smallest area that is (a, b)-tileable?
Solution: If a = 0, then a 1 × 2b rectangle is tileable in an obvious way. If a > 0, then
a 2a × 2b rectangle is tileable by dividing the rectangle into quarters and pairing each
square with a square from the diagonally opposite quarter. The answer is therefore
max{1, 2a} × 2b. Minimality of area follows from the next question.

b b

1
3. [20] Prove that every (a, b)-tileable rectangle contains a rectangle of these dimensions.
Solution: An m × n rectangle, m ≤ n, does not contain a max{1, 2a} × 2b rectangle if
and only if m < 2a or n < 2b. But if either of these is the case, then the square closest
to the center of the rectangle cannot be paired with any other square of the rectangle
to form a domino of type (a, b), so the rectangle cannot be (a, b)-tileable.

4. [30] Prove that an m × n rectangle is (b, b)-tileable if and only if 2b | m and 2b | n.


Solution: Color the first b rows of an m×n rectangle black, the next b white, the next
b black, etc. Any (b, b) domino covers one square of each color, so for the rectangle to
be (b, b)-tileable, there must be the same number of black squares as white squares.
This is possible only when 2b | m. Similarly, we must have 2b | n. It is easy to exhibit
a tiling of all such rectangles, proving the claim. (It is also possible to prove this using
the lemma described below.)

5. [35] Prove that an m × n rectangle is (0, b)-tileable if and only if 2b | m or 2b | n.


Solution: It is easy to exhibit a tiling of such a rectangle. The other direction follows
from below. (It is also possible to prove this using a coloring argument as above:
starting in one corner, divide the board into b × b blocks and color them checkerboard
fashion. The details are left to the reader.)

6. [40] Let k be an integer such that k | a and k | b. Prove that if an m × n rectangle is


(a, b)-tileable, then 2k | m or 2k | n.
Solution: We prove the following lemma.

Lemma. Let k be a positive integer such that k|a and k|b. Then an m ¥ mצn rectangle
§ mis¨
0 0 a b 0
(a, b)-tileable
¥ ¦ if and
§ ¨only if an m × n rectangle is ( ,
k k
)-tileable for k
≤ m ≤ k
and nk ≤ n0 ≤ nk . (Here, bxc denotes the greatest integer less than or equal to x,
while dxe denotes the least integer greater than or equal to x.)

Proof. Number the rows and columns in order. For each pair 0 ≤ i, j < k, consider
the set of squares in a row congruent to i modulo k and in a column congruent to j
modulo k. If one square of a type (a, b) domino lies in this set, then so does the other.
We can therefore partition the rectangle into these sets and then tile ¥ m ¦these 0sets§instead.
¨
Each
¥n¦ such set
§ is¨a rectangular array of dimensions m 0
× n 0
, with k
≤m ≤ m k
and
0 n a b
k
≤ n ≤ k , and a type (a, b) domino on the original rectangle is a type ( k , k )
domino on this new array. Since all possible pairs (m0 , n0 ) occur, the result follows.
¥ ¦ §m¨
Suppose 2k - m and 2k - n. Then at least one of mk
and k
is odd, so we can choose
m0 odd. Likewise we can choose n0 odd. But then an m0 × n0 rectangle has odd area
and so cannot be tileable, implying that the m × n rectangle is not tileable.

An Interlude — Discovering One’s Roots [100]

A kth root of unity is any complex number ω such that ω k = 1.

2
7. [15] Find a real, irreducible quartic polynomial with leading coefficient 1 whose roots
are all twelfth roots of unity.
Solution: All twelfth roots of unity are roots of
x12 − 1 = (x6 − 1)(x6 + 1)
= (x3 − 1)(x3 + 1)(x6 + 1)
= (x − 1)(x2 + x + 1)(x + 1)(x2 − x + 1)(x2 + 1)(x4 − x2 + 1),
so the answer is x4 − x2 + 1.
8. [25] Let x and y be two kth roots of unity. Prove that (x + y)k is real.
Solution: Note that
k µ ¶
X
k k
(x + y) = xi y k−i
i=0
i
k µ ¶
1X k
= (xi y k−i + xk−i y i )
2 i=0 i

by pairing the ith and (k − i)th terms. But xk−i y i = (xi y k−i )−1 since x and y are kth
roots of unity. Moreover, since x and y have absolute value 1, so does xi y k−i , so xk−i y i
is in fact its complex conjugate. It follows that their sum is real, thus so is (x + y)k .
This can also be shown geometrically. The argument of x (the angle between the vector
x and the positive x-axis) is an integer multiple of 2π
k
, as is the argument of y. Since
x + y bisects the angle between x and y, its argument is an integer multiple of πk .
Multiplying this angle by k gives a multiple of π, so (x + y)k is real.
9. [30] Let x and y be two distinct roots of unity. Prove that x + y is also a root of unity
if and only if xy is a cube root of unity.
Solution: This is easiest to see geometrically. The vectors corresponding to x, y, and
−x − y sum to 0, so they form a triangle. In order for them all to be roots of unity,
they must all have length one, so the triangle must be equilateral. Therefore the angle
between x and y is ± 2π
3
, that is, xy is a cube root of unity.
10. [30] Let x, y, and z be three roots of unity. Prove that x + y + z is also a root of unity
if and only if x + y = 0, y + z = 0, or z + x = 0.
Solution: Again, we consider the geometric picture. Arrange the vectors x, y, z, and
−x − y − z so as to form a quadrilateral. If they are all roots of unity, they form a
quadrilateral all of whose side lengths are 1. If the quadrilateral is degenerate, then two
of the vectors sum to 0, which implies the result. But even if it is not degenerate, the
quadrilateral must be a rhombus, and since opposite sides of a rhombus are parallel,
this again implies that two of the four roots of unity sum to 0.

Early Re-tile-ment [150]


Let S = {s0 , . . . , sn } be a finite set of integers, and define S + k = {s0 + k, . . . , sn + k}.
We say that S and T are equivalent, written S ∼ T , if T = S + k for some k. Given a
(possibly infinite) set of integers A, we say that S tiles A if A can be partitioned into subsets
equivalent to S. Such a partition is called a tiling of A by S.

3
11. [20] Find all sets S with minimum element 1 that tile A = {1, . . . , 12}.
Solution: This can be done by brute force. Alternatively, note that if P (x) and Q(x)
are polynomials with coefficients either 0 or 1 with P (x)Q(x) = x + x2 + · · · + x12 ,
then the set consisting of the exponents of nonzero terms in P tiles A. Either way,
we find that S is one of the following: {1}, {1, 2}, {1, 3}, {1, 4}, {1, 7}, {1, 2, 3},
{1, 3, 5}, {1, 5, 9}, {1, 2, 3, 4}, {1, 2, 7, 8}, {1, 4, 7, 10}, {1, 2, 3, 4, 5, 6}, {1, 2, 3, 7, 8, 9},
{1, 2, 5, 6, 9, 10}, {1, 3, 5, 7, 9, 11}, or A itself.

12. [35] Let A be a finite set with more than one element. Prove that the number of
nonequivalent sets S which tile A is always even.
Solution: Suppose A can be partitioned into sets S0 , . . . , Sm , each equivalent to S.
(This partition is unique, simply by choosing S0 to contain the smallest element of A,
S1 the smallest element of A not in S0 , etc.) Then if Sj = S + tj , each element of A
can be written uniquely as si + tj for some i and j. But then the set T containing all
tj also tiles A by translation by the si . We cannot have S and T equivalent, for if so,
since A has more than one element, both S and T would as well. This would imply
that s0 + t1 = s1 + t0 , an overlap in the tiling of A. We can thus pair together S and
T , each of which tile A, so that the total number of sets tiling A must be even.

13. [25] Exhibit a set S which tiles the integers Z but not the natural numbers N.
Solution: One example is {1, 3, 4, 6}. Since its elements are all distinct modulo 4, it
tiles Z by translation by multiples of 4. On the other hand, it is easy to see that it
cannot tile N: 1 is contained in {1, 3, 4, 6}, but then there is no possible set for 2 to
be contained in that does not overlap. Another example is {1, 2, 6}.

14. [30] Suppose that S tiles the set of all integer cubes. Prove that S has only one
element.
Solution: Let the difference between the smallest and largest element of S be a. Then
the set equivalent to S that contains b3 can only contain integers between b3 − a and
b3 + a, inclusive. But for sufficiently large b, b3 is the only cube in this range, so S can
only have one element.

15. [40] Suppose that S tiles the set of odd prime numbers. Prove that S has only one
element.
Solution: Consider the set S0 equivalent to S that contains 3. If it contains 5 but
not 7, then the set S1 equivalent to S containing 7 must contain 9, which is not prime.
Likewise, S0 cannot contain 7 but not 5, because then the set S1 containing 5 must
contain 9. Suppose S0 contains 3, 5, and 7. Then any other set S1 of the tiling contains
elements p, p + 2, and p + 4. But not all of these can be prime, because one of them is
divisible by 3. Finally, suppose S0 contains 3 and has second-smallest element p > 7.
Then the set S1 containing 5 does not contain 7 but does contain p + 2, and the set S2
containing 7 contains p + 4. But as before, not all of p, p + 2, and p + 4 can be prime.
Therefore S has no second-smallest element, so it has only one element.

4
IXth Annual Harvard-MIT Mathematics Tournament
Saturday 25 February 2006

Algebra Test: Solutions


1. Larry can swim from Harvard to MIT (with the current of the Charles River) in 40
minutes, or back (against the current) in 45 minutes. How long does it take him to
row from Harvard to MIT, if he rows the return trip in 15 minutes? (Assume that the
speed of the current and Larry’s swimming and rowing speeds relative to the current
are all constant.) Express your answer in the format mm:ss.
Answer: 14:24
Solution: Let the distance between Harvard and MIT be 1, and let c, s, r denote
the speeds of the current and Larry’s swimming and rowing, respectively. Then we are
given
1 9 1 8 1 24
s+c= = , s−c= = , r−c= = ,
40 360 45 360 15 360
so
9 − 8 + 24 25
r + c = (s + c) − (s − c) + (r − c) = = ,
360 360
and it takes Larry 360/25 = 14.4 minutes, or 14:24, to row from Harvard to MIT.
2. Find all real solutions (x, y) of the system x2 + y = 12 = y 2 + x.
√ √ ! √ √ !
1+3 5 1−3 5 1−3 5 1+3 5
Answer: (3, 3), (−4, −4), , , ,
2 2 2 2
2 2
Solution: We have x +y = y +x which can be written as (x−y)(x+y −1) = 0. The
case x = y yields x2 + x − 12 = 0, hence (x, y) = (3, 3) or (−4, −4). The√case y = 1 −

x
2 2 1± 1+44 1±3 5
yields x + 1 − x − 12 = x − x − 11 = 0 which has solutions x = 2
= 2 .
The other two solutions follow.
3. The train schedule in Hummut is hopelessly unreliable. Train A will enter Intersection
X from the west at a random time between 9:00 am and 2:30 pm; each moment in that
interval is equally likely. Train B will enter the same intersection from the north at
a random time between 9:30 am and 12:30 pm, independent of Train A; again, each
moment in the interval is equally likely. If each train takes 45 minutes to clear the
intersection, what is the probability of a collision today?
13
Answer: 48
Solution: Suppose we fix the time at which Train B arrives at Intersection X; then
call the interval during which Train A could arrive (given its schedule) and collide with
Train B the “disaster window.”
We consider two cases:
(i) Train B enters Intersection X between 9:30 and 9:45. If Train B arrives at 9:30,
the disaster window is from 9:00 to 10:15, an interval of 1 14 hours. If Train B
arrives at 9:45, the disaster window is 1 21 hours long. Thus, the disaster window
has an average length of (1 14 + 1 12 ) ÷ 2 = 118
. From 9:00 to 2:30 is 5 21 hours. The
probability of a collision is thus 11
8
÷ 5 12 = 14 .

1
(ii) Train B enters Intersection X between 9:45 and 12:30. Here the disaster window
is always 1 21 hours long, so the probability of a collision is 1 12 ÷ 5 12 = 11
3
.

From 9:30 to 12:30 is 3 hours. Now case (i) occurs with probability 14 ÷ 3 = 12 1
, and
11
case (ii) occurs with probability 12 . The overall probability of a collision is therefore
1
· 1 + 11
12 4
· 3 = 48
12 11
1
+ 14 = 48
13
.

4. Let a1 , a2 , . . . be a sequence defined by a1 = a2 = 1 and an+2 = an+1 + an for n ≥ 1.


Find ∞
X an
n+1
.
n=1
4

1
Answer: 11
Solution: Let X denote the desired sum. Note that
1 1 2 3 5
X= 2
+ 3 + 4 + 5 + 6 + ...
4 4 4 4 4
1 1 2 3 5 8
4X = 1
+ 2 + 3 + 4 + 5 + 6 + ...
4 4 4 4 4 4
1 1 2 3 5 8 13
16X = 0 + 1 + 2 + 3 + 4 + 5 + 6 + . . .
4 4 4 4 4 4 4
so that X + 4X = 16X − 1, and X = 1/11.

5. Tim has a working analog 12-hour clock with two hands that run continuously (instead
of, say, jumping on the minute). He also has a clock that runs really slow—at half
the correct rate, to be exact. At noon one day, both clocks happen to show the exact
time. At any given instant, the hands on each clock form an angle between 0◦ and
180◦ inclusive. At how many times during that day are the angles on the two clocks
equal?
Answer: 33
Solution: A tricky thing about this problem may be that the angles on the two
clocks might be reversed and would still count as being the same (for example, both
angles could be 90◦ , but the hour hand may be ahead of the minute hand on one clock
and behind on the other).
Let x, −12 ≤ x < 12, denote the number of hours since noon. If we take 0◦ to mean
upwards to the “XII” and count angles clockwise, then the hour and minute hands of
the correct clock are at 30x◦ and 360x◦ , and those of the slow clock are at 15x◦ and
180x◦ . The two angles are thus 330x◦ and 165x◦ , of course after removing multiples of
360◦ and possibly flipping sign; we are looking for solutions to

330x◦ ≡ 165x◦ (mod 360◦ ) or 330x◦ ≡ −165x◦ (mod 360◦ ).

In other words,
360 | 165x or 360 | 495x.
Or, better yet,
165 11 495 11
x = x and/or x= x
360 24 360 8

2
must be an integer. Now x is any real number in the range [−12, 12), so 11x/8 ranges
in [−16.5, 16.5), an interval that contains 33 integers. For any value of x such that
11x/24 is an integer, of course 11x/8 = 3 × (11x/24) is also an integer, so the answer
is just 33.
√ √ √
6. Let a, b, c be the roots of x3 − 9x2 + 11x − 1 = 0, and let s = a + b + c. Find
s4 − 18s2 − 8s.
Answer: −37
Solution: First of all, as the left side of the first given equation takes values −1, 2,
−7, and 32 when x = 0,√1, 2, and
√ 3, √ respectively, we know that a, b, and c are distinct
positive reals. Let t = ab + bc + ca, and note that
s2 = a + b + c + 2t = 9 + 2t,

t2 = ab + bc + ca + 2 abcs = 11 + 2s,
s4 = (9 + 2t)2 = 81 + 36t + 4t2 = 81 + 36t + 44 + 8s = 125 + 36t + 8s,
18s2 = 162 + 36t,
so that s4 − 18s2 − 8s = −37.
7. Let
f (x) = x4 − 6x3 + 26x2 − 46x + 65.
Let the roots of f (x) be ak + ibk for k = 1, 2, 3, 4. Given that the ak , bk are all integers,
find |b1 | + |b2 | + |b3 | + |b4 |.
Answer: 10
Solution: The roots of f (x) must come in complex-conjugate pairs. We can then
say that a1 = a2 and b1 = −b2 ; a3 = a4 and b3 = −b4 . The constant term of f (x) is
the product of these, so 5 · 13 = (a1 2 + b1 2 )(a3 2 + b3 2 ). Since ak and bk are integers
for all k, and it is simple to check that 1 and i are not roots of f (x), we must have
a1 2 + b1 2 = 5 and a3 2 + b3 2 = 13. The only possible ways to write these sums with
positive integers is 12 + 22 = 5 and 22 + 32 = 13, so the values of a1 and b1 up to sign
are 1 and 2; and a3 and b3 up to sign are 2 and 3. From the x3 coefficient of f (x), we
get that a1 + a2 + a3 + a4 = 6, so a1 + a3 = 3. From the limits we already have, this
tells us that a1 = 1 and a3 = 2. Therefore b1 , b2 = ±2 and b3 , b4 = ±3, so the required
sum is 2 + 2 + 3 + 3 = 10.
8. Solve for all complex numbers z such that z 4 + 4z 2 + 6 = z.
√ √
1 ± i 7 −1 ± i 11
Answer: ,
2 2
2
Solution: Rewrite the given equation as (z 2 + 2) + 2 = z. Observe that a solution
to z 2 + 2 = z is a solution
√ of the quartic by substitution of the left hand side into itself.
1±i 7 2
This gives z = 2 . But now, we know that z 2 − z + 2 divides into (z 2 + 2) − z + 2 =
z 4 + 4z 2 − z + 6. Factoring it out, we obtain (z 2 − z + 2) √
(z 2 + z + 3) = z 4 + 4z 2 − z + 6.
Finally, the second term leads to the solutions z = −1±i2 11 .
9. Compute the value of the infinite series

X n4 + 3n2 + 10n + 10
n=2
2n · (n4 + 4)

3
Answer: 11 10
Solution: We employ the difference of squares identity, uncovering the factorization
of the denominator: n4 + 4 = (n2 + 2)2 − (2n)2 = (n2 − 2n + 2)(n2 + 2n + 2). Now,

n4 + 3n2 + 10n + 10 3n2 + 10n + 6


= 1 +
n4 + 4 n4 + 4
4 1
= 1+ 2 − 2
n − 2n + 2 n + 2n + 2
∞ 4 2 ∞
X n + 3n + 10n + 10 X 1 4 1
=⇒ n 4
= n
+ n 2
− n 2
n=2
2 · (n + 4) n=2
2 2 · (n − 2n + 2) 2 · (n + 2n + 2)

1 X 1 1
= + −
2 n=2 2n−2 · ((n − 1)2 + 1) 2n · ((n + 1)2 + 1)

1 1 1
The last series telescopes to 2
+ 10
, which leads to an answer of 2
+ 12 + 1
10
= 11
10
.

10. Determine the maximum value attained by

x4 − x2
x6 + 2x3 − 1
over real numbers x > 1.
Answer: 16
Solution: We have the following algebra:

x4 − x2 x − x1
= 1
x6 + 2x3 − 1 x3 + 2 − x3
1
x− x
=
1 3 1
 
x− x
+2+3 x− x
x − x1 1
≤ 1
 1
=
3 x− x +3 x− x 6
3
where x − x1 + 1 + 1 ≥ 3 x − x1 in the denominator was deduced by the √AM-GM


inequality. As a quick check, equality holds where x − x1 = 1 or when x = 1+2 5 .

4
IXth Annual Harvard-MIT Mathematics Tournament
Saturday 25 February 2006

Calculus Test: Solutions

1. A nonzero polynomial f (x) with real coefficients has the property that f (x) = f 0 (x)f 00 (x).
What is the leading coefficient of f (x)?
1
Answer: 18
Solution: Suppose that the leading term of f (x) is cxn , where c 6= 0. Then the
leading terms of f 0 (x) and of f 00 (x) are cnxn−1 and cn(n − 1)xn−2 , respectively, so
cxn = cnxn−1 · cn(n − 1)xn−2 , which implies that n = (n − 1) + (n − 2), or n = 3, and
1
c = cn · cn(n − 1) = 18c2 , or c = 18 .
ex cos x − 1 − x
2. Compute lim .
x→0 sin(x2 )
Answer: 21
Solution: Let’s compute all the relevant Maclaurin series expansions, up to the
quadratic terms:
1
x cos x = x + . . . , ex cos x = 1 + x + x2 + . . . , sin(x2 ) = x2 + . . . ,
2
so
1 2
ex cos x − 1 − x 2
x + ... 1
lim 2
= lim 2
= .
x→0 sin(x ) x→0 x + . . . 2
3. At time 0, an ant is at (1, 0) and a spider is at (−1, 0). The ant starts walking
counterclockwise along the unit circle, and the spider starts creeping to the right along
the x-axis. It so happens that the ant’s horizontal speed is always half the spider’s.
What will the shortest distance ever between the ant and the spider be?

14
Answer: 4
Solution: Picture an instant in time where the ant and spider have x-coordinates a
and s, respectively. If 1 ≤ s ≤ 3, then a ≤ 0, and the distance between the bugs is at
least 1. If s > 3, then, needless to say the distance between the bugs is at least 2. If
−1 ≤ s ≤ 1, then s = 1 − 2a, and the distance between the bugs is
r
p √ (8a − 3)2 + 7
(a − (1 − 2a))2 + (1 − a2 ) = 8a2 − 6a + 2 = ,
8
p
which attains the minimum value of 7/8 when a = 3/8.

X k4
4. Compute .
k=1
k!
Answer: 15e
Solution: Define, for non-negative integers n,

X kn
Sn := ,
k=0
k!

1
where 00 = 1 when it occurs. Then S0 = e, and, for n ≥ 1,
∞ ∞ ∞ ∞ n−1 
kn kn (k + 1)n (k + 1)n−1

X X X X X n−1
Sn = = = = = Si ,
k=0
k! k=1
k! k=0
(k + 1)! k=0
k! i=0
i

so we can compute inductively that S1 = e, S2 = 2e, S3 = 5e, and S4 = 15e.

5. Compute Z 1
dx
√ √
0 x+ 3x

Answer: 5 − 6 ln 2
Solution: Writing x = u6 so that dx = 6u5 du, we have
Z 1 Z 1
dx 6u5 du
√ √ = 3 2
0 x+ 3x 0 u +u
Z 1 3
u du
= 6
u+1
Z0 1  
2 1
= 6 u −u+1− du
0 u+1
1 !
u 3 u2
= 6 − + u − ln |u + 1| = 5 − 6 ln(2)
3 2 0

6. A triangle with vertices at (1003, 0), (1004, 3), and (1005, 1) in the xy-plane is revolved
all the way around the y-axis. Find the volume of the solid thus obtained.
Answer: 5020π
Solution: Let T ⊂ R2 denote the triangle, including its interior. Then T ’s area is
5/2, and its centroid is (1004, 4/3), so
Z
5
x dx dy = · 1004 = 2510.
(x,y)∈T 2

We are interested in the volume


Z
2πx dx dy,
(x,y)∈T

but this is just 2π · 2510 = 5020π.

7. Find all positive real numbers c such that the graph of f : R → R given by f (x) =
x3 − cx has the property that the circle of curvature at any local extremum is centered
at a point on the x-axis.

Answer: 23
0 2
p
Solution:
p The equation 0 =
p f (x) = 3x − c has two real roots: ± c/3. Let
00
a := c/3. As f (−a) = −6 c/3 < 0, f has a unique local maximum at x = −a.

2
Because f has half-turn symmetry about the origin, it suffices to consider this local
extremum. The radius of curvature at any local extremum is
1 1
r(x) = = ,
|f 00 (x)| 6|x|
so the condition in the problem is equivalent to

r(−a) = f (−a)
1
= −a(a2 − c)
6a
1 = 6a2 (c − a2 ) = 2c(2c/3)

c = 3/2.

8. Compute
Z π/3
x tan2 (x)dx
0
√ 2
Answer: π 3 3 − π18 − ln 2
Solution: We have
Z π/3 Z π/3  
2 1
x tan (x)dx = x −1 + dx
0 0 cos2 (x)
π/3 Z π/3
x2 xdx
=− +
2 0 0 cos2 (x)
π/3 π/3 Z π/3 !
x2 dx
= − + x tan(x) − tan(x)dx (u = x; dv = )
2 0 0 0 cos2 (x)
π/3 √
x2 π2 π 3
= − + x tan(x) + ln |cos(x)| = − +
− ln(2)
2 0 18 3

9. Compute the sum of all real numbers x such that

2x6 − 3x5 + 3x4 + x3 − 3x2 + 3x − 1 = 0

Answer: − 12
Solution: The carefully worded problem statement suggests that repeated roots
might be involved (not to be double counted), as well as complex roots (not to be
counted). Let P (x) = 2x6 − 3x5 + 3x4 + x3 − 3x2 + 3x − 1. Now, a is a double root of
the polynomial P (x) if and only if P (a) = P 0 (a) = 0. Hence, we consider the system

P (a) = 2a6 − 3a5 + 3a3 + a3 − 3a2 + 3a − 1 = 0


P 0 (a) = 12a5 − 15a4 + 12a3 + 3a2 − 6a + 3 = 0
=⇒ 3a4 + 8a3 − 15a2 + 18a − 7 = 0
37a3 − 57a2 + 57a − 20 = 0
a2 − a + 1 = 0

3
We have used polynomial long division to deduce that any double root must be a root
of a2 − a + 1! With this information, we can see that P (x) = (x2 − x + 1)2 (2x2 + x − 1).
The real roots are easily computed via the quadratic formula, leading to an answer of
− 12 . In fact the repeated roots were complex.

10. Suppose f and g are differentiable functions such that

xg(f (x))f 0 (g(x))g 0 (x) = f (g(x))g 0 (f (x))f 0 (x)

for all real x. Moreover, f is nonnegative and g is positive. Furthermore,


Z a
e−2a
f (g(x))dx = 1 −
0 2

for all reals a. Given that g(f (0)) = 1, compute the value of g(f (4)).
Answer: e−16 or e116
Solution: Differentiating the given integral with respect to a gives f (g(a)) = e−2a .
Now
d [ln (f (g(x)))] f 0 (g(x))g 0 (x) g 0 (f (x))f 0 (x) d [ln (g(f (x)))]
x =x = =
dx f (g(x)) g(f (x)) dx

where the second equals sign follows from the given. Since ln (f (g(x))) = −2x, we have
2
−x2 +C = ln (g(f (x))), so g(f (x)) = Ke−x . It follows that K = 1 and g(f (4)) = e−16 .

4
IXth Annual Harvard-MIT Mathematics Tournament
Saturday 25 February 2006

Combinatorics Test: Solutions

Combinatorics Test
1. Vernonia High School has 85 seniors, each of whom plays on at least one of the school’s
three varsity sports teams: football, baseball, and lacrosse. It so happens that 74 are
on the football team; 26 are on the baseball team; 17 are on both the football and
lacrosse teams; 18 are on both the baseball and football teams; and 13 are on both the
baseball and lacrosse teams. Compute the number of seniors playing all three sports,
given that twice this number are members of the lacrosse team.
Answer: 11
Solution: Suppose that n seniors play all three sports and that 2n are on the
lacrosse team. Then, by the principle of inclusion-exclusion, 85 = (74 + 26 + 2n) −
(17 + 18 + 13) + (n) = 100 + 2n − 48 + n = 52 + 3n. It is easily seen that n = 11.

2. Compute
2
X n60
X n3
X n2
X n1
X
··· 1.
n60 =0 n59 =0 n2 =0 n1 =0 n0 =0

Answer: 1953
Solution: The given sum counts the number of non-decreasing 61-tuples of integers
(n0 , . . . , n60 ) from the set {0, 1, 2}. Such 61-tuples are in one-to-one correspondence
with strictly increasing 61-tuples of integers (m0 , . . . , m60 ) from the set {0, 1, 2, . . . , 62}:
simply letmk = nk + k. But the number of such (m0 , . . . , m60 ) is almost by definition
63
= 63

61 2
= 1953.

3. A moth starts at vertex A of a certain cube and is trying to get to vertex B, which is
opposite A, in five or fewer “steps,” where a step consists in traveling along an edge
from one vertex to another. The moth will stop as soon as it reaches B. How many
ways can the moth achieve its objective?
Answer: 48
Solution: Let X, Y, Z be the three directions in which the moth can intially go. We
can symbolize the trajectory of the moth by a sequence of stuff from Xs, Y s, and Zs
in the obvious way: whenever the moth takes a step in a direction parallel or opposite
to X, we write down X, and so on.
The moth can reach B in either exactly 3 or exactly 5 steps. A path of length 3 must
be symbolized by XY Z in some order. There are 3! = 6 such orders. A trajectory
of length 5 must by symbolized by XY ZXX, XY ZY Y , or XY ZZZ, in some order,
5!
There are 3 · 3!1!1! = 3 · 20 = 60 possibilities here. However, we must remember to
subtract out those trajectories that already arrive at B by the 3rd step: there are
3 · 6 = 18 of those. The answer is thus 60 − 18 + 6 = 48.

1
4. A dot is marked at each vertex of a triangle ABC. Then, 2, 3, and 7 more dots are
marked on the sides AB, BC, and CA, respectively. How many triangles have their
vertices at these dots?
Answer: 357
15

Solution: Altogether there are 3 + 2 + 3 + 7 = 15 dots, and thus 3
= 455
2+2 2+3
 2+7
combinations of 3 dots. Of these combinations, 3 + 3 + 3 = 4 + 10 + 84 = 98
do not give triangles because they are collinear (the rest do give triangles). Thus
455 − 98 = 357 different triangles can be formed.

5. Fifteen freshmen are sitting in a circle around a table, but the course assistant (who
remains standing) has made only six copies of today’s handout. No freshman should
get more than one handout, and any freshman who does not get one should be able to
read a neighbor’s. If the freshmen are distinguishable but the handouts are not, how
many ways are there to distribute the six handouts subject to the above conditions?
Answer: 125
Solution: Suppose that you are one of the freshmen; then there’s a 6/15 chance that
you’ll get one of the handouts. We may ask, given that you do get a handout, how
many ways are there to distribute the rest? We need only multiply the answer to that
question by 15/6 to answer the original question.
Going clockwise around the table from you, one might write down the sizes of the gaps
between people with handouts. There are six such gaps, each of size 0–2, and the sum
of their sizes must be 15 − 6 = 11. So the gap sizes are either 1, 1, 1, 2, 2, 2 in some
6!
order, or 0, 1, 2, 2, 2, 2 in some order. In the former case, 3!3! = 20 orders are possible;
6!
in the latter, 1!1!4! = 30 are. Altogether, then, there are 20 + 30 = 50 possibilities.
Multiplying this by 15/6, or 5/2, gives 125.

6. For how many ordered triplets (a, b, c) of positive integers less than 10 is the product
a × b × c divisible by 20?
Answer: 102
Solution: One number must be 5. The other two must have a product divisible by
4. Either both are even, or one is divisible by 4 and the other is odd. In the former
case, there are 48 = 3 × 4 × 4 possibilities: 3 positions for the 5, and any of 4 even
numbers to fill the other two. In the latter case, there are 54 = 3 × 2 × 9 possibilites: 3
positions and 2 choices for the multiple of 4, and 9 ways to fill the other two positions
using at least one 5.

7. Let n be a positive integer, and let Pushover be a game played by two players, standing
squarely facing each other, pushing each other, where the first person to lose balance
loses. At the HMPT, 2n+1 competitors, numbered 1 through 2n+1 clockwise, stand in
a circle. They are equals in Pushover: whenever two of them face off, each has a 50%
probability of victory. The tournament unfolds in n + 1 rounds. In each round, the
referee randomly chooses one of the surviving players, and the players pair off going
clockwise, starting from the chosen one. Each pair faces off in Pushover, and the losers
leave the circle. What is the probability that players 1 and 2n face each other in the
last round? Express your answer in terms of n.

2
2n − 1
Answer:
8n
Solution: At any point during this competition, we shall say that the situation is
living if both players 1 and 2n are still in the running. A living situation is far if
those two players are diametrically opposite each other, and near otherwise, in which
case (as one can check inductively) they must be just one person shy of that maximal
separation. At the start of the tournament, the situation is living and near. In each
of rounds 1 to n, a far situation can never become near, and a near situation can stay
near or become far with equal likelihood.
In each of rounds 1 to n − 1, a living situation has a 1/4 probability of staying living.
Therefore, at the end of round k, where 1 ≤ k ≤ n − 1, the situation is near with
probability 1/8k , and far with probability 1/4k − 1/8k . In round n, a far situation has
a 1/4 probability of staying living, whereas a near situation has only a 1/8 probability
of staying living. But if the situation is living at the beginning of the last round, it
can only be far, so we can say with complete generality that, at the end of round k,
where 1 ≤ k ≤ n, the situation is living and far with probability 1/4k − 1/8k . We are
interested in finding the probability that the situation is living at the end of round n
n
(and hence far); that probability is thus 41n − 81n = 2 8−1
n .

8. In how many ways can we enter numbers from the set {1, 2, 3, 4} into a 4 × 4 array so
that all of the following conditions hold?
(a) Each row contains all four numbers.
(b) Each column contains all four numbers.
(c) Each “quadrant” contains all four numbers. (The quadrants are the four corner
2 × 2 squares.)

Answer: 288
Solution: Call a filled 4 × 4 array satisfying the given conditions cool. There are 4!
possibilities for the first row; WLOG, let it be 1 2 3 4. Since each quadrant has to
contain all four numbers, we have exactly four possibilities for the second row, namely:
(i) 3 4 1 2
(ii) 3 4 2 1
(iii) 4 3 1 2
(iv) 4 3 2 1
I claim that the number of cool arrays with (i) is equal to those with (iv), and that
the number of cool arrays with (ii) is equal to those with (iii). Let’s first consider (i)
and (iv). Now, (i) is

1234
3412

while (iv) is

1234
4321

3
In (iv), switch 3 and 4 (relabeling doesn’t affect the coolness of the array); then, it
becomes
1243
3421
Now, interchange the last two columns, which also does not affect the coolness. This
gives us (i). Hence, the cool arrays with (i) and the cool arrays with (iv) have a 1:1
correspondence. Using the exact same argument, we can show that the number of cool
arrays with (ii) equals those with (iii).
So we only need consider cases (i) and (ii). It is easy to verify that there are four cool
arrays with (i), determined precisely by, say, the first two entries of the third row; and
two with (ii), determined precisely by, say the first entry of the third row. Hence, the
answer is 4! × (4 + 2) × 2 = 288.
9. Eight celebrities meet at a party. It so happens that each celebrity shakes hands with
exactly two others. A fan makes a list of all unordered pairs of celebrities who shook
hands with each other. If order does not matter, how many different lists are possible?
Answer: 3507
Solution: Let the celebrities get into one or more circles so that each circle has
at least three celebrities, and each celebrity shook hands precisely with his or her
neighbors in the circle.
Let’s consider the possible circle sizes:
• There’s one big circle with all 8 celebrities. Depending on the ordering of the
people in the circle, the fan’s list can still vary. Literally speaking, there are 7!
different circles 8 people can make: fix one of the people, and then there are 7
choices for the person to the right, 6 for the person after that, and so on. But
this would be double-counting because, as far as the fan’s list goes, it makes no
difference if we “reverse” the order of all the people. Thus, there are 7!/2 = 2520
different possible lists here.
• 5+3. In this case there are 85 ways to split into the two circles, 4!2 essentially


different ways of ordering the 5-circle, and 2!2 ways for the 3-circle, giving a total
count of 56 · 12 · 1 = 672.
• 4+4. In this case there are 84 /2 = 35 ways to split into the two circles (we divide


by 2 because here, unlike in the 5 + 3 case, it does not matter which circle is
which), and 3!2 = 3 ways of ordering each, giving a total count of 35 · 3 · 3 = 315.
Adding them up, we get 2520 + 672 + 315 = 3507.
10. Somewhere in the universe, n students are taking a 10-question math competition.
Their collective performance is called laughable if, for some pair of questions, there
exist 57 students such that either all of them answered both questions correctly or
none of them answered both questions correctly. Compute the smallest n such that
the performance is necessarily laughable.
Answer: 253
Solution: Let ci,j denote the number of students correctly answering questions i and
j (1 ≤ i < j ≤ 10), and let wi,j denote the number of students getting both questions

4
wrong. An individual student answers k questions correctly and 10 − k questions
incorrectly. This student answers k2 pairs of questions correctly and 10−k

2
pairs of
questions incorrectly. Now observe that
   
k 10 − k
+ = k 2 − 10k + 45 = (k − 5)2 + 20 ≥ 20
2 2

Therefore, X
ci,j + wi,j ≥ 20n
1≤i<j≤10

Now if the performance is not laughable,


10
 then ci,j ≤ 56 and wi,j ≤ 56 for all 1 ≤
i < j ≤ 10. Observe that there are 2 2 = 90 of these variables. Hence, in a boring
performance, X
20n ≤ ci,j + wi,j ≤ 90 · 56 = 5040
1≤i<j≤10

or n ≤ 252. In particular this implies


 that if n ≥ 253, the performance is laughable.
This is the best bound because 10 5
= 252, and if each of 252 students correctly
answers a different 5 element subset of the 10 questions, then ci,j = wi,j = 56 for all
1 ≤ i < j ≤ 10.

5
IXth Annual Harvard-MIT Mathematics Tournament
Saturday 25 February 2006

Geometry Test: Solutions


1. Octagon ABCDEF GH is equiangular. Given that AB = 1, BC = 2, CD = 3,
DE = 4, and EF = F G = 2, compute the perimeter of the octagon.

Answer: 20 + 2
Solution: Extend sides AB, CD, EF, GH to form a rectangle: let X be the intersec-
tion of lines GH and AB; Y that of AB and CD; Z that of CD and EF ; and W that
of EF and GH.
√ √
As BC = 2, we have BY = Y C √= 2. As DE = 4, we have DZ = ZE = 2 2. As
F G = 2, we have F W = W G = 2.
We can
√ compute the dimensions of the rectangle: W X√ = Y Z = Y C + CD + DZ =
3 + 3 2, and XY√ = ZW = ZE + √ EF + F W = √ 2 + 3 2. Thus, HX = XA = XY −
AB −BY = 1+2 2, and so AH = 2HX = 4+ 2, and GH = W X −W G−HX = 2.
The perimeter of the octagon can now be computed by adding up all its sides.
2. Suppose ABC is a scalene right triangle, and P is the point on hypotenuse AC such
that ∠ABP = 45◦ . Given that AP = 1 and CP = 2, compute the area of ABC.
Answer: 95
Solution: Notice that BP bisects the right angle at B. Thus, we write AB = 2x,
BC = x. By the Pythagorean theorem, 5x2 = 9, from which the area 12 (x)(2x) = x2 =
9
5
.
3. Let A, B, C, and D be points on a circle such that AB = 11 and CD = 19. Point
P is on segment AB with AP = 6, and Q is on segment CD with CQ = 7. The line
through P and Q intersects the circle at X and Y . If P Q = 27, find XY .
Answer: 31
Solution: Suppose X, P, Q, Y lie in that order. Let P X = x and QY = y. By power
of a point from P , x · (27 + y) = 30, and by power of a point from Q, y · (27 + x) =
84. Subtracting the first from the second, 27 · (y − x) = 54, so y = x + 2. Now,
x · (29 + x) = 30, and we find x = 1, −30. Since −30 makes no sense, we take x = 1
and obtain XY = 1 + 27 + 3 = 31.
4. Let ABC be a triangle such that AB = 2, CA = 3, and BC = 4. A semicircle with its
diameter on BC is tangent to AB and AC. Compute the area of the semicircle.
Answer: 27π40
Solution: Let O, D, and E be the midpoint of the diameter and the points of
tangency with AB and AC respectively. Then [ABC] = [AOB] + [AOC] = 12 (AB +
AC)r, where r is the radius of the semicircle. Now by Heron’s formula, [ABC] =
q √ √
9 3 15 3 15
2
· 21 · 32 · 5
2
= 4
. We solve for r = 10
and compute 12 πr2 = 27π
40
.

5. Triangle ABC has side lengths AB = 2 5, BC = 1, and CA = 5. Point D is on side
AC such that CD = 1, and F is a point such that BF = 2 and CF = 3. Let E be the
intersection of lines AB and DF . Find the area of CDEB.

1
22
Answer: 35
Solution: Draw segment AF . Then notice AF = 4, and we have a right triangle.
4 8
Now draw line CE, let it intersect AF at G. By Ceva, F G = and AG = . Using
3 3
AE [AEF ] 4
mass points we find that = 6 so = 6, and since [ABF ] = 4, [BEF ] = .
EB [BEF ] 7
1 6
It’s easy to see that [CDF ] = [ACF ] = , so
5 5
6 4 22
[BCDE] = [CDF ] − [BEF ] = − = .
5 7 35

6. A circle of radius t is tangent to the hypotenuse, the incircle, and one leg of an isosceles
right triangle with inradius r = 1 + sin π8 . Find rt.

2+ 2
Answer:
4
Solution: The distance between the point of tangency of the two circles and the
nearest vertex of the triangle is seen to be both r(csc π8 − 1) and t(csc π8 + 1), so

r2 (csc π8 − 1) (1 + sin π8 )2 (1 − sin π8 ) π


rt = π = π = 1 − sin2
csc 8 + 1 1 + sin 8 8
2 π π
√ √
1 1 − 2 sin 8 1 cos 4 1 2 2+ 2
= + = + = + = .
2 2 2 2 2 4 4

7. Suppose ABCD is an isosceles trapezoid in which AB k CD. Two mutually externally


tangent circles ω1 and ω2 are inscribed in ABCD such that ω1 is tangent to AB, BC,
and CD while ω2 is tangent to AB, DA, and CD. Given that AB = 1, CD = 6,
compute the radius of either circle.
Answer: 37
Solution: Let the radius of both circles be r, and let ω1 be centered at O1 . Let ω1 be
tangent to AB, BC, and CD at P , Q, and R respectively. Then, by symmetry, P B =
1
2
− r and RC = 3 − r. By equal tangents from B and C, BQ = 21 − r and QC = 3 − r.
Now, ∠BO1 C is right because m∠O1 BC +m∠BCO1 = 21 (m∠P BC +m∠BCR) = 90◦ .
Since O1 Q ⊥ BC, r2 = O1 Q2 = BQ · QC = ( 12 − r)(3 − r) = r2 − 27 r + 32 . Solving, we
find r = 73 .

8. Triangle ABC has a right angle at B. Point D lies on side BC such that 3∠BAD =
∠BAC. Given AC = 2 and CD = 1, compute BD.
Answer: 38
Solution: Let BD = x. We reflect D over AB √ to D0 . Then√DD0 = 2x, but AD
bisects
p CAD0 , so 4x = √
AD0 = AD. Also, AD = x2 + AB 2 = x2 + AC 2 − BC 2 =
x2 + 4 − (x + 1)2 = 3 − 2x. We have the quadratic 16x2 = 3 − 2x which gives
x = 3/8.

9. Four spheres, each of radius r, lie inside a regular tetrahedron with side length 1 such
that each sphere is tangent to three faces of the tetrahedron and to the other three
spheres. Find r.

2

6−1
Answer:
10
Solution: Let O be the center of the sphere that is tangent to the faces ABC, ABD,
and BCD. Let P , Q be the feet of the perpendiculars from O to ABC and ABD
respectively. Let R be the foot of the perpendicular from P to AB. Then, OP RQ
is a quadrilateral such that ∠P , ∠Q are right angles and OP = OQ = r. Also, ∠R
is the dihedral angle
√ between faces
√ ABC and ABD, so cos √ ∠R = 1/3. We √ can then
√ QR = 2r, so BR = 6r. Hence, 1 = AB = 2( 6r) + 2r = 2r( 6 + 1), so
compute
r = ( 6 − 1)/10.

10. Triangle ABC has side lengths AB = 65, BC = 33, and AC = 56. Find the radius of
the circle tangent to sides AC and BC and to the circumcircle of triangle ABC.
Answer: 24
Solution: Let Γ be the circumcircle of triangle ABC, and let E be the center of
the circle tangent to Γ and the sides AC and BC. Notice that ∠C = 90◦ because
332 + 562 = 652 . Let D be the second intersection of line CE with Γ, so that D is
the midpoint of the√arc AB away from C. Because ∠BCD = 45◦ , one can easily
calculate
√ CD = 89
√ √ 2/2. The power of E with respect to Γ is both r(65 − r) and
r 2 · (89 2/2 − r 2) = r(89 − 2r), so r = 89 − 65 = 24.

3
IXth Annual Harvard-MIT Mathematics Tournament
Saturday 25 February 2006

General Test, Part 1: Solutions

1. How many positive integers x are there such that 3x has 3 digits and 4x has four digits?
Answer: 84
Solution: Note that x must be between 250 and 333, inclusive. There are 84 integers
in that interval.

2. What is the probability that two cards randomly selected (without replacement) from
a standard 52-card deck are neither of the same value nor the same suit?
Answer: 12 17
Solution: After choosing a first card, the second needs to be in one of the other
3·12
three suits and of a different value. Hence, the answer is 52−1 = 12
17
.

3. A square and an equaliteral triangle together have the property that the area of each
is the perimeter of the other. Find the square’s area.

Answer: 12 3 4
Solution:
√ 2 Let s, t be the side lengths of the square and triangle, respectively. Then
3t
4s = 4 , and so
√ 2 !2
3t 3t4
3t = s2 = = 8,
16 2
√3

so that t3 = 28 and s2 = 3t = 3 28 = 12 3 4.

4. Find

q p
31 + 31 + 31 + . . .
q p √ .
1 + 1 + 1 + ...

Answer: 6 − 5
Solution: Let the numerator be x and the denominator y. Then x2 = 31 + x, so, as
x > 0, we have √ √
1 + 1 + 4 · 31 1+5 5
x= = .
2 2
Similarly we compute that
√ √
1+ 1+4·1 1+ 5
y= = ,
2 2
so that √ √ √ √
x 1+5 5 1+5 5 1− 5 −24 + 4 5 √
= √ = √ · √ = = 6 − 5.
y 1+ 5 1+ 5 1− 5 −4

1
5. In the plane, what is the length of the shortest path from (−2, 0) to (2, 0) that avoids
the interior of the unit circle (i.e., circle of radius 1) centered at the origin?
√ π
Answer: 2 3 +
3
Solution: The path goes in a line segment tangent to the circle, then an arc of the
circle, then another line segment tangent to the circle. Since one of these tangent lines
and a radius of the circle give two legs of a right triangle √with hypotenuse
√ the line from
2 2
(0,0) to (-2,0) or (2,0), the length of each tangent line is 2 − 1 = 3. Also, because
these are 30◦ -60◦ -90◦ right triangles, the angle of the arc is 60◦ and has length π/3.
6. Six celebrities meet at a party. It so happens that each celebrity shakes hands with
exactly two others. A fan makes a list of all unordered pairs of celebrities who shook
hands with each other. If order does not matter, how many different lists are possible?
Answer: 70
Solution: Let the celebrities get into one or more circles so that each circle has
at least three celebrities, and each celebrity shook hands precisely with his or her
neighbors in the circle.
Either there is one big circle of all 6 celebrities or else there are two small circles of 3
celebrities each.
If there is one big circle of 6, then depending on the ordering of the people in the circle,
the fan’s list can still vary. Literally speaking, there are 5! different circles 6 people can
make: fix one of the people, and then there are 5 choices for the person to the right, 4
for the person after that, and so on. But this would be double-counting because, as far
as the fan’s list goes, it makes no difference if we “reverse” the order of all the people.
There are thus 5!/2 = 60 different possible lists here.
If there are two small circles of 3, then there are 63 different ways the members of


the “first” small circle may be selected. But this, too, is double-counting, because it
makes no difference which circle is termed the “first” and which the “second.” There
6

are therefore 3 /2 = 10 essentially different ways to split up the people into the two
circles. In a circle of just three, each person shakes hands with both the others, so
naturally the order of people in the circle doesn’t matter. There are thus 10 different
possible lists here.
(Note that, translated into the language of graph theory, the problem is asking for the
number of graphs on six labeled vertices such that each vertex has degree two.)
7. The train schedule in Hummut is hopelessly unreliable. Train A will enter Intersection
X from the west at a random time between 9:00 am and 2:30 pm; each moment in that
interval is equally likely. Train B will enter the same intersection from the north at
a random time between 9:30 am and 12:30 pm, independent of Train A; again, each
moment in the interval is equally likely. If each train takes 45 minutes to clear the
intersection, what is the probability of a collision today?
Answer: 13 48
Solution: Suppose we fix the time at which Train B arrives at Intersection X; then
call the interval during which Train A could arrive (given its schedule) and collide with
Train B the “disaster window.”
We consider two cases:

2
(i) Train B enters Intersection X between 9:30 and 9:45. If Train B arrives at 9:30,
the disaster window is from 9:00 to 10:15, an interval of 1 14 hours. If Train B
arrives at 9:45, the disaster window is 1 21 hours long. Thus, the disaster window
has an average length of (1 14 + 1 12 ) ÷ 2 = 118
. From 9:00 to 2:30 is 5 21 hours. The
probability of a collision is thus 11
8
÷ 5 12 = 14 .
(ii) Train B enters Intersection X between 9:45 and 12:30. Here the disaster window
is always 1 12 hours long, so the probability of a collision is 1 12 ÷ 5 12 = 11
3
.

From 9:30 to 12:30 is 3 hours. Now case (i) occurs with probability 14 ÷ 3 = 12 1
, and
11
case (ii) occurs with probability 12 . The overall probability of a collision is therefore
1
· 1 + 11
12 4
· 3 = 48
12 11
1
+ 14 = 48
13
.

8. A dot is marked at each vertex of a triangle ABC. Then, 2, 3, and 7 more dots are
marked on the sides AB, BC, and CA, respectively. How many triangles have their
vertices at these dots?
Answer: 357
15

Solution: Altogether there are 3 + 2 + 3 + 7 = 15 dots, and thus 3
= 455
2+2 2+3
 2+7
combinations of 3 dots. Of these combinations, 3 + 3 + 3 = 4 + 10 + 84 = 98
do not give triangles because they are collinear (the rest do give triangles). Thus
455 − 98 = 357 different triangles can be formed.

9. Take a unit sphere S, i.e., a sphere with radius 1. Circumscribe a cube C about S,
and inscribe a cube D in S, so that every edge of cube C is parallel to some edge of
cube D. What is the shortest possible distance from a point on a face of C to a point
on a face of D?
√ √
3− 3 3
Answer: =1− , or equivalent
3 3
Solution: Using the Pythagorean
√ theorem, we know that the length of a diagonal of
a cube of edge length s is s 3. Since √ D is inscribed in a sphere that has diameter 2,
this means that its side length is 2/ 3.
The distance from a face of D to a face of C will be the distance between them along
any line perpendicular to both of them; take such a line passing through the center of
S. The distance from
√ the center to any face of D along this line will be half the side
length of D, or 1/ 3. The distance from the center to the edge√of C is the radius of

S, which is 1. Therefore the desired distance is 1 − 1/ 3 = 1 − 33 .

10. A positive integer n is called “flippant” if n does not end in 0 (when written in decimal
notation) and, moreover, n and the number obtained by reversing the digits of n are
both divisible by 7. How many flippant integers are there between 10 and 1000?
Answer: 17
Solution: We use the notation “|” to mean “divides.”
There is only one flippant 2-digit number, namely 77. Indeed, if 10a + b is flippant
(where a, b are integers 1–9), then 7 | 10a + b and 7 | 10b + a. Thus,

7 | 3(10a + b) − (10b + a) = 29a − 7b = a + 7(4a − b),

so that 7 | a, and similarly 7 | b, so we’d better have a = b = 7.

3
There are 16 flippant 3-digit numbers. First consider the 12 palindromic ones (ones
where the hundreds and units digits are the same): 161, 252, 343, 434, 525, 595, 616,
686, 707, 777, 868, and 959. Now consider the general case: suppose 100a + 10b + c is
flippant, where a, b, c are integers 1–9. Then 7 | 100a + 10b + c and 7 | 100c + 10b + a,
so 7 | (100a + 10b + c) − (100c + 10b + a) = 99(a − c), and so 7 | a − c. In order for this
not to result in a palindromic integer, we must have a − c = ±7 and, moreover, both
100a + 10b + a and 100c + 10b + c must be palindromic flippant integers. Consulting
our list above, we find 4 more flippant integers: 168, 259, 861, and 952.

4
IXth Annual Harvard-MIT Mathematics Tournament
Saturday 25 February 2006

General Test, Part 2: Solutions

1. Larry can swim from Harvard to MIT (with the current of the Charles River) in 40
minutes, or back (against the current) in 45 minutes. How long does it take him to
row from Harvard to MIT, if he rows the return trip in 15 minutes? (Assume that the
speed of the current and Larry’s swimming and rowing speeds relative to the current
are all constant.) Express your answer in the format mm:ss.
Answer: 14:24
Solution: Let the distance between Harvard and MIT be 1, and let c, s, r denote
the speeds of the current and Larry’s swimming and rowing, respectively. Then we are
given
1 9 1 8 1 24
s+c= = , s−c= = , r−c= = ,
40 360 45 360 15 360
so
9 − 8 + 24 25
r + c = (s + c) − (s − c) + (r − c) = = ,
360 360
and it takes Larry 360/25 = 14.4 minutes, or 14:24, to row from Harvard to MIT.

2. Find
22 32 42 20062
· · · · · · · .
22 − 1 32 − 1 42 − 1 20062 − 1
4012
Answer:
2007
Solution:
2006 2006 2006
Y k 2006
Y k2 Y k2 Y k 2006 2 4012
2
= = = · = .
k=2
k − 1 k=2 (k − 1)(k + 1) k=2 k − 1 k=2 k + 1 1 2007 2007

3. Let C be the unit circle. Four distinct, smaller congruent circles C1 , C2 , C3 , C4 are
internally tangent to C such that Ci is externally tangent to Ci−1 and Ci+1 for i =
1, . . . , 4 where C5 denotes C1 and C0 represents C4 . Compute the radius of C1 .

Answer: 2−1
Solution: Let O and O0 be the centers of C and C1 respectively, and let C1 be
tangent to C, C2 , C4 at P , Q, and R respectively. Observe that QORO0 is a square √ and
0 0
that P, O , and O are√collinear. Thus, if r is the desired radius, 1 = r + OO = r + r 2,
1
so that r = √2+1 = 2 − 1.

4. Vernonia High School has 85 seniors, each of whom plays on at least one of the school’s
three varsity sports teams: football, baseball, and lacrosse. It so happens that 74 are
on the football team; 26 are on the baseball team; 17 are on both the football and
lacrosse teams; 18 are on both the baseball and football teams; and 13 are on both the
baseball and lacrosse teams. Compute the number of seniors playing all three sports,
given that twice this number are members of the lacrosse team.

1
Answer: 11
Solution: Suppose that n seniors play all three sports and that 2n are on the
lacrosse team. Then, by the principle of inclusion-exclusion, 85 = (74 + 26 + 2n) −
(17 + 18 + 13) + (n) = 100 + 2n − 48 + n = 52 + 3n. It is easily seen that n = 11.

2 2
a + b
5. If a, b are nonzero real numbers such that a + b = 8ab, find the value of .
a − b

15
Answer:
3
Solution: Note that
s r r √
a + b (a + b)2 a 2 + b2 + 2ab 10ab 15
a − b = (a − b)2 = a2 + b2 − 2ab = = .

6ab 3

6. Octagon ABCDEF GH is equiangular. Given that AB = 1, BC = 2, CD = 3,


DE = 4, and EF = F G = 2, compute the perimeter of the octagon.

Answer: 20 + 2
Solution: Extend sides AB, CD, EF, GH to form a rectangle: let X be the intersec-
tion of lines GH and AB; Y that of AB and CD; Z that of CD and EF ; and W that
of EF and GH.
√ √
As BC = 2, we have BY = Y C √= 2. As DE = 4, we have DZ = ZE = 2 2. As
F G = 2, we have F W = W G = 2.
We can
√ compute the dimensions of the rectangle: W X√ = Y Z = Y C + CD + DZ =
3 + 3 2, and XY√ = ZW = ZE + √ EF + F W = √ 2 + 3 2. Thus, HX = XA = XY −
AB −BY = 1+2 2, and so AH = 2HX = 4+ 2, and GH = W X −W G−HX = 2.
The perimeter of the octagon can now be computed by adding up all its sides.

7. What is the smallest positive integer n such that n2 and (n + 1)2 both contain the digit
7 but (n + 2)2 does not?
Answer: 27
Solution: The last digit of a square is never 7. No two-digit squares begin with 7.
There are no 3-digit squares beginning with the digits 17, 27, 37, or 47. In fact, the
smallest square containing the digit 7 is 576 = 242 . Checking the next few numbers,
we see that 252 = 625, 262 = 676, 272 = 729, 282 = 784, and 292 = 841, so the answer
is 27.

8. Six people, all of different weights, are trying to build a human pyramid: that is, they
get into the formation

A
BC
DEF

We say that someone not in the bottom row is “supported by” each of the two closest
people beneath her or him. How many different pyramids are possible, if nobody can
be supported by anybody of lower weight?

2
Answer: 16
Solution: Without loss of generality, let the weights of the people be 1, 2, 3, 4, 5,
and 6. Clearly we must have A = 1. Then, equally clearly, either B or C must be 2.
Suppose B = 2: Then either C or D must be 3. If C = 3, we have 3! = 6 possibilities
to fill the bottom row. If D = 3, then C = 4 and we have 2! = 2 possibilities to fill E
and F . Altogether there are 6 + 2 = 8 possibilities in this case.
Suppose C = 2: then, similarly, there are 8 possibilities here.
Altogether there are 8 + 8 = 16 possibilities.

9. Tim has a working analog 12-hour clock with two hands that run continuously (instead
of, say, jumping on the minute). He also has a clock that runs really slow—at half
the correct rate, to be exact. At noon one day, both clocks happen to show the exact
time. At any given instant, the hands on each clock form an angle between 0◦ and
180◦ inclusive. At how many times during that day are the angles on the two clocks
equal?
Answer: 33
Solution: A tricky thing about this problem may be that the angles on the two
clocks might be reversed and would still count as being the same (for example, both
angles could be 90◦ , but the hour hand may be ahead of the minute hand on one clock
and behind on the other).
Let x, −12 ≤ x < 12, denote the number of hours since noon. If we take 0◦ to mean
upwards to the “XII” and count angles clockwise, then the hour and minute hands of
the correct clock are at 30x◦ and 360x◦ , and those of the slow clock are at 15x◦ and
180x◦ . The two angles are thus 330x◦ and 165x◦ , of course after removing multiples of
360◦ and possibly flipping sign; we are looking for solutions to

330x◦ ≡ 165x◦ (mod 360◦ ) or 330x◦ ≡ −165x◦ (mod 360◦ ).

In other words,
360 | 165x or 360 | 495x.
Or, better yet,
165 11 495 11
x = x and/or x= x
360 24 360 8
must be an integer. Now x is any real number in the range [−12, 12), so 11x/8 ranges
in [−16.5, 16.5), an interval that contains 33 integers. For any value of x such that
11x/24 is an integer, of course 11x/8 = 3 × (11x/24) is also an integer, so the answer
is just 33.

10. Fifteen freshmen are sitting in a circle around a table, but the course assistant (who
remains standing) has made only six copies of today’s handout. No freshman should
get more than one handout, and any freshman who does not get one should be able to
read a neighbor’s. If the freshmen are distinguishable but the handouts are not, how
many ways are there to distribute the six handouts subject to the above conditions?
Answer: 125
Solution: Suppose that you are one of the freshmen; then there’s a 6/15 chance that
you’ll get one of the handouts. We may ask, given that you do get a handout, how

3
many ways are there to distribute the rest? We need only multiply the answer to that
question by 15/6 to answer the original question.
Going clockwise around the table from you, one might write down the sizes of the gaps
between people with handouts. There are six such gaps, each of size 0–2, and the sum
of their sizes must be 15 − 6 = 11. So the gap sizes are either 1, 1, 1, 2, 2, 2 in some
6!
order, or 0, 1, 2, 2, 2, 2 in some order. In the former case, 3!3! = 20 orders are possible;
6!
in the latter, 1!1!4! = 30 are. Altogether, then, there are 20 + 30 = 50 possibilities.
Multiplying this by 15/6, or 5/2, gives 125.

4
IXth Annual Harvard-MIT Mathematics Tournament
Saturday 25 February 2006

Guts Round Solutions

1. A bear walks one mile south, one mile east, and one mile north, only to find itself
where it started. Another bear, more energetic than the first, walks two miles south,
two miles east, and two miles north, only to find itself where it started. However, the
bears are not white and did not start at the north pole. At most how many miles
apart, to the nearest .001 mile, are the two bears’ starting points?

Answer: 3.477
Solution: Say the first bear walks a mile south, an integer n > 0 times around
the south pole, and then a mile north. The middle leg of the first bear’s journey is
1
a circle of circumference 1/n around the south pole, and therefore about 2nπ miles
north of the south pole. (This is not exact even if we assume the Earth is perfectly
spherical, but it is correct to about a micron.) Adding this to the mile that the bear
1
walked south/north, we find that it started about 1 + 2nπ miles from the south pole.
2
Similarly, the second bear started about 2 + 2mπ miles from the south pole for some
integer m > 0, so they must have started at most
1 2 3
3+ + ≤3+ ≈ 3.477
2nπ 2mπ 2π
miles apart.
2. Compute the positive integer less than 1000 which has exactly 29 positive proper
divisors. (Here we refer to positive integer divisors other than the number itself.)

Answer: 720
Solution: Recall that the number N = pe11 pe22 · · · pekk (where the pi are distinct
primes) has exactly (e1 + 1)(e2 + 1) · · · (ek + 1) positive integer divisors including itself.
We seek N < 1000 such that this expression is 30. Since 30 = 2 · 3 · 5, we take
e1 = 1, e2 = 2, e3 = 4. Then we see that N = 51 32 42 = 720 is satisfactory.
3. At a nursey, 2006 babies sit in a circle. Suddenly each baby pokes the baby immediately
to either its left or its right, with equal probability. What is the expected number of
unpoked babies?

Answer: 1003 2
Solution: The probability that any given baby goes unpoked is 1/4. So the answer
is 2006/4 = 1003/2.
4. Ann and Anne are in bumper cars starting 50 meters apart. Each one approaches the
other at a constant ground speed of 10 km/hr. A fly starts at Ann, flies to Anne, then
back to Ann, and so on, back and forth until it gets crushed when the two bumper
cars collide. When going from Ann to Anne, the fly flies at 20 km/hr; when going in
the opposite direction the fly flies at 30 km/hr (thanks to a breeze). How many meters
does the fly fly?

1
Answer: 55
Solution: Suppose that at a given instant the fly is at Ann and the two cars are
12d apart. Then, while each of the cars travels 4d, the fly travels 8d and meets Anne.
Then the fly turns around, and while each of the cars travels d, the fly travels 3d and
meets Ann again. So, in this process described, each car travels a total of 5d while the
fly travels 11d. So the fly will travel 11
5
times the distance traveled by each bumper
11 50
car: 5 · 2 = 55 meters.

5. Find the number of solutions in positive integers (k; a1 , a2 , . . . , ak ; b1 , b2 , . . . , bk ) to the


equation
a1 (b1 ) + a2 (b1 + b2 ) + · · · + ak (b1 + b2 + · · · + bk ) = 7.

Answer: 15
Solution: Let k, a1 , . . . , ak , b1 , . . . , bk be a solution. Then b1 , b1 +b2 , . . . , b1 +· · ·+bk is
just some increasing sequence of positive integers. Considering the ai as multiplicities,
the ai ’s and bi ’s uniquely determine a partition of 7. Likewise, we can determine ai ’s
and bi ’s from any partition of 7, so the number of solutions is p(7) = 15.

6. Suppose ABC is a triangle such that AB = 13, BC = 15, and CA = 14. Say D is
the midpoint of BC, E is the midpoint of AD, F is the midpoint of BE, and G is the
midpoint of DF . Compute the area of triangle EF G.

Answer: 21 4 p
Solution: By Heron’s formula, [ABC] = 21(21 − 15)(21 − 14)(21 − 13) = 84.
Now, unwinding the midpoint conditions yields [EF G] = [DEF2
]
= [BDE]
4
= [ABD]
8
=
[ABC] 84 21
16
= 16 = 4 .

7. Find all real numbers x such that


jxk jxk
x2 + + = 10.
2 3


Answer: − 14
Solution: Evidently x2 must be an √ integer. Well, there
√ aren’t that many things to
check,
√ are there? Among positive
√ x, 8 is too small and 9 is too big; among negative
x, − 15 is too small and − 13 is too big.

8. How many ways are there to label the faces of a regular octahedron with the integers 1–
8, using each exactly once, so that any two faces that share an edge have numbers that
are relatively prime? Physically realizable rotations are considered indistinguishable,
but physically unrealizable reflections are considered different.

Answer: 12
Solution: Well, instead of labeling the faces of a regular octahedron, we may label
the vertices of a cube. Then, as no two even numbers may be adjacent, the even
numbers better form a regular tetrahedron, which can be done in 2 ways (because

2
rotations are indistiguishable but reflections are different). Then 3 must be opposite
6, and the remaining numbers — 1, 5, 7 — may be filled in at will, in 3! = 6 ways.
The answer is thus 2 × 6 = 12.

9. Four unit circles are centered at the vertices of a unit square, one circle at each vertex.
What is the area of the region common to all four circles?


Answer: π3 + 1 − 3
Solution: The desired region consists of a small square and four “circle segments,”
i.e. regions of a circle bounded by a chord and an arc. The side of this small square
is just the chord of a unit circle that cuts off an angle of 30◦ , and the circle segments
are bounded by that chord and the circle. Using the law of cosines (in an isosceles
triangle with unit leg length and vertex
√ angle 30◦ ), we find that the square of the
length of the chord is equal to 2 − 3. We can also compute the area of each circle
π 1 π 1
segment, namely − (1)(1) sin 30◦ = − . Hence, the desired region has area
12  2 12 4
√ √

π 1 π
2− 3+4 − = + 1 − 3.
12 4 3
10. Let f (x) = x2 − 2x. How many distinct real numbers c satisfy f (f (f (f (c)))) = 3?

Answer: 9
Solution: We see the size of the set f −1 (f −1 (f −1 (f −1 (3)))). Note that f (x) =
(x − 1)2 − 1 = 3 has two solutions: x = 3 and x = −1, and that the fixed points
f (x) = x are x = 3 and x = 0. Therefore, the number of real solutions is equal to the
number of distinct real numbers c such that c = 3, c = −1, f (c) = −1 or f (f (c)) = −1,
or f (f (f (c))) = −1. The equation f (x) = −1 has exactly one root x = 1. Thus, the
last three equations are√ equivalent to c = 1, f (c) = 1, and f (f (c)) = 1. f (c) = 1 has
two solutions, c = 1 ± 2, and for each of these two values c there are two preimages.
It follows that the answer is 1 + 1 + 1 + 2 + 4 = 9.
n2 +7n+136
11. Find all positive integers n > 1 for which n−1
is the square of a positive integer.

Answer: 5, 37
2
Solution: Write n +7n+136
n−1
= n + 8n+136
n−1
144
= n + 8 + n−1 144
= 9 + (n − 1) + (n−1) . We seek
2
to find p and q such that pq = 144 and p + q + 9 = k . The possibilities are seen to be
1 + 144 + 9 = 154, 2 + 72 + 9 = 83, 3 + 48 + 9 = 60, 4 + 36 + 9 = 49, 6 + 24 + 9 = 39,
8 + 18 + 9 = 35, 9 + 16 + 9 = 34, and 12 + 12 + 9 = 33. Of these, {p, q} = {4, 36} is
the only solution to both equations. Hence n − 1 = 4, 36 and n = 5, 37.

12. For each positive integer n let Sn denote the set {1, 2, 3, . . . , n}. Compute the number
of triples of subsets A, B, C of S2006 (not necessarily nonempty or proper) such that A
is a subset of B and S2006 − A is a subset of C.

Answer: 24012
Solution: Let Ao , Bo , Co be sets satisfying the said conditions. Note that 1 ∈ Ao
implies that 1 ∈ Bo and 1 ∈/ S2006 − Ao so that 1 may or may not be in Co . Also,

3
1 ∈
/ Ao implies that 1 ∈ S2006 − Ao ⊂ Co while 1 may or may not be in Bo . Thus
there are four possibilities for the distribution of 1, and since the same argument holds
independently for 2, 3, . . . , 2006, the answer is 42006 or 24012 .
13. Let Z be as in problem 15. Let X be the greatest integer such that |XZ| ≤ 5. Find
X.

Answer: 2
Solution: Problems 13–15 go together. See below.
14. Let X be as in problem 13. Let Y be the number of ways to order X crimson flowers,
X scarlet flowers, and X vermillion flowers in a row so that no two flowers of the same
hue are adjacent. (Flowers of the same hue are mutually indistinguishable.) Find Y .

Answer: 30
Solution: Problems 13–15 go together. See below.

15. Let Y be as in problem 14. Find the maximum Z such that three circles of radius Z
can simultaneously fit inside an equilateral triangle of area Y without overlapping each
other.

Answer: 10 3 − 15 √
Solution: We first find that, in problem 15, each of the circles of radius Z is the
incircle of a 30◦ -60◦ -90◦ triangle formed √by cutting the equilateral one in half. The
equilateral triangle itself has sidelength 2√
4
Y
3
, so the said inradius is
√ √
√ 1+ 3−2 1 2 Y
Z= · · √ 4
,
2 2 3
so that √ √ √
(−1 + 3)2 4−2 3 2 3−3
Z= √ Y = √ Y = Y.
4 3 4 3 6
Now we guess that X = 2 and see that, miraculously, everything works: in the problem
14, say a crimson flower is placed first. Then there are 2 possibilities for C C ,4
for C C , 2 for C C , and 2 for C C, giving a total of 10. Of course, the
first flower can be of any of the three hues, so Y = 3 · 10 = 30. We compute Z and
check X in a straightforward manner.

If X > 2, then Y > 30, and Z > 10 3−15, with the result that X ≤ 2, a contradiction.
Assuming X < 2 results in a similar contradiction.
q
16. A sequence a1 , a2 , a3 , . . . of positive reals satisfies an+1 = 1+a 2
n
. Determine all a1 such
√ √
6+ 2
that ai = 4
for some positive integer i.
√ √ √
2+ 6 3 1
Answer: , ,
2 2 2
Solution: Clearly a1 < 1, or else 1 ≤ a1 ≤ a2 ≤ a3 r ≤ . . . . We can therefore
θ 1 + cos θ
write a1 = cos θ for some 0 < θ < 90◦ . Note that cos = , and cos 15◦ =
2 2

4
√ √
6+ 2
. Hence, the possibilities for a1 are cos 15◦ , cos 30◦ , and cos 60◦ , which are
√ 4 √ √
2+ 6 3 1
, , and .
2 2 2
17. Begining at a vertex, an ant is crawls between the vertices of a regular octahedron.
After reaching a vertex, it randomly picks a neighboring vertex (sharing an edge) and
walks to that vertex along the adjoining edge (with all possibilities equally likely.)
What is the probability that after walking along 2006 edges, the ant returns to the
vertex where it began?

2005
Answer: 23·22006+1

Solution: For each nonnegative integer n, let an , bn , and cn denote the respective
probabilities that the ant is where it began, at a neighbor of where it began, or is
opposite where it began after moving along n edges. We seek a2006 . We have a0 = 1
and b0 = c0 = 0. We also have the recursive system
bn−1
an =
4
bn−1
bn = an−1 + + cn−1
2
bn−1
cn =
4

for integers n ≥ 1. Substituting into the second equation we have bn = bn−12


+ bn−2
2
2 x 1 −1
for n ≥ 2. Solving the characteristic equation x − 2 − 2 = 0 for x = 1, 2 , we write
bn = a · 1n + b(−1/2)n . Using b0 = 0, b1 = 1, we compute
2
bn = · (1 − (−1/2)n )
3
b2005
 22005 +1
From which we find a2006 = 4
= 16 1 + 22005
1
= 3·22006 .

18. Cyclic quadrilateral ABCD has side lengths AB = 1, BC = 2, CD = 3 and DA = 4.


Points P and Q are the midpoints of BC and DA. Compute P Q2 .

Answer: 116 35
Solution: Construct AC, AQ, BQ, BD, and let R denote the intersection of AC
and BD. Because ABCD is cyclic, we have that 4ABR ∼ 4DCR and 4ADR ∼
4BCR. Thus, we may write AR = 4x, BR = 2x, CR = 6x, DR = 12x. Now, Ptolemy
applied to ABCD yields 140x2 = 1 · 3 + 2 · 4 = 11. Now BQ is a median in triangle
2 2 −AD 2 2 2 −DA2
ABD. Hence, BQ2 = 2BA +2BD 4
. Likewise, CQ2 = 2CA +2CD 4
. But P Q is a
2 2BQ2 +2CQ2 −BC 2 AB 2 +BD2 +CD2 +CA2 −BC 2 −AD2
median in triangle BQC, so P Q = 4
= 4
=
11
(196+100)x2 +12 +32 −22 −42 148x2 −5 148· 140 −5 116
4
= 2
= 2
= 35
.
Another solution is possible. Extend AD and BC past A and B to their intersection S.
Use similar triangles SAB and SCD, and similar triangles SAC and SBD to compute
SA and SB, then apply the Law of Cosines twice, first to compute the cosine of ∠A
and then to compute P Q2 .

5
19. Let ABC be a triangle with AB = 2, CA = 3, BC = 4. Let D be the point diametri-
cally opposite A on the circumcircle of ABC, and let E lie on line AD such that D is
the midpoint of AE. Line l passes through E perpendicular to AE, and F and G are
the intersections of the extensions of AB and AC with l. Compute F G.

Answer: 1024
45 √
3 15
Solution: Using Heron’s formula we arrive at [ABC] = 4
. Now invoking
 the rela-

abc 2·3
tion [ABC] = 4R
where R is the circumradius of ABC, we compute R2 = [ABC]2
=
64
15
.
Now observe that ∠ABD is right, so that BDEF is a cyclic quadrilateral. Hence
AB · AF = AD · AE = 2R · 4R = 512 15
. Similarly, AC · AG = 512
15
. It follows that
BCGF is a cyclic quadrilateral, so that triangles ABC and AGF are similar. Then
AF 512
F G = BC · AC = 4 · 2·15·3 = 1024
45

20. Compute the number of real solutions (x, y, z, w) to the system of equations:

x = z + w + zwx z = x + y + xyz
y = w + x + wxy w = y + z + yzw

Answer: 5
w+z
Solution: The first equation rewrites as x = 1−wz , which is a fairly strong reason
to consider trigonometric substitution. Let x = tan(a), y = tan(b), z = tan(c), and
w = tan(d), where −90◦ < a, b, c, d < 90◦ . Under modulo 180◦ , we find a ≡ c + d; b ≡
d + a; c ≡ a + b; d ≡ b + c. Adding all of these together yields a + b + c + d ≡ 0.
Then a ≡ c + d ≡ −a − b so b ≡ −2a. Similarly, c ≡ −2b; d ≡ −2c; d ≡ −2a. Hence,
c ≡ −2b ≡ 4a, d ≡ −2c ≡ −8a, and a ≡ −2d ≡ 16a, so the only possible solutions
are (a, b, c, d) ≡ (t, −2t, 4t, −8t) where 15t ≡ 0. Checking, these, we see that actually
5t ≡ 0, which yields 5 solutions. Our division by 1 − yz is valid since 1 − yz = 0
iff yz = 1, but x = y + z + xyz so y = −z, which implies that yz ≤ 0 < 1, which
is impossible. (The solutions we have computed are in fact (0,0,0,0) and the cyclic
permutations of (tan(36◦ ), tan(−72◦ ), tan(−36◦ ), tan(72◦ )).)

21. Find the smallest positive integer k such that z 10 + z 9 + z 6 + z 5 + z 4 + z + 1 divides


z k − 1.

Answer: 84
Solution: Let Q(z) denote the polynomial divisor. We need that the roots of Q are
k-th roots of unity. With this in mind, we might observe that solutions to z 7 = 1 and
z 6= 1 are roots of Q, which leads to its factorization. Alternatively, we note that

(z − 1)Q(z) = z 11 − z 9 + z 7 − z 4 + z 2 − 1 = (z 4 − z 2 + 1)(z 7 − 1)

Solving for the roots of the first factor, z 2 = 1+i2 3 = ±cisπ/3 (we use the notation
cis(x) = cos(x) + i sin(x)) so that z = ±cis(±π/6). These are primitive 12-th roots
of unity. The other roots of Q(z) are the primitive 7-th roots of unity (we introduced
z = 1 by multiplication.) It follows that the answer is lcm[12, 7] = 84.

6
22. Let f (x) be a degree 2006 polynomial with complex roots c1 , c2 , . . . , c2006 , such that
the set
{|c1 |, |c2 |, . . . , |c2006 |}
consists of exactly 1006 distinct values. What is the minimum number of real roots of
f (x)?

Answer: 6
Solution: The complex roots of the polynomial must come in pairs, ci and ci , both of
which have the same absolute value. If n is the number of distinct absolute values |ci |
corresponding to those of non-real roots, then there are at least 2n non-real roots of
f (x). Thus f (x) can have at most 2006 − 2n real roots. However, it must have at least
1006 − n real roots, as |ci | takes on 1006 − n more values. By definition of n, these all
correspond to real roots. Therefore 1006 − n ≤ # real roots ≤ 2006 − 2n, so n ≤ 1000,
and # real roots ≥ 1006 − n ≥ 6. It is easy to see that equality is attainable.

23. Let a0 , a1 , a2 , . . . be a sequence of real numbers defined by a0 = 21, a1 = 35, and


an+2 = 4an+1 − 4an + n2 for n ≥ 2. Compute the remainder obtained when a2006 is
divided by 100.

Answer: 0
Solution: No pattern is evident in the first few terms, so we look for a formula for
an . If we write an = An2 + Bn + C + bn and put bn+2 = 4bn+1 − 4bn . Rewriting the
original recurrence, we find

An2 + (4A + B)n + (4A + 2B + C) + bn+2


= 4 An2 + (2A + B)n + (A + B + C) + bn+1 − 4 An2 + Bn + C + bn + n2
 

= n2 + 8An + (4A + 4B) + 4bn+1 − 4bn

Solving, A = 1, B = 4, C = 8. With this information, we can solve for b0 = 1 and


b1 = 6. Since the characteristic equation of the recurrence of the bi is x2 − 4x + 4 =
(x − 2)2 = 0, we have bn = (Dn + E) · 2n for some constants D and E. Using the
known values b0 and b1 , we compute D = 2 and E = 1, and finally

an = n2 + 4n + 8 + (2n + 1) · 2n

Now, taking modulo 100, we have a2006 ≡ 62 + 4 · 6 + 8 + 13 · 22006 (mod 100). Evidently
22006 ≡ 0 (mod 4), but by Euler’s theorem 2φ(25) ≡ 220 ≡ 1 (mod 25), and so 22006 ≡
26 ≡ 14 (mod 25). Now the Chinese remainder theorem yields 22006 ≡ 64 (mod 100),
and we compute a2006 ≡ 36 + 24 + 8 + 13 · 64 ≡ 0 (mod 100).

24. Two 18-24-30 triangles in the plane share the same circumcircle as well as the same
incircle. What’s the area of the region common to both the triangles?

Answer: 132
Solution: Notice, first of all, that 18-24-30 is 6 times 3-4-5, so the triangles are
right. Thus, the midpoint of the hypotenuse of each is the center of their common
circumcircle, and the inradius is 21 (18 + 24 − 30) = 6. Let one of the triangles be ABC,

7
where ∠A < ∠B < ∠C = 90◦ . Now the line ` joining the midpoints of sides AB and
AC is tangent to the incircle, because it is the right distance (12) from line BC. So,
the hypotenuse of the other triangle lies along `. We may formulate this thus: The
hypotenuse of each triangle is parallel to the shorter leg, and therefore perpendicular
to the longer leg, of the other. Now it is not hard to see, as a result of these parallel-
and perpendicularisms, that the other triangle “cuts off” at each vertex of 4ABC a
smaller, similar right triangle. If we compute the dimensions of these smaller triangles,
we find that they are as follows: 9-12-15 at A, 6-8-10 at B, and 3-4-5 at C. The total
area chopped off of 4ABC is thus
9 · 12 6 · 8 3 · 4
+ + = 54 + 24 + 6 = 84.
2 2 2
The area of 4ABC is 18 · 24/2 = 216. The area of the region common to both the
original triangles is thus 216 − 84 = 132.
25. Points A, C, and B lie on a line in that order such that AC = 4 and BC = 2. Circles
ω1 , ω2 , and ω3 have BC, AC, and AB as diameters. Circle Γ is externally tangent
to ω1 and ω2 at D and E respectively, and is internally tangent to ω3 . Compute the
circumradius of triangle CDE.

Answer: 23
Solution: Let the center of ωi be Oi for i = 1, 2, 3 and let O denote the center of
Γ. Then O, D, and O1 are collinear, as are O, E, and O2 . Denote by F the point of
tangency between Γ and ω3 ; then F, O, and O3 are collinear. Writing r for the radius
of Γ we have OO1 = r + 2, OO2 = r + 1, OO3 = 3 − r. Now since O1 O3 = 1 and
O3 O2 = 2, we apply Stewart’s theorem:
OO12 · O2 O3 + OO22 · O1 O3 = OO32 · O1 O2 + O1 O3 · O3 O2 · O1 O2
2(r + 2)2 + (r + 1)2 = 3(3 − r)2 + 1 · 2 · 3
We find r = 76 . Now the key observation is that the circumcircle of triangle CDE is
the incircle of triangle OO1 O2 . We easily compute the sides of OO1 O2 to be 13
7 7
, 20 , and
3. By Heron’s formula, the area of OO1 O2 is 18 7
, but the semiperimeter is 277
, so the
2
desired radius is 3 .
26. Let a ≥ b ≥ c be real numbers such that
a2 bc + ab2 c + abc2 + 8 = a + b + c
a2 b + a2 c + b2 c + b2 a + c2 a + c2 b + 3abc = −4
a2 b2 c + ab2 c2 + a2 bc2 = 2 + ab + bc + ca
If a + b + c > 0, then compute the integer nearest to a5 .

Answer: 1279
Solution: We factor the first and third givens, obtaining the system
a2 bc + ab2 c + abc2 − a − b − c = (abc − 1)(a + b + c) = −8
a2 b + a2 c + b2 c + b2 a + c2 a + c2 b + 3abc = (ab + bc + ca)(a + b + c) = −4
a2 b2 c + ab2 c2 + a2 bc2 − ab − bc − ca = (abc − 1)(ab + bc + ca) = 2

8
Writing X = a+b+c, Y = ab+bc+ca, Z = abc−1, we have XZ = −8, XY = −4, Y Z =
2. Multiplying the three yields (XY Z)2 = 64 from which XY Z = ±8. Since we are
given X > 0, multiplying the last equation by X we have 2X = XY Z = ±8. Evidently
XY Z = 8 from which X = 4, Y = −1, Z = −2. We conclude that a, b, c are the roots
of the polynomial P (t) = t3 − 4t2 − t + 1. Thus, P (a) = a3 − 4a2 − a + 1 = 0, and
also P (b) = P (c) = 0. Now since P (1/2) = − 38 , P (0) = 1 and P (−2/3) = − 27
11
, we
5 5 1
conclude that −2/3 < c < 0 < b < 1/2 < a. It follows that |b + c | < 2 . Thus, we
compute a5 + b5 + c5 .
Defining Sn = an + bn + cn , we have Sn+3 = 4Sn+2 + Sn+1 − Sn for n ≥ 0. Evidently
S0 = 3, S1 = 4, S2 = (a + b + c)2 − 2(ab + bc + ca) = 18. Then S3 = 4 · 18 + 4 − 3 = 73,
S4 = 4 · 73 + 18 − 4 = 306, and S5 = 4 · 306 + 73 − 18 = 1279. Since |b5 + c5 | < 21 , we
conclude that |S5 − a5 | < 12 , or that 1279 is the integer nearest to a5 .

27. Let N denote the number of subsets of {1, 2, 3, . . . , 100} that contain more prime
numbers than multiples of 4. Compute the largest integer k such that 2k divides N .

Answer: 52
Solution: Let S denote a subset with the said property. Note that there are 25
multiples of 4 and 25 primes in the set {1, 2, 3, . . . , 100}, with no overlap between the
two. Let T denote the subset of 50 numbers that are neither prime nor a multiple of 4,
and let U denote the 50 other numbers. Elements of T can be arbitrarily included in
or excluded by S. Consider S ∩ U = S1 and U − S = S2 (the set difference is defined
to be all elements of U that are not in S.) S1 and S2 are two disjoint sets such that
U = S1 ∩ S2 . If S1 contains more multiples of 4 than primes, then S2 contains more
primes than multiples of 4, and conversely. Furthermore, S1 contains an equal number
of primes and multiples of 4 if and only if S2 contains equal numbers as well. Let V
denote an arbitrary subset of T . It follows from examining pairs of sets V ∪ S1 and
V ∪ S2 that
25  2
!
1 X 25
N = 250 · 250 −
2 k
  k=0 
50
= 249 · 250 −
25

Since 50! is divisible by 2 exactly 25 + 12 + 6 + 3 + 1 = 47 times while 25! is divisible


by 2 exactly 12 + 6 + 3 + 1 = 22 times, it follows that 5025
is divisible by 2 exactly 3
times, so the answer is 49 + 3 = 52.

28. A pebble is shaped as the intersection of a cube of side length 1 with the solid sphere
tangent to all of the cube’s edges. What is the surface area of this pebble?

6 2−5
Answer: π
2
Solution: Imagine drawing the sphere and the cube. Take a cross section, with a
plane parallel to two of the cube’s faces, passing through the sphere’s center. In this
cross section, the sphere looks like a circle, and the cube looks like a square (of side
length 1) inscribed in that circle. We can now calculate that the sphere has diameter

9
√ 2
d := √ 2 and surface area S := πd = 2π, and that the sphere protrudes a distance of
2−1
x := 2
out from any given face of the cube.
It is known that the surface area chopped off from a sphere by any plane is proportional
to the perpendicular distance thus chopped off. Thus, each face of the cube chops of
a fraction xd of the sphere’s surface. The surface area of the pebble contributed by the
sphere is thus S · (1 − 6 · xd ), whereas the cube contributes 6 circles of radius 12 , with
2
total area 6 · π 21 = 23 π. The pebble’s surface area is therefore
√ ! √
 x 3 2−1 3 6 2−5
S· 1−6· + π = 2π · 1 − 6 · √ + π= π.
d 2 2 2 2 2

29. Find the area in the first quadrant bounded by the hyperbola x2 − y 2 = 1, the x-axis,
and the line 3x = 4y.

Answer: ln47
Solution: Convert to polar coordinates: the hyperbola becomes

1 = r2 (cos2 θ − sin2 θ) = r2 cos(2θ),

so, letting α := arctan(3/4), the area is


Z α 2 α
1 α
Z
r 1
S := dθ = sec(2θ)dθ = ln |sec(2θ) + tan(2θ)| .
0 2 2 0 4 0

Now
2 tan α 3/2 24
tan(2α) = 2
= = ,
1 − tan α 7/16 7
25
q
sec(2α) = 1 + tan2 (2α) = ,
7
so  
1 25 24 ln 7
S= ln + − ln |1 + 0| =
.
4 7 7 4

30. ABC is an acute triangle with incircle ω. ω is tangent to sides BC, CA, and AB at
D, E, and F respectively. P is a point on the altitude from A such that Γ, the circle
with diameter AP , is tangent to ω. Γ intersects AC and AB at X and Y respectively.
Given XY = 8, AE = 15, and that the radius of Γ is 5, compute BD · DC.

Answer: 675 4
Solution: By the Law of Sines we have sin ∠A = XY AP
= 45 . Let I, T , and Q denote the
center of ω, the point of tangency between ω and Γ, and the center of Γ respectively.
Since we are told ABC is acute, we can compute tan ∠ A2 = 12 . Since ∠EAI = A2
and AE is tangent to ω, we find r = AE 2
= 15
2
. Let H be the foot of the altitude
from A to BC. Define hT to be the homothety about T which sends Γ to ω. We
have hT (AQ) = DI, and conclude that A, T , and D are collinear. Now since AP is
a diameter of Γ, ∠P AT is right, implying that DT HP is cyclic. Invoking Power of

10
a Point twice, we have 225 = AE 2 = AT · AD = AP · AH. Because we are given
radius of Γ we can find AP = 10 and AH = 45 2
= ha . If we write a, b, c, s in the usual
manner with respect to triangle ABC, we seek BD · DC = (s − b)(s − c). But recall
that Heron’s formula gives us
p
s(s − a)(s − b)(s − c) = K
r s 2
where K is the area of triangle ABC. Writing K = rs, we have (s − b)(s − c) = s−a .
15 s 1
Knowing r = 2 , we need only compute the ratio a . By writing K = 2 aha = rs, we
r2 s
2 as
find as = h2ra = 23 . Now we compute our answer, s−a = 15
2
· s −1 = 675
4
.
a

31. Let A be as in problem 33. Let W be the sum of all positive integers that divide A.
Find W .

Answer: 8
Solution: Problems 31–33 go together. See below.
32. In the alphametic W E × EY E = SCEN E, each different letter stands for a different
digit, and no word begins with a 0. The W in this problem has the same value as the
W in problem 31. Find S.

Answer: 5
Solution: Problems 31–33 go together. See below.
33. Let W , S be as in problem 32. Let A be the least positive integer such that an acute
triangle with side lengths S, A, and W exists. Find A.

Answer: 7
Solution: There are two solutions to the alphametic in problem 32: 36×686 = 24696
and 86 × 636 = 54696. So (W, S) may be (3, 2) or (8, 5). If (W, S) = (3, 2), then by
problem (3) A = 3, but then by problem 31 W = 4, a contradiction. So, (W, S) must
be (8, 5). By problem 33, A = 7, and this indeed checks in problem 31.
34. In bridge, a standard 52-card deck is dealt in the usual way to 4 players. By convention,
each hand is assigned a number of “points” based on the formula
4 × (# A’s) + 3 × (# K’s) + 2 × (# Q’s) + 1 × (# J’s).
Given that a particular hand has exactly 4 cards that are A, K, Q, or J, find the
probability that its point value is 13 or higher.

197
Answer: 1820
Solution: Obviously, we can ignore the cards lower than J. Simply enumerate the
ways to get at least 13 points: AAAA (1), AAAK (16), AAAQ (16), AAAJ (16),
AAKK (36), AAKQ (96), AKKK (16). The numbers in parentheses represent the
number of ways to choose the suits, given the choices for the values. We see that there
are a total of1 + 16 + 16 + 16 + 36 + 96 + 16 = 197 ways to get at least 13. There are
a total of 16
4
= 1820 possible ways to choose 4 cards from the 16 total A’s, K’s, Q’s,
and J’s. Hence the answer is 197/1820.

11
35. A sequence is defined by A0 = 0, A1 = 1, A2 = 2, and, for integers n ≥ 3,
An−1 + An−2 + An−3 1
An = + 4
3 n − n2
Compute limN →∞ AN .

2
Answer: 136
− π12 .
Solution: If we sum the given equation for n = 3, 4, 5, . . . , N , we obtain
N N
X X An−1 + An−2 + An−3 1
An = +
n=3 n=3
3 n4 − n2

This reduces dramatically to


N
2AN −1 AN −2 2A1 A0 X 1
AN + + = A2 + + + (*)
3 3 3 3 n=3
n − n2
4

Let limN →∞ AN = L. Under this limit, the left hand side of (*) is simply 2L. We
compute the sum on the right with the help of partial fractions
N ∞
X 1 X 1 1
lim 4 2
= 2
− 2
N →∞
n=3
n −n n=3
n −1 n
∞  ! ∞
X 1 1 1 1 1 X 1
= − + 2+ 2−
n=3
2 n−1 n+1 1 2 n=1
n2
5 π2
 
1 1 1
= + + −
2 2 3 4 6
2
5 π
= −
3 6
 2
 2
With this we easily find L = 12 · 2 + 23 · 1 + 13 · 0 + 53 − π6 = 13 6
− π12 , and we are done.

36. Four points are independently chosen uniformly at random from the interior of a regular
dodecahedron. What is the probability that they form a tetrahedron whose interior
contains the dodecahedron’s center?

Answer: 81
Solution: Situate the origin O at the dodecahedron’s center, and call the four random
points Pi , where 1 ≤ i ≤ 4.
To any tetrahedron P1 P2 P3 P4 we can associate a quadruple ((ijk) ), where (ijk) ranges
over all conjugates of the cycle (123) in the alternating group A4 : ijk is the sign of
the directed volume [OPi Pj Pk ]. Assume that, for a given tetrahedron P1 P2 P3 P4 , all
members of its quadruple are nonzero (this happens with probability 1). For 1 ≤ i ≤
4, if we replace Pi with its reflection through the origin, the three members of the
tetrahedron’s quadruple that involve Pi all flip sign, because each [OPi Pj Pk ] is a linear

12
−−→
function of the vector OPi . Thus, if we consider the 16 sister tetrahedra obtained
by choosing independently whether to flip each Pi through the origin, the quadruples
range through all 16 possibilities (namely, all the quadruples consisting of ±1s). Two
of these 16 tetrahedra, namely those with quadruples (1, 1, 1, 1) and (−1, −1, −1, −1),
will contain the origin.
So the answer is 2/16 = 1/8.

37. Compute

X 2n + 5
n=1
2n · (n3 + 7n2 + 14n + 8)

Answer: 137
24
− 8 ln 2
Solution: First, we manipulate using partial fractions and telescoping:
∞ ∞  
X 2n + 5 1 X 1 2 1 1
= · − −
n=1
2n · (n3 + 7n2 + 14n + 8) 2 n=1 2n n + 1 n + 2 n + 4

1 1X 1
= −
4 2 n=1 2n · (n + 4)
P∞ rn
Now, consider the function f (r, k) := n=1 nk . We have
∞ ∞ ∞
∂ X rn X ∂ rn rn−1
  X
∂f (r, k) 1
= k
= k
= k−1
= f (r, k − 1)
∂r ∂r n=1 n n=1
∂r n n=1
n r

df (r, 1) 1 X rn 1 r 1
= 0
= · =
dr r n r 1−r 1−r
Z n=1
dr
f (r, 1) = = − ln(1 − r) + f (0, 1)
1−r

By inspection, f (0, 1) = 0, so f 21 , 1 = ∞ 1
 P
n=1
P∞n·2n
= ln(2). It is easy to compute the
1 1
desired sum in terms of f 2 , 1 , and we find n=1 2n (n+4) = 16 ln(2) − 131
12
. Hence, our
final answer is 137
24
− 8 ln(2).

38. Suppose ABC is a triangle with incircle ω, and ω is tangent to BC and CA at D and
E respectively. The bisectors of ∠A and ∠B intersect line DE at F and G respectively,
such that BF = 1 and F G = GA = 6. Compute the radius of ω.

Answer: 2 5 5
Solution: Let α, β, γ denote the measures of 12 ∠A, 12 ∠B, 12 ∠C, respectively. We have
m∠CEF = 90◦ − γ, m∠F EA = 90◦ + γ, m∠AF G = m∠AF E = 180◦ − α − (90◦ + γ) =
β = m∠ABG, so ABF G is cyclic. Now AG = GF implies that BG bisects ∠ABF .
Since BG by definition bisects ∠ABC, we see that F must lie on BC. Hence, F = D.
If I denotes the incenter of triangle ABC, then ID is perpendicular to BC, but since
A, I, F are collinear, we have that AD ⊥ BC. Hence, ABC is isoceles with AB = AC.
Furthermore, BC = 2BF = 2. Moreover, since ABF G is cyclic, ∠BGA is a right angle.
Construct F 0 on minor arc GF such that BF 0 = 6 and F 0 G = 1, and let AB = x. By

13

the Pythagorean theorem, AF 0 = BG = x2 − 36, so that Ptolemy applied to ABF 0 G
yields x2 − 36 = x + 36. We have (x − 9)(x + 8) = 0. Since x is a length we find
x = 9. Now
√ we have √ AB = AC = 9. Pythagoras applied to triangle ABD√now yields √
AD = 9 − 1 = 4 5, which enables us to compute [ABC] = 12 · 2 · 4 5 = 4 5.
2 2

Since the
√ area of a triangle
√ is also equal to its semiperimeter times its inradius, we
2 5
have 4 5 = 10r or r = 5 .
REMARK. In fact, ABF G is always a cyclic quadrilateral for which AB plays a di-
ameter. That is, we could have proven this fact without using F G = GA.
39. A fat coin is one which, when tossed, has a 2/5 probability of being heads, 2/5 of
being tails, and 1/5 of landing on its edge. Mr. Fat starts at 0 on the real line. Every
minute, he tosses a fat coin. If it’s heads, he moves left, decreasing his coordinate by
1; if it’s tails, he moves right, increasing his coordinate by 1. If the coin lands on its
edge, he moves back to 0. If Mr. Fat does this ad infinitum, what fraction of his time
will he spend at 0?

Answer: 13
Solution: For n ∈ Z, let an be the fraction of the time Mr. Fat spends at n. By
symmetry, an = a−n for all n.
For n > 0, we have an = 25 an−1 + 52 an+1 , or an+1 = 52 an − an−1 . This Fibonacci-like
recurrence can be solved explicitly to obtain
an = α · 2|n| + β · 2−|n|
for all n ∈ Z. Now we also have X
an = 1,
n∈Z

so we better have α = 0, so that a0 = β and a±1 =Pβ2 . Now we also have a0 =


2
a + 25 a1 + 15 , so β = 31 . This matches perfectly with n∈Z an = 1.
5 −1

40. Compute

X 3k + 1
3 + k2
· (−1)k+1 .
k=1
2k
2
Answer: π12 + π2 − 2 + ln 2
Solution: Via partial fraction decomposition we write the sum as
∞  
X 1 2 1
− + 2 (−1)k+1
k=1
k 1 + 2k k
Now recall that

X 1 π2
= = S1
k=1
k2 6

X (−1)k+1
= ln(2) = S2
k=1
k

X (−1)k+1 π
= = S3
k=1
2k − 1 4

14
Manipulating (1), we deduce
∞ ∞
! ∞
X (−1)k+1 X 1 X 1
= −2·
k=1
k2 k=1
k2 k=1
(2k)2
2 2 2
π π π
= − 2/4 · = = S4
6 6 12
It is then easily seen that the answer is equal to S2 + 2 · S3 − 2 + S4 .

41. Let Γ denote the circumcircle of triangle ABC. Point D is on AB such that CD bisects
∠ACB. Points P and Q are on Γ such that P Q passes through D and is perpendicular
to CD. Compute P Q, given that BC = 20, CA = 80, AB = 65.


Answer: 4 745
Solution: Suppose that P lies between A and B and Q lies between A and C, and let
line P Q intersect lines AC and BC at E and F respectively. As usual, we write a, b, c
for the lengths of BC, CA, AB. By the angle bisector theorem, AD/DB = AC/CB
 bc  so
bc ac 2 ac
that AD = a+b and BD = a+b . Now by Stewart’s theorem, c · CD + a+b a+b c =
2 2
a2 bc 2 −c )
a+b
+ ab c
a+b
from which CD2 = ab((a+b)
(a+b)2
. Now observe that triangles CDE and
CA ED F B
CDF are congruent, so ED = DF . By Menelaus’ theorem, AE DF BC
= 1 so that
CA AE b(b−a) 2ab
= F B . Since CF = CE while b > a, it follows that AE = a+b so that EC = a+b .
BC
√ √ 2 2
ab(c −(a−b) )
Finally, DE = CE 2 − CD2 = a+b
. Plugging in a = 20, b = 80, c = 65,
we see that AE = 48, EC = 32, DE = 10 as well as AD = 52, BD = 13. Now let
P D = x, QE = y. By power of a point about D and E, we have x(y + 10) = 676 and
y(x + 10) = 1536. Subtracting one from the√other, we see that y = x + 86. Therefore,

x2 + 96x − 676 = 0, from which x = −48 + 2 745. Finally, P Q = x + y + 10 = 4 745.

42. Suppose hypothetically that a certain, very corrupt political entity in a universe holds
an election with two candidates, say A and B. A total of 5,825,043 votes are cast,
but, in a sudden rainstorm, all the ballots get soaked. Undaunted, the election officials
decide to guess what the ballots say. Each ballot has a 51% chance of being deemed a
vote for A, and a 49% chance of being deemed a vote for B. The probability that B
will win is 10−X . What is X rounded to the nearest 10?

Answer: 510
Solution: Let N = 2912521, so that the number of ballots cast is 2N + 1. Let P
be the probability that B wins, and let α = 51% and β = 49% and γ = β/α < 1. We
have
N   N  
−X
X 2N + 1 N −i N +1+i N N +1
X 2N + 1 i
10 =P = α β =α β γ
i=0
N −i i=0
N −i
(think of 2i + 1 as representing B’s margin of victory). Now
N 
22N +1
  X 
2N + 1 2N + 1 i
< < γ < 22N +1 ,
2N + 1 N i=0
N − i

15
So

−X = log P = N log α+(N +1) log β+(2N +1) log 2− = N log(2α)+(N +1) log(2β)−,

where 0 <  < log(2N + 1) < 7. With a calculator, we find that

−X ≈ 25048.2 − 25554.2 −  = −506.0 − ,

so X ≈ 510.

43. Write down at least one, and up to ten, different 3-digit prime numbers. If you somehow
fail to do this, we will ignore your submission for this problem. Otherwise, you’re
entered into a game with other teams. In this game, you start with 10 points, and
each number you write down is like a bet: if no one else writes that number, you gain
1 point, but if anyone else writes that number, you lose 1 point. Thus, your score on
this problem can be anything from 0 to 20.

Solution: There are 143 three-digit primes. None of the following necessarily applies
to the actual contest, but it might be useful to think about. Suppose that you’re
trying to maximize your expected score on this problem. Then you should write down
a number if you think the probability that someone else is writing it is less than 1/2
(of course, limit yourself to 10 numbers). You should avoid writting down any number
if you think the probability that someone else is writing it is more than 1/2 (of course,
write down at least 1 number). Suppose that you expect a total of M different numbers
are going to be written down, but have no idea what numbers they might be. If you
think M ≥ 72, you should write down 10 numbers at random; if M ≤ 71, you should
write just 1 number.

44. On the Euclidean plane are given 14 points:

A = (0, 428) B = (9, 85) C = (42, 865) D = (192, 875)


E = (193, 219) F = (204, 108) G = (292, 219) H = (316, 378)
I = (375, 688) J = (597, 498) K = (679, 766) L = (739, 641)
M = (772, 307) N = (793, 0)

A fly starts at A, visits all the other points, and comes back to A in such a way as to
minimize the total distance covered. What path did the fly take? Give the names of
the points it visits in order. Your score will be

20 + bthe optimal distancec − byour distancec

or 0, whichever is greater.

Answer: The optimal path is ACDIKLJM N HGEF B(A), or the reverse, of course.
In this way the total distance covered by the fly is just over 3591.22.
Solution: This problem is an instance of the Traveling Salesman Problem, which
is NP-hard. There is an obvious algorithm in O(n!) time (where n is the number of
points), but faster algorithms exist. Nonetheless, the best strategy for solving this
problem is probably to draw the points and exercise your geometric intuition.

16
45. On your answer sheet, clearly mark at least seven points, as long as

(i) No three are collinear.


(ii) No seven form a convex heptagon.

Please do not cross out any points; erase if you can do so neatly. If the graders
deem that your paper is too messy, or if they determine that you violated one of those
conditions, your submission for this problem will be disqualified. Otherwise, your score
will be the number of points you marked minus 6, even if you actually violated one of
the conditions but were able to fool the graders.

Solution: This is the heptagon case of what is known as the “Happy Ending” or
“Erdős-Szekeres” problem, which in general asks, For any integer n ≥ 3, what is the
smallest N (n), such that any N (n) points in the plane in general position determine
a convex n-gon? It is known that such an N (n) always exists and is finite (in fact a
specific upper bound has been found). The best known lower bound is N (n) ≥ 2n−2 +1;
Erdős and Szekeres conjectured that this bound is tight. The n ≤ 5 cases have been
known for some time. According to the Wikipedia, the n = 6 case is solved but
unpublished, and for n ≥ 7, the problem remains open.
For a discussion, see

W. Morris and V. Soltan. The Erdős-Szekeres Problem on Points in Convex


Postion—A Survey, Bulletin of the American Math Monthly. 37 (2000),
437–458.

This article is available at

http://www.ams.org/bull/2000-37-04/S0273-0979-00-00877-6/home.html.

If N (7) = 33, the highest sure score on this problem would be 32 − 6 = 26. It is not
known whether there exist arbitrarily large sets of points that will fool the graders.

The unexamined life is not worth living.

17
IXth Annual Harvard-MIT Mathematics Tournament
Saturday 25 February 2006

Team Round A: Solutions

Mobotics [120]
Spring is finally here in Cambridge, and it’s time to mow our lawn. For the purpose of these
problems, our lawn consists of little clumps of grass arranged at the points of a certain grid
(to be specified later). Our machinery consists of a fleet of identical mowbots (or “mobots”
for short). A mobot is a lawn-mowing machine. To mow our lawn, we begin by choosing a
formation: we place as many mobots as we want at various clumps of grass and orient each
mobot’s head in a certain direction. At the blow of a whistle, each mobot starts moving in
the direction we’ve chosen, mowing every clump of grass in its path (including the clump it
starts on) until it goes off the lawn.
Because the spring is so young, our lawn is rather delicate. Consequently, we want to
make sure that every clump of grass is mowed once and only once. We will not consider
formations that do not meet this criterion.
One more thing: two formations are considered “different” if there exists a clump of grass
for which either (1) for exactly one of the formations does a mobot starts on that clump, or
(2) there are mobots starting on this clump for both the formations, but they’re oriented in
different directions.

1. [15] For this problem, our lawn consists of a row of n clumps of grass. This row
runs in an east-west direction. In our formation, each mobot may be oriented toward
the north, south, east, or west. One example of an allowable formation if n = 6 is
symbolized below:
· ·← ↑ ↑ ↓
(The mobot on the third clump will move westward, mowing the first three clumps.
Each of the last three clumps is mowed by a different mobot.) Here’s another allowable
formation for n = 6, considered different from the first:
↑ ↑
· ·← →

Compute the number of different allowable formations for any given n.

Solution: Let a be the number of clumps mowed by a mobot oriented to go west,


and let b be the number of clumps mowed by a mobot oriented to go east. (So in the
two examples given, (a, b) would be (3, 0) and (3, 1), respectively.) We may ask how
many allowable formations with a given ordered pair (a, b) there are, and then sum
our answers up over all possible ordered pairs (a, b).
Given any particular (a, b), first of all, a and b had better be non-negative integers
summing to at most n. As long as that’s true, there will be n − a − b clumps of grass
which are each mowed by a single mobot oriented to go either north or south: there

1
are thus 2n−a−b possibilities. So our answer is

n X
X n−a n
X n
X
n−a−b n−a n−a−1 0
2 = (2 +2 + ··· + 2 ) = (2n−a+1 − 1)
a=0 b=0 a=0 a=0
n
X
= −(n + 1) + 2n−a+1 = −(n + 1) + (2n+1 + 2n + 2n−1 + · · · + 21 )
a=0
= −(n + 1) + (2n+2 − 2) = 2n+2 − n − 3.

2. [25] For this problem, our lawn is an m × n rectangular grid of clumps, that is, with m
rows running east-west and n columns running north-south. To be even more explicit,
we might say our clumps are at the lattice points

{(x, y) ∈ Z2 | 0 ≤ x < n and 0 ≤ y < m}.

However, mobots are now allowed to be oriented to go either north or east only. So
one allowable formation for m = 2, n = 3 might be as follows:

· → ·

→ ·

(m + n)!
Prove that the number of allowable formations for given m and n is .
m! n!

Solution: There is a one-to-one correspondence between allowable formations and


paths from (0, 0) to (n, m) made up of n moves 1 unit to the right and m moves 1
unit up. The correspondence works as follows: There must be a mobot at (0, 0), so
start the path there. If that mobot is oriented to move up, then move to the right;
if that mobot is oriented to move to the right, then move up. In doing so, you will
meet another mobot, upon which you can repeat the above process, until you leave
the lawn. Once you leave the lawn, there will be only one way to proceed to (n, m) —
either keep going right, or keep going up. (For the example in the problem, our path
would be (0, 0)–(1, 0)–(1, 1)–(1, 2)–(2, 2)–(3, 2).)
Conversely, given any path from (0, 0) to (n, m), we can derive back a mobot formation:
place a mobot at every lattice point of the lawn that the path touches, and don’t orient
that mobot in the same direction as the path takes when leaving that point.
It is easy to check that this is indeed a one-to-one correspondence as claimed. Every
path from (0, 0) to (n, m) consists of n moves to the right and m moves up done in an
 (m+n)!
arbitrary order, and there are precisely m+n
m
= m! n! orders.

3. [40] In this problem, we stipulate that m ≥ n, and the lawn is shaped differently. The
clumps are now at the lattice points in a trapezoid:

{(x, y) ∈ Z2 | 0 ≤ x < n and 0 ≤ y < m + 1 − n + x},

As in problem 2, mobots can be set to move either north or east. For given m and n,
determine with proof the number of allowable formations.

2
Solution: For exactly the same reasons as in problem 2, we have a one-to-one
correspondence between the allowable formations and paths (going 1 unit up or right
at a time) from (0, 0) to (n, m) avoiding points (x, y) with y > m + 1 − n + x.
The number of these paths equals the total number of up/right paths from (0, 0) to
(n, m) minus the number of up/right paths from (0, 0) to (n, m) that do pass through
at least one point (x, y) with y > m + 1 − n + x. The first of these numbers is m+n

m
,
as before. It remains to calculate the second of these numbers.
To that end, we first note that the reflection of (n, m) across the line y = m + 2 − n + x
is (n − 2, m + 2). Now there is a one-to-one correspondence between up/right paths
from (0, 0) to (m, n) that pass through something with y ≥ m + 2 − n + x with up/right
paths from (0, 0) to (n − 2, m + 2): indeed, taking a path of one kind, we may isolate
the first point on it that lies on the line y = m + 2 − n + x, and reflect the rest of the
path through that line, to obtain a path of the other kind. There are therefore m+n m+2
paths of either kind.
The answer is therefore    
m+n m+n

m m+2
or, if you happen to prefer,
(m + n)! [(m + 2)(m + 1) − n(n − 1)]
.
(m + 2)! n!

4. [15] In this problem and the next, the lawn consists of points in a triangular grid of
size n, so that for n = 3 the lawn looks like

·
· ·
· · ·

Mobots are allowed to be oriented to the east, 30◦ west of north, or 30◦ west of south.
Under these conditions, for any given n, what is the minimum number of mobots
needed to now the lawn?
Answer: n
Solution: It is evident that n mobots are enough; just place one at the left edge of
each row, and set them to move to the right. Suppose for the sake of contradiction
that, for some n, it is possible to mow the lawn with fewer than n mobots. Consider
the minimum n for which this is the case. Clearly n > 1. Now consider the mobot
that mows the southwest corner of the lawn. This mobot is confined to either the
bottom row or the left edge of the lawn; let’s assume it’s the former (the latter case is
very similar). This mobot starts in the bottom row. Let’s throw it away. Now if any
remaining mobots start in the bottom row, they must not be set to move east, and
we may advance them by one step in the direction they’re set to go in, so that they
land either off the lawn or in the second-to-last row. In doing so we have produced a
starting formation of fewer than n − 1 mobots in the first n − 1 rows that will mow
those rows. This contradicts the assumed minimality of n.

3
5. [25] With the same lawn and the same allowable mobot orientations as in the previous
problem, let us call a formation “happy” if it is invariant under 120◦ rotations. (A
rotation applies both to the positions of the mobots and to their orientations.) An
example of a happy formation for n = 2 might be

-
. →

Find the number of happy formations for a given n.

Solution: If n ≡ 1 (mod 3), then there is a clump of grass at the center of the lawn;
otherwise there are 3 blades of grass equally closest to the center. In the former case,
whatever mobot mows this center blade of grass cannot possibly have a counterpart
under a 120◦ rotation: if this mobot starts in the center, it must be oriented a certain
way, and whatever way that is, cannot remain invariant under such a rotation; but
if the mobot does not start in the center, then it will collide in the center with its
counterparts under the 120◦ rotations.
In the case that n 6≡ 1 (mod 3), by considering how the three central blades of grass
can be mowed, we easily see that there are two possibilities, both of which involve
starting one mobot at each of these three positions. Both possibilities distinctly have
the effect of cutting up the rest of the lawn (i.e., the part of the lawn not mowed by
any of these three mobots) into three congruent pieces. The point is that these pieces
are equivalent to trapezoids as in problem 3, both in their shape, and in the allowed
directions of mobot motion. Luckily, we know how to count the ways to configure each
such trapezoid. Call the trapezoid defined in problem 3 an “(m, n)-trapezoid.”
If n ≡ 0 (mod 3), then these pieces are either all ( 2n
3
, n−3
3
)-trapezoids or all ( 2n−3
3
, n3 )-
trapezoids, depending on how the central mobots are configured. In this case, the
answer is    3    3
n−1 n−1 n−1 n−1
2n − 2n+6 + 2n−3 − 2n+3 .
3 3 3 3

If, on the other hand, n ≡ 2 (mod 3), then these pieces are either all ( 2n−1
3
, n−2
3
)-
2n−4 n+1
trapezoids or all ( 3 , 3 )-trapezoids, again depending on how the central mobots
are configured. In this case, the answer is
   3    3
n−1 n−1 n−1 n−1
2n−1 − 2n+1 + 2n−4 − 2n−2 .
3 3 3 3

Polygons [110]
6. [15] Let n be an integer at least 5. At most how many diagonals of a regular n-gon
can be simultaneously drawn so that no two are parallel? Prove your answer.
Answer: n
Solution: Let O be the center of the n-gon. Let us consider two cases, based on the
parity of n:

4
• n is odd. In this case, for each diagonal d, there is exactly one vertex D of the
n-gon, such that d is perpendicular to line OD; and of course, for each vertex
D, there is at least one diagonal d perpendicular to OD, because n ≥ 5. The
problem of picking a bunch of d’s so that no two are parallel is thus transmuted
into one of picking a bunch of d’s so that none of the corresponding D’s are the
same. Well, go figure.
• n is even. What can I say? For each diagonal d, the perpendicular dropped from
O to d either passes through two opposite vertices of the n-gon, or else bisects
two opposite sides. Conversely, for each line joining opposite vertices or bisecting
opposite sides, there is at least one diagonal perpendicular to it, because n ≥ 6.
By reasoning similar to the odd case, we find the answer to be n.

7. [25] Given a convex n-gon, n ≥ 4, at most how many diagonals can be drawn such
that each drawn diagonal intersects every other drawn diagonal either in the interior
of the n-gon or at a vertex? Prove your answer.
Answer: If n = 4, then 2; otherwise, n.
Solution: First of all, assume without loss of generality that the n-gon is regular
(this has no effect as far as diagonal intersection is concerned). Also, treat n = 4 as a
special case; obviously the answer is 2 here.
If n is odd, simply draw n diagonals, connecting each vertex to the ones (n − 1)/2
vertices away (in either direction).
If n is even, first draw the n/2 diagonals connecting pairs of vertices n/2 vertices
apart. Then, there are n diagonals connecting pairs of vertices n/2 − 1 vertices apart;
they come in n/2 pairs of parallel diagonals; from each such pair, randomly pick one
diagonal and draw it.
To see that these constructions work, note that two diagonals, each connecting pairs
of vertices at least n/2 − 1 vertices apart, can fail to intersect or share a vertex only if
they are parallel.
The previous problem shows that these constructions are optimal.

8. [15] Given a regular n-gon with sides of length 1, what is the smallest radius r such
that there is a non-empty intersection of n circles of radius r centered at the vertices
of the n-gon? Give r as a formula in terms of n. Be sure to prove your answer.
1 180◦
Answer: r = csc
2 n
Solution: It is easy to see that, with this r, all the circles pass through the center
of the n-gon. The following proves that this r is necessary even if the word “circle” is
replaced by the word “disk.”
For n even, it is easy to see using symmetry that containing the center point is neces-
sary and sufficient. For n odd, there is more work to do. Again, containing the center
point is sufficient. To see its necessity, consider three circles: a circle at a vertex A,
and the two circles on the segment BC opposite A. Circles B and C intersect in a
region R symmetric about the perpendicular bisector of BC, with the closest point
of R to A being on this line. Hence, circle A must intersect R at some point on the
perpendicular bisector of BC; and thus we see the entire perpendicular bisector of BC

5
inside of the n-gon is contained in the circles. Now, this bisector contains the center
of the n-gon, so some circle must contain the center. But by symmetry, if one circle
contains the center, all do. Thus, in any case, it is necessary and sufficient for r to
be large enough so the center is contained in a circle. Basic trigonometry gives the
answer, which equals the distance between a vertex and the center.

9. [40] Let n ≥ 3 be a positive integer. Prove that given any n angles 0 < θ1 , θ2 , . . . , θn <
180◦ , such that their sum is 180(n − 2) degrees, there exists a convex n-gon having
exactly those angles, in that order.

Solution: We induct on n. The statement holds trivially for n = 3, as all triangles


are convex. Now, suppose that the statement is true for n − 1, where n ≥ 4. Let
θ1 , θ2 , . . . , θn be n angles less than 180◦ whose sum equals 180(n − 2) degrees. The
statement is clearly true if n = 4 and θ1 = θ3 = 180◦ − θ2 = 180◦ − θ4 since we can
easily form a parallelogram, so assume otherwise.
I claim that there exist two adjacent angles whose sum is greater than 180◦ . Assume
otherwise. Then, we have θi +θi+1 ≤ 180 for i = 1, 2, . . . , n, where θn+1 = θ1 . Summing
these inequalities over all i yields 2 · 180(n − 2) ≤ 180n, which is equivalent to n ≤ 4.
Of course, we can have n = 4 if and only if we have equality in each of the above
inequalities, forcing us to have a parallelogram contrary to our assumption.
Hence, we have two adjacent angles with sum greater than 180◦ . Without loss of
generality, let these angles be θn−1 and θn , relabeling if necessary. By the inductive
hypothesis, we may construct an (n−1)-gon with angles θ1 , θ2 , . . . , θn−2 , θn−1 +θn −180◦ ,
as these angles are each less than 180◦ and their sum equals 180(n−3) degrees. Consider
the vertex with angle θn−1 + θn − 180◦ . Note that we can “clip off” a triangle with
angles θn−1 + θn − 180◦ , 180◦ − θn−1 , and 180◦ − θn at this vertex, yielding an n-gon
with the desired angles, completing the inductive step.

10. [15] Suppose we have an n-gon such that each interior angle, measured in degrees, is a
positive integer. Suppose further that all angles are less than 180◦ , and that all angles
are different sizes. What is the maximum possible value of n? Prove your answer.
Answer: 26
Solution: Let’s work with the exterior angles (each is 180 minus the interior angle).
Then the conditions on the exterior angles are identical to the conditions on the interior
angles: each is a positive integer between 1 and 179 inclusive. The sum of the exterior
angles is exactly 360. However, the sum of 1 through 27 is 27 · 28/2 = 378, which is
too large. We can get 26 using angles of 1 through 25 (sum 325) and an angle of 35.
The previous problem shows that this is actually possible.

What do the following problems have in common? [170]


11. [15] The lottery cards of a certain lottery contain all nine-digit numbers that can be
formed with the digits 1, 2 and 3. There is exactly one number on each lottery card.
There are only red, yellow and blue lottery cards. Two lottery numbers that differ
from each other in all nine digits always appear on cards of different color. Someone

6
draws a red card and a yellow card. The red card has the number 122 222 222 and the
yellow card has the number 222 222 222. The first prize goes to the lottery card with
the number 123 123 123. What color(s) can it possibly have? Prove your answer.
Answer: The card with the number 123 123 123 is red.
Solution: First, it can in fact be red, if, say, cards are colored based on the first digit
only (1 = red, 2 = yellow, 3 = blue). We now endeavor to show it must be red.
Consider the cards 333 133 133 and 331 331 331: they each differ in all their digits
from 122 222 222 and from 222 222 222, so they must both be blue. Now 211 311 311
differs in all its digits from both 122 222 222 and 333 133 133, so it must be yellow.
Finally, 123 123 123 differs in all its digits from both 331 331 331 and 211 311 311, so
it must be red.

12. [25] A 3 × 3 × 3 cube is built from 27 unit cubes. Suddenly five of those cubes mys-
teriously teleport away. What is the minimum possible surface area of the remaining
solid? Prove your answer.
Answer: 50
Solution: Orient the cube so that its edges are parallel to the x-, y-, and z-axes. A
set of three unit cubes whose centers differ only in their x-coordinate will be termed
an “x-row”; there are thus nine x-rows. Define “y-row” and “z-row” similarly.
To achieve 50, simply take away one x-row and one y-row (their union consists of
precisely five unit cubes).
To show that 50 is the minimum: Note that there cannot be two x-rows that are
both completely removed, as that would imply removing six unit cubes. (Similar
statements apply for y- and z-rows, of course.) It is also impossible for there to be one
x-row, one y-row, and one z-row that are all removed, as that would imply removing
seven unit cubes. Every x-, y-, or z-row that is not completely removed contributes
at least 2 square units to the surface area. Thus, the total surface area is at least
9 · 2 + 8 · 2 + 8 · 2 = 50.

13. [40] Having lost a game of checkers and my temper, I dash all the pieces to the ground
but one. This last checker, which is perfectly circular in shape, remains completely on
the board, and happens to cover equal areas of red and black squares. Prove that the
center of this piece must lie on a boundary between two squares (or at a junction of
four).

Solution: Suppose, for the sake of contradiction, that the problem is false. Evidently,
at least one boundary between adjacent squares must lie within our checker, or else the
checker would exist entirely within one square, meaning it would cover only one color.
Note also that a checker’s diameter is smaller than the side of any square of the board,
so there are at most two such boundaries within our checker (one in each direction).
Let ` be this, or one of these, boundaries. Draw a diameter d of the checker parallel
to `. Presumably, the strip of the checker between ` and d is part red, part black.
These red and black areas are unequal, however, because the center of the checker does
not lie on any boundary between squares. But, if we were to swap colors within this
strip, then the checker would have equal red and black areas, because then it would be

7
colored in a way such that flipping it across d swaps the colors. This shows that, the
way it is currently colored, the checker does not have equal red and black areas. This
gives us the desired contradiction.

14. [40] A number n is called bummed out if there is exactly one ordered pair of positive
integers (x, y) such that
bx2 /yc + by 2 /xc = n.
Find all bummed out numbers.
Answer: 2, 6, 8, 10
Solution: Suppose n is bummed out. If (a, b) is one solution for (x, y) to the given
equation bx2 /yc+by 2 /xc = n, then (b, a) is another, so the unique solution (a, b) better
have the property that a = b and n = 2a ≥ 2. In particular, n is an even positive
integer.
Now, if n = 2a ≥ 12, then setting x = a − 1 ≥ 5, y = a + 1 ≥ 7, we have
 2  2    
x y 4 4
+ = a−3+ + a+3+ = 2a = n,
y x a+1 a−1

so n cannot be bummed out.


Moreover, b12 /2c + b22 /1c = 4, so 4 is not bummed out. The only possibilities left are
2, 6, 8, and 10.
To check these, note that
 2  2
x y x2 x2 y 2 3x
n= + > −2 + + + ≥ −2 + √
3
y x 2y 2y x 4
so √
3
4
x< (n + 2) < .53(n + 2),
3
and similarly for y. So we only have to check x, y ≤ b.53(10 + 2)c = 6:

x\y 1 2 3 4 5 6
1 2 4 9 16 25 36
2 4 4 5 9 12 18
3 9 5 6 7 9 13
4 16 9 7 8 9 11
5 25 12 9 9 10 11
6 36 18 13 11 11 12

15. [50] Find, with proof, all positive integer palindromes whose square is also a palin-
drome.
Answer: A palindrome satisfies the requirement if and only if the sum of the squares
of its digits is less than 10. We may categorize these numbers this way:

• 3
• Any palindromic combination of 1s and 0s with at most nine 1s.

8
• Any palindrome consisting of a single 2 in the middle and 1s and 0s elsewhere,
with at most four 1s.
• 2000 . . . 0002
• 2000 . . . 0001000 . . . 0002

d
X
Solution: Let n := ai · 10i be a palindrome, where the ai are digits with ai = ad−i
i=0
and ad 6= 0. Then, if we let X
bk := ai aj
i+j=k

for all 0 ≤ k ≤ 2d, then


2d
X
2
n = bk · 10k
k=0

(this
Pd is2 not necessarily the decimal expansion of n2 , however). We have to show that
2
i=0 ai < 10 if and only if n is a palindrome.

Suppose di=0 a2i < 10. Then, by the AM-GM inequality, we have
P

X a2i + a2j d d
X X a2i X a2j 10 10
bk = ai aj ≤ ≤ + < + = 10.
i+j=k i+j=k
2 i=0
2 j=0
2 2 2

Thus, loosely speaking, no carrying is ever done in computing n × n by long multi-


plication, so the digit in the 10k place in n2 is precisely bk , and it’s easy to see that
bk = b2d−k and that b2d = a2d 6= 0. So n2 is indeed a palindrome, as desired.
Now suppose di=0 a2i ≥ 10. Here note that
P

X d
X d
X
bd = ai aj = ai ad−i = a2i ≥ 10.
i+j=d i=0 i=0

Thus, it cannot be true that, for all k, bk represents the 10k digit of n2 , because no
digit can be greater than or equal to 10. Let ` be the greatest such that b` does not
represent the 10` digit of n2 . We are trying to prove that n2 cannot be a palindrome.
Consider three cases:

• ad = a0 ≥ 4. In this case we must have ` ≥ 2d, because b2d = a2d > 10.
If a0 = 4, then n2 ends in the digit 6, but lies in the interval [16 · 102d , 25 · 102d ),
and so starts with either a 1 or a 2; thus, n2 cannot be a palindrome. Similarly,
if a0 = 5, then n2 ends in 5 but starts with 2 or 3; if a0 = 6, then n2 ends in 6
but starts with 3 or 4; if a0 = 7, then n2 ends in 9 but starts with 4, 5, or 6; if
a0 = 8, then n2 ends in 4 but starts with 6, 7 or 8; if a0 = 9, then n2 ends in 1
but starts with 8 or 9.
• ` ≥ 2d and ad = a0 ≤ 3.
Here we do something similar, but with a slight twist. The units digit of n2 is
a20 . Because ` ≥ 2d, n2 must be in the interval [(a20 + 1) · 102d , (a0 + 1)2 · 102d ),

9
which is certainly a subset of the interval [(a20 + 1) · 102d , a20 · 102d+1 ). No integer
in even this larger interval manages to start with the digit a20 , so n2 cannot be
palindromic.
• ` < 2d.
Here we can rest assured that n2 does have (2d + 1) digits — that is, the first
digit is in the 102d place. In order for n2 to be a palindrome, the digits in the 10k
and 102d−k places must always be the same.
Now b` , b`+1 , . . . , b2d had all better be less than 10, or else ` would be greater than
what it is. Thus, the numbers just listed do appear as the lowest digits of n2 in
left-to-right order, although they don’t appear as the highest (2d + 1 − `) digits
of n2 in right-to-left order. Thus, n2 cannot be a palindrome.

10
IXth Annual Harvard-MIT Mathematics Tournament
Saturday 25 February 2006

Team Round B: Solutions

Mobotics [135]
Spring is finally here in Cambridge, and it’s time to mow our lawn. For the purpose of these
problems, our lawn consists of little clumps of grass arranged in an m × n rectangular grid,
that is, with m rows running east-west and n columns running north-south. To be even more
explicit, we might say our clumps are at the lattice points
{(x, y) ∈ Z2 | 0 ≤ x < n and 0 ≤ y < m}.
Our machinery consists of a fleet of identical mowbots (or “mobots” for short). A mobot
is a lawn-mowing machine. To mow our lawn, we begin by choosing a formation: we place
as many mobots as we want at various clumps of grass and orient each mobot’s head in a
certain direction, either north or east (not south or west). At the blow of a whistle, each
mobot starts moving in the direction we’ve chosen, mowing every clump of grass in its path
(including the clump it starts on) until it goes off the lawn.
Because the spring is so young, our lawn is rather delicate. Consequently, we want to
make sure that every clump of grass is mowed once and only once. We will not consider
formations that do not meet this criterion.
One more thing: two formations are considered “different” if there exists a clump of grass
for which either (1) for exactly one of the formations does a mobot starts on that clump, or
(2) there are mobots starting on this clump for both the formations, but they’re oriented in
different directions.
As an example, one allowable formation for m = 2, n = 3 might be as follows:
· → ·

→ ·
1. [25] Prove that the maximum number of mobots you need to mow your lawn is m+n−1.

Solution: This is attainable if we place one mobot at each clump in the first row,
oriented north, and one mobot in the first column of each row except the first, oriented
east.
To show that at most m + n − 1 mobots can be used, note that each mobot must mow
at least one of the m + n − 1 clumps in union of the first row and last column.
2. [40] Prove that the minimum number of mobots you need to mow your lawn is
min{m, n}.

Solution: This is attainable if we place one mobot at each clump in the first column,
oriented toward the east, or one mobot at each clump in the last row, oriented toward
the north (whichever of the two is more efficient).
To show that at least min{m, n} mobots must be used, note that each of the clumps
(0, 0), (1, 1), (2, 2), . . . , (min{m, n} − 1, min{m, n} − 1) must be mowed by a different
mobot.

1
3. [15] Prove that, given any formation, each mobot may be colored in one of three colors
— say, white, black, and blue — such that no two adjacent clumps of grass are mowed
by different mobots of the same color. Two clumps of grass are adjacent if the distance
between them is 1. In your proof, you may use the Four-Color Theorem if you’re
familiar with it.

Solution: We can divide the coordinate plane into regions: Let’s say a point belongs
to Region 0 if the closest lattice point to it is not on the lawn, and each mobot M
owns a region that is the set of points for which the closest lattice point is on the lawn
and mowed by M . Applying the Four-Color Theorem to these regions, we note that all
the conditions demanded in the problem are satisfied. In particular at most 3 colors
are used on the mobots because every mobot region borders Region 0 and hence is not
colored the same color as Region 0.

4. [15] For n = m = 4, find a formation with 6 mobots for which there are exactly 12
ways to color the mobots in three colors as in problem 3. (No proof is necessary.)

Solution: Place north-oriented mobots at (0, 0), (1, 0), (2, 2), and (3, 2), and east-
oriented mobots at (2, 0) and (2, 1). Other formations are possible.

5. [40] For n, m ≥ 3, prove that a formation has exactly six possible colorings satisfying
the conditions in problem 3 if and only if there is a mobot that starts at (1, 1).

Solution: Let’s trace a path through all the mobots. There must be a mobot at
(0, 0), so start the path there. If that mobot is oriented to move up, then move to
the right; if that mobot is oriented to move to the right, then move up. In doing so,
you will meet another mobot, upon which you can repeat the above process, until you
leave the lawn.
In tracing this path, we’ve effectively ordered the mobots, starting with (0, 0). At any
point in this ordering, we have a number of choices for how to color the current mobot
without violating the condition with any previously colored mobot. Specifically, there
are three ways to color the mobot at (0, 0). If a mobot has x- or y-coordinate 0, there
are 2 ways to color it: it need only not be the same color as the previous mobot on
our trail. If a mobot has both x- and y-coordinates positive, then there is only 1 way
to color it: it cannot be the same color as whatever mobots mows the clump directly
south or directly west of its starting point.
Thus, if there is a mobot at (1, 1), our path must begin with either (0, 0)–(0, 1)–(1, 1)
or (0, 0)–(1, 0)–(1, 1), and in either case there are only 6 ways to do the coloring.
Otherwise, if there is no mobot at (1, 1), our path must begin with either (0, 0)–(0, 1)–
(0, 2) or (0, 0)–(1, 0)–(2, 0), and in either case there are already 3 × 2 × 2 = 12 ways to
do the coloring thus far.

Polygons [130]
6. [15] Suppose we have a regular hexagon and draw all its sides and diagonals. Into how
many regions do the segments divide the hexagon? (No proof is necessary.)

2
Answer: 24
Solution: An accurate diagram and a careful count yields the answer.
7. [25] Suppose we have an √octagon with all angles of 135◦ , and consecutive sides of
alternating length 1 and 2. We draw all its sides and diagonals. Into how many
regions do the segments divide the octagon? (No proof is necessary.)
Answer: 84
Solution: The easiest way to see the answer is to view the octagon as five unit
squares in a cross arrangement, with four half-squares wedged at the corners. The
center square is divided into 8 regions. The other 4 squares are each divided into
15 regions. The 4 half-squares are each divided into 4 regions. The answer is thus
8 + 4 × 15 + 4 × 4 = 84.
8. [25] A regular 12-sided polygon is inscribed in a circle of radius 1. How many chords
of the circle that join two of the vertices of the 12-gon have lengths whose squares are
rational? (No proof is necessary.)
Answer: 42
Solution: The chords joining vertices subtend minor arcs of 30◦ , 60◦ , 90◦ , 120◦ , 150◦ ,
or 180◦ . There are 12 chords of each of the first five kinds and 6 diameters. For a
chord with central angle θ, we can draw radii from the two endpoints of the chord to
the center of the circle. By the law of cosines, the square of the length of the chord is
1 + 1 − 2 cos θ, which is rational when θ is 60◦ , 90◦ , 120◦ , or 180◦ . The answer is thus
12 + 12 + 12 + 6 = 42.
9. [25] Show a way to construct an equiangular hexagon with side lengths 1, 2, 3, 4, 5,
and 6 (not necessarily in that order).

Solution: The trick is to view an equiangular hexagon as an equilateral triangle


with its corners cut off. Consider an equilateral triangle with side length 9, and cut
off equilateral triangles of side length 1, 2, and 3 from its corners. This yields an
equiangular hexagon with sides of length 1, 6, 2, 4, 3, 5 in that order.
10. [40] Given a convex n-gon, n ≥ 4, at most how many diagonals can be drawn such
that each drawn diagonal intersects every other drawn diagonal strictly in the interior
of the n-gon? Prove that your answer is correct.
Answer: bn/2c
Solution: If n is even, simply draw all n/2 diagonals connecting a vertex to the one
n/2 vertices away. If n is odd, pretend one of the vertices does not exist, and do the
above for the (n − 1)-gon remaining.
To show this is optimal, consider any given drawn diagonal: it divides the remaining
n − 2 vertices into two camps, one of which therefore has size at most bn/2c − 1, and
one cannot draw two diagonals sharing a vertex.

What do the following problems have in common? [135]


11. [15] Find the largest positive integer n such that 1! + 2! + 3! + · · · + n! is a perfect
square. Prove that your answer is correct.

3
Answer: 3
Solution: Clearly 1! + 2! + 3! = 9 works. For n ≥ 4, we have

1! + 2! + 3! + · · · + n! ≡ 1! + 2! + 3! + 4! ≡ 3 (mod 5),

but there are no squares congruent to 3 modulo 5.

12. [15] Find all ordered triples (x, y, z) of positive reals such that x + y + z = 27 and
x2 + y 2 + z 2 − xy − yz − zx = 0. Prove that your answer is correct.
Answer: (9, 9, 9)
(x − y)2 + (y − z)2 + (z − x)2
Solution: We have x2 + y 2 + z 2 − xy − yz − zx = = 0,
2
27
which implies x = y = z = = 9.
3
13. [25] Four circles with radii 1, 2, 3, and r are externally tangent to one another. Compute
r. (No proof is necessary.)
Answer: 6/23
Solution: Let A, B, C, P be the centers of the circles with radii 1, 2, 3, and r,
respectively. Then, ABC is a 3-4-5 right triangle. Using the law of cosines in 4P AB
yields
32 + (1 + r)2 − (2 + r)2 3−r
cos ∠P AB = =
2 · 3 · (1 + r) 3(1 + r)
Similarly,
42 + (1 + r)2 − (3 + r)2 2−r
cos ∠P AC = =
2 · 4 · (1 + r) 2(1 + r)
We can now use the equation (cos ∠P AB)2 + (cos ∠P AC)2 = 1, which yields 0 =
23r2 + 132r − 36 = (23r − 6)(r + 6), or r = 6/23.

14. [40] Find the prime factorization of

20062 · 2262 − 6692 · 3599 + 15932 · 1337.

(No proof is necessary.)


Answer: 2 · 3 · 7 · 13 · 29 · 59 · 61 · 191
Solution: Upon observing that 2262 = 669 + 1593, 3599 = 1593 + 2006, and 1337 =
2006 − 669, we are inspired to write a = 2006, b = 669, c = −1593. The expression in
question then rewrites as a2 (b − c) + b2 (c − a) + c2 (a − b). But, by experimenting in
the general case (e.g. setting a = b), we find that this polynomial is zero when two
of a, b, c are equal. Immediately we see that it factors as (b − a)(c − b)(a − c), so the
original expression is a way of writing (−1337) · (−2262) · (3599). Now, 1337 = 7 · 191,
2262 = 2 · 3 · 13 · 29, and 3599 = 602 − 12 = 59 · 61.

15. [40] Let a, b, c, d be real numbers so that c, d are not both 0. Define the function

ax + b
m(x) =
cx + d

4
on all real numbers x except possibly −d/c, in the event that c 6= 0. Suppose that the
equation x = m(m(x)) has at least one solution that is not a solution of x = m(x).
Find all possible values of a + d. Prove that your answer is correct.
Answer: 0
Solution: That 0 is a possible value of a + d can be seen by taking m(x) = −x, i.e.,
a = −d = 1, b = c = 0. We will now show that 0 is the only possible value of a + d.
(a2 + bc)x + (a + d)b
The equation x = m(m(x)) implies x = , which in turn implies
(a + d)cx + (bc + d2 )

(a + d)[cx2 + (−a + d)x − b] = 0.

Suppose for the sake of contradiction that a + d 6= 0. Then the above equation would
further imply

cx2 + (−a + d)x − b = 0, x(cx + d) = ax + b,

which would imply x = m(x) for any x except possibly −d/c. But of course −d/c is
not a root of x = m(m(x)) anyway, so in this case, all solutions of x = m(m(x)) are
also solutions of x = m(x), a contradiction. So our assumption was wrong, and in fact
a + d = 0, as claimed.

You might also like